You are on page 1of 775

MORE THAN 3000 QUESTION

QUESTION BANK OF
Dr. MOHAMED YAHIA

:‫بنـــــــــــــــــــك أسئـــــــــــــلة‬

‫ محمد يحيــــــــى‬.‫د‬
PHYSIOLOGY – MICROBIOLOGY – BIOCHEMISTRY –
PATHOLOGY – PHARMACOLOGY - ANATOMY

WRITTEN BY: Dr. MOHAMED YAHIA ABUELGASEM KHATEM


QUESTION BANK OF Dr. MOHAMED YAHIA

CHAPTER 1

1168 Question

2 ‫صفحة‬ Dr. MOHAMED YAHIA 0900987639


QUESTION BANK OF Dr. MOHAMED YAHIA

PRETEST PHYSIOLOGY

CARDIOVASCULAR SYSTEM
1. Which of the following is part of the reflex response to an increase in arterial
pressure?

a) Decreased firing of carotid sinus baroreceptors


b) Increased sympathetic activity to the ventricles
c) Increased parasympathetic activity to the SA Node
d) Increased parasympathetic activity to the arterioles of skeletal muscles & skin
e) Increased parasympathetic stimulation to the ventricles

The answer is: C

2. Correct sequences of steps in short-term compensation for hemorrhage


include:

a) Decreased arterial pressure → Increased baroreceptor firing rate


b) Increased formation of Angiotensin II → Increased renin released by kidneys
c) Decreased excretion of Na+ and water → Increased aldosterone formation
d) Decreased firing of baroreceptors → Increased sympathetic activity
e) Decreased atrial volume → Increased volume receptor firing rate

The answer is: D

3. Venous Return:

a) Is increased on standing
b) Decreases by deep inspiration
c) Is decreased by venoconstriction
d) When increased, activates Bainbridge reflex
e) When increased, increases end-systolic volume

The answer is: D

4. Which of the following is not a characteristic of cardiac muscle?

a) It's a syncytium of muscle fibers


b) There are intercalated discs
c) Gap junctions cause spread of depolarization
d) Has myosin and actin muscles
e) There are paracellular spaces

The answer is: E

3 ‫صفحة‬ Dr. MOHAMED YAHIA 0900987639


QUESTION BANK OF Dr. MOHAMED YAHIA

5. In a normal ECG:

a) The P wave indicates the condition of the conductive system


b) The P wave represents depolarization of atrial myocardium
c) QRS complex is mainly negative due to spread of depolarization down
d) QRS is due to depolarization in the ventricular septum
e) The T wave is a positive wave in all chest leads

The answer is: B

6. At rest the LV end-systolic volume is:

a) 10 mL
b) 30 mL
c) 50 mL
d) 120 mL
e) 140 Ml

The answer is: C

7. The best site to measure mixed venous PO2 is:

a) Superior Vena Cava


b) Right Atrium
c) Pulmonary Artery
d) Pulmonary Vein
e) Left Ventricle

The answer is: C

8. The atrial contraction component of ventricular filling is:

a) 5%
b) 10%
c) 30%
d) 50%
e) 80%

The answer is: C

9. The lowest intrinsic discharge activity resides in the:

a) SA Node
b) AV Node
c) Bundle of His
d) Bundle of branches
e) Purkinje fibers

The answer is: E

4 ‫صفحة‬ Dr. MOHAMED YAHIA 0900987639


QUESTION BANK OF Dr. MOHAMED YAHIA

10. The velocity of blood flow is the slowest in:

a) Capillaries
b) Pulmonary vein
c) Small arteries
d) Inferior Vena Cava
e) Arterioles

The answer is: A

11. The volume of blood is greatest in:

a) Systemic capillaries
b) Veins
c) Arteries
d) The spleen
e) The heart

The answer is: B

12. Cardiac Output (in liters per minute) divided by the heart rate (in beats per
minute) equals to:

a) Cardiac Index
b) Cardiac Efficiency
c) Mean Arterial Pressure
d) Stroke Volume
e) Blood Velocity

The answer is: D

13. The segment of the vascular bed responsible for local regulation of blood flow
in most tissues is:

a) Distributing arteries
b) Large veins
c) Capillaries
d) Venules
e) Arterioles

The answer is: E

14. First heart sound occurs at the beginning of:

a) Isometric relaxation
b) Isotonic relaxation
c) Isovolumetric contraction
d) Isovolumetric relaxation

5 ‫صفحة‬ Dr. MOHAMED YAHIA 0900987639


QUESTION BANK OF Dr. MOHAMED YAHIA

e) Atrial contraction

The answer is: C

15. Stimulation of baroreceptors leads to:

a) Tachycardia
b) Increased stroke volume
c) Stimulation of vasomotor center
d) Vasoconstriction
e) Decreased arterial blood pressure

The answer is: E

16. The fourth heart sound is due to:

a) Closure of mitral and tricuspid valve


b) Iso-volumetric contraction
c) Iso-volumetric relaxation
d) Ventricular filling
e) Atrial systole

The answer is: D

41. Frank Straling's law of the heart:

a) Explains the tachycardia caused by increased venouse return


b) Explains the tachycardia of exercise
c) Does not operate when the person is at rest
d) Explains the increased venous return when end-diastolic volume is
increased
e) Explains the increased stroke volume when end-diastolic volume is
increased

The answer is: E

17. Stroke Volume:

a) Increases as a result of increased afterload


b) Equals end-diastolic volume minus end-systolic volume
c) Increases as heart rate is increased by electrical pacing
d) Is increased by parasympathetic stimulation
e) Is increased by Ca2+ channel blocks

The answer is: B

6 ‫صفحة‬ Dr. MOHAMED YAHIA 0900987639


QUESTION BANK OF Dr. MOHAMED YAHIA

18. A drug that increases the heart rate from 70 to 100 beats per minute could
be:

a) Stimulation of B-1 adrenergic receptors


b) Inhibitor of alpha adrenergic receptors
c) Stimulation of muscarinic cholinergic receptors
d) Inhibitor B-2 adrenergic receptors
e) Digitalis

The answer is: A

19. Which one of the following is the best index of preload?

a. Blood volume
b. Central venous pressure
c. Pulmonary capillary wedge pressure
d. Left ventricular end-diastolic volume
e. Left ventricular end-diastolic pressure

The answer is: D

20. Which one of the following is the best index of afterload?

a. Left ventricular end-diastolic pressure


b. Left ventricular mean systolic pressure
c. Pulmonary capillary wedge pressure
d. Total peripheral resistance
e. Mean arterial blood pressure

The answer is: B

21. The highest blood flow per gram of left ventricular myocardium would occur

a. When aortic pressure is highest


b. When left ventricular pressure is highest
c. At the beginning of isovolumic contraction
d. When aortic blood flow is highest
e. At the beginning of diastole

The answer is: E

22. During ventricular ejection, the pressure difference smallest in magnitude is


between the

a. Pulmonary artery and left atrium


b. Right ventricle and right atrium
c. Left ventricle and aorta
d. Left ventricle and left atrium

7 ‫صفحة‬ Dr. MOHAMED YAHIA 0900987639


QUESTION BANK OF Dr. MOHAMED YAHIA

e. Aorta and capillaries

The answer is: C

23. Which of the following would result from a regurgitant aortic valve in a
nonfailing heart?

a. A decrease in diastolic pressure


b. A decrease in cardiac energy consumption
c. A systolic murmur
d. A decrease in heart rate
e. A decrease in systolic blood pressure

The answer is: A

24. The pacemaker prepotential:

a) Is due to a slow decrease in K+ influx


b) Is a slow increase in Resting Membrane Potential
c) Maintained by opening of long acting Ca2+ channels
d) Is augmented by opening of transient Ca2+ channels
e) Occurs only in the SA node

The answer is: E

25. Parasympathetic stimulation results in:

a) Decreases K+ efflux in cardiac muscle


b) Increases Ca2+ influx in SA node
c) Bradycardia
d) Increases Na+ influx
e) Increasing slope of prepotential

The answer is: C

26. Which of the following is true of the electrical activity of cardiac muscle?

a) Increased extracellular K+ causes depolarization


b) Repolarization is due to Na+ current
c) Extracellular Na+ affects the pacemaker potential
d) Plateu of action potential is due to Ca2+ influx
e) Initial repolarization is due to delayed K+ efflux

The answer is: D

27. Stroke volume is increased by

a. A decrease in venous compliance


b. An increase in afterload

8 ‫صفحة‬ Dr. MOHAMED YAHIA 0900987639


QUESTION BANK OF Dr. MOHAMED YAHIA

c. A decrease in contractility
d. An increase in heart rate
e. A decrease in coronary blood flow

The answer is: A

28. Which of the following is true of the cardiac cycle?

a) Ventricular diastole follows atrial systole


b) Ventricular systole causes an immediate rise in aortic pressure
c) The beginning of systole causes closure of AV valves
d) Closure of semi-lunar valves gives rise to first heart sound
e) Isovolumetric relaxation occurs at the end of diastole

The answer is: C

29. Pressure in the pulmonary artery is:

a) Higher than that in the right atrium


b) About 25 mmHg during systole
c) About 80 mmHg during diastole
d) Is lower than that in the right ventricle
e) Is regulated by parasympathetic innervations

The answer is: D

30. The following is not correct about baroreceptors:

a) Found in the wall of carotid sinuses


b) Found in the aortic arch
c) Send impulses through the glosso-pharyngeal nerve
d) Normally fires during late systole

The answer is: d

31. Chemoreceptors are:

a) Found in aortic and carotid sinuses


b) Stimulated best by hypercapnoea
c) Send impulses to vasomotor center through vagus nerve
d) Have a high blood flow rate
e) Are not sensitive to hydrogen ion concentration

The answer is: B

32. An increased preload would most likely be caused by an increase in

a. Arteriolar tone
b. Venous tone

9 ‫صفحة‬ Dr. MOHAMED YAHIA 0900987639


QUESTION BANK OF Dr. MOHAMED YAHIA

c. Myocardial contractility
d. Heart rate
e. Capillary permeability

The answer is: B

33. Propagation of the action potential through the heart is fastest in the

a. SA node
b. Atrial muscle
c. AV node
d. Purkinje fibers
e. Ventricular muscle

The answer is: D

34. Closure of the aortic valve occurs at the onset of which phase of the cardiac
cycle?

a. Isovolumetric contraction
b. Rapid ejection
c. Protodiastole
d. Isovolumetric relaxation
e. Rapid filling

The answer is: D

35. Which one of the following is most likely to be observed in a patient with
untreated atrial fibrillation?

a. An increased venous A wave


b. An increased left atrial pressure
c. A decreased heart rate
d. An increased stroke volume
e. An increased arterial blood pressure

The answer is: B

36. Normal splitting of the second heart sound (S 2 ) into two components is
increased during inspiration because

a. The closing of the aortic valve is delayed


b. The opening of the mitral valve is delayed
c. The closing of the pulmonic valve is delayed
d. The stroke volume of the left ventricle is increased
e. The heart rate is decreased

The answer is: C

10 ‫صفحة‬ Dr. MOHAMED YAHIA 0900987639


QUESTION BANK OF Dr. MOHAMED YAHIA

37. Which of the following statements about the third heart sound (S 3 ) is
correct?

a. It is usually diminished in congestive heart failure


b. It is produced by turbulence during rapid ventricular filling in early diastole
c. It is produced by turbulence following atrial contraction
d. It is often associated with the “floppy” mitral valve syndrome
e. It is produced by flow through the patent foramen ovale

The answer is: B

38. In a resting, healthy man, the ejection fraction is approximately?

a. 0.1
b. 0.2
c. 0.3
d. 0.6
e. 0.9

The answer is: D

39. Sustained elevation of cardiac output will occur with which of the following
conditions?

a. Hypertension
b. Aortic regurgitation
c. Anemia
d. Third-degree heart block
e. Cardiac tamponade

The answer is: C

40. A diastolic murmur accompanied by an increased preload can be caused by

a. Mitral stenosis
b. Mitral regurgitation
c. Atherosclerosis
d. Aortic regurgitation
e. Tachycardia

The answer is: D

41. Increasing vagal stimulation of the heart will cause an increase in

a. Heart rate
b. PR interval
c. Ventricular contractility
d. Ejection fraction

11 ‫صفحة‬ Dr. MOHAMED YAHIA 0900987639


QUESTION BANK OF Dr. MOHAMED YAHIA

e. Cardiac output

The answer is: B

42. Which of the following results in increased stroke volume?

a) An increase in end-diastolic volume


b) An increased after-load
c) Parasympathetic stimulation
d) An increase in the heart rate
e) Venodilation

The answer is: A

43. During the cardiac cycle, closure of the aortic valve occurs at:

a) The end of isovolumetric contraction


b) The beginning of rapid ejection phase
c) The beginning of isometric relaxation
d) The end of systole
e) The end of rapid filling phase

The answer is: C

44. SA node is the pacemaker of the heart because:

a) Location in the right atrium


b) Neural control
c) Natural leakiness to Cl-
d) Natural leakiness to K+
e) Fastest rate of discharge

The answer is: E

45. Stroke Volume:

a) Is the volume of blood pumped by the heart per minute


b) Decreases by increased end-diastolic volume
c) Is decreased by increased afterload
d) From the left ventricle is more than from the right ventricle
e) Multiplied by heart rate gives cardiac index

The answer is: C

46. In the ECG, the T wave denotes:

a) Atrial contraction
b) Atrial depolarization
c) Ventricular repolarization

12 ‫صفحة‬ Dr. MOHAMED YAHIA 0900987639


QUESTION BANK OF Dr. MOHAMED YAHIA

d) SA node depolarization
e) Ventricular contraction

The answer is: C

47. During exercise, there is an increase in a person’s

a. Stroke volume
b. Diastolic pressure
c. Venous compliance
d. Pulmonary arterial resistance
e. Total peripheral resistance

The answer is: A

48. Phase-4 depolarization of SA nodal cells is caused by

a. An increase in the flow of sodium into the cell


b. A decrease in the flow of potassium out of the cell
c. An increase in the activity of the Na/Ca exchanger
d. A decrease in the flow of chloride out of the cell
e. A decrease in the activity of the Na-K pump

The answer is: B

49. Cardiovascular changes that occur during inspiration include decreased

a. Right ventricular filling


b. Right ventricular output
c. Pressure gradient from extrathoracic veins to the right atrium
d. Systemic blood pressure
e. Left ventricular contractility

The answer is: D

50. Blood pressure increases and heart rate decreases in response to

a. Exercise
b. Increased body temperature
c. Exposure to high altitude
d. Increased intracranial pressure
e. Hemorrhage

The answer is: D

51. During exercise, cardiac output is augmented by

a. Sympathetic stimulation of resistance vessels


b. Dilation of venous vessels

13 ‫صفحة‬ Dr. MOHAMED YAHIA 0900987639


QUESTION BANK OF Dr. MOHAMED YAHIA

c. Decreased end-diastolic volume


d. Decreased mean systemic arterial pressure
e. Increased ventricular contractility

The answer is: E

52. When flow through the mitral valve is restricted by mitral stenosis,

a. Exercise can induce acute pulmonary edema


b. Left ventricular preload increases
c. Left atrial pressure diminishes
d. Right ventricular end-diastolic pressure decreases
e. Central venous pressure decreases

The answer is: A

53. Stroke volume can be decreased by

a. Increasing ventricular contractility


b. Increasing heart rate
c. Increasing central venous pressure
d. Decreasing total peripheral resistance
e. Decreasing systemic blood pressure

The answer is: B

54. An ectopic extrasystole caused by a ventricular focus is characterized by

a. Interruption of the regular SA node discharge


b. Retrograde conduction of the action potential to the atria
c. A skipped ventricular contraction
d. A skipped atrial contraction
e. A larger than normal force of contraction

The answer is: C

55. The electrocardiogram is most effective in detecting a decrease in

a. Ventricular contractility
b. Mean blood pressure
c. Total peripheral resistance
d. Ejection fraction
e. Coronary blood flow

The answer is: E

56. Stroke volume can be increased by

a. Decreasing ventricular compliance

14 ‫صفحة‬ Dr. MOHAMED YAHIA 0900987639


QUESTION BANK OF Dr. MOHAMED YAHIA

b. Increasing venous compliance


c. Decreasing total peripheral resistance
d. Increasing heart rate
e. Decreasing atrial contractility

The answer is: C

57. Which one of the following correctly describes an event that normally occurs
during the PR interval?

a. The ventricle is contracting


b. The cardiac action potential passes through the AV node
c. There is no change in the voltage tracing on the ECG
d. The mitral and aortic valves are both closed
e. The second heart sound is heard

The answer is: B

58. Which one of the following will be observed in a patient with aortic
regurgitation?

a. An increased ejection fraction


b. An increased mean blood pressure
c. An increased diastolic blood pressure
d. A decreased pulse pressure
e. A decreased left atrial end-diastolic pressure

The answer is: C

59. The diagnosis of a first-degree heart block is made if

a. The PR interval of the ECG is increased


b. The P wave of the ECG is never followed by a QRS complex
c. The P wave of the ECG is sometimes followed by a QRS complex
d. The T wave of the ECG is inverted
e. The ST segment of the ECG is elevated

The answer is: A

60. Increased pressure in the carotid sinus leads to:

a) An increase in vasomotor tone


b) Increase in vagal tone
c) Reflex vasoconstriction
d) Venoconstriction
e) Tachycardia

The answer is: B

15 ‫صفحة‬ Dr. MOHAMED YAHIA 0900987639


QUESTION BANK OF Dr. MOHAMED YAHIA

61. The Cardiac output is:

a) The volume of blood pumped by the heart in one minute


b) Equal to the heart rate multiplied by the stroke volume
c) Measured by dilution method
d) Increased on standing up
e) Greater in left ventricle than the right

The answer is: B

62. The chemoreceptors are found in:

a) The lungs
b) Glomus bodies
c) Aortic arch
d) Carotid sinus
e) Medulla

The answer is: E

63. Local vasodilation can be produced by:

a) Neural reflexes
b) Myogenic autoregulation
c) An increase in PO2
d) A decrease in H+ concentration
e) Synthesis of ATP

The answer is: B

64. Stimulation of parasympathetic nerves to the heart:

a) Causes tachycardia
b) Makes the prepotential more horizontal
c) Decreases the rate of potassium efflux in the sino-atrial node
d) Prolongs the refractory period
e) Shortens the duration of the cardiac cycle

The answer is: B

65. The upstroke of the SA nodal action potential is produced by opening a


channel that is

a. Primarily permeable to Na +
b. Primarily permeable to Ca 2+
c. Primarily permeable to K +
d. Primarily permeable to Cl −
e. Equally permeable to Na + and K +

16 ‫صفحة‬ Dr. MOHAMED YAHIA 0900987639


QUESTION BANK OF Dr. MOHAMED YAHIA

The answer is: B

67. The channel responsible for the initiation of phase-4 depolarization in SA


nodal cells

a. Is primarily permeable to Na +
b. Is opened by membrane depolarization
c. Is opened by vagal nerve stimulation
d. Is primarily permeable to K +
e. Is closed by norepinephrine

The answer is: A

68. Sympathetic stimulation of the heart results in

a. An increase in the activity of the SR calcium pump


b. An increase in the duration of systole
c. An increase in the duration of diastole
d. A decrease in the affinity of troponin for calcium
e. A decrease in the concentration of Ca 2+ during systole

The answer is: A

69. An exercise stress test to rule out ischemic heart disease is positive if

a. The systolic blood pressure rises


b. The ST segment of the ECG is depressed
c. The heart rate fails to increase
d. A diastolic murmur is heard
e. The diastolic blood pressure decreases

The answer is: B

70. After a mild hemorrhage, compensatory responses initiated by the


baroreceptor reflex keeps blood pressure at or close to its normal value. Which
one of the following values is less after compensation for the hemorrhage than it
was before the hemorrhage?

a. Venous compliance
b. Heart rate
c. Ventricular contractility
d. Total peripheral resistance
e. Coronary blood flow

The answer is: A

71. The constriction of a blood vessel to one-half of its resting diameter would
increase its resistance to blood flow by a factor of

17 ‫صفحة‬ Dr. MOHAMED YAHIA 0900987639


QUESTION BANK OF Dr. MOHAMED YAHIA

a. 2
b. 4
c. 8
d. 12
e. 16

The answer is: E

72. During aerobic exercise, blood flow remains relatively constant within

a. The skin
b. The heart
c. The brain
d. The skeletal muscles
e. The kidneys

The answer is: C

73. Which of the following conditions causes pulse pressure to increase?

a. Tachycardia
b. Hypertension
c. Hemorrhage
d. Aortic stenosis
e. Heart failure

The answer is: B

74. Sudden standing evokes the baroreceptor reflex. Which one of the following
will be greater after a person suddenly stands up than it was before the person
stood?

a. The end-diastolic volume


b. The renal blood flow
c. The venous return
d. The pulse pressure
e. The ejection fraction

The answer is: E

39. Central venous pressure is increased by

a. Decreasing blood volume


b. Increasing venous compliance
c. Increasing total peripheral resistance
d. Decreasing heart rate
e. Decreasing plasma aldosterone concentration

18 ‫صفحة‬ Dr. MOHAMED YAHIA 0900987639


QUESTION BANK OF Dr. MOHAMED YAHIA

The answer is: D

75. Net filtration from capillaries will increase following

a. Increased contraction of the precapillary sphincter


b. Decreased arterial pressure
c. Increased plasma protein concentration
d. Decreased capillary permeability
e. Increased postcapillary resistance

The answer is: E

76. A reduction in carotid sinus pressure would cause a decrease in

a. Heart rate
b. Myocardial contractility
c. Total peripheral resistance
d. Venous compliance
e. Cardiac output

The answer is: D

77. Which one of the following organs has the highest arteriovenous O 2
difference under normal resting conditions?

a. Brain
b. Heart
c. Skeletal muscle
d. Kidney
e. Stomach

The answer is: B

78. The percentage of the total cardiac output distributed to any single organ is
most dependent on

a. The contractile state of the heart


b. The magnitude of mean blood pressure
c. The magnitude of diastolic pressure
d. The ratio of an organ’s vascular resistance to total peripheral resistance (TPR)
e. The magnitude of cardiac output

The answer is: D

79. At which of the following sites does the blood flow lose the greatest amount of
energy?

a. Mitral valve
b. Large arteries

19 ‫صفحة‬ Dr. MOHAMED YAHIA 0900987639


QUESTION BANK OF Dr. MOHAMED YAHIA

c. Arterioles
d. Capillaries
e. Venules

The answer is: C

80. The greatest percentage of blood volume is found in the

a. Heart
b. Aorta
c. Distributing arteries and arterioles
d. Capillaries
e. Venules and veins

The answer is: E

81. Turbulence is more likely to occur in a blood vessel if

a. The velocity of blood within the vessel increases


b. The viscosity of blood within the vessel increases
c. The diameter of the vessel decreases
d. The density of the blood decreases
e. The length of the vessel increases

The answer is: A

82. Which of the following will be lower than normal in a patient with an
abnormally high intracranial pressure?

a. Ventricular contractility
b. Heart rate
c. Mean blood pressure
d. Stroke volume
e. Total peripheral resistance

The answer is: B

83. Which one of the following increases during aerobic exercise?

a. Diastolic blood pressure


b. Cerebral vascular resistance
c. Mixed venous oxygen tension
d. Blood flow to the kidney
e. Circulating blood volume

The answer is: B

20 ‫صفحة‬ Dr. MOHAMED YAHIA 0900987639


QUESTION BANK OF Dr. MOHAMED YAHIA

84. Pulse pressure increases when

a. Heart rate increases


b. Stroke volume decreases
c. Aortic compliance increases
d. Aortic stenosis develops
e. Mean arterial pressure increases

The answer is: E

85. The distribution of blood among the various organs of the body is regulated
by regulating the resistance of the

a. Arteries
b. Arterioles
c. Precapillary sphincters
d. Postcapillary venules
e. Veins

The answer is: B

86. Which of the following results in a decreased stroke volume?

a) An increase in end-diastolic volume


b) An increased afterload
c) Parasympathetic stimulation
d) A decrease in total peripheral resistance
e) Increased residual volume

The answer is: B

87. Stroke volume is decreased when:

a) The sympathetic nerves are stimulated


b) The arterial blood pressure falls
c) Vagal centers are stimulated
d) The end-diastolic volume is increased
e) A patient stands up

The answer is: E

88. Which of the following is the most important in determining the total
peripheral resistance?

a) a)Blood viscosity
b) Concentration of plasma protein
c) Arteriolar diameter
d) Cardiac Output

21 ‫صفحة‬ Dr. MOHAMED YAHIA 0900987639


QUESTION BANK OF Dr. MOHAMED YAHIA

e) Metabolic autoregulation

The answer is: C

89. Which of the following substances will be most likely to dilate systemic
arterioles:

a) Endothelin
b) ADH
c) Histamine
d) Noreadrenaline
e) Aldosterone

The answer is: C

90. Resistance to venous return:

a) Is higher during exercise than at rest


b) Equals to the slope of the venous return curve
c) Is reduced by sympathetic stimulation
d) Increases by increase in circulatory filling pressure
e) Increased by constriction of arterioles

The answer is: E

91. Factors that increase the stroke volume include:

a) An increased afterload
b) A high end-diastolic volume
c) An increased heart rate
d) Parasympathetic stimulation
e) Ca2+ channel blockers

The answer is: B

92. After a loss of one liter of blood:

a) The heart rate decreases slightly


b) Baroreceptors increase their discharge
c) The chemoreceptors have no role
d) The end diastolic volume increases
e) There is increased vasomotor tone

The answer is: E

93. A patient is diagnosed with anaphylactic shock rather than hypovolumic


shock because

a. Cardiac output is higher than normal

22 ‫صفحة‬ Dr. MOHAMED YAHIA 0900987639


QUESTION BANK OF Dr. MOHAMED YAHIA

b. Ventricular contractility is greater than normal


c. Total peripheral resistance is greater than normal
d. Serum creatinine is elevated
e. Heart rate is greater than normal

The answer is: A

94. Which one of the following will increase if massaging the neck stretches the
carotid sinus baroreceptor?

a. Total peripheral resistance


b. Right atrial pressure
c. Venous tone
d. Ventricular contractility
e. Vagal nerve activity

The answer is: E

RESPIRATORY SYSTEM
1. The physiological dead space:

a) Is exactly equal to the anatomic dead space in normal people


b) Increases during exercise
c) Is measured by single breath nitrogen analysis
d) Is higher in elderly than young adults
e) Is the same as the physiological shunt
The answer is: D

2. A high alveolo-arterial PO2 difference most likely results from:

a) Low fractional concentration of O2 in the inspired air


b) Hypoventilation
c) Arteriovenous shunt
d) Pulmonary edema
e) Polycythemia
The answer is: D

3. Which of the following combinations are typical findings in compensated


respiratory alkalosis:

a) Low PaCO2, low bicarbonate level and normal pH


b) Low PaCO2, normal bicarbonate level and high pH
c) Low PaCO2, low bicarbonate level and high pH
d) Low PaCO2, normal bicarbonate level and normal pH
e) Low PaCO2, high bicarbonate level and high Ph

The answer is: A

23 ‫صفحة‬ Dr. MOHAMED YAHIA 0900987639


QUESTION BANK OF Dr. MOHAMED YAHIA

4. Compliance of the lung:

a) Is higher in small children than in adults


b) Is about 500 ml/cm H2O
c) Is normally twice the compliance of the chest
d) Is higher in elderly than young adults
e) Is decreased in emphysematous changes

The answer is: D

5. Dennervation of carotid and aortic bodies leads to all the following except:

a) Decreased sensitivity of ventilation to change in PCO2


b) Decreased sensitivity of ventilation to change in pH
c) Complete absence of response of ventilation to change in PO2
d) Absence response of ventilation to exercise
e) Decreased sensitivity of ventilaton to stagnant hypoxia

The answer is: A

6. Exchange of which of the following gases across the respiratory membrane is


normally diffusion limited:

a) Oxygen
b) Carbon dioxide
c) Carbon monoxide
d) Nitrous oxide
e) Nitrogen

The answer is: C

7. Minute ventilation is:

a) The volume of air moved in or out of the alveoli per minute


b) The volume of air moved in or out of the lung per breath
c) Alveolar ventilation plus dead space ventilation
d) Invariably increases if the respiratory rate increases
e) Is measured using Bohr's equation (CO2 analysis in expired air)

The answer is: A

8. The ventilatory response to hypoxia:

a) Is independent of arterial PCO2


b) Is mediated by the central chemoreceptors
c) Decreases gradually with time
d) Is largely mediated by the aortic bodies in humans
e) None of the above

24 ‫صفحة‬ Dr. MOHAMED YAHIA 0900987639


QUESTION BANK OF Dr. MOHAMED YAHIA

The answer is: D

9. With respect to oxygen and carbon dioxide transport in the blood:

a) For the same partial pressure a unit of blood carries the same amount of O2
and CO2
b) High PCO2 favors O2 binding to hemoglobin
c) Metabolic acidosis reduces PCO2
d) Oxygenation of hemoglobin increases the affinity of hemoglobin to CO2
e) Per unit volume of arterial blood there is more O2 than CO2

The answer is: C

10. The residual volume can be calculated by subtracting the expiratory reserve
volume from:

a) Vital Capacity
b) Inspiratory capacity
c) Functional residual capacity
d) Total lung capacity
e) Alveolar ventilation

The answer is: C

11. Ventilation is most sensitive to changes in:

a) PO2
b) pH of arterial blood
c) PCO2
d) Body temperature
e) % saturation of hemoglobin

The answer is: C

12. Inspiration is initiated by discharge from:

a) The dorsal inspiratory group of neurons


b) The ventral group of neurons
c) The preBottzinger complex
d) Apneustic center
e) Pneumotaxic center

The answer is: A

13. Voluntary hyperventilation by a normal person is expected to lead to all the


following except:

a) Alkalosis
b) Hypocapnia

25 ‫صفحة‬ Dr. MOHAMED YAHIA 0900987639


QUESTION BANK OF Dr. MOHAMED YAHIA

c) Low blood bicarbonate concentration in arterial blood


d) A great increase in oxygen concentration in arterial blood
e) Low ionized plasma calcium

The answer is: C

14. Obstructive lung disease is typically characterized by:

a) Reduced total lung capacity


b) Reduced FEV1
c) Reduced functional residual capacity
d) Slightly increased FEV1/FVC ratio
e) Reduced residual volume

The answer is: B

15. All the following are expected to decrease airway resistance except:

a) Atropine
b) Adrenaline
c) Parasympathetic stimulation
d) Sympathetic stimulation
e) Shifting from nose to mouth breathing

The answer is: C

16. Which one of the following would increase in obstructive, but not in
restrictive, lung disease?

a. Vital capacity
b. Maximum breathing capacity
c. FEV 1
d. Functional residual capacity
e. Breathing frequency

The answer is: D

17. During the early stages of an asthmatic attack,

a. Arterial carbon dioxide tension decreases


b. The equal pressure point moves toward the mouth
c. Lung compliance increases
d. Airway resistance decreases
e. Arterial oxygen tension increases

The answer is: A

18. Which one of the following will decrease in a person with ventilation-
perfusion (V/Q) abnormalities?

26 ‫صفحة‬ Dr. MOHAMED YAHIA 0900987639


QUESTION BANK OF Dr. MOHAMED YAHIA

a. Anion gap
b. Arterial pH
c. Arterial carbon dioxide tension
d. Alveolar-arterial (A-a) gradient for oxygen
e. Alveolar ventilation

The answer is: C

19. Which one of the following is higher at the apex of the lung than at the base
when a person is standing?

a. V/Q ratio
b. Blood flow
c. Ventilation
d. Pa CO 2
e. Lung compliance

The answer is: A

20. In areas of the lung with lower than normal V/Q ratios, the

a. Capillary CO 2 tension is lower than normal


b. Pulmonary vascular resistance is higher than normal
c. Alveolar O 2 tension is higher than normal
d. Water vapor pressure is higher than normal
e. Gas exchange ratio is higher than normal

The answer is: B

21. A decrease in the recoil force of the lung is likely to:

a) Decrease total lung capacity


b) Increase vital capacity
c) Increase functional residual capacity
d) Decrease the residual volume
e) Decrease compliance of the lung

The answer is: C

22. Central chemoreceptors differ from chemoreceptors in that they:

a) Respond to changes in PaCO2


b) Respond to changes in pH
c) Do not respond to changes in PO2
d) Become more sensitive to CO2 with chronic exposure
e) They are less sensitive to changes in PaCO2 than Ph

The answer is: C

27 ‫صفحة‬ Dr. MOHAMED YAHIA 0900987639


QUESTION BANK OF Dr. MOHAMED YAHIA

23. Surfactant:

a) Helps to equalize pressure within interconnected alveoli


b) Reduces surface tension more effectively as alveolus becomes smaller
c) Increases lung compliance
d) Increases the slope of the pressure-volume curve
e) All of the above

The answer is: E

24. At the end of maximum inspiration:

a) Intra-alveolar pressure is zero


b) Intra-pleural pressure is zero
c) Recoil force of the chest is greater than the recoil force of the lung
d) Surface tension is lowest
e) Average alveolar PO2 is lower than at the end of maximum expiration

The answer is: C

25. Which of the following conditions causes a decrease in arterial O 2 saturation


without a decrease in O 2 tension?

a. Anemia
b. Carbon monoxide poisoning
c. A low V/Q ratio
d. Hypoventilation
e. Right-to-left shunt

The answer is: A

26. The bulk of CO 2 is transported in arterial blood as

a. Dissolved CO 2
b. Carbonic acid
c. Carbaminohemoglobin
d. Bicarbonate
e. Carboxyhemoglobin

The answer is: D

27. Peripheral and central chemoreceptors may both contribute to the increased
ventilation that occurs as a result of

a. A decrease in arterial oxygen content


b. A decrease in arterial blood pressure
c. An increase in arterial carbon dioxide tension
d. A decrease in arterial oxygen tension

28 ‫صفحة‬ Dr. MOHAMED YAHIA 0900987639


QUESTION BANK OF Dr. MOHAMED YAHIA

e. An increase in arterial Ph

The answer is: C

28. Complete transection of the brainstem above the pons would

a. Result in cessation of all breathing movements


b. Prevent any voluntary holding of breath
c. Prevent the central chemoreceptors from exerting any control over ventilation
d. Prevent the peripheral chemoreceptors from exerting any control over
ventilation
e. Abolish the Hering-Breuer reflex

The answer is: B

29. A deficiency of pulmonary surfactant would

a. Decrease surface tension in the alveoli


b. Decrease the change in intrapleural pressure required to achieve a given
tidal volume
c. Decrease lung compliance
d. Decrease the work of breathing
e. Increase functional residual capacity (FRC)

The answer is: C

30. Which one of the following components of a pulmonary function test will be
closest to normal in a patient with restrictive lung disease?

a. FEV 1
b. MVV
c. FVC
d. FEV 1 /FVC
e. TLC

The answer is: B

31. Measurement of the lecithin-sphingomyelin (L-S) ratio in amniotic fluid


assesses

a. The placenta’s ability to oxygenate the fetus


b. Fetal adrenal function
c. Fetal kidney development
d. Fetal brain development
e. Fetal lung maturity

The answer is: E

32. When the respiratory muscles are relaxed, the lungs are at

29 ‫صفحة‬ Dr. MOHAMED YAHIA 0900987639


QUESTION BANK OF Dr. MOHAMED YAHIA

a. Residual volume (RV)


b. Expiratory reserve volume (ERV)
c. Functional residual capacity (FRC)
d. Inspiratory reserve volume (IRV)
e. Total lung capacity (TLC)

The answer is: C

33. Which one of the following is the most likely cause of a high arterial P CO 2 ?

a. Increased metabolic activity


b. Increased alveolar dead space
c. Depressed medullary respiratory centers
d. Alveolar capillary block
e. Increased alveolar ventilation

The answer is: C

34. PCO2 in arterial blood:

a) Invariably increases with hypoxia


b) Stimulates ventilation mainly through peripheral chemoreceptors
c) Is the major controller of ventilation
d) Increases early in exercise
e) Increases on ascent to high altitude

The answer is: C

35. Which of the following is likely to decrease airway resistance?

a) Beta-adrenergic blockers
b) Muscarinic cholinergic agonists
c) Breathing at higher lung volumes
d) Leukotrienes
e) Histamine
The answer is: C

36. Deficiency of surfactant is likely to result in:

a) Increased compliance
b) Decreased work of breathing
c) Reduced surface tension of fluid in the alveoli
d) Decreased compliance
e) Asthma

The answer is: D

30 ‫صفحة‬ Dr. MOHAMED YAHIA 0900987639


QUESTION BANK OF Dr. MOHAMED YAHIA

37. In which of the following conditions is the percent saturation of hemoglobin


in arterial blood likely to be normal :

a) Venoarterial shunts
b) Pulmonary edema
c) Carbon monoxide poisoning
d) Anemia
e) Methemoglobinemia
The answer is: D

38. Immediate acclimatization to high altitude is by:

a) Polycythemia
b) Hyperventilation
c) Increased cardiac output
d) Shift of hemoglobin dissociation curve to the left
e) Increase formation of fetal hemoglobin

The answer is: B

39. At the end of maximal expiration the volume of air in the lungs is:

a) FRC - RV
b) IRV + RV
c) FRC - TV
d) TLC - VC
e) VC – ERV – IRV
The answer is: D

40. While repaying the O2 dept:

a) The [H+] is increasing


b) PCO2 is decreasing
c) Muscle blood flow is increasing
d) Respiratory rate is increasing
e) Minute volume is decreasing

The answer is: B

41. Because the small airways produce a small fraction of the airway resistance,
their obstruction is difficult to detect. Which one of the following changes is
indicative of small rather than large airway obstruction?

a. An increase in maximum airflow during a forced expiration


b. A decrease in the maximum expiratory pressure that can be generated
c. An increase in the turbulent airflow produced during inspiration
d. A decrease in the forced vital capacity
e. An increase in the closing volume of the lung

31 ‫صفحة‬ Dr. MOHAMED YAHIA 0900987639


QUESTION BANK OF Dr. MOHAMED YAHIA

The answer is: E

45. Pulmonary vascular resistance decreases if

a. The lungs are inflated to total lung capacity


b. Sympathetic stimulation to the pulmonary vessels is increased
c. Alveolar oxygen tension is decreased
d. Plasma hydrogen ion concentration is decreased
e. Cardiac output is increased

The answer is: E

46. Which of the following would normally be less in the fetus than in the
mother?

a. Pa CO 2
b. Pulmonary vascular resistance
c. Affinity of hemoglobin for oxygen
d. Pa O 2
e. Arterial hydrogen ion concentration

The answer is: D

47. An increase in the P 50 of an oxyhemoglobin curve would result from a


decrease in

a. Metabolism
b. pH
c. Temperature
d. Oxygen
e. 2,3-DPG

The answer is: B

48. During moderate aerobic exercise,

a. Pa O 2 increases
b. Pa CO 2 decreases
c. Arterial pH decreases
d. Alveolar ventilation increases
e. Blood lactate level increases

The answer is: B

49. Reduction of functional hemoglobin associated with anemia,


methemoglobinemia, or carbon monoxide poisoning does not produce hyper-
pnea because the

a. Blood flow to the carotid body is decreased

32 ‫صفحة‬ Dr. MOHAMED YAHIA 0900987639


QUESTION BANK OF Dr. MOHAMED YAHIA

b. Total arterial oxygen content is maintained within the normal range


c. Carotid body chemoreceptors are stimulated
d. Central chemoreceptors are stimulated
e. P O 2 of arterial blood is normal

The answer is: E

50. Pulmonary alveoli are kept dry by factors that include the

a. Phagocytic activity of alveolar macrophages


b. Negative interstitial fluid pressure
c. Low vapor pressure of water in inspired air
d. Secretion of surfactant
e. Tight junctions between the alveolar capillary endothelial cells

The answer is: E

51. In which one of the following conditions will the diffusing capacity of the lung
increase?

a. Formation of pulmonary emboli


b. Fibrotic lung disease
c. Polycythemia
d. Congestive heart failure
e. COPD

The answer is: C

52. The percentage of hemoglobin saturated with oxygen will increase if

a. The arterial P CO 2 is increased


b. The hemoglobin concentration is increased
c. The temperature is increased
d. The arterial P O 2 is increased
e. The arterial pH is decreased

The answer is: D

53. Which of the following will return toward normal during acclimatization to
high altitude?

a. Arterial hydrogen ion concentration


b. Arterial carbon dioxide tension
c. Arterial bicarbonate ion concentration
d. Arterial hemoglobin concentration
e. Alveolar ventilation

The answer is: A

33 ‫صفحة‬ Dr. MOHAMED YAHIA 0900987639


QUESTION BANK OF Dr. MOHAMED YAHIA

54. Which one of the following statements characterizes pulmonary compliance?

a. It decreases with advancing age


b. It is inversely related to the elastic recoil properties of the lung
c. It increases in patients with pulmonary edema
d. It is equivalent to DP/DV
e. It increases when there is a deficiency of surfactant

The answer is: B

55. The activity of the central chemoreceptors is stimulated by

a. An increase in the P CO 2 of blood flowing through the brain


b. A decrease in the P O 2 of blood flowing through the brain
c. A decrease in the oxygen content of blood flowing through the brain
d. A decrease in the metabolic rate of the surrounding brain tissue
e. An increase in the pH of the CSF

The answer is: A

56. In an acclimatized person at high altitudes, oxygen delivery to the tissues may
be adequate at rest because of

a. An increase in hemoglobin concentration


b. The presence of an acidosis
c. A decrease in the number of tissue capillaries
d. The presence of a normal arterial P O 2
e. The presence of a lower-than-normal arterial P CO 2

The answer is: A

57. Which of the following will increase as a result of stimulating


parasympathetic nerves to the bronchial smooth muscle?

a. Lung compliance
b. Airway diameter
c. Elastic work of breathing
d. Resistive work of breathing
e. Anatomic dead space

The answer is: D

58. During a normal inspiration, more air goes to the alveoli at the base of the
lung than to the alveoli at the apex of the lung because

a. The alveoli at the base of the lung have more surfactant


b. The alveoli at the base of the lung are more compliant
c. The alveoli at the base of the lung have higher V/Q ratios

34 ‫صفحة‬ Dr. MOHAMED YAHIA 0900987639


QUESTION BANK OF Dr. MOHAMED YAHIA

d. There is a more negative intrapleural pressure at the base of the lung


e. There is more blood flow to the base of the lung

The answer is: B

59. A spirometer can be used to measure directly

a. Functional residual capacity


b. Inspiratory capacity
c. Residual volume
d. Total lung capacity
e. Physiological dead space

The answer is: B

60. The affinity of hemoglobin for oxygen is increased by

a. Metabolic acidosis
b. Exercise
c. Hypoxemia
d. Anemia
e. Carbon monoxide poisoning

The answer is: E

61. The oxygen consumption of the respiratory muscles is decreased by

a. A decrease in lung compliance


b. A decrease in airway resistance
c. An increase in the rate of respiration
d. A decrease in the production of pulmonary surfactant
e. An increase in tidal volume

The answer is: B

62. Which one of the following characteristics is the most similar between the
pulmonary and skeletal muscle capillaries?

a. The interstitial protein concentration


b. The interstitial hydrostatic pressure
c. The capillary oncotic pressure
d. The capillary hydrostatic pressure
e. The capillary permeability to proteins

The answer is: C

63. An increase in pulmonary blood flow during exercise

a. Causes alveolar oxygen tension to decrease

35 ‫صفحة‬ Dr. MOHAMED YAHIA 0900987639


QUESTION BANK OF Dr. MOHAMED YAHIA

b. Causes the V/Q ratio at the top of the lung to increase


c. Causes pulmonary arterial resistance to decrease
d. Causes diffusion capacity to decrease
e. Causes arterial oxygen saturation to decrease

The answer is: C

64. Which one of the following will be greater than normal in a patient with a
low V/Q ratio?

a. Pa CO 2
b. Pa O 2
c. A-a gradient
d. Oxygen dissolved in blood
e. Oxygen combined with hemoglobin

The answer is: A

65. Which one of the following contributes to the normal difference between the
alveolar and arterial oxygen tension (A-a) gradient?

a. The low P O 2 in the mixed venous blood


b. The inability of pulmonary capillary blood to equilibrate with alveolar gas
c. The low solubility of oxygen in blood
d. The shape of the oxyhemoglobin saturation curve
e. The high affinity of hemoglobin for oxygen in arterial blood

The answer is: D

66. A patient with inadequate surfactant will have a relatively normal

a. FEV 1
b. FVC
c. FEV 1 /FVC
d. MVV
e. V/Q ratio

The answer is: C

67. When a person ascends to a high altitude, alveolar ventilation increases.


Alveolar ventilation continues to increase over the next several days because

a. The central chemoreceptors become more sensitive to low oxygen tensions


b. The peripheral chemoreceptors increase their firing rate
c. The plasma concentration of 2,3-DPG increases
d. The pH of the cerebrospinal fluid decreases
e. The oxygen-carrying capacity of hemoglobin increases

36 ‫صفحة‬ Dr. MOHAMED YAHIA 0900987639


QUESTION BANK OF Dr. MOHAMED YAHIA

The answer is: D

68. The clinical sign of cyanosis is caused by

a. An increase in the affinity of hemoglobin for oxygen


b. A decrease in the percent of red blood cells (hematocrit)
c. An increase in the concentration of carbon monoxide in the venous blood
d. A decrease in the concentration of iron in the red blood cells
e. An increase in the concentration of deoxygenated hemoglobin

The answer is: E

69. Which one of the following gases diffuses across the alveoli-capillary
membrane by a diffusion-limited transport process?

a. Oxygen
b. Nitrogen
c. Carbon dioxide
d. Carbon monoxide
e. Nitrous oxide (N 2 O)

The answer is: D

70. Hyperventilation normally occurs during

a. Pregnancy
b. Sleep
c. Morphine administration
d. Exercise
e. Metabolic alkalosis

The answer is: A

71. A person ascends to the top of a mountain where the atmospheric pressure is
below normal. Which one of the following blood gases was drawn from the
person at the top of the mountain?

PO2 P CO 2

a. 50 30
b. 60 40
c. 80 50
d. 100 40
e. 120 30

The answer is: A

72. Hyperventilation in response to a stressful situation leads to

37 ‫صفحة‬ Dr. MOHAMED YAHIA 0900987639


QUESTION BANK OF Dr. MOHAMED YAHIA

a. A decrease in the blood flow to the brain


b. An increase in the activity of the central chemoreceptors
c. A decrease in pH of the arterial blood
d. An increase in the resistance of the pulmonary blood vessels
e. A decrease in the excitability of nerve and muscle cells

The answer is: A

73. A patient with reduced VC, FRC, and RV is found to have a normal pH. A
tentative diagnosis of diffuse interstitial fibrosis is made. Which of the following
characteristics are consistent with this disease?

a. An increase in lung compliance


b. A decrease in respiratory rate
c. An increase in the V/Q ratio
d. A decrease in Pa CO 2
e. An increase in the FEV 1 /FVC ratio

The answer is: E

74. Airway resistance is lowest

a. During a forced expiration


b. At the total lung capacity
c. At the residual volume
d. During vagal stimulation
e. When breathing gas with low oxygen

The answer is: B

GIT
1. The physiological actions of CCK include:

a) Increased oesophageal motility


b) Closing the sphincter of oddi
c) Increased bile synthesis
d) Contraction of pancreatic acini
e) Increased enzyme secretion from the pancreas
The answer is: E

2. The enzymes produced by the exocrine pancreas include the following except:

a) Proelastase
b) DNase
c) Amylase
d) Enteropeptidase
e) Lipase

38 ‫صفحة‬ Dr. MOHAMED YAHIA 0900987639


QUESTION BANK OF Dr. MOHAMED YAHIA

The answer is: D

3. The secretion of the exocrine pancreas is increased by the following except:

a) Vagal stimulation
b) Acetylcholine
c) Secretin
d) CCK
e) Atropine
The answer is: E

4. The jejunum is the main site for absorption of the following except:

a) Glucose
b) Amino acids
c) Fatty acids
d) Bile salts
e) Water

The answer is: D

5. A tumor producing large amounts of the intestinal hormone GIP may result
in:

a) Hyperglycemia
b) Hypoglycemia
c) Diarrhea
d) Intestinal colic
e) Hyperacidity in the stomach

The answer is: B

6. Chronic administration of which of the following types of drugs would lead to


a sustained increase in serum gastrin levels?

a. H 2 receptor antagonist
b. Proton pump inhibitor
c. Anticholinergic
d. Antacid
e. Beta blocker

The answer is: B

7. Which one of the following statements about small intestinal motility is


correct?

a. Contractile frequency is constant from duodenum to terminal ileum


b. Peristalsis is the major contractile pattern during feeding

39 ‫صفحة‬ Dr. MOHAMED YAHIA 0900987639


QUESTION BANK OF Dr. MOHAMED YAHIA

c. Migrating motor complexes occur during the digestive period


d. Vagotomy abolishes contractile activity during the digestive period
e. Contractile activity is initiated in response to bowel wall distention

The answer is: E

8. Which one of the following statements about gastric emptying is correct?

a. Solids empty more rapidly than liquids


b. Vagotomy accelerates the emptying of solids
c. Indigestible food empties during the digestive period
d. Acidification of the antrum decreases gastric emptying
e. Vagotomy decreases accommodation of the proximal stomach

The answer is: E

9. Vitamin B 12 is absorbed primarily in the

a. Stomach
b. Duodenum
c. Jejunum
d. Ileum
e. Colon

The answer is: D

10. The principal paracrine secretion involved in the inhibitory feedback


regulation of gastric acid secretion is

a. Gastrin
b. Somatostatin
c. Histamine
d. Enterogastrone
e. Acetylcholine

The answer is: B

11. Which one of the following is the putative inhibitory neurotransmitter


responsible for relaxation of gastrointestinal smooth muscle?

a. Dopamine
b. Vasoactive intestinal peptide
c. Somatostatin
d. Substance P
e. Acetylcholine

The answer is: B

12. Cholera toxin causes diarrhea by inhibiting

40 ‫صفحة‬ Dr. MOHAMED YAHIA 0900987639


QUESTION BANK OF Dr. MOHAMED YAHIA

a. Neutral NaCl absorption in the small intestine


b. Electrogenic Na absorption from the small intestine
c. Na-glucose coupled absorption from the small intestine
d. Na/H exchange in the small intestine
e. Electrogenic Na absorption from the colon

The answer is: A

13. Which one of the following statements best describes water and electrolyte
absorption in the GI tract?

a. Most water and electrolytes come from ingested fluids


b. The small intestine and colon have similar absorptive capacities
c. Osmotic equilibration of chyme occurs in the stomach
d. The majority of absorption occurs in the jejunum
e. Water absorption is independent of Na + absorption

The answer is: D

14. Hypokalemic metabolic acidosis can occur with excess fluid loss from the

a. Stomach
b. Ileum
c. Colon
d. Pancreas
e. Liver

The answer is: C

15. Which gastrointestinal motor activity is most affected by vagotomy?

a. Secondary esophageal peristalsis


b. Distention-induced intestinal segmentation
c. Orad stomach accommodation
d. Caudad stomach peristalsis
e. Migrating motor complexes

The answer is: C

16. The following statements are true about bile except:

a) 25% is produced by bile duct cells


b) The bile stored in the gall bladder contains secondary bile acids
c) Its secretion is increased by secretin
d) Bile pigments are needed for micelle formation
e) Some lipids will be digested and absorbed in the absence of bile

The answer is: B

41 ‫صفحة‬ Dr. MOHAMED YAHIA 0900987639


QUESTION BANK OF Dr. MOHAMED YAHIA

17. After a cholecystoctomy, patients may have problems with fat-rich foods
becuase:

a) The liver stops producing enough bile


b) Bile pigments are reduced in amount
c) The enterohepatic circulation has stopped
d) Bile is not well concentrated
e) The sphincter of Oddi fails to open in response to a fatty meal

The answer is: D

18. The actions of secretin include:

a) Increased salivary secretion


b) Relaxation of the oesophageal sphincters
c) Stimulation of gastric secretion
d) Stimulation of secretion of bicarbonate by pancreatic duct cells
e) Stimulation of secretion of bicarbonate by pancreatic acinar cells

The answer is: E

19. The function of the migrating motor complex (MMC) is to:

a) Empty non-digestable material from stomach and small intestine during


fasting
b) Initiate mass movements in the large intestine
c) Increase intestinal peristalsis in response to overfeeding
d) Increase ileal motility in response to entry of chyme into the stomach
e) Overcome intestinal obstruction

The answer is: A

20. The hormone involved in the initiation of the migrating motor complex is

a. Gastrin
b. Motilin
c. Secretin
d. Cholecystokinin
e. Enterogastrone

The answer is: B

21. The rate of gastric emptying increases with an increase in

a. Intragastric volume
b. Intraduodenal volume
c. Fat content of duodenum
d. Osmolality of duodenum

42 ‫صفحة‬ Dr. MOHAMED YAHIA 0900987639


QUESTION BANK OF Dr. MOHAMED YAHIA

e. Acidity of duodenum

The answer is: A

22. Basal acid output is increased by

a. Acidification of the antrum


b. Administration of an H 2 receptor antagonist
c. Vagotomy
d. Alkalinization of the antrum
e. Acidification of the duodenum

The answer is: D

23. Which one of the following processes applies to the proximal stomach?

a. Accommodation
b. Peristalsis
c. Retropulsion
d. Segmentation
e. Trituration

The answer is: A

24. After secretion of trypsinogen into the duodenum, the enzyme is converted
into its active form, trypsin, by

a. Enteropeptidase
b. Procarboxypeptidase
c. Pancreatic lipase
d. Previously secreted trypsin
e. An alkaline Ph

The answer is: A

25. The major mechanism for absorption of sodium from the small intestine is

a. Na + -H + exchange
b. Cotransport with potassium
c. Electrogenic transport
d. Neutral NaCl absorption
e. Solvent drag

The answer is: A

26. Pharmacological blockade of histamine H 2 receptors in the gastric mucosa

a. Inhibits both gastrin- and acetylcholine-mediated secretion of acid


b. Inhibits gastrin-induced but not meal-stimulated secretion of acid

43 ‫صفحة‬ Dr. MOHAMED YAHIA 0900987639


QUESTION BANK OF Dr. MOHAMED YAHIA

c. Has no effect on either gastrin-induced or meal-stimulated secretion of acid


d. Prevents activation of adenyl cyclase by gastrin
e. Causes an increase in potassium transport by gastric parietal (oxyntic) cells

The answer is: A

27. Removal of proximal segments of the small intestine results in a decrease in

a. Basal acid output


b. Maximal acid output
c. Gastric emptying of liquids
d. Gastric emptying of solids
e. Pancreatic enzyme secretion

The answer is: E

28. Dietary fat, after being processed, is extruded from the mucosal cells of the
gastrointestinal tract into the lymphatic ducts in the form of

a. Monoglycerides
b. Diglycerides
c. Triglycerides
d. Chylomicrons
e. Free fatty acids

The answer is: D

29. Gas within the colon is primarily derived from which one of the following
sources?

a. CO 2 liberated by the interaction of HCO 3 − and H +


b. Diffusion from the blood
c. Fermentation of undigested oligosaccharides by bacteria
d. Swallowed atmospheric air
e. Air pockets within foodstuffs

The answer is: C

30. Removal of the pyloric sphincter is associated with

a. A decrease in gastric compliance


b. An increase in maximal output of acid
c. An increase in basal output of acid
d. An increase in the rate of gastric emptying of solids
e. An increase in the serum gastrin level

The answer is: D

31. Removal of the terminal ileum will result in

44 ‫صفحة‬ Dr. MOHAMED YAHIA 0900987639


QUESTION BANK OF Dr. MOHAMED YAHIA

a. A decrease in absorption of amino acids


b. An increase in the water content of the feces
c. An increase in the concentration of bile acid in the enterohepatic circulation
d. A decrease in the fat content of the feces
e. An increase in the absorption of iron

The answer is: B

32. Vitamins synthesized by intestinal bacteria and absorbed in significant


quantities include

a. Vitamin B 6
b. Vitamin K
c. Thiamine
d. Riboflavin
e. Folic acid

The answer is: E

33. Which one of the following statements about the colon is correct?

a. Absorption of Na + in the colon is under hormonal (aldosterone) control


b. Bile acids enhance absorption of water from the colon
c. Net absorption of HCO 3 − occurs in the colon
d. Net absorption of K + occurs in the colon
e. The luminal potential in the colon is positive

The answer is: A

34. Contraction of the gallbladder is correctly described by which of the


following statements?

a. It is inhibited by a fat-rich meal


b. It is inhibited by the presence of amino acids in the duodenum
c. It is stimulated by atropine
d. It occurs in response to cholecystokinin
e. It occurs simultaneously with the contraction of the sphincter of Oddi

The answer is: D

35. Acidification of the duodenum will

a. Decrease pancreatic secretion of bicarbonate


b. Increase secretion of gastric acid
c. Decrease gastric emptying
d. Increase contraction of the gallbladder
e. Increase contraction of the sphincter of Oddi

45 ‫صفحة‬ Dr. MOHAMED YAHIA 0900987639


QUESTION BANK OF Dr. MOHAMED YAHIA

The answer is: D

36. Which one of the following statements about small intestine crypt cells is
correct?

a. They evidence well-developed microvilli


b. They are responsible for net NaCl and water absorption
c. They contain significant quantities of brush border hydrolases
d. They are responsible for net NaCl and water secretion
e. They demonstrate little or no proliferative activity

The answer is: A

37. Which of the following sugars is absorbed from the small intestine by
facilitated diffusion?

a. Glucose
b. Galactose
c. Fructose
d. Sucrose
e. Lactose

The answer is: C

38. As compared to long-chain fatty acids, medium-chain fatty acids

a. Are also packaged as chylomicrons


b. Can be used as a source of calories in patients with malabsorptive disease
c. Are more abundant in the diet
d. Are less water-soluble
e. Are also returned to the circulation via the lymph

The answer is: B

39. Severe inflammation of the ileum may be accompanied by

a. Increased vitamin B 12 absorption


b. Decreased bile acid pool size
c. Increased colon absorption of water
d. Decreased release of secretin
e. Increased absorption of dietary fats

The answer is: B

40. Which one of the following statements about the process of vitamin B 12
absorption is correct?

a. In humans, intrinsic factor is secreted from chief cells of the gastric gland
b. Vitamin B 12 binds preferentially to intrinsic factor in the stomach

46 ‫صفحة‬ Dr. MOHAMED YAHIA 0900987639


QUESTION BANK OF Dr. MOHAMED YAHIA

c. In adults, vitamin B 12 absorption occurs along the length of the small


intestine
d. Absorption may be reduced in a patient with pancreatic insufficiency
e. Absorption occurs via passive diffusion into the enterocyte

The answer is: D

41. Patients may experience nausea and a sense of early satiety following

a. Surgical resection of the proximal small bowel


b. A vagotomy of the distal stomach
c. Surgical resection of the proximal stomach
d. Surgical removal of the gastric antrum
e. A vagotomy of the orad stomach

The answer is: B

42. Which one of the following statements about bile acids is correct?

a. Conjugation with glycine enhances passive absorption of bile acids


b. Bile acids constitute approximately 80% of the total solutes in bile
c. Deoxycholic acid and lithocholic acid are examples of primary bile acids
d. Bile acid synthesis is catalyzed by the microsomal enzyme 7α-hydroxylase
e. Bile acids are essentially water-insoluble

The answer is: D

43. Withdrawal from chronic administration of an antisecretory compound is


followed by rebound gastric acid hypersecretion. Which drug could account for
the observed result?

a. A H 1 receptor antagonist
b. A proton pump inhibitor
c. A cholinergic receptor antagonist
d. An antacid
e. A CCK B receptor antagonist

The answer is: B

44. The migrating motor complex in humans

a. Occurs only in the small intestine


b. Requires an intact intrinsic nervous system for coordinated propagation
c. Is the result of food-mediated distension of the small intestine
d. Mixes intestinal contents with bile and the digestive enzymes
e. Is correlated with periodic increases in plasma gastrin levels

The answer is: B

47 ‫صفحة‬ Dr. MOHAMED YAHIA 0900987639


QUESTION BANK OF Dr. MOHAMED YAHIA

45. The delivery of chyme into the proximal small intestine will

a. Increase gastric acid secretion


b. Decrease pancreatic bicarbonate secretion
c. Increase gastric emptying of solids
d. Decrease circulating CCK levels
e. Increase small intestine segmentation

The answer is: E

46. A person is found to have increased basal and maximal acid outputs,
decreased serum calcium levels, and microcytic anemia. Inflammation in which
area of the gastrointestinal tract would explain these findings?

a. Colon
b. Jejunum
c. Duodenum
d. Gallbladder
e. Stomach

The answer is: C

47. Which one of the following statements best describes water and electrolyte
absorption in the gastrointestinal tract?

a. Most water and electrolytes derive from the oral intake of fluids
b. The small and large intestines have similar absorptive capacities
c. Net secretion of potassium occurs from the ileum
d. Osmotic equilibration of chyme occurs in the duodenum
e. Cholera toxin inhibits sodium-coupled nutrient transport

The answer is: D

48. The paracrine secretion responsible for inhibiting gastric acid secretion is

a. Histamine
b. Enterogastrone
c. Somatostatin
d. Pepsin
e. Enterooxyntin

The answer is: C

49. A medical student presents to the emergency room with a two-day history of
severe vomiting and orthostatic hypotension. What kind of metabolic
abnormalities would you expect?

a. Hypokalemia, hypochloremia, and metabolic acidosis

48 ‫صفحة‬ Dr. MOHAMED YAHIA 0900987639


QUESTION BANK OF Dr. MOHAMED YAHIA

b. Hyperkalemia, hyperchloremia, and metabolic alkalosis


c. Normal serum electrolytes and metabolic acidosis
d. Normal serum electrolytes and metabolic alkalosis
e. Hypokalemic, hypochloremic, metabolic alkalosis

The answer is: E

50. The metabolic effects of insulin include

a. Decreased glucose utilization


b. Decreased lipolysis
c. Increased proteolysis
d. Increased gluconeogenesis
e. Increased ketogenesis

The answer is: B

51. Which one of the following statements about bile acids is correct?

a. They are essentially water-insoluble


b. The majority of bile acids is absorbed by passive diffusion
c. Glycine conjugates are more soluble than taurine conjugates
d. The amount lost in the stool each day represents the daily loss of cholesterol
e. The bile acid dependent fraction of bile is stimulated by the hormone secretin

The answer is: D

52. In contrast to secretory diarrhea, osmotic diarrhea

a. Is characterized by an increase in stool osmolarity


b. Is the result of increased crypt cell secretion
c. Is the result of decreased electroneutral sodium absorption
d. Is caused by bacterial toxins
e. Occurs only in the colon

The answer is: A

53. The transport protein responsible for entry of glucose into the intestinal
enterocyte is called

a. Glut-2
b. Glut-5
c. SGLT1
d. SGLT2
e. SGLT5

The answer is: D

54. Short-chain fatty acid absorption occurs almost exclusively from the

49 ‫صفحة‬ Dr. MOHAMED YAHIA 0900987639


QUESTION BANK OF Dr. MOHAMED YAHIA

a. Stomach
b. Duodenum
c. Jejunum
d. Ileum
e. Colon

The answer is: E

55. Which one of the following statements about medium-chain fatty acids is
correct?

a. They are more water-soluble than long-chain fatty acids


b. Within the enterocyte, they are used for triglyceride resynthesis
c. They are packaged into chylomicrons
d. They are transported in the lymph
e. They require emulsification prior to enterocyte uptake

The answer is: A

Endocrine system
1. A tumor that produces large amounts of catecholamines can be detected by
examining the urine for:

a) K+
b) Uric acid
c) Choline
d) VMA
e) Calcium

The answer is: D

2. The following hormone is secreted by the anterior pituitary gland:

a) Cortisol
b) TRH
c) CRH
d) Vasopressin
e) TSH

The answer is: E

3. A ten year old boy has high levels of the growth hormone. He is likely to
develop:

a) Acromegaly

50 ‫صفحة‬ Dr. MOHAMED YAHIA 0900987639


QUESTION BANK OF Dr. MOHAMED YAHIA

b) Hyperglycemia
c) Mental retardation
d) Muscle atrophy
e) Hyperkalaemia

The answer is: B

4. Stimuli for aldosterone regulation includes:

a) High sodium in the distal renal tubule


b) High serum potassium
c) Hypervolaemia
d) ACTH as a potent stimulator
e) Increased renal blood flow

The answer is: B

5. Thyroid hormone actions include:

a) Augmentation of sympathetic effect on GI motility


b) Release of insulin
c) Lipogenesis
d) Increasing the numbers of Beta adrenergic receptors
e) Increasing diastolic arterial blood pressure

The answer is: D

6. The most sensitive regulatory mechanism of ADH secretion is dependent on:

a) The volume receptors


b) Renin-angiotensin system
c) Hypothalamic osmoreceptors
d) Renal glomerulotubular feedback
e) Renal principal cells

The answer is: C

7. Clinical uses of Oxytocin include:

a) Stimulation of milk ejection in breastfeeding women


b) Stimulation of ejaculation in infertile men
c) Stimulation of uterine contraction in pregnant women
d) Induction of breast development in adolescent girls
e) Induction of vasoconstriction in hypotension

The answer is: C

51 ‫صفحة‬ Dr. MOHAMED YAHIA 0900987639


QUESTION BANK OF Dr. MOHAMED YAHIA

8. All the following is true about control of secretion of TSH except:

a) It is inhibited by T4
b) It is increased in cold weather
c) It is increased in Grave's disease
d) It is increased in TRH
e) Has no marked diurnal rhythm

The answer is: D

9. The actions of ACTH include the following except:

a) Stimulation of melanocytes
b) Increased secretion of aldosterone
c) Induction of growth of the adrenal gland
d) Induction of secretion of adrenomedullins
e) Feedback inhibition of CRH

The answer is: B

10. A most effective method of treating type I diabetes mellitus is:

a) The patient should not eat any carbohydrates


b) Exercise
c) Reduce weight
d) Drugs to stimulate the B cells of the pancreas
e) Daily injections of insulin

The answer is: E

11. Primary hyperaldosteronism leads to:

a) Moderate hyperglycemia
b) Sodium natriuresis
c) Oedema
d) Alkalosis
e) Increased rennin secretion

The answer is: D

12. Which of the following is a manifestation of Addison's disease:

a) Hypopigmentation
b) Good appetite
c) Hypoglycemia
d) Obesity
e) Hypertension

The answer is: C

52 ‫صفحة‬ Dr. MOHAMED YAHIA 0900987639


QUESTION BANK OF Dr. MOHAMED YAHIA

13. Excess cortisol in Cushing's syndrome is associated with:

a) Hypotension
b) Protein depletion
c) Hypoglycemia
d) Dry thick skin
e) Increased body hair

The answer is: B

14. Insulin secretion is inhibited by:

a) Alpha adrenergic receptor stimulation


b) Vagal stimulation
c) Amino acids
d) GIP
e) Gastrin

The answer is: A

15. Calcitonin:

a) Increases serum calcium by stimulation of osteoclasts


b) Prevents postprandial hypercalcemia
c) Is important in long-term regulation of serum ionized calcium
d) Is stimulated by a hypothalamic releasing hormone
e) Is released during stress

The answer is: B

16. Hypomagnesemia is a recognized cause of:

a) Diabetes mellitus
b) Diabetes insipidus
c) Hypervitaminosis D
d) Tetany
e) Hypothyroidism

The answer is: E

17. The supraoptic nucleus of the hypothalamus is believed to control secretion


of which of the following hormones?

a. Antidiuretic hormone (arginine vasopressin)


b. Oxytocin
c. Growth hormone
d. Adrenocorticotropic hormone
e. Follicle-stimulating hormone

53 ‫صفحة‬ Dr. MOHAMED YAHIA 0900987639


QUESTION BANK OF Dr. MOHAMED YAHIA

The answer is: A

18. Parathyroid hormone (PTH) is accurately described by which of the


following statements?

a. It is secreted in response to an increase in plasma Ca 2+ concentration


b. It acts directly on bone cells to increase Ca 2+ deposition
c. It acts directly on intestinal cells to increase Ca 2+ absorption
d. It causes a decrease in cAMP concentration within renal proximal tubular cells
e. It is essential for life

The answer is: E

19. When a person is in the fasting state,

a. Liver glycogen levels are increased


b. The excretion of urea in the urine decreases
c. Basal metabolic rate (BMR) decreases
d. Glucose is the only fuel used by the central nervous system
e. Gluconeogenesis is inhibited

The answer is: C

20. True statements regarding atrial natriuretic hormone (ANH) include that it

a. Is synthesized from cholesterol


b. Is secreted in response to hypovolemia
c. Increases total peripheral resistance
d. Decreases glomerular filtration rate
e. Inhibits aldosterone secretion

The answer is: E

21. Which one of the following statements about spermatogenesis is correct?

a. Production and release of spermatozoa is cyclical


b. Sertoli cells are required for mitotic and meiotic activity of germ cells
c. Spermatogenesis requires continuous release of gonadotropin-releasing
hormone (GRH)
d. Leydig cell secretion of testosterone requires follicle-stimulating hormone
(FSH)
e. Luteinizing hormone (LH) acts directly on Sertoli cells to promote cell
division

The answer is: B

22. Release of oxytocin by the posterior pituitary gland is controlled by:

a) Oxytocin releasing hormone

54 ‫صفحة‬ Dr. MOHAMED YAHIA 0900987639


QUESTION BANK OF Dr. MOHAMED YAHIA

b) Neural signals from hypothalamus


c) Neural signals from the thalamus
d) Changes in body temperature
e) Changes in arterial blood pressure

The answer is: B

23. The following can be observed in a patient who has Grave's disease
(hyperthyroidism):

a) Drop of the upper eyelid


b) High level of TSH
c) Intolerance to cold
d) High systolic blood pressure
e) Constipation

The answer is: D

24. The thyrotropin releasing hormone, stimulates the secretion of:

a) Growth hormone
b) Prolactin
c) Adrenocorticotrophic hormone
d) Melanocyte stimulating hormone
e) Luteinizing hormone

The answer is: B

25. The following is true about estrogen:

a) It is essential for development of female external genitalia


b) Is released only during the follicular phase of the ovarian cycle
c) Is released from granulosa cells
d) Its secretion is decreased by inhibin
e) Its receptors are located on cell membranes

The answer is: C

26. The secretion of growth hormone is increased by

a. Hyperglycemia
b. Exercise
c. Somatostatin
d. Hypothermia
e. Free fatty acid

The answer is: B

55 ‫صفحة‬ Dr. MOHAMED YAHIA 0900987639


QUESTION BANK OF Dr. MOHAMED YAHIA

27. The source of estrogen and progesterone during the last seven months of
pregnancy is the

a. Ovary
b. Placenta
c. Corpus luteum
d. Anterior pituitary
e. Posterior pituitary

The answer is: B

28. The source of estrogen and progesterone during the first two months of
pregnancy is the

a. Ovary
b. Placenta
c. Corpus luteum
d. Anterior pituitary
e. Posterior pituitary

The answer is: C

29. Which one of the following hormones is secreted by the posterior pituitary
gland?

a. Adrenocorticotropic hormone (ACTH)


b. Oxytocin
c. Thyroid-stimulating hormone (TSH)
d. Growth hormone (GH)
e. Prolactin

The answer is: B

30. Which one of the following statements about prolactin is correct?

a. Prolactin initiates ovulation


b. Prolactin causes milk ejection during suckling
c. Prolactin inhibits the growth of breast tissue
d. Prolactin secretion is tonically inhibited by the hypothalamus
e. Prolactin secretion is increased by dopamine

The answer is: D

31. The principal steroid secreted by the fetal adrenal cortex is

a. Cortisol
b. Corticosterone
c. Dehydroepiandrosterone

56 ‫صفحة‬ Dr. MOHAMED YAHIA 0900987639


QUESTION BANK OF Dr. MOHAMED YAHIA

d. Progesterone
e. Pregnenolone

The answer is: C

32. The effect of insulin on glucose transport is to

a. Permit transport against a concentration gradient


b. Enhance transport into adipocytes
c. Enhance transport across the tubular epithelium of the kidney
d. Enhance transport into the brain
e. Enhance transport through the intestinal mucosa

The answer is: B

33. Compared with the resting state, during prolonged exercise, the caloric needs
of skeletal muscle are met by

a. Release of free fatty acids from adipose tissue


b. An increase in hepatic glycogenolysis
c. An increase in gluconeogenesis in muscle
d. Increased intestinal uptake of glucose and amino acids
e. Increased chylomicron formation

The answer is: B

34. Iodides are stored in the thyroid follicles mainly in the form of

a. Thyroxine
b. Thyroglobulin
c. Monoiodotyrosine
d. Diiodotyrosine
e. 3,5,3′-triiodothyronine

The answer is: B

35. Almost all of the active thyroid hormone entering the circulation is in the
form of

a. Triiodothyronine
b. Thyroxine
c. Thyroglobulin
d. Thyrotropin
e. Long-acting thyroid stimulator (LATS)

The answer is: B

36. Physiologically active thyroxine exists in which of the following forms?

57 ‫صفحة‬ Dr. MOHAMED YAHIA 0900987639


QUESTION BANK OF Dr. MOHAMED YAHIA

a. Bound to albumin
b. Bound to prealbumin
c. Bound to globulin
d. As a glucuronide
e. Unbound

The answer is: E

37. Ovulation is caused by a sudden increase in the secretion of

a. Estrogen
b. Progesterone
c. LH
d. FSH
e. Prolactin

The answer is: C

38. Activation of hormone-sensitive lipase in adipocytes

a. Causes increased hydrolysis of cholesterol esters


b. Is mediated by a cyclic AMP–dependent protein kinase
c. Is prevented by cortisol
d. Is stimulated by insulin
e. Results in accumulation of monoglycerides and diglycerides in adipocytes

The answer is: B

39. Which of the following is a correct statement about the production of human
sperm?

a. Spermatogonia undergo meiosis


b. Spermatogenesis occurs in the epididymis
c. Normally, 10 to 20 million sperm are produced daily
d. FSH is required
e. Complete maturation of spermatozoa occurs in about 1 month

The answer is: D

40. Functions of the Sertoli cells in the seminiferous tubules include

a. Secretion of FSH into the tubular lumen


b. Secretion of testosterone into the tubular lumen
c. Maintenance of the blood-testis barrier
d. Synthesis of estrogen after puberty
e. e. Expression of surface LH receptors

58 ‫صفحة‬ Dr. MOHAMED YAHIA 0900987639


QUESTION BANK OF Dr. MOHAMED YAHIA

The answer is: C

41. The following are actions of insulin, except:

a) Inhibition of protein degradation


b) Inhibition of gluconeogenesis
c) Inhibition of ketogenesis
d) Reduction of extracellular K+
e) Increased renal tubular absorption of glucose

The answer is: E

42. The following factors have a direct marked effect on the adrenal cortex to
stimulate the secretion of aldosterone except:

a) Angiotensin III
b) Angiotensin II
c) Hyperkalemia
d) Hypernatremia
e) High levels of ACTH

The answer is: D

43. A major regulator of bone growth is:

a) Parathyroid hormone
b) Calcitonin
c) Growth hormone
d) Prolactin
e) Active vitamin D

The answer is: E

44. If a patient dies of hypercalcemia, the most likely cause would be:

a) Excessive bleeding due to failure of clotting


b) Paralysis of skeletal muscles due to failure of interaction between actin and
myosin
c) Uncontrolled contraction of skeletal muscles due to hyperexcitability of
nerves and muscles
d) Failure of the SA node to generate impulses
e) Hypo-osmolality

The answer is: C

45. An indication that ovulation has taken place is

a. An increase in serum FSH levels


b. A drop in body temperature

59 ‫صفحة‬ Dr. MOHAMED YAHIA 0900987639


QUESTION BANK OF Dr. MOHAMED YAHIA

c. An increase in serum LH levels


d. An increase in serum progesterone levels
e. An increase in serum estrogen levels

The answer is: D

46. The hormone involved in the ejection of milk from a lactating mammary
gland is

a. Prolactin
b. FSH
c. LH
d. Growth hormone
e. Oxytocin

The answer is: E

47. Insulin can be described as:

a) Gluconeogenic
b) Ketogenic
c) Protein anabolic
d) Diuretic
e) Vasoconstrictive

The answer is: C

48. The main effect of somatostatin secreted by the D cells of the islets of
Langerhans appears to be:

a) Inhibition of the secretion of growth hormone


b) Stimulation of glucagon secretion
c) Inhibition of gastric acid secretion
d) Inhibition of insulin secretion
e) Relaxation of the sphincter of Oddi

The answer is: D

49. Factors that act on the cells of the adrenal gland to cause secretion of
aldosterone include:

a) Renin
b) Angiotensinogen
c) Angiotensin I
d) Atrial natriuretic peptide (ANP)
e) Potassium ions

The answer is: E

60 ‫صفحة‬ Dr. MOHAMED YAHIA 0900987639


QUESTION BANK OF Dr. MOHAMED YAHIA

50. The following effects can be caused by cortisol except:

a) Excessive loss of K+ in urine


b) Hypertension
c) Hyperglycemia
d) Increased production of CRH by the hypothalamus in cases of Cushing's
syndrome
e) Immunosuppression

The answer is: D

51. T3 is different from T4 in that:

a) It binds to receptors located on cell membranes


b) Its concentration increases in starvation
c) Its volume of distribution is high
d) It increases the efficiency of energy utilization
e) Most of it in plasma is bound to TBG

The answer is: D

52. Plasma levels of calcium can be increased most rapidly by the direct action of
parathyroid hormone on the

a. Kidney
b. Intestine
c. Thyroid gland
d. Bones
e. Skeletal musculature

The answer is: D

53. Correct statements about human growth hormone include which of the
following?

a. It is synthesized in the hypothalamus


b. It stimulates production of somatomedins by the liver
c. Its release is stimulated by somatostatin
d. It causes a decrease in lipolysis
e. It is deficient in acromegaly

The answer is: B

54. The basic effects of growth hormone on body metabolism include

a. Decreasing the rate of protein synthesis


b. Increasing the rate of use of carbohydrate
c. Decreasing the mobilization of fats

61 ‫صفحة‬ Dr. MOHAMED YAHIA 0900987639


QUESTION BANK OF Dr. MOHAMED YAHIA

d. Increasing the use of fats for energy


e. Inhibition of insulin-like growth factor 1

The answer is: D

55. Which of the following is true about the actions of glucagon?

a. It stimulates glycogenolysis in muscle


b. It inhibits insulin secretion
c. It stimulates gluconeogenesis in the liver
d. It inhibits adenyl cyclase
e. It inhibits phospholipase C

The answer is: A

56. The actions of insulin include

a. Converting glycogen to glucose


b. Stimulating gluconeogenesis
c. Increasing plasma amino acid concentration
d. Enhancing potassium entry into cells
e. Reducing urine formation

The answer is: D

57. The secretion of ACTH is correctly described in which of the following


statements?

a. It shows circadian rhythm in humans


b. It is decreased during periods of stress
c. It is inhibited by aldosterone
d. It is stimulated by glucocorticoids
e. It is stimulated by epinephrine

The answer is: A

58. Injection of thyroid hormone into a human subject will

a. Decrease the rate of oxygen consumption


b. Increase muscle protein synthesis
c. Decrease the need for vitamins
d. Increase the plasma concentration of cholesterol
e. Decrease the rate of lipolysis

The answer is: B

59. The effects of primary hyperaldosteronism (Conn’s syndrome) include

a. Hypertension

62 ‫صفحة‬ Dr. MOHAMED YAHIA 0900987639


QUESTION BANK OF Dr. MOHAMED YAHIA

b. Hyperkalemia
c. Decreased extracellular fluid volume
d. Increased concentrating ability of the kidney
e. Increased hematocrit

The answer is: A

60. Hyperparathyroidism decreases the plasma levels of

a. Phosphate
b. Sodium
c. Calcium
d. Potassium
e. Calcitonin

The answer is: A

61. Adrenalectomy is associated with

a. Hyperglycemia with decreased insulin sensitivity


b. Increased mobilization and utilization of fatty tissues
c. Augmented water excretion by the kidneys and sodium loss in the urine
d. Improved resistance to infection or shock
e. Euphoria

The answer is: C

62. Hyperthyroidism is characterized by

a. Anorexia
b. Increased basal metabolic rate
c. Bradycardia
d. Increased weight gain
e. Decreased sweating

The answer is: B

63. Insulin-independent glucose uptake occurs in

a. Adipose tissue
b. Cardiac muscle
c. Skeletal muscle
d. The brain
e. The uterus

The answer is: D

64. Which one of the following statements is correct?

63 ‫صفحة‬ Dr. MOHAMED YAHIA 0900987639


QUESTION BANK OF Dr. MOHAMED YAHIA

a. About 100 mg of iron is absorbed per day


b. Iron is absorbed rapidly from the small intestine
c. Iron is transported in the blood bound to transferrin
d. In general, iron must be oxidized from the ferrous to the ferric state for
efficient absorption
e. Iron absorption is a passive process regulated by plasma iron levels

The answer is: C

65. The actions of angiotensin II include

a. Vasodilation of peripheral arterioles


b. Promotion of salt excretion by renal tubules
c. Decreased aldosterone secretion
d. Increased renin secretion
e. Initiation of thirst

The answer is: E

66. A person with hypothyroidism would exhibit

a. Tachycardia
b. Increased metabolic rate
c. Heat intolerance
d. Sleepiness
e. Decreased body mass index

The answer is: D

67. Assuming a regular menstrual cycle of 28 to 30 days, ovulation would be


expected to occur between

a. Days 6 and 8
b. Days 10 and 12
c. Days 14 and 16
d. Days 18 and 20
e. Days 22 and 24

The answer is: C

68. Secretion of growth hormone is stimulated by

a. Increased plasma glucose levels


b. Deep sleep
c. Free fatty acids
d. Somatostatin

64 ‫صفحة‬ Dr. MOHAMED YAHIA 0900987639


QUESTION BANK OF Dr. MOHAMED YAHIA

e. Hyperglycemia

The answer is: B

69. Interaction of insulin with its membrane receptor

a. Affects transmembrane ion transport


b. Inhibits tyrosine phosphorylation of the receptor molecule
c. Reduces cellular glucose uptake
d. Enhances binding of additional glucose molecules
e. Stimulates synthesis of additional receptor molecules

The answer is: A

70. The uptake of triglycerides into adipose tissue from plasma lipoproteins

a. Is regulated by the activity of lipoprotein lipase


b. Is increased by catecholamines
c. Is decreased by glucose
d. Is decreased by insulin
e. Requires receptor-mediated endocytosis

The answer is: A

71. Products of the anterior pituitary gland include:

a) Melatonin
b) Acetylcholine
c) Beta endorphin
d) Inhibin
e) Somatomedins

The answer is: E

72. Secretion of gonadotropins is ordinarily inhibited by which of the following

a) Estradiol
b) Testosterone
c) Calcium
d) Oxytocin
e) A or B

The answer is: E

73. In humans, each of the following facilitates ovulation except:

a) Pulsatile secretion of gonadotropin-releasing hormone


b) Positive feedback effect of estradiol on LH secretion
c) Negative feedback effect of estradiol on LH secretion

65 ‫صفحة‬ Dr. MOHAMED YAHIA 0900987639


QUESTION BANK OF Dr. MOHAMED YAHIA

d) Action of FSH on granulose cells


e) Secretion of LH in a surge pattern

The answer is: C

74. Ionized calcium:

a) Forms 60% of total blood calcium


b) Cannot be filtered by the kidneys
c) Low levels stimulate calcitonin secretion
d) Levels in blood are increased by increase in H+ concentration
e) Concentration inside the cell is the same as the interstitial fluid

The answer is: D

75. Known effects of ACTH include all of the following except:

a) Increased secretion of adrenal androgens


b) Increased secretion of aldosterone transiently
c) Increased pigmentation of body tissues
d) Stimulation of growth of zona reticularis of the adrenal cortex
e) Direct stimulation of surfactant secretion

The answer is: E

76. Prolactin secretion:

a) Has diurnal rhythm


b) Can be inhibited by L-dopa
c) Is inhibited by chlorpromazine
d) Is stimulated by bromocriptine
e) Is continuously stimulated by prolactin releasing hormone

The answer is: B

77. Implantation of the zygote in the uterine wall

a. Precedes formation of the zona pellucida


b. Involves infiltration of the endometrium by the syncytiotrophoblast
c. Occurs 3 to 5 days after fertilization
d. Occurs when the embryo consists of approximately 128 cells
e. Is inhibited by secretion of progesterone from the corpus luteum

The answer is: B

78. Biological actions of estrogens include

a. Inhibition of follicular growth


b. Delayed bone loss at menopause

66 ‫صفحة‬ Dr. MOHAMED YAHIA 0900987639


QUESTION BANK OF Dr. MOHAMED YAHIA

c. Increased glucose tolerance


d. Decreased serum LDL cholesterol
e. Decreased progesterone receptors

The answer is: B

79. The hormone primarily responsible for development of ovarian follicles prior
to ovulation is

a. Chorionic gonadotropin
b. Estradiol
c. Follicle-stimulating hormone
d. Luteinizing hormone
e. Progesterone

The answer is: C

80. Thyroid hormones

a. Are short-acting hormones


b. Have no effect on basal metabolic activity
c. Are stored in extracellular sites
d. Exist in the plasma primarily as free hormones
e. Increase the metabolic activity of the brain

The answer is: C

81. Melatonin

a. Is synthesized in the anterior pituitary gland


b. Regulates skin pigmentation in humans
c. Secretion is increased by darkness
d. Secretion is inhibited by norepinephrine from the sympathetic nervous system
e. Is synthesized from the amino acid L –arginine

The answer is: C

82. Insulin deficiency leads to

a. Increased cellular uptake of glucose


b. Decreased intracellular α-glycerophosphate in liver and fat cells
c. Enhanced glucose uptake and use except by brain tissue
d. Decreased fatty acid release from adipose tissue
e. Indirect depression of use of glucose by excess fatty acids in the blood

The answer is: B

83. The islets of Langerhans

67 ‫صفحة‬ Dr. MOHAMED YAHIA 0900987639


QUESTION BANK OF Dr. MOHAMED YAHIA

a. Are found primarily in the head of the pancreas


b. Constitute approximately 30% of the pancreatic weight
c. Contain six distinct endocrine cell types
d. Have a meager blood supply
e. Secrete insulin and glucagon

The answer is: E

84. Progesterone

a. Is secreted by the corpus luteum


b. Secretion by the placenta increases at week 6 of gestation
c. Plasma levels increase during the menses
d. Plasma levels remain constant after implantation
e. Plasma levels decrease subsequent to ovulation

The answer is: A

85. The anti-inflammatory effect of cortisol treatment is thought to be due to

a. Increased capillary membrane permeability


b. Increased formation of leukotrienes
c. Increased release of pyrogen from granulocytes
d. Activation of phospholipase A 2
e. Stabilization of cellular lysosomal membranes

The answer is: E

86. Administration of estrogens in women will

a. Limit the growth of ovarian follicles


b. Produce cyclic changes in the vagina and endometrium
c. Cause cervical mucus to become thicker and more acidic
d. Retard ductal proliferation in the breast
e. Produce glandular proliferation in the breast

The answer is: B

87. The gastrointestinal hormone involved in initiation of migrating motor


complexes is

a. Gastrin
b. Enterooxyntin
c. Somatostatin
d. Motilin
e. Secretin

The answer is: D

68 ‫صفحة‬ Dr. MOHAMED YAHIA 0900987639


QUESTION BANK OF Dr. MOHAMED YAHIA

88. The principal androgen responsible for transforming undifferentiated fetal


external genitalia into male external genitalia is

a. Müllerian-inhibiting substance
b. Androstenedione
c. Dihydrotestosterone
d. Testosterone
e. Androsterone

The answer is: C

89. Which one of the following statements about growth and development is
correct?

a. Thyroid hormones are essential for normal growth during puberty


b. Linear growth ceases earlier in boys than in girls
c. Serum IGF-1 levels decrease throughout childhood
d. Growth hormone is essential for prenatal linear growth
e. Prepubertal growth rates are constant throughout the year

The answer is: A

90. The hormone most structurally related to gastrin is

a. Secretin
b. Cholecystokinin
c. Motilin
d. Vasoactive intestinal polypeptide
e. Glucagon

The answer is: B

91. Zollinger-Ellison syndrome patients experience severe, intractable peptic


ulcer disease due to gastric acid hypersecretion. The cause of the increased acid
output is increased plasma levels of

a. Secretin
b. Somatostatin
c. Gastrin
d. VIP
e. Motilin

The answer is: C

92. Which one of the following hormones initiates a biological effect by activation
of cell membrane receptors?

69 ‫صفحة‬ Dr. MOHAMED YAHIA 0900987639


QUESTION BANK OF Dr. MOHAMED YAHIA

a. Progesterone
b. Estrogens
c. Cortisol
d. Epinephrine
e. Thyroxine

The answer is: D

93. Which one of the following hormones interacts with a cytoplasmic receptor,
then localizes in the nucleus and directs protein and nucleotide synthesis?

a. Thyrotropin-releasing hormone
b. Epinephrine
c. Luteinizing hormone
d. Cortisol
e. Insulin

The answer is: D

94. The hormone responsible for stimulating milk letdown is

a. Oxytocin
b. Progesterone
c. Estradiol
d. Insulin
e. Prolactin

The answer is: A

95. A patient with chronic asthma is started on glucocorticoid therapy. The


treatment may result in bone loss because glucocorticoids

a. Inhibit bone formation


b. Increase calcium absorption from the GI tract
c. Increase osteoblast growth
d. Inhibit bone resorption
e. Suppress vitamin D activation

The answer is: A

96. Hormone replacement therapy is being considered for a 55-year-old woman


undergoing menopause. Which one of the following is a benefit of the therapy?

a. A return to normal menstrual cycle pattern


b. Hot flashes
c. A reduced risk of stroke
d. Increased risk of osteoporosis
e. Increased levels of low-density cholesterol (LDL)

70 ‫صفحة‬ Dr. MOHAMED YAHIA 0900987639


QUESTION BANK OF Dr. MOHAMED YAHIA

The answer is: C

97. Insulin regulates glucose transport into muscle and fat cells via which glucose
transporter?

a. GLUT-1
b. GLUT-2
c. GLUT-3
d. GLUT-4
e. GLUT-5

The answer is: D

98. A patient with hypercalcemia would be expected to experience

a. Diarrhea
b. Polyuria
c. Metabolic alkalosis
d. Euphoria
e. Hyperphagia

The answer is: B

99. The hallmark of pheochromocytoma is

a. Hypertension
b. Hypoglycemia
c. Dry skin
d. Lethargy
e. Bradycardia

The answer is: A

100. Plasma catecholamine levels will increase when

a. Blood pressure increases


b. Anxiety increases
c. Blood volume increases
d. Blood glucose increases
e. Moving from a standing to a supine position

The answer is: B

101. Cortisol administration to a patient with adrenal insufficiency will

a. Increase insulin sensitivity in muscle


b. Enhance wound healing
c. Increase corticotropin-releasing hormone secretion
d. Increase ACTH secretion

71 ‫صفحة‬ Dr. MOHAMED YAHIA 0900987639


QUESTION BANK OF Dr. MOHAMED YAHIA

e. Increase gluconeogenesis

The answer is: E

102. The signs and symptoms of a patient with primary adrenal insufficiency
include

a. Pallor
b. Low ACTH levels
c. High cortisol levels
d. Hyperkalemia
e. Hypertension

The answer is: D

103. Which one of the following statements about semen is correct?

a. The bulk of semen volume is contributed by the prostate gland


b. It prevents sperm capacitation
c. It buffers vaginal acidity
d. It activates sperm motility in the male tract
e. Sperm counts have increased over the last 20 years

The answer is: B

104. Which one of the following statements about atrial natriuretic factor is
correct?

a. Secretion increases when central venous pressure increases


b. It constricts afferent renal arterioles
c. It acts only on the distal nephron to increase urine flow
d. It enhances ADH secretion
e. Secretion is stimulated by hyponatremia

The answer is: A

105. A patient is diagnosed with acromegaly. Patients with this disease typically
have

a. Decreased gluconeogenesis
b. Hypoglycemia
c. Insulin resistance
d. Decreased protein synthesis
e. Decreased lipolysis

The answer is: C

106. Prolactin secretion is tonically suppressed in nonpregnant women by

72 ‫صفحة‬ Dr. MOHAMED YAHIA 0900987639


QUESTION BANK OF Dr. MOHAMED YAHIA

a. Estrogens
b. Progesterone
c. Dopamine
d. FSH
e. LH

The answer is: C

107. A woman tests positive for pregnancy. In order for the pregnancy to
proceed uneventfully, which of the following must occur?

a. The corpus luteum must secrete progesterone to sustain the endometrium


b. The pituitary must secrete hCG to maintain the corpus luteum
c. The pituitary must secrete prolactin to sustain the placenta
d. The placenta must secrete FSH to maintain ovarian function
e. The placenta must secrete LH to maintain ovarian function

The answer is: A

108. Symptoms of hypoglycemia develop when the plasma glucose concentration


fall to between 45 and 50 mg% and may include

a. Dry skin
b. Bradycardia
c. Insomnia
d. Loss of fine motor skills
e. Satiety

The answer is: D

109. Radiation treatment for a pituitary tumor in an 8-year-old boy resulted in


the complete loss of pituitary function. As a result, the child is likely to
experience

a. Hypothyroidism and goiter


b. Increased ACTH
c. Absent sexual maturation
d. Accelerated growth spurts
e. Increased TSH levels

The answer is: C

110. Which one of the following statements is correct?

a. Thyroid hormones are highly soluble in water


b. Secretion of TSH is regulated primarily by the pituitary level of T3
c. TSH is secreted from the posterior pituitary

73 ‫صفحة‬ Dr. MOHAMED YAHIA 0900987639


QUESTION BANK OF Dr. MOHAMED YAHIA

d. TSH initiates thyroid hormone secretion via activation of nuclear receptors in


thyroid gland cells
e. T 4 is the physiologically active hormone

The answer is: B

111. Aldosterone secretion by the adrenal gland is stimulated by

a. Hypokalemia
b. Angiotensin I
c. Decreased renal afferent arteriole pressure
d. Increased parasympathetic nerve activity
e. Decreased sodium delivery to the distal convoluted tubule

The answer is: C

112. Maternal physiology changes dramatically throughout pregnancy. Which


one of the following changes would be expected?

a. Reduced circulating gonadotrophin levels


b. Increased conversion of glucose to glycogen
c. Hypercapnia
d. Increased hematocrit
e. Decreased plasma cortisol levels

The answer is: A

113. Glucagon secretion is increased by

a. Hyperglycemia
b. Arginine
c. Vagal stimulation of the pancreas
d. Insulin
e. Somatostatin

The answer is: B

114. Which one of the following conditions experienced by pregnant women may
be due to the effects of pregnancy?

a. Increased incidence of heartburn


b. Decreased facial acne
c. Increased gastric emptying leading to intestinal cramping and diarrhea
d. Decreased afterload
e. Increased cardiac output

The answer is: A

74 ‫صفحة‬ Dr. MOHAMED YAHIA 0900987639


QUESTION BANK OF Dr. MOHAMED YAHIA

115. Which one of the following is associated with prolonged fasting (3 or more
days)?

a. Decreased lipolysis
b. Increased urinary excretion of nitrogen
c. Decreased gluconeogenesis
d. Increased glucose utilization by the brain
e. Increased secretion of insulin

The answer is: C

116. An abnormal decrease in the circulating levels of which placental hormone


is indicative of a nonviable pregnancy?

a. Human placental lactogen


b. Estrogen
c. Corticotropin-releasing hormone
d. Human chorionic gonadotropin
e. Progesterone

The answer is: D

RENAL SYSTEM
1. Which of the following is true about renal blood flow?

a) The kidneys receive 12-15% of cardiac output


b) Is kept constant within a small range by sympathetic vasomotor tone
c) Measured by renal clearance of inulin
d) The dilution method is used for its measurement
e) Goes mainly to the renal cortex

The answer is: E

2. The characteristics of the Loop of Henle include:

a) Descending limb is not permeable to water


b) Urine becomes more dilute as it reaches the tip of the loop
c) Ascending limb is permeable to water
d) In thin ascending limb urea is actively transported to interstitial fluid
e) Na is reabsorbed by Na-K ATPase in ascending limb

The answer is: D

3. The following is true about the Vasta Recta:

a) Descending vasta recta reabsorbs water from the interstitial fluid


b) The descending capillaries loose Na and Urea

75 ‫صفحة‬ Dr. MOHAMED YAHIA 0900987639


QUESTION BANK OF Dr. MOHAMED YAHIA

c) Plasma becomes hyperosmotic as it descends in vasta recta


d) Ascending capillaries loose water to interstitial fluid
e) They are responsible for the countercurrent multiplier mechanism

The answer is: C

4. The functions of the renal proximal convoluted tubes include:

a) Reabsorption of all bicarbonate filtered at the glomerulus


b) Production of renin
c) Reaborption of 80% of filtered bicarbonate
d) Reabsorption of bicarbonate resulting in fixed acid secretion
e) Hydrogen ion secretion by facilitated diffusion

The answer is: C

5. For those substances that are actively reabsorbed or secreted, the highest limit
of the amount that can be transported per unit time by the kidney tubules:

a) Depends on the rate at which the transport mechanism operates


b) Is directly related to the plasma concentration of the substance
c) Depends on the tubular transport maximum
d) Is dependent upon tubular load
e) Depends on the GFR

The answer is: C

6. Water reabsorption in the collecting duct:

a) Is dependent on aldosterone
b) Is increased by ADH
c) Has the largest percentage of total water reabsorbed
d) Is facilitated by sodium reabsorption
e) Independent of osmotic difference across the tubules

The answer is: B

7. The consumption of oxygen by the kidney

a. Decreases as blood flow increases


b. Is regulated by erythropoietin
c. Remains constant as blood flow increases
d. Directly reflects the level of sodium transport
e. Is greatest in the medulla

The answer is: D

8. The anion gap will increase with an increase in the plasma concentration of

76 ‫صفحة‬ Dr. MOHAMED YAHIA 0900987639


QUESTION BANK OF Dr. MOHAMED YAHIA

a. Sodium
b. Potassium
c. Chloride
d. Bicarbonate
e. Lactate

The answer is: E

9. In the presence of ADH, the filtrate will be isotonic to plasma in the

a. Descending limb of the loop of Henle


b. Ascending limb of the loop of Henle
c. Cortical collecting tubule
d. Medullary collecting tubule
e. Renal pelvis

The answer is: C

10. Sodium reabsorption from the distal tubule will be increased if there is an
increase in

a. Plasma potassium concentration


b. Plasma volume
c. Mean arterial pressure
d. Urine flow rate
e. Plasma osmolality

The answer is: A

11. ADH will be released from the posterior pituitary when there is a decrease in

a. Plasma Na + concentration
b. Plasma volume
c. Plasma K + concentration
d. Plasma pH
e. Plasma Ca 2+ concentration

The answer is: B

12. If 600 mL of water is ingested rapidly, plasma volume will increase by


approximately

a. 400 mL
b. 200 mL
c. 100 mL
d. 50 mL

77 ‫صفحة‬ Dr. MOHAMED YAHIA 0900987639


QUESTION BANK OF Dr. MOHAMED YAHIA

e. 25 Ml

The answer is: D

13. Renin secretion by the kidney is increased by

a. Increasing mean blood pressure


b. Increasing glomerular filtration rate
c. Increasing sympathetic nerve activity
d. Increasing angiotensin II synthesis
e. Increasing atrial natriuretic hormone secretion

The answer is: C

14. Na + is reabsorbed from the basolateral surface of the renal epithelial cells by

a. Na/H exchange
b. Na-glucose cotransport
c. Na-K pump
d. Facilitated diffusion
e. Solvent drag

The answer is: C

15. Which of the following is most likely to cause an increase in the glomerular
filtration rate?

a. Contraction of mesangial cells


b. Blockage of the ureter
c. Release of renin from the juxtaglomerular apparatus
d. Dilation of the afferent arterioles
e. Volume depletion

The answer is: D

16. If the clearance of Substance X is greater than the clearance on inulin, it is


most likely that substance X is:

a) Bound to tubular proteins


b) Bound to plasma proteins
c) Secreted
d) Reabsorbed
e) Neither secreted nor reabsorbed

The answer is: C

17. Renin is produced and secreted from granules located in the:

a) Macula densa

78 ‫صفحة‬ Dr. MOHAMED YAHIA 0900987639


QUESTION BANK OF Dr. MOHAMED YAHIA

b) Mesangial cells
c) Intercalated cells
d) Afferent arteriole
e) Interstitial cells

The answer is: D

18. The tonicity of urine as it enters the renal collecting duct is:

a) Isotonic
b) Hypotonic or isotonic, but never hypertonic
c) Hypotonic
d) Hypertonic
e) Hypertonic or isotonic, but never hypotonic

The answer is: B

19. On assuming the upright posture, one would expect:

a) Stimulation of the baroreceptore reflex


b) Increased pulmonary artery pressure
c) A decrease in pulse rate
d) Venodilation
e) An increase in renal blood flow

The answer is: E

20. The following factors when increased, they will increase the glomerular
filteration rate except:

a) Renal blood flow


b) Afferent arteriolar diameter
c) Efferent arteriolar diameter
d) Permeability of glomerular membrane
e) Capillary surface area

The answer is: C

21. The following hormones can be formed by the kidney except:

a) Erythropoietin
b) Angiotensin II
c) Prostaglandins
d) ADH
e) Calcitrol

The answer is: E

22. The daily production of hydrogen ion from CO 2 is primarily buffered by

79 ‫صفحة‬ Dr. MOHAMED YAHIA 0900987639


QUESTION BANK OF Dr. MOHAMED YAHIA

a. Extracellular bicarbonate
b. Red blood cell bicarbonate
c. Red blood cell hemoglobin
d. Plasma proteins
e. Plasma phosphate

The answer is: B

23. Glomerular filtration rate would be decreased by

a. Constriction of the efferent arteriole


b. An increase in afferent arteriolar pressure
c. Compression of the renal capsule
d. A decrease in the concentration of plasma protein
e. An increase in renal blood flow

The answer is: C

24. The secretion of H + in the proximal tubule is primarily associated with

a. Excretion of potassium ion


b. Excretion of hydrogen ion
c. Reabsorption of calcium ion
d. Reabsorption of bicarbonate ion
e. Reabsorption of phosphate ion

The answer is: D

25. If a substance appears in the renal artery but not in the renal vein,

a. Its clearance is equal to the glomerular filtration rate


b. It must be reabsorbed by the kidney
c. Its urinary concentration must be higher than its plasma concentration
d. Its clearance is equal to the renal plasma flow
e. It must be filtered by the kidney

The answer is: D

26. Destruction of the supraoptic nuclei of the brain will produce which of the
following changes in urinary volume and concentration? (Assume that fluid
intake equals fluid loss.)

a. An increased urinary volume and a very dilute urine


b. An increased urinary volume and concentrated urine
c. A normal urinary volume and concentration
d. Decreased urinary volume and a very dilute urine

80 ‫صفحة‬ Dr. MOHAMED YAHIA 0900987639


QUESTION BANK OF Dr. MOHAMED YAHIA

e. Decreased urinary volume and a concentrated urine

The answer is: A

27. Which one of the following returns closest to normal during chronic
respiratory acidosis?

a. Alveolar ventilation
b. Arterial P CO 2
c. Arterial P O 2
d. Plasma concentration of bicarbonate
e. Arterial concentration of hydrogen ion

The answer is: E

28. The pH of the tubular fluid in the distal nephron can be lower than that in
the proximal tubule because

a. A greater sodium gradient can be established across the wall of the distal
nephron than across the wall of the proximal tubule
b. More buffer is present in the tubular fluid of the distal nephron than in the
proximal tubule
c. More hydrogen ion is secreted into the distal nephron than into the proximal
tubule
d. The brush border of the distal nephron contains more carbonic anhydrase than
that of the proximal tubule
e. The tight junctions of the distal nephron are less leaky to solute than those of
the proximal tubule

The answer is: E

29. Which of the following statements about renin is true?

a. It is secreted by cells of the proximal tubule


b. Its secretion leads to loss of sodium and water from plasma
c. Its secretion is stimulated by increased mean renal arterial pressure
d. It converts angiotensinogen to angiotensin I
e. It converts angiotensin I to angiotensin II

The answer is: D

30. Renal correction of hyperkalemia will result in

a. Alkalosis
b. Acidosis
c. Increased secretion of HCO 3 −
d. Increased secretion of H +
e. Increased excretion of Na +

81 ‫صفحة‬ Dr. MOHAMED YAHIA 0900987639


QUESTION BANK OF Dr. MOHAMED YAHIA

The answer is: B

31. Sodium reabsorption in the proximal tubules from the luminal surface is
marked by all of the following except:

a) Sodium glucose co-transporters


b) Sodium-hydrogen exchanger
c) Na-K pump
d) Solvent drag
e) Sodium amino acid transporters

The answer is: D

32. Glomerular filtration rate would be increased by:

a) A decrease in afferent arteriolar pressure


b) A decrease in renal blood flow
c) A decrease in efferent arteriolar resistance
d) Decrease in the concentration of plasma proteins
e) Obstruction of ureters

The answer is: D

33. Potassium secretion in the distal tubule will be increased if there is an


increase in:

a) Plasma (H+)
b) Plasma volume
c) Plasma sodium concentration
d) Plasma potassium concentration
e) Plasma calcium concentration

The answer is: D

33. The following is true about glucose reabsorption in the renal tubules:

a) Transport maximum is 80 mg/min


b) The transport maximum is dependent on the amount of glucose filtered
c) It is mainly a symport with K+
d) It is reduced with metabolic inhibitors that reduce the activity Na/K
ATPase
e) About 50% of filtered glucose is reabsorbed

The answer is: D

35. The effective renal plasma flow, which equals the clearance of PAH, is less
than the true renal plasma flow because

a. The fraction of PAH filtered is less than the filtration fraction

82 ‫صفحة‬ Dr. MOHAMED YAHIA 0900987639


QUESTION BANK OF Dr. MOHAMED YAHIA

b. The plasma entering the renal vein contains a small amount of PAH
c. The cortical and medullary collecting ducts are able to reabsorb some PAH
d. The calculated clearance of PAH depends on the urinary flow rate
e. The measured value of the plasma PAH concentration is less than the actual
PAH concentration

The answer is: B

36. Most of the glucose that is filtered through the glomerulus undergoes
reabsorption in the

a. Proximal tubule
b. Descending limb of the loop of Henle
c. Ascending limb of the loop of Henle
d. Distal tubule
e. Collecting duct

The answer is: A

37. Glomerular filtration rate (GFR) and renal blood flow (RBF) will both be
increased if

a. The efferent and afferent arterioles are both dilated


b. The efferent and afferent arterioles are both constricted
c. Only the afferent arteriole is constricted
d. Only the efferent arteriole is constricted
e. The afferent arteriole is constricted and the efferent arteriole is dilated

The answer is: A

38. A man drinks 2 L of water to replenish the fluids lost by sweating during a
period of exercise. Compared with the situation prior to the period of sweating,

a. His intracellular fluid will be hypertonic


b. His extracellular fluid will be hypertonic
c. His intracellular fluid volume will be greater
d. His extracellular fluid volume will be greater
e. His intracellular and extracellular fluid volumes will be unchanged

The answer is: C

39. The amount of potassium excreted by the kidney will decrease if

a. Distal tubular flow increases


b. Circulating aldosterone levels increase
c. Dietary intake of potassium increases
d. Na + reabsorption by the distal nephron decreases
e. The excretion of organic ions decreases

83 ‫صفحة‬ Dr. MOHAMED YAHIA 0900987639


QUESTION BANK OF Dr. MOHAMED YAHIA

The answer is: D

40. A respiratory acidosis that results in an increase in the concentration of


hydrogen ion in arterial blood from 40 meq/L (pH 7.4) to 50 meq/L (pH 7.3)
would

a. Stimulate the peripheral chemoreceptors


b. Decrease the amount of ammonium excreted in the urine
c. Inhibit the central chemoreceptors
d. Increase the pH of the urine
e. Decrease the concentration of HCO 3 − in arterial blood

The answer is: A

41. Which of the following substances will be more concentrated at the end of the
proximal tubule than at the beginning of the proximal tubule?

a. Glucose
b. Creatinine
c. Sodium
d. Bicarbonate
e. Phosphate

The answer is: B

42. When a person is dehydrated, hypotonic fluid will be found in the

a. Glomerular filtrate
b. Proximal tubule
c. Loop of Henle
d. Cortical collecting tubule
e. Distal collecting duct

The answer is: D

43. The electrically neutral active transport of sodium from the lumen of the
kidney occurs in the

a. Proximal tubule
b. Descending limb of the loop of Henle
c. Ascending limb of the loop of Henle
d. Cortical collecting duct
e. Medullary collecting duct

The answer is: C

44. In metabolic acidosis caused by diabetic ketoacidosis, which of the following


would be greater than normal?

84 ‫صفحة‬ Dr. MOHAMED YAHIA 0900987639


QUESTION BANK OF Dr. MOHAMED YAHIA

a. Concentration of plasma HCO 3 −


b. Anion gap
c. Arterial P CO 2
d. Plasma pH
e. Blood volume

The answer is: B

45. Decreasing the resistance of the afferent arteriole in the glomerulus of the
kidney will decrease

a. The renal plasma flow


b. The filtration fraction
c. The oncotic pressure of the peritubular capillary blood
d. The glomerular filtration rate
e. None of the above

The answer is: E

46. If GFR increases, proximal tubular reabsorption of salt and water will
increase by a process called glomerulotubular balance. Contributions to this
process include

a. An increase in peritubular capillary hydrostatic pressure


b. A decrease in peritubular sodium concentration
c. An increase in peritubular oncotic pressure
d. An increase in proximal tubular flow
e. An increase in peritubular capillary flow

The answer is: C

47. Renin release from the juxtaglomerular apparatus is inhibited by

a. Beta-adrenergic agonists
b. Prostaglandins
c. Aldosterone
d. Stimulation of the macula densa
e. Increased pressure within the afferent arterioles

The answer is: E

48. Patients with renal insufficiency develop very high plasma concentrations of
urea (uremia) because of

a. An increased synthesis of urea by the liver


b. An increased reabsorption of urea by the proximal tubules
c. A decreased secretion of urea by the distal tubules
d. A decreased glomerular filtration rate

85 ‫صفحة‬ Dr. MOHAMED YAHIA 0900987639


QUESTION BANK OF Dr. MOHAMED YAHIA

e. An increased renal blood flow

The answer is: D

49. Which one of the following statements about aldosterone is correct?

a. It produces its effect by activating cAMP


b. It produces its effect by increasing distal tubular permeability to
sodium
c. It causes an increased reabsorption of hydrogen ion
d. It has its main effect on the proximal tubule
e. It is secreted in response to an increase in blood pressure

The answer is: B

50. The effect of antidiuretic hormone (ADH) on the kidney is to

a. Increase the permeability of the distal nephron to water


b. Increase the glomerular filtration rate
c. Increase the excretion of Na +
d. Increase the excretion of water
e. Increase the diameter of the renal artery

The answer is: A

51. Which of the following decreases the glomerular filteration rate:

a) An increase in Bowman's capsule pressure


b) A decrease in glomerular capillary hydrostatic pressure
c) An increase in tubular fluid hydrostatic pressure
d) A decrease in efferent arteriolar resistance
e) A decrease in plasma colloid pressure

The answer is: A

52. The macula densa in the juxtaglomerular apparatus regulate:

a) Plasma pH
b) Glomerular fileration rate
c) Reabsorption of potassium by the proximal tubules
d) Red blood cell production
e) Phosphate excretion

The answer is: B

53. Hypokalemia can result from

a. Metabolic acidosis
b. Diarrhea

86 ‫صفحة‬ Dr. MOHAMED YAHIA 0900987639


QUESTION BANK OF Dr. MOHAMED YAHIA

c. Adrenal insufficiency
d. Hypovolemia
e. Hyperosmotic extracellular fluid

The answer is: B

54. The ability of the kidney to excrete a concentrated urine will increase if

a. The reabsorption of Na + by the proximal tubule decreases


b. The flow of filtrate through the loop of Henle increases
c. The glomerular capillary pressure increases
d. The activity of the Na-K pump in the loop of Henle decreases
e. The permeability of the collecting duct to water increases

The answer is: B

55. The glomerular filtration rate will increase if

a. Sympathetic nerve activity to the kidney increases


b. The afferent arteriolar resistance increases
c. The efferent arteriolar resistance decreases
d. The plasma protein concentration decreases
e. Urine flow through the urethra is blocked

The answer is: D

56. Metabolic alkalosis will be observed in a patient with

a. Hyperaldosteronism
b. Hyperventilation
c. Persistent diarrhea
d. Renal failure
e. Diabetes

The answer is: A

57. The syndrome of inappropriate antidiuretic hormone secretion (SIADH) is


caused by the excess release of ADH. SIADH will cause an increase in

a. Concentration of plasma sodium


b. Intracellular volume
c. Urinary flow
d. Plasma oncotic pressure
e. Plasma osmolarity

The answer is: B

87 ‫صفحة‬ Dr. MOHAMED YAHIA 0900987639


QUESTION BANK OF Dr. MOHAMED YAHIA

58. Which one of the following comparisons between the distal nephron and the
proximal tubule is correct?

a. The distal nephron is more permeable to hydrogen ion than the


proximal tubule
b. The distal nephron is less responsive to aldosterone than the proximal tubule
c. The distal nephron has a more negative intraluminal potential than the
proximal tubule
d. The distal nephron secretes less potassium than does the proximal tubule
e. The distal nephron secretes more hydrogen ion than does the proximal tubule

The answer is: A

59. An increase in the concentration of NaCl in the intraluminal fluid with the
ascending limb of the loop of Henle causes the macula densa to release

a. ADH
b. Aldosterone
c. Adenosine
d. Renin
e. Angiotensinogen

The answer is: C

60. Metabolic acidosis is caused by

a. Hypoaldosteronism
b. Hyperventilation
c. Hypokalemia
d. Hypovolemia
e. Hypercalcemia

The answer is: A

61. The Glomerular Filteration Rate (GFR) can be calculated by all of the
following except:

a) PAH clearance
b) Creatinine clearance
c) Urea clearance
d) Inulin clearance
e) Formula using plasma creatinine concentration

The answer is: A

62. The transport of glucose across the luminal membrane in the proximal part
of the renal tubule is:

88 ‫صفحة‬ Dr. MOHAMED YAHIA 0900987639


QUESTION BANK OF Dr. MOHAMED YAHIA

a) Through insulin dependent glucose transporters


b) Cannot take place against the concentration gradient
c) Is not affected by Na+/K+ pump inhibitors
d) Is a co-transport with sodium
e) Is decreased in diabetes mellitus

The answer is: D

63. The following contributes to the high osmolarity of the renal medulla:

a) Low blood flow in the medulla


b) The very low rate of tubular flow in the medulla
c) The increase in the permeability of the collecting ducts caused by
ADH
d) Aldosterone
e) The sodium dependent organic substance transporter
The answer is: A

64. The restriction of protein to pass through the filtration membrane is due to:

a) The size of the pores in the capillary wall


b) The size of the spaces between the cells of the epithelial layer
c) The negative charge of basement membrane
d) The positive charge on the membrane
e) The presence of the phagocytic mesangial cells

The answer is: C

65. The following hormones act on the proximal tubules of the kidney:

a) ADH
b) Aldosterone
c) 25 hydroxy cholecalciferol
d) Angiotensin II
e) Dihydrotestosterone

The answer is: D

66. The following decreases the GFR:

a) Increased constriction of the afferent arteriole


b) Decrease in the pressure of the afferent arteriole
c) Decrease in concentration of plasma protein
d) Decreased hydrostatic pressure in Bowman's capsule
e) Decrease in the radius of efferent arterioles

The answer is: A

89 ‫صفحة‬ Dr. MOHAMED YAHIA 0900987639


QUESTION BANK OF Dr. MOHAMED YAHIA

67. The tubular fluid as it leaves the proximal tubules is:

a) Hypotonic
b) Hypertonic
c) Has the same sodium concentration as the filtrate
d) Has a volume that is 20% of the filtrate
e) Has very low inulin concentration following inulin administration

The answer is: C

68. The clearance of a substance was found to be less than that of inulin. This
substance is:

a) Not bound to plasma proteins


b) Actively reabsorbed
c) Secreted
d) Neither reabsorbed nor secreted
e) Secreted at a rate greater than reabsorbed

The answer is: B

69. High osmalarity of the extracellular fluid results in:

a) Decreased thirst stimulation


b) Increased rennin secretion
c) Increased ADH secretion
d) Increased urine formation
e) Increased stimulation of macula densa cells

The answer is: C

70. In the proximal tubules:

a) 20% of filtered amino acid is reabsorbed


b) Sodium reabsorption is aldosterone dependent
c) Equal percentage of sodium and water is reabsorbed
d) Potassium is mainly secreted
e) Calcium cannot be reabsorbed

The answer is: C

71. Renin secretion by the kidney is increased by increasing:

a) Tubular fluid flow rate


b) Mean systemic blood pressure
c) Atrial natriuretic peptide
d) Renal sympathetic nerve activity
e) Discharge from baroreceptors

90 ‫صفحة‬ Dr. MOHAMED YAHIA 0900987639


QUESTION BANK OF Dr. MOHAMED YAHIA

The answer is: D

72. The segment of the nephron that contributes most to potassium excretion
when dietary potassium is altered is:

a) Proximal convulated tube


b) Descending limb of Henle's loop
c) Proximal straight tubule
d) Distal tubule
e) Thick ascending limb of Henle's loop

The answer is: D

73. The ability of the kidney to excrete a concentrated urine will increase if:

a) The permeability of the proximal tubules decreases


b) The rate of blood flow through the medulla decreases
c) The rate of flow through the loop of Henle increases
d) The activity of the Na-K pump in the loop of Henle decreases

The answer is: C

74. Aldosterone secretion is increased when there is an increase in the plasma


concentration of

a. ACTH
b. Chloride
c. Sodium
d. Hydrogen
e. Potassium

The answer is: E

75. In which one of the following situations is urinary flow less than normal?

a. Diabetes insipidus
b. Diabetes mellitus
c. Sympathetic stimulation
d. Increased renal arterial pressure
e. Infusion of mannitol

The answer is: C

76. Most of the volatile acid entering the blood is buffered by

a. Bicarbonate
b. Plasma proteins
c. Hemoglobin
d. Phosphates

91 ‫صفحة‬ Dr. MOHAMED YAHIA 0900987639


QUESTION BANK OF Dr. MOHAMED YAHIA

e. Lactate

The answer is: A

77. Potassium-sparing diuretics inhibit Na + reabsorption in the

a. Proximal tubule
b. Thin descending limb of Henle’s loop
c. Thick descending limb of Henle’s loop
d. Distal convoluted tubule
e. Cortical collecting duct

The answer is: E

78. Which one of the following values will be above normal in a diabetic patient
with a blood glucose concentration of 600 meq/L?

a. Urine flow
b. Intracellular volume
c. Plasma sodium concentration
d. Arterial pH
e. Alveolar P CO 2

The answer is: A

79. Which one of the following will be increased in a patient suffering from
persistent diarrhea?

a. The filtered load of HCO 3 −


b. The production of ammonia by the proximal tubule
c. H + secretion by the distal nephron
d. The anion gap
e. The production of new bicarbonate by the distal nephron

The answer is: C

80. In addition to increasing the permeability of the collecting duct to water,


ADH increases the permeability of the collecting duct to

a. Hydrogen
b. Ammonium
c. Potassium
d. Sodium
e. Urea

The answer is: E

81. The filtration fraction is increased by

92 ‫صفحة‬ Dr. MOHAMED YAHIA 0900987639


QUESTION BANK OF Dr. MOHAMED YAHIA

a. Increasing renal blood flow


b. Increasing afferent arteriolar resistance
c. Increasing efferent arteriolar resistance
d. Increasing plasma oncotic pressure
e. Increasing the pressure within Bowman’s capsule

The answer is: C

82. Which one of the following conditions causes a higher than normal plasma
bicarbonate concentration?

a. Volume depletion
b. Renal failure
c. Hypoxemia
d. Diarrhea
e. Hypoaldosteronism

The answer is: A

83. Which one of the following substances causes renal blood flow to decrease?

a. Nitric oxide
b. Bradykinin
c. Prostaglandins
d. Adenosine
e. Dopamine

The answer is: D

84. Diuretics, such as acetazolamide, which produce their effect by inhibiting


carbonic anhydrase, inhibit the reabsorption of sodium in

a. The proximal tubule


b. The thick ascending limb of Henle’s loop
c. The distal convoluted tubule
d. The cortical collecting duct
e. The outer medullary collecting duct

The answer is: A

85. The extracellular potassium of a hyperkalemic patient can be decreased by


administering

a. Atropine
b. Epinephrine
c. Glucagon
d. Lactic acid
e. Isotonic saline

93 ‫صفحة‬ Dr. MOHAMED YAHIA 0900987639


QUESTION BANK OF Dr. MOHAMED YAHIA

The answer is: E

86. Which one of the following blood-gas values is consistent with metabolic
acidosis?

Pa CO2 HCO3- pH

(mmHg) (mM)

a. 25 30 7.7
b. 35 20 7.3
c. 40 25 7.4
d. 50 30 7.1
e. 60 20 7.1

The answer is: B

87. Hyperkalemia may be observed in patients with

a. Volume depletion
b. Diuretic therapy
c. Administration of insulin
d. Metabolic alkalosis
e. Stimulation of adrenal medulla

The answer is: B

88. Which one of the following substances is less concentrated at the end of the
proximal tubule than at the beginning of the proximal tubule?

a. Creatinine
b. Hydrogen
c. Chloride
d. Phosphate
e. Sodium

The answer is: D

89. What percentage of the filtered load of sodium is reabsorbed by the proximal
tubule?

a. 15%
b. 25%
c. 45%
d. 65%
e. 95%

The answer is: D

94 ‫صفحة‬ Dr. MOHAMED YAHIA 0900987639


QUESTION BANK OF Dr. MOHAMED YAHIA

59. ANP (atrial natruretic hormone) decreases Na reabsorption within

a. The proximal tubule


b. The thick ascending limb of Henle’s loop
c. The distal convoluted tubule
d. The cortical collecting duct
e. The inner medullary collecting duct

The answer is: E

90. Free water clearance by the kidney is increased by

a. Heart failure
b. Renal failure
c. Diuretic therapy
d. Diabetes mellitis
e. Diabetes insipidus

The answer is: E

91. The renal clearance of phosphate is increased by

a. Aldosterone
b. Parathyroid hormone
c. Norepinephrine
d. Vasopressin
e. Angiotensin

The answer is: B

92. Hyponatremia will result from an excess secretion of

a. Vasopressin
b. Atrial natriuretic hormone
c. Norepinephrine
d. Insulin
e. Aldosterone

The answer is: A

93. Which one of the following will produce the greatest increase in potassium
secretion?

a. A decrease in urinary flow rates


b. An increase in distal nephron sodium concentration
c. A decrease in circulating blood volume
d. An increase in sympathetic nerve activity
e. A decrease in renal blood flow

95 ‫صفحة‬ Dr. MOHAMED YAHIA 0900987639


QUESTION BANK OF Dr. MOHAMED YAHIA

The answer is: B

94. A decrease in the concentration of NaCl in the intraluminal fluid causes the
juxtaglomerular apparatus to release

a. ADH
b. Aldosterone
c. Adenosine
d. Renin
e. Angiotensinogen

The answer is: D

95. Which one of the following will be observed in a patient with chronic renal
failure?

a. A decrease in the fractional excretion of sodium


b. An increase in the free water clearance
c. An increase in the anion gap
d. A decrease in the excretion of creatinine
e. An increase in the net acid excretion

The answer is: D

96. The extracellular fluid volume of a normal individual is approximately

a. 5% of body mass
b. 10% of body mass
c. 20% of body mass
d. 40% of body mass
e. 60% of body mass

The answer is: C

97. Which one of the following signs of renal failure is caused by the loss of a
hormone produced by the kidney?

a. Edema
b. Hypertension
c. Anemia
d. Uremia
e. Acidosis

The answer is: C

96 ‫صفحة‬ Dr. MOHAMED YAHIA 0900987639


QUESTION BANK OF Dr. MOHAMED YAHIA

BRS PHYSIOLOGY

CARDIOVASCULAR SYSTEM
1. A 53-year-old woman is found, by arteriography, to have 50% narrowing of
her left renal artery. What is the expected change in blood flow through the
stenotic artery?

A. Decrease to 1⁄2
B. Decrease to 1⁄4
C. Decrease to 1⁄8
D. Decrease to 1 ⁄ 16
E. No change

The answer is: D

2. When a person moves from a supine position to a standing position, which of


the following compensatory changes occurs?

(A) Decreased heart rate


(B) Increased contractility
(C) Decreased total peripheral resistance (TPR)
(D) Decreased cardiac output
(E) Increased PR intervals

The answer is: B

3. At which site is systolic blood pressure the highest?

(A) Aorta
(B) Central vein
(C) Pulmonary artery
(D) Right atrium
(E) Renal artery
(F) Renal vein

The answer is: E

4. A person’s electrocardiogram (ECG) has no P wave, but has a normal QRS


complex and a normal T wave. Therefore, his pacemaker is located in the

(A) sinoatrial (SA) node


(B) atrioventricular (AV) node
(C) bundle of His
(D) Purkinje system

97 ‫صفحة‬ Dr. MOHAMED YAHIA 0900987639


QUESTION BANK OF Dr. MOHAMED YAHIA

(E) ventricular muscle

The answer is: E

5. If the ejection fraction increases, there will be a decrease in

(A) cardiac output


(B) end-systolic volume
(C) heart rate
(D) pulse pressure
(E) stroke volume
(F) systolic pressure

The answer is: C

Questions 6 and 7: An electrocardiogram (ECG) on a person shows ventricular


extrasystoles.

6. The extrasystolic beat would produce

(A) increased pulse pressure because contractility is increased


(B) increased pulse pressure because heart rate is increased
(C) decreased pulse pressure because ventricular filling time is increased
(D) decreased pulse pressure because stroke volume is decreased
(E) decreased pulse pressure because the PR interval is increased

The answer is: D

7. After an extrasystole, the next “normal” ventricular contraction produces

(A) increased pulse pressure because the contractility of the ventricle is increased
(B) increased pulse pressure because total peripheral resistance (TPR) is
decreased
(C) increased pulse pressure because compliance of the veins is decreased
(D) decreased pulse pressure because the contractility of the ventricle is
increased
(E) decreased pulse pressure because TPR is decreased

The answer is: A

8. An increase in contractility is demonstrated on a Frank–Starling diagram by

(A) increased cardiac output for a given end-diastolic volume


(B) increased cardiac output for a given end-systolic volume
(C) decreased cardiac output for a given end-diastolic volume
(D) decreased cardiac output for a given end-systolic volume

The answer is: A

98 ‫صفحة‬ Dr. MOHAMED YAHIA 0900987639


QUESTION BANK OF Dr. MOHAMED YAHIA

9. The tendency for blood flow to be turbulent is increased by

(A) increased viscosity


(B) increased hematocrit
(C) partial occlusion of a blood vessel
(D) decreased velocity of blood flow

The answer is: C

0. Following a sympathectomy, a 66-year old man experiences orthostatic


hypotension. The explanation for this occurrence is

(A) an exaggerated response of the renin–angiotensin–aldosterone system


(B) a suppressed response of the renin–angiotensin–aldosterone system
(C) an exaggerated response of the baroreceptor mechanism
(D) a suppressed response of the baroreceptor mechanism

The answer is: D

11. The ventricles are completely depolarized during which isoelectric portion of
the electrocardiogram (ECG)?

(A) PR interval
(B) QRS complex
(C) QT interval
(D) ST segment
(E) T wave

The answer is: D

12. In which of the following situations is pulmonary blood flow greater than
aortic blood flow?

(A) Normal adult


(B) Fetus
(C) Left-to-right ventricular shunt
(D) Right-to-left ventricular shunt
(E) Right ventricular failure
(F) Administration of a positive inotropic Agent

The answer is: C

13. A 30-year-old female patient’s electrocardiogram (ECG) shows two P waves


preceding each QRS complex. The interpretation of this pattern is

(A) decreased firing rate of the pacemaker in the sinoatrial (SA) node
(B) decreased firing rate of the pacemaker in the atrioventricular (AV) node

99 ‫صفحة‬ Dr. MOHAMED YAHIA 0900987639


QUESTION BANK OF Dr. MOHAMED YAHIA

(C) increased firing rate of the pacemaker in the SA node


(D) decreased conduction through the AV node
(E) increased conduction through the His Purkinje system

The answer is: D

14. An acute decrease in arterial blood pressure elicits which of the following
compensatory changes?

(A) Decreased firing rate of the carotid sinus nerve


(B) Increased parasympathetic outflow to the heart
(C) Decreased heart rate
(D) Decreased contractility
(E) Decreased mean systemic pressure

The answer is: A

15. The tendency for edema to occur will be increased by

(A) arteriolar constriction


(B) increased venous pressure
(C) increased plasma protein concentration
(D) muscular activity

The answer is: B

16. Inspiration “splits” the second heart sound because

(A) the aortic valve closes before the pulmonic valve


(B) the pulmonic valve closes before the aortic valve
(C) the mitral valve closes before the tricuspid valve
(D) the tricuspid valve closes before the mitral valve
(E) filling of the ventricles has fast and slow components

The answer is: A

17. During exercise, total peripheral resistance (TPR) decreases because of the
effect of

(A) the sympathetic nervous system on splanchnic arterioles


(B) the parasympathetic nervous system on skeletal muscle arterioles
(C) local metabolites on skeletal muscle arterioles
(D) local metabolites on cerebral arterioles
(E) histamine on skeletal muscle arterioles

The answer is: C

18. An increase in arteriolar resistance, without a change in any other


component of the cardiovascular system, will produce

100 ‫صفحة‬ Dr. MOHAMED YAHIA 0900987639


QUESTION BANK OF Dr. MOHAMED YAHIA

(A) a decrease in total peripheral resistance (TPR)


(B) an increase in capillary filtration
(C) an increase in arterial pressure
(D) a decrease in afterload

The answer is: C

19. The following measurements were obtained in a male patient:

❖ Central venous pressure: 10 mm Hg


❖ Heart rate: 70 beats/min
❖ Pulmonary vein [O 2 ] = 0.24 mL O 2 /mL
❖ Pulmonary artery [O 2 ] = 0.16 mL O 2 /mL
❖ Whole body O 2 consumption: 500 mL/min

What is this patient’s cardiac output?

(A) 1.65 L/min


(B) 4.55 L/min
(C) 5.00 L/min
(D) 6.25 L/min
(E) 8.00 L/min

The answer is: D

20. Which of the following is the result of an inward Na + current?

(A) Upstroke of the action potential in the sinoatrial (SA) node


(B) Upstroke of the action potential in Purkinje fibers
(C) Plateau of the action potential in ventricular muscle
(D) Repolarization of the action potential in ventricular muscle
(E) Repolarization of the action potential in the SA node

The answer is: B

21. The greatest pressure decrease in the circulation occurs across the arterioles
because

(A) they have the greatest surface area


(B) they have the greatest cross-sectional area
(C) the velocity of blood flow through them is the highest
(D) the velocity of blood flow through them is the lowest
(E) they have the greatest resistance

The answer is: E

22. Pulse pressure is

(A) the highest pressure measured in the arteries

101 ‫صفحة‬ Dr. MOHAMED YAHIA 0900987639


QUESTION BANK OF Dr. MOHAMED YAHIA

(B) the lowest pressure measured in the arteries


(C) measured only during diastole
(D) determined by stroke volume
(E) decreased when the capacitance of the arteries decreases
(F) the difference between mean arterial pressure and central venous pressure

The answer is: D

23. In the sinoatrial (SA) node, phase 4 depolarization (pacemaker potential) is


attributable to

(A) an increase in K + conductance


(B) an increase in Na + conductance
(C) a decrease in Cl − conductance
(D) a decrease in Ca 2+ conductance
(E) simultaneous increases in K + and Cl – conductances

The answer is: B

24. Which receptor mediates constriction of arteriolar smooth muscle?

(A) α 1 Receptors
(B) β 1 Receptors
(C) β 2 Receptors
(D) Muscarinic receptors

The answer is: A

25. During which phase of the cardiac cycle is aortic pressure highest?

(A) Atrial systole


(B) Isovolumetric ventricular contraction
(C) Rapid ventricular ejection
(D) Reduced ventricular ejection
(E) Isovolumetric ventricular relaxation
(F) Rapid ventricular filling
(G) Reduced ventricular filling (diastasis)

The answer is: D

26. Myocardial contractility is best correlated with the intracellular


concentration of

(A) Na +
(B) K+
(C) Ca 2+
(D) Cl −
(E) Mg 2+

102 ‫صفحة‬ Dr. MOHAMED YAHIA 0900987639


QUESTION BANK OF Dr. MOHAMED YAHIA

The answer is: C

27. Which of the following is an effect of histamine?

(A) Decreased capillary filtration


(B) Vasodilation of the arterioles
(C) Vasodilation of the veins
(D) Decreased P c
(E) Interaction with the muscarinic receptors on the blood vessels

The answer is: B

28. Carbon dioxide (CO 2 ) regulates blood flow to which one of the following
organs?

(A) Heart
(B) Skin
(C) Brain
(D) Skeletal muscle at rest
(E) Skeletal muscle during exercise

The answer is: C

29. Cardiac output of the right side of the heart is what percentage of the cardiac
output of the left side of the heart?

(A) 25%
(B) 50%
(C) 75%
(D) 100%
(E) 125%

The answer is: D

30. The physiologic function of the relatively slow conduction through the
atrioventricular (AV) node is to allow sufficient time for

(A) runoff of blood from the aorta to the arteries


(B) venous return to the atria
(C) filling of the ventricles
(D) contraction of the ventricles
(E) repolarization of the ventricles

The answer is: C

31. Blood flow to which organ is controlled primarily by the sympathetic nervous
system rather than by local metabolites?

(A) Skin

103 ‫صفحة‬ Dr. MOHAMED YAHIA 0900987639


QUESTION BANK OF Dr. MOHAMED YAHIA

(B) Heart
(C) Brain
(D) Skeletal muscle during exercise

The answer is: A

32. Which of the following parameters is decreased during moderate exercise?

(A) Arteriovenous O 2 difference


(B) Heart rate
(C) Cardiac output
(D) Pulse pressure
(E) Total peripheral resistance (TPR)

The answer is: E

33. When propranolol is administered, blockade of which receptor is responsible


for the decrease in cardiac output that occurs?

(A) α 1 Receptors
(B) β 1 Receptors
(C) β 2 Receptors
(D) Muscarinic receptors
(E) Nicotinic receptors

The answer is: B

34. During which phase of the cardiac cycle is ventricular volume lowest?

(A) Atrial systole


(B) Isovolumetric ventricular contraction
(C) Rapid ventricular ejection
(D) Reduced ventricular ejection
(E) Isovolumetric ventricular relaxation
(F) Rapid ventricular filling
(G) Reduced ventricular filling (diastasis)

The answer is: E

35. Which of the following changes will cause an increase in myocardial O 2


consumption?

(A) Decreased aortic pressure


(B) Decreased heart rate
(C) Decreased contractility
(D) Increased size of the heart
(E) Increased influx of Na + during the upstroke of the action potential

104 ‫صفحة‬ Dr. MOHAMED YAHIA 0900987639


QUESTION BANK OF Dr. MOHAMED YAHIA

The answer is: D

36. Which of the following substances crosses capillary walls primarily through
water-filled clefts between the endothelial cells?

(A) O2
(B) CO 2
(C) CO
(D) Glucose

The answer is: D

37. A 24-year-old woman presents to the emergency department with severe


diarrhea. When she is supine (lying down), her blood pressure is 90/60 mm Hg
(decreased) and her heart rate is 100 beats/min (increased). When she is moved
to a standing position, her heart rate further increases to 120 beats/min. Which
of the following accounts for the further increase in heart rate upon standing?

(A) Decreased total peripheral resistance


(B) Increased venoconstriction
(C) Increased contractility
(D) Increased afterload
(E) Decreased venous return

The answer is: E

38. A 60-year-old businessman is evaluated by his physician, who determines


that his blood pressure is significantly elevated at 185/130 mm Hg. Laboratory
tests reveal an increase in plasma rennin activity, plasma aldosterone level, and
left renal vein renin level. His right renal vein renin level is decreased. What is
the most likely cause of the patient’s hypertension?

(A) Aldosterone-secreting tumor


(B) Adrenal adenoma secreting aldosterone and cortisol
(C) Pheochromocytoma
(D) Left renal artery stenosis
(E) Right renal artery stenosis

The answer is: D

39. Propranolol has which of the following effects?

(A) Decreases heart rate


(B) Increases left ventricular ejection fraction
(C) Increases stroke volume
(D) Decreases splanchnic vascular resistance
(E) Decreases cutaneous vascular resistance

105 ‫صفحة‬ Dr. MOHAMED YAHIA 0900987639


QUESTION BANK OF Dr. MOHAMED YAHIA

The answer is: A

40. Which receptor mediates slowing of the heart?

(A) α 1 Receptors
(B) β 1 Receptors
(C) β 2 Receptors
(D) Muscarinic receptors

The answer is: D

41. Which of the following agents or changes has a negative inotropic effect on
the heart?

(A) Increased heart rate


(B) Sympathetic stimulation
(C) Norepinephrine
(D) Acetylcholine (ACh)
(E) Cardiac glycosides

The answer is: A

42. The low-resistance pathways between myocardial cells that allow for the
spread of action potentials are the

(A) gap junctions


(B) T tubules
(C) sarcoplasmic reticulum (SR)
(D) intercalated disks
(E) mitochondria

The answer is: A

43. Which agent is released or secreted after a hemorrhage and causes an


increase in renal Na + reabsorption?

(A) Aldosterone
(B) Angiotensin I
(C) Angiotensinogen
(D) Antidiuretic hormone (ADH)
(E) Atrial natriuretic peptide

The answer is: A

44. During which phase of the cardiac cycle does the mitral valve open?

(A) Atrial systole

106 ‫صفحة‬ Dr. MOHAMED YAHIA 0900987639


QUESTION BANK OF Dr. MOHAMED YAHIA

(B) Isovolumetric ventricular contraction


(C) Rapid ventricular ejection
(D) Reduced ventricular ejection
(E) Isovolumetric ventricular relaxation
(F) Rapid ventricular filling
(G) Reduced ventricular filling (diastasis)

The answer is: E

4. A hospitalized patient has an ejection fraction of 0.4, a heart rate of 95


beats/min, and a cardiac output of 3.5 L/min. What is the patient’s end-diastolic
volume?

(A) 14 mL
(B) 37 mL
(C) 55 mL
(D) 92 mL
(E) 140 Ml

The answer is: D

RESPIRATORY SYSTEM
1. Which of the following lung volumes or capacities can be measured by
spirometry?

(A) Functional residual capacity (FRC)


(B) Physiologic dead space
(C) Residual volume (RV)
(D) Total lung capacity (TLC)
(E) Vital capacity (VC)

The answer is: E

2. An infant born prematurely in gestational week 25 has neonatal respiratory


distress syndrome. Which of the following would be expected in this infant?

(A) Arterial P O 2 of 100 mm Hg


(B) Collapse of the small alveoli
(C) Increased lung compliance
(D) Normal breathing rate
(E) Lecithin:sphingomyelin ratio of greater than 2:1 in amniotic fluid

The answer is: B

3. In which vascular bed does hypoxia cause vasoconstriction?

(A) Coronary

107 ‫صفحة‬ Dr. MOHAMED YAHIA 0900987639


QUESTION BANK OF Dr. MOHAMED YAHIA

(B) Pulmonary
(C) Cerebral
(D) Muscle
(E) Skin

The answer is: B

Questions 4 and 5: A 12-year-old boy has a severe asthmatic attack with


wheezing. He experiences rapid breathing and becomes cyanotic. His arterial P
O 2 is 60 mm Hg and his P CO 2 is 30 mm Hg.

4. Which of the following statements about this patient is most likely to be true?

(A) Forced expiratory volume/forced vital capacity (FEV 1 /FVC) is increased


(B) Ventilation/perfusion (V/Q) ratio is increased in the affected areas of his
lungs
(C) His arterial P CO 2 is higher than normal because of inadequate gas
exchange
(D) His arterial P CO 2 is lower than normal because hypoxemia is causing him
to hyperventilate
(E) His residual volume (RV) is decreased

The answer is: D

5. To treat this patient, the physician should administer

(A) an α 1 -adrenergic antagonist


(B) a β 1 -adrenergic antagonist
(C) a β 2 -adrenergic agonist
(D) a muscarinic agonist
(E) a nicotinic agonist

The answer is: C

6. Which of the following is true during inspiration?

A. Intrapleural pressure is positive


B. The volume in the lungs is less than the functional residual capacity (FRC)
C. Alveolar pressure equals atmospheric pressure
D. Alveolar pressure is higher than atmospheric pressure
E. Intrapleural pressure is more negative than it is during expiration

The answer is: E

7. Which volume remains in the lungs after a tidal volume (TV) is expired?

(A) Tidal volume (TV)


(B) Vital capacity (VC)

108 ‫صفحة‬ Dr. MOHAMED YAHIA 0900987639


QUESTION BANK OF Dr. MOHAMED YAHIA

(C) Expiratory reserve volume (ERV)


(D) Residual volume (RV)
(E) Functional residual capacity (FRC)
(F) Inspiratory capacity
(G) Total lung capacity

The answer is: E

8. When a person is standing, blood flow in the lungs is

(A) equal at the apex and the base


(B) highest at the apex owing to the effects of gravity on arterial pressure
(C) highest at the base because that is where the difference between arterial and
venous pressure is greatest
(D) lowest at the base because that is where alveolar pressure is greater than
arterial pressure

The answer is: C

9. Which of the following is the site of highest airway resistance?

(A) Trachea
(B) Largest bronchi
(C) Medium-sized bronchi
(D) Smallest bronchi
(E) Alveoli

The answer is: C

10. A 49-year-old man has a pulmonary embolism that completely blocks blood
flow to his left lung. As a result, which of the following will occur?

(A) Ventilation/perfusion (V/Q) ratio in the left lung will be zero


(B) Systemic arterial P O 2 will be elevated
(C) V/Q ratio in the left lung will be lower than in the right lung
(D) Alveolar P O 2 in the left lung will be approximately equal to the P O 2 in
inspired air
(E) Alveolar P O 2 in the right lung will be approximately equal to the P O 2 in
venous blood

The answer is: D

11. In the hemoglobin–O 2 dissociation curves shown above, the shift curve from
left to right could be caused by

(A) increased pH
(B) decreased 2,3-diphosphoglycerate (DPG) concentration
(C) strenuous exercise

109 ‫صفحة‬ Dr. MOHAMED YAHIA 0900987639


QUESTION BANK OF Dr. MOHAMED YAHIA

(D) fetal hemoglobin (HbF)


(E) carbon monoxide (CO) poisoning

The answer is: C

12. The shift curve from left to right is associated with

(A) increased P 50
(B) increased affinity of hemoglobin for O 2
(C) impaired ability to unload O 2 in the tissues
(D) increased O 2 -carrying capacity of hemoglobin
(E) decreased O 2 -carrying capacity of Hemoglobin

The answer is: A

13. Which volume remains in the lungs after a maximal expiration?

(A) Tidal volume (TV)


(B) Vital capacity (VC)
(C) Expiratory reserve volume (ERV)
(D) Residual volume (RV)
(E) Functional residual capacity (FRC)
(F) Inspiratory capacity
(G) Total lung capacity

The answer is: D

14. Compared with the systemic circulation, the pulmonary circulation has a

(A) higher blood flow


(B) lower resistance
(C) higher arterial pressure
(D) higher capillary pressure
(E) higher cardiac output

The answer is: B

15. Compared with the apex of the lung, the base of the lung has

(A) a higher pulmonary capillary P O 2


(B) a higher pulmonary capillary P CO 2
(C) a higher ventilation/perfusion (V/Q) ratio
(D) the same V/Q ratio

The answer is: B

16. Hypoxemia produces hyperventilation by a direct effect on the

(A) phrenic nerve

110 ‫صفحة‬ Dr. MOHAMED YAHIA 0900987639


QUESTION BANK OF Dr. MOHAMED YAHIA

(B) J receptors
(C) lung stretch receptors
(D) medullary chemoreceptors
(E) carotid and aortic body chemoreceptors

The answer is: E

17. Which of the following changes occurs during strenuous exercise?

(A) Ventilation rate and O 2 consumption increase to the same extent


(B) Systemic arterial P O 2 decreases to about 70 mm Hg
(C) Systemic arterial P CO 2 increases to about 60 mm Hg
(D) Systemic venous P CO 2 decreases to about 20 mm Hg
(E) Pulmonary blood flow decreases at the expense of systemic blood flow

The answer is: A

18. If an area of the lung is not ventilated because of bronchial obstruction, the
pulmonary capillary blood serving that area will have a P O 2 that is

(A) equal to atmospheric P O 2


(B) equal to mixed venous P O 2
(C) equal to normal systemic arterial P O 2
(D) higher than inspired P O 2
(E) lower than mixed venous P O 2

The answer is: B

19. In the transport of CO 2 from the tissues to the lungs, which of the following
occurs in venous blood?

(A) Conversion of CO 2 and H 2 O to H + and HCO 3 – in the red blood cells


(RBCs)
(B) Buffering of H + by oxyhemoglobin
(C) Shifting of HCO 3 – into the RBCs from plasma in exchange for Cl –
(D) Binding of HCO 3 – to hemoglobin
(E) Alkalinization of the RBCs

The answer is: A

20. Which of the following causes of hypoxia is characterized by a decreased


arterial P O 2 and an increased A–a gradient?

(A) Hypoventilation
(B) Right-to-left cardiac shunt
(C) Anemia
(D) Carbon monoxide poisoning
(E) Ascent to high altitude

111 ‫صفحة‬ Dr. MOHAMED YAHIA 0900987639


QUESTION BANK OF Dr. MOHAMED YAHIA

The answer is: B

21. A 42-year-old woman with severe pulmonary fibrosis is evaluated by her


physician and has the following

arterial blood gases: pH = 7.48, Pa O 2 = 55 mm Hg, and Pa CO 2 = 32 mm Hg.


Which statement best explains the observed value of Pa CO 2 ?

(A) The increased pH stimulates breathing via peripheral chemoreceptors


(B) The increased pH stimulates breathing via central chemoreceptors
(C) The decreased Pa O 2 inhibits breathing via peripheral chemoreceptors
(D) The decreased Pa O 2 stimulates breathing via peripheral chemoreceptors
(E) The decreased Pa O 2 stimulates breathing via central chemoreceptors

The answer is: D

22. A 38-year-old woman moves with her family from New York City (sea level)
to Leadville Colorado (10,200 feet above sea level). Which of the following will
occur as a result of residing at high altitude?

(A) Hypoventilation
(B) Arterial P O 2 greater than 100 mm Hg
(C) Decreased 2,3-diphosphoglycerate (DPG) concentration
(D) Shift to the right of the hemoglobin–O 2 dissociation curve
(E) Pulmonary vasodilation
(F) Hypertrophy of the left ventricle
(G) Respiratory acidosis

The answer is: D

23. The pH of venous blood is only slightly more acidic than the pH of arterial
blood because

(A) CO 2 is a weak base


(B) there is no carbonic anhydrase in venous blood
(C) the H + generated from CO 2 and H 2 O is buffered by HCO 3 – in venous
blood
(D) the H + generated from CO 2 and H 2 O is buffered by deoxyhemoglobin in
venous blood
(E) oxyhemoglobin is a better buffer for H + than is deoxyhemoglobin

The answer is: D

24. In a maximal expiration, the total volume expired is

(A) tidal volume (TV)


(B) vital capacity (VC)
(C) expiratory reserve volume (ERV)

112 ‫صفحة‬ Dr. MOHAMED YAHIA 0900987639


QUESTION BANK OF Dr. MOHAMED YAHIA

(D) residual volume (RV)


(E) functional residual capacity (FRC)
(F) inspiratory capacity
(G) total lung capacity

The answer is: B

25. A person with a ventilation/perfusion (V/Q) defect has hypoxemia and is


treated with supplemental O 2 . The supplemental O 2 will be most helpful if the
person’s predominant V/Q defect is

(A) dead space


(B) shunt
(C) high V/Q
(D) low V/Q
(E) V/Q = 0
(F) V/Q = ×

The answer is: D

26. Which person would be expected to have the largest A–a gradient?

(A) Person with pulmonary fibrosis


(B) Person who is hypoventilating due to morphine overdose
(C) Person at 12,000 feet above sea level
(D) Person with normal lungs breathing 50% O 2
(E) Person with normal lungs breathing 100% O 2

The answer is: A

GIT
1. Which of the following substances is released from neurons in the GI tract and
produces smooth muscle relaxation?

(A) Secretin
(B) Gastrin
(C) Cholecystokinin (CCK)
(D) Vasoactive intestinal peptide (VIP)
(E) Gastric inhibitory peptide (GIP)

The answer is: D

2. Which of the following is the site of secretion of intrinsic factor?

(A) Gastric antrum


(B) Gastric fundus
(C) Duodenum

113 ‫صفحة‬ Dr. MOHAMED YAHIA 0900987639


QUESTION BANK OF Dr. MOHAMED YAHIA

(D) Ileum
(E) Colon

The answer is: A

3. Vibrio cholerae causes diarrhea because it

(A) increases HCO 3 – secretory channels in intestinal epithelial cells


(B) increases Cl – secretory channels in crypt cells
(C) prevents the absorption of glucose and causes water to be retained in the
intestinal lumen isosmotically
(D) inhibits cyclic adenosine monophosphate (cAMP) production in intestinal
epithelial cells
(E) inhibits inositol 1,4,5-triphosphate (IP 3 ) production in intestinal epithelial
cells

The answer is: D

4. Cholecystokinin (CCK) has some gastrin-like properties because both CCK


and gastrin

(A) are released from G cells in the stomach


(B) are released from I cells in the duodenum
(C) are members of the secretin-homologous family
(D) have five identical C-terminal amino acids
(E) have 90% homology of their amino acids

The answer is: D

5. Which of the following is transported in intestinal epithelial cells by a Na + -


dependent cotransport process?

(A) Fatty acids


(B) Triglycerides
(C) Fructose
(D) Alanine
(E) Oligopeptides

The answer is: D

6. A 49-year-old male patient with severe Crohn’s disease has been unresponsive
to drug therapy and undergoes ileal resection. After the surgery, he will have
steatorrhea because

(A) the liver bile acid pool increases


(B) chylomicrons do not form in the intestinal lumen
(C) micelles do not form in the intestinal lumen
(D) dietary triglycerides cannot be digested

114 ‫صفحة‬ Dr. MOHAMED YAHIA 0900987639


QUESTION BANK OF Dr. MOHAMED YAHIA

(E) the pancreas does not secrete lipase

The answer is: C

7. Cholecystokinin (CCK) inhibits

(A) gastric emptying


(B) pancreatic HCO 3 – secretion
(C) pancreatic enzyme secretion
(D) contraction of the gallbladder
(E) relaxation of the sphincter of Oddi

The answer is: A

8. Which of the following abolishes “receptive relaxation” of the stomach?

(A) Parasympathetic stimulation


(B) Sympathetic stimulation
(C) Vagotomy
(D) Administration of gastrin
(E) Administration of vasoactive intestinal peptide (VIP)
(F) Administration of cholecystokinin (CCK)

The answer is: C

9. Secretion of which of the following substances is inhibited by low pH?

(A) Secretin
(B) Gastrin
(C) Cholecystokinin (CCK)
(D) Vasoactive intestinal peptide (VIP)
(E) Gastric inhibitory peptide (GIP)

The answer is: B

10. Micelle formation is necessary for the intestinal absorption of

(A) glycerol
(B) galactose
(C) leucine
(D) bile acids
(E) vitamin B 12
(F) vitamin D

The answer is: F

11. Which of the following changes occurs during defecation?

(A) Internal anal sphincter is relaxed

115 ‫صفحة‬ Dr. MOHAMED YAHIA 0900987639


QUESTION BANK OF Dr. MOHAMED YAHIA

(B) External anal sphincter is contracted


(C) Rectal smooth muscle is relaxed
(D) Intra-abdominal pressure is lower than when at rest
(E) Segmentation contractions predominate

The answer is: A

12. Which of the following is characteristic of saliva?

(A) Hypotonicity relative to plasma


(B) A lower HCO 3 – concentration than plasma
(C) The presence of proteases
(D) Secretion rate that is increased by vagotomy
(E) Modification by the salivary ductal cells involves reabsorption of K + and
HCO 3 –

The answer is: A

13. Which of the following substances is secreted in response to an oral glucose


load?

(A) Secretin
(B) Gastrin
(C) Cholecystokinin (CCK)
(D) Vasoactive intestinal peptide (VIP)
(E) Glucose-dependent insulinotropic peptide (GIP)

The answer is: E

14. Which of the following is true about the secretion from the exocrine
pancreas?

(A) It has a higher Cl – concentration than does plasma


(B) It is stimulated by the presence of HCO 3 – in the duodenum
(C) Pancreatic HCO 3 – secretion is increased by gastrin
(D) Pancreatic enzyme secretion is increased by cholecystokinin (CCK)
(E) It is hypotonic

The answer is: D

15. Which of the following substances must be further digested before it can be
absorbed by specific carriers in intestinal cells?

(A) Fructose
(B) Sucrose
(C) Alanine
(D) Dipeptides
(E) Tripeptides

116 ‫صفحة‬ Dr. MOHAMED YAHIA 0900987639


QUESTION BANK OF Dr. MOHAMED YAHIA

The answer is: B

16. Slow waves in small intestinal smooth muscle cells are

(A) action potentials


(B) phasic contractions
(C) tonic contractions
(D) oscillating resting membrane potentials
(E) oscillating release of cholecystokinin (CCK)

The answer is: D

17. A 24-year-old male graduate student participates in a clinical research study


on intestinal motility. Peristalsis of the small intestine

(A) mixes the food bolus


(B) is coordinated by the central nervous system (CNS)
(C) involves contraction of smooth muscle behind and in front of the food bolus
(D) involves contraction of smooth muscle behind the food bolus and relaxation
of smooth muscle in front of the bolus
(E) involves relaxation of smooth muscle simultaneously throughout the small
intestine

The answer is: D

18. A patient with a duodenal ulcer is treated successfully with the drug
cimetidine. The basis for cimetidine’s inhibition of gastric H + secretion is that it

(A) blocks muscarinic receptors on parietal cells


(B) blocks H 2 receptors on parietal cells
(C) increases intracellular cyclic adenosine monophosphate (cAMP) levels
(D) blocks H + ,K + -adenosine triphosphatase (ATPase)
(E) enhances the action of acetylcholine (ACh) on parietal cells

The answer is: B

19. Which of the following substances inhibits gastric emptying?

(A) Secretin
(B) Gastrin
(C) Cholecystokinin (CCK)
(D) Vasoactive intestinal peptide (VIP)
(E) Gastric inhibitory peptide (GIP)

The answer is: C

20. When parietal cells are stimulated, they secrete

(A) HCl and intrinsic factor

117 ‫صفحة‬ Dr. MOHAMED YAHIA 0900987639


QUESTION BANK OF Dr. MOHAMED YAHIA

(B) HCl and pepsinogen


(C) HCl and HCO 3 –
(D) HCO 3 – and intrinsic factor
(E) mucus and pepsinogen

The answer is: A

21. A patient with Zollinger–Ellison syndrome would be expected to have which


of the following changes?

(A) Decreased serum gastrin levels


(B) Increased serum insulin levels
(C) Increased absorption of dietary lipids
(D) Decreased parietal cell mass
(E) Peptic ulcer disease

The answer is: E

22. Which of the following is the site of Na + –bile acid cotransport?

(A) Gastric antrum


(B) Gastric fundus
(C) Duodenum
(D) Ileum
(E) Colon

The answer is: D

ENDOCRINE SYSTEM
1. A 41-year-old woman has hypocalcemia, hyperphosphatemia, and decreased
urinary phosphate excretion. Injection of parathyroid hormone (PTH) causes an
increase in urinary cyclic adenosine monophosphate (cAMP). The most likely
diagnosis is

(A) primary hyperparathyroidism


(B) vitamin D intoxication
(C) vitamin D deficiency
(D) hypoparathyroidism after thyroid surgery
(E) pseudohypoparathyroidism

The answer is: D

2. Which of the following hormones acts on its target tissues by a steroid


hormone mechanism of action?

(A) Thyroid hormone


(B) Parathyroid hormone (PTH)

118 ‫صفحة‬ Dr. MOHAMED YAHIA 0900987639


QUESTION BANK OF Dr. MOHAMED YAHIA

(C) Antidiuretic hormone (ADH) on the collecting duct


(D) β 1 adrenergic agonists
(E) Glucagon

The answer is: A

3. A 38-year-old man who has galactorrhea is found to have a prolactinoma. His


physician treats him with bromocriptine, which eliminates the galactorrhea. The
basis for the therapeutic action of bromocriptine is that it

(A) antagonizes the action of prolactin on the breast


(B) enhances the action of prolactin on the breast
(C) inhibits prolactin release from the anterior pituitary
(D) inhibits prolactin release from the hypothalamus
(E) enhances the action of dopamine on the anterior pituitary

The answer is: C

4. Which of the following hormones originates in the anterior pituitary?

(A) Dopamine
(B) Growth hormone–releasing hormone (GHRH)
(C) Somatostatin
(D) Gonadotropin-releasing hormone (GnRH)
(E) Thyroid-stimulating hormone (TSH)
(F) Oxytocin
(G) Testosterone

The answer is: E

5. Which of the following functions of the Sertoli cells mediates negative


feedback control of follicle-stimulating hormone (FSH) secretion?

(A) Synthesis of inhibin


(B) Synthesis of testosterone
(C) Aromatization of testosterone
(D) Maintenance of the blood–testes barrier

The answer is: A

6. Which of the following substances is derived from pro-opiomelanocortin


(POMC)?

(A) Adrenocorticotropic hormone (ACTH)


(B) Follicle-stimulating hormone (FSH)
(C) Melatonin
(D) Cortisol
(E) Dehydroepiandrosterone

119 ‫صفحة‬ Dr. MOHAMED YAHIA 0900987639


QUESTION BANK OF Dr. MOHAMED YAHIA

The answer is: A

7. Which of the following inhibits the secretion of growth hormone by the


anterior pituitary?

(A) Sleep
(B) Stress
(C) Puberty
(D) Somatomedins
(E) Starvation
(F) Hypoglycemia

The answer is: D

8. Selective destruction of the zona glomerulosa of the adrenal cortex would


produce a deficiency of which hormone?

(A) Aldosterone
(B) Androstenedione
(C) Cortisol
(D) Dehydroepiandrosterone
(E) Testosterone

The answer is: A

9. Which of the following explains the suppression of lactation during


pregnancy?

(A) Blood prolactin levels are too low for milk production to occur
(B) Human placental lactogen levels are too low for milk production to occur
(C) The fetal adrenal gland does not produce sufficient estriol
(D) Blood levels of estrogen and progesterone are high
(E) The maternal anterior pituitary is suppressed

The answer is: D

10. Which step in steroid hormone biosynthesis, if inhibited, blocks the


production of all androgenic compounds but does not block the production of
glucocorticoids?

(A) Cholesterol → pregnenolone


(B) Progesterone → 11-deoxycorticosterone
(C) 17-Hydroxypregnenolone → dehydroepiandrosterone
(D) Testosterone → estradiol
(E) Testosterone → dihydrotestosterone

The answer is: C

120 ‫صفحة‬ Dr. MOHAMED YAHIA 0900987639


QUESTION BANK OF Dr. MOHAMED YAHIA

11. A 46-year-old woman has hirsutism, hyperglycemia, obesity, muscle wasting,


and increased circulating levels of adrenocorticotropic hormone (ACTH). The
most likely cause of her symptoms is

(A) primary adrenocortical insufficiency (Addison’s disease)


(B) pheochromocytoma
(C) primary overproduction of ACTH (Cushing’s disease)
(D) treatment with exogenous glucocorticoids
(E) hypophysectomy

The answer is: C

12. Which of the following decreases the conversion of 25-hydroxycholecalciferol


to 1,25-dihydroxycholecalciferol?

(A) A diet low in Ca 2+


(B) Hypocalcemia
(C) Hyperparathyroidism
(D) Hypophosphatemia
(E) Chronic renal failure

The answer is: E

13. Increased adrenocorticotropic hormone (ACTH) secretion would be expected


in patients

(A) with chronic adrenocortical insufficiency (Addison’s disease)


(B) with primary adrenocortical hyperplasia
(C) who are receiving glucocorticoid for immunosuppression after a renal
transplant
(D) with elevated levels of angiotensin II

The answer is: A

14. Which of the following would be expected in a patient with Graves’ disease?

(A) Cold sensitivity


(B) Weight gain
(C) Decreased O 2 consumption
(D) Decreased cardiac output
(E) Drooping eyelids
(F) Atrophy of the thyroid gland
(G) Increased thyroid-stimulating hormone (TSH) levels
(H) Increased triiodothyronine (T 3 ) levels

The answer is: H

121 ‫صفحة‬ Dr. MOHAMED YAHIA 0900987639


QUESTION BANK OF Dr. MOHAMED YAHIA

15. Blood levels of which of the following substances is decreased in Graves’


disease?

(A) Triiodothyronine (T 3 )
(B) Thyroxine (T 4 )
(C) Diiodotyrosine (DIT)
(D) Thyroid-stimulating hormone (TSH)
(E) Iodide (I – )

The answer is: D

16. Which of the following hormones acts by an inositol 1,4,5-triphosphate (IP 3


)–Ca 2+ mechanism of action?

(A) 1,25-Dihydroxycholecalciferol
(B) Progesterone
(C) Insulin
(D) Parathyroid hormone (PTH)
(E) Gonadotropin-releasing hormone (GnRH)

The answer is: E

17. Which step in steroid hormone biosynthesis is stimulated by


adrenocorticotropic hormone (ACTH)?

(A) Cholesterol → pregnenolone


(B) Progesterone → 11-deoxycorticosterone
(C) 17-Hydroxypregnenolone → dehydroepiandrosterone
(D) Testosterone → estradiol
(E) Testosterone → dihydrotestosterone

The answer is: A

18. The source of estrogen during the second and third trimesters of pregnancy
is the

(A) corpus luteum


(B) maternal ovaries
(C) fetal ovaries
(D) placenta
(E) maternal ovaries and fetal adrenal gland
(F) maternal adrenal gland and fetal liver
(G) fetal adrenal gland, fetal liver, and placenta

The answer is: G

122 ‫صفحة‬ Dr. MOHAMED YAHIA 0900987639


QUESTION BANK OF Dr. MOHAMED YAHIA

19. Which of the following causes increased aldosterone secretion?

(A) Decreased blood volume


(B) Administration of an inhibitor of angiotensin-converting enzyme (ACE)
(C) Hyperosmolarity
(D) Hypokalemia

The answer is: A

20. Secretion of oxytocin is increased by

(A) milk ejection


(B) dilation of the cervix
(C) increased prolactin levels
(D) increased extracellular fluid (ECF) volume
(E) increased serum osmolarity

The answer is: B

21. Propylthiouracil can be used to reduce the synthesis of thyroid hormones in


hyperthyroidism because it inhibits oxidation of

(A) Triiodothyronine (T 3 )
(B) Thyroxine (T 4 )
(C) Diiodotyrosine (DIT)
(D) Thyroid-stimulating hormone (TSH)
(E) Iodide (I – )

The answer is: E

22. A 39-year-old man with untreated diabetes mellitus type I is brought to the
emergency room. An injection of insulin would be expected to cause an increase
in his

(A) urine glucose concentration


(B) blood glucose concentration
(C) blood K + concentration
(D) blood pH
(E) breathing rate

The answer is: D

23. Which of the following results from the action of parathyroid hormone
(PTH) on the renal tubule?

(A) Inhibition of 1α-hydroxylase


(B) Stimulation of Ca 2+ reabsorption in the distal tubule
(C) Stimulation of phosphate reabsorption in the proximal tubule

123 ‫صفحة‬ Dr. MOHAMED YAHIA 0900987639


QUESTION BANK OF Dr. MOHAMED YAHIA

(D) Interaction with receptors on the luminal membrane of the proximal tubular
cells
(E) Decreased urinary excretion of cyclic adenosine monophosphate (cAMP)

The answer is: B

24. Which step in steroid hormone biosynthesis occurs in the accessory sex target
tissues of the male and is catalyzed by 5α-reductase?

(A) Cholesterol → pregnenolone


(B) Progesterone → 11-deoxycorticosterone
(C) 17-Hydroxypregnenolone → dehydroepiandrosterone
(D) Testosterone → estradiol
(E) Testosterone → dihydrotestosterone

The answer is: E

25. Which of the following pancreatic secretions has a receptor with four
subunits, two of which have tyrosine kinase activity?

(A) Insulin
(B) Glucagon
(C) Somatostatin
(D) Pancreatic lipase

The answer is: A

26. A 16-year-old, seemingly normal female is diagnosed with androgen


insensitivity disorder. She has never had a menstrual cycle and is found to have a
blind ending vagina; no uterus, cervix, or ovaries; a 46 XY genotype; and
intraabdominal testes. Her serum testosterone is elevated. Which of the following
characteristics is caused by lack of androgen receptors?

(A) 46 XY genotype
(B) Testes
(C) Elevated serum testosterone
(D) Lack of uterus and cervix
(E) Lack of menstrual cycles

The answer is: C

RENAL SYSTEM:
1. Secretion of K + by the distal tubule will be decreased by

(A) metabolic alkalosis


(B) a high-K + diet
(C) hyperaldosteronism

124 ‫صفحة‬ Dr. MOHAMED YAHIA 0900987639


QUESTION BANK OF Dr. MOHAMED YAHIA

(D) spironolactone administration


(E) thiazide diuretic administration

The answer is: D

2. Subjects A and B are 70-kg men. Subject A drinks 2 L of distilled water, and
subject B drinks 2 L of isotonic NaCl. As a result of these ingestions, subject B
will have a

(A) greater change in intracellular fluid (ICF) volume


(B) higher positive free-water clearance (C H 2 O )
(C) greater change in plasma osmolarity
(D) higher urine osmolarity
(E) higher urine flow rate

The answer is: D

A 45-year-old woman develops severe diarrhea while on vacation. She has the
following arterial blood values:

❖ pH = 7.25
❖ P co2 = 24 mm Hg
❖ [HCO 3 – ] = 10 mEq/L
❖ Venous blood samples show decreased blood [K + ] and a normal anion gap.

3. The correct diagnosis for this patient is

(A) metabolic acidosis


(B) metabolic alkalosis
(C) respiratory acidosis
(D) respiratory alkalosis
(E) normal acid–base status

The answer is: A

4. Which of the following statements about this patient is correct?

(A) She is hypoventilating


(B) The decreased arterial [HCO 3 – ] is a result of buffering of excess H + by
HCO 3 –
(C) The decreased blood [K + ] is a result of exchange of intracellular H + for
extra- cellular K +
(D) The decreased blood [K + ] is a result of increased circulating levels of
aldosterone
(E) The decreased blood [K + ] is a result of decreased circulating levels of
antidiuretic hormone (ADH)

The answer is: C

125 ‫صفحة‬ Dr. MOHAMED YAHIA 0900987639


QUESTION BANK OF Dr. MOHAMED YAHIA

5. At plasma concentrations of glucose higher than occur at transport maximum


(T m ), the

(A) clearance of glucose is zero


(B) excretion rate of glucose equals the filtration rate of glucose
(C) reabsorption rate of glucose equals the filtration rate of glucose
(D) excretion rate of glucose increases with increasing plasma glucose
concentrations
(E) renal vein glucose concentration equals the renal artery glucose concentration

The answer is: D

6. A negative free-water clearance (–C H 2 O ) will occur in a person who

(A) drinks 2 L of distilled water in 30 minutes


(B) begins excreting large volumes of urine with an osmolarity of 100 mOsm/L
after a severe head injury
(C) is receiving lithium treatment for depression, and has polyuria that is
unresponsive to the administration of antidiuretic hormone (ADH)
(D) has an oat cell carcinoma of the lung, and excretes urine with an osmolarity
of 1000 mOsm/L

The answer is: D

7. Which of the following would produce an increase in the reabsorption of


isosmotic fluid in the proximal tubule?

(A) Increased filtration fraction


(B) Extracellular fluid (ECF) volume expansion
(C) Decreased peritubular capillary protein concentration
(D) Increased peritubular capillary hydrostatic pressure
(E) Oxygen deprivation

The answer is: A

8. Which of the following substances or combinations of substances could be


used to measure interstitial fluid volume?

(A) Mannitol
(B) D 2 O alone
(C) Evans blue
(D) Inulin and D 2 O
(E) Inulin and radioactive albumin

The answer is: E

9. At plasma para-aminohippuric acid (PAH) concentrations below the


transport maximum (T m ), PAH

126 ‫صفحة‬ Dr. MOHAMED YAHIA 0900987639


QUESTION BANK OF Dr. MOHAMED YAHIA

(A) reabsorption is not saturated


(B) clearance equals inulin clearance
(C) secretion rate equals PAH excretion rate
(D) concentration in the renal vein is close to zero
(E) concentration in the renal vein equals PAH concentration in the renal artery

The answer is: D

10. Compared with a person who ingests 2 L of distilled water, a person with
water deprivation will have a

(A) higher free-water clearance (C H 2 O )


(B) lower plasma osmolarity
(C) lower circulating level of antidiuretic hormone (ADH)
(D) higher tubular fluid/plasma (TF/P) osmolarity in the proximal tubule
(E) higher rate of H 2 O reabsorption in the collecting ducts

The answer is: E

11. Which of the following would cause an increase in both glomerular filtration
rate (GFR) and renal plasma flow (RPF)?

(A) Hyperproteinemia
(B) A ureteral stone
(C) Dilation of the afferent arteriole
(D) Dilation of the efferent arteriole
(E) Constriction of the efferent arteriole

The answer is: C

12. A patient has the following arterial blood values:

❖ pH = 7.52
❖ PCO 2 = 20 mm Hg
❖ [HCO 3 – ] = 16 mEq/L

Which of the following statements about this patient is most likely to be correct?

(A) He is hypoventilating
(B) He has decreased ionized [Ca 2+ ] in blood
(C) He has almost complete respiratory compensation
(D) He has an acid–base disorder caused by overproduction of fixed acid
(E) Appropriate renal compensation would cause his arterial [HCO 3 – ] to
increase

The answer is: B

127 ‫صفحة‬ Dr. MOHAMED YAHIA 0900987639


QUESTION BANK OF Dr. MOHAMED YAHIA

13. Which of the following would best distinguish an otherwise healthy person
with severe water deprivation from a person with the syndrome of inappropriate
antidiuretic hormone (SIADH)?

(A) Free-water clearance (C H 2 O )


(B) Urine osmolarity
(C) Plasma osmolarity
(D) Circulating levels of antidiuretic hormone (ADH)
(E) Corticopapillary osmotic gradient

The answer is: C

14. Which of the following causes a decrease in renal Ca 2+ clearance?

(A) Hypoparathyroidism
(B) Treatment with chlorothiazide
(C) Treatment with furosemide
(D) Extracellular fluid (ECF) volume expansion
(E) Hypermagnesemia

The answer is: B

15. A patient arrives at the emergency room with low arterial pressure, reduced
tissue turgor, and the following arterial blood values:

❖ pH = 7.69
❖ [HCO 3 – ] = 57 mEq/L
❖ PCO 2 = 48 mm Hg

Which of the following responses would also be expected to occur in this patient?

(A) Hyperventilation
(B) Decreased K + secretion by the distal tubules
(C) Increased ratio of H 2 PO 4 – to HPO 4 –2 in urine
(D) Exchange of intracellular H + for extra- cellular K +

The answer is: D

16. A woman has a plasma osmolarity of 300 mOsm/L and a urine osmolarity of
1200 mOsm/L. The correct diagnosis is

(A) syndrome of inappropriate antidiuretic hormone (SIADH)


(B) water deprivation
(C) central diabetes insipidus
(D) nephrogenic diabetes insipidus
(E) drinking large volumes of distilled water

The answer is: B

128 ‫صفحة‬ Dr. MOHAMED YAHIA 0900987639


QUESTION BANK OF Dr. MOHAMED YAHIA

17. Which of the following substances has the highest renal clearance?

(A) Para-aminohippuric acid (PAH)


(B) Inulin
(C) Glucose
(D) Na +
(E) Cl –

The answer is: A

18. A woman runs a marathon in 90°F weather and replaces all volume lost in
sweat by drinking distilled water. After the marathon, she will have

(A) decreased total body water (TBW)


(B) decreased hematocrit
(C) decreased intracellular fluid (ICF) volume
(D) decreased plasma osmolarity
(E) increased intracellular osmolarity

The answer is: D

19. Which of the following causes hyperkalemia?

(A) Exercise
(B) Alkalosis
(C) Insulin injection
(D) Decreased serum osmolarity
(E) Treatment with β-agonists

The answer is: A

20. Which of the following is a cause of metabolic alkalosis?

(A) Diarrhea
(B) Chronic renal failure
(C) Ethylene glycol ingestion
(D) Treatment with acetazolamide
(E) Hyperaldosteronism
(F) Salicylate poisoning

The answer is: E

21. Which of the following is an action of parathyroid hormone (PTH) on the


renal tubule?

(A) Stimulation of adenylate cyclase


(B) Inhibition of distal tubule K + secretion
(C) Inhibition of distal tubule Ca 2+ reabsorption

129 ‫صفحة‬ Dr. MOHAMED YAHIA 0900987639


QUESTION BANK OF Dr. MOHAMED YAHIA

(D) Stimulation of proximal tubular phosphate reabsorption


(E) Inhibition of production of 1,25-dihy- droxycholecalciferol

The answer is: A

22. A man presents with hypertension and hypokalemia. Measurement of his


arterial blood gases reveals a pH of 7.5 and a calculated HCO 3 – of 32 mEq/L.
His serum cortisol and urinary vanillylmandelic acid (VMA) are normal, his
serum aldosterone is increased, and his plasma rennin activity is decreased.
Which of the following is the most likely cause of his hypertension?

(A) Cushing’s syndrome


(B) Cushing’s disease
(C) Conn’s syndrome
(D) Renal artery stenosis
(E) Pheochromocytoma

The answer is: C

23. Which set of arterial blood values describes a heavy smoker with a history of
emphysema and chronic bronchitis who is becoming increasingly somnolent?

pH HCO 3 P CO 2 (mm Hg)

(A) 7.65 48 45
(B) 7.50 15 20
(C) 7.40 24 40
(D) 7.32 30 60
(E) 7.31 16 33

The answer is: D

24. Which set of arterial blood values describes a patient with partially
compensated respiratory alkalosis after 1 month on a mechanical ventilator?

pH HCO 3 – P CO 2 (mm Hg)

(A) 7.65 48 45
(B) 7.50 15 20
(C) 7.40 24 40
(D) 7.32 30 60
(E) 7.31 16 33

The answer is: B

25. Which set of arterial blood values describes a patient with chronic renal
failure (eating a normal protein diet) and decreased urinary excretion of NH 4 +
?

130 ‫صفحة‬ Dr. MOHAMED YAHIA 0900987639


QUESTION BANK OF Dr. MOHAMED YAHIA

pH HCO 3 – P CO 2 (mm Hg)

(A) 7.65 48 45
(B) 7.50 15 20
(C) 7.40 24 40
(D) 7.32 30 60
(E) 7.31 16 33

The answer is: E

26 . Which set of arterial blood values describes a patient with untreated


diabetes mellitus and increased urinary excretion of NH 4 + ?

pH HCO 3 – P CO 2 (mm Hg)

(A) 7.65 48 45
(B) 7.50 15 20
(C) 7.40 24 40
(D) 7.32 30 60
(E) 7.31 16 33

The answer is: E

27. Which set of arterial blood values describes a patient with a 5-day history of
vomiting?

pH HCO 3 – P CO 2 (mm Hg)

(A) 7.65 48 45
(B) 7.50 15 20
(C) 7.40 24 40
(D) 7.32 30 60
(E) 7.31 16 33

The answer is: A

28. A person who takes an aspirin (salicylic acid) overdose is treated in the
emergency room. The treatment produces a change in urine pH that increases
the excretion of salicylic acid. What was the change in urine pH, and what is the
mechanism of increased salicylic acid excretion?

(A) Acidification, which converts salicylic acid to its HA form


(B) Alkalinization, which converts salicylic acid to its A – form
(C) Acidification, which converts salicylic acid to its A – form
(D) Alkalinization, which converts salicylic acid to its HA form

The answer is: B

131 ‫صفحة‬ Dr. MOHAMED YAHIA 0900987639


QUESTION BANK OF Dr. MOHAMED YAHIA

‫شيت الجامعة‬
CARDIOVASCULAR SYSTEM
1. Which of the following is part of the reflex response to an increase in arterial
pressure?

a) Decreased firing of carotid sinus baroreceptors

b) Increased sympathetic activity to the ventricles

c) Increased parasympathetic activity to the SA Node

d) Increased parasympathetic activity to the arterioles of skeletal muscles &

skin

e) Increased parasympathetic stimulation to the ventricles

2. Correct sequences of steps in short-term compensation for hemorrhage


include:

a) Decreased arterial pressure → Increased baroreceptor firing rate

b) Increased formation of Angiotensin II → Increased renin released by

kidneys

c) Decreased excretion of Na+ and water → Increased aldosterone formation

d) Decreased firing of baroreceptors → Increased sympathetic activity

e) Decreased atrial volume → Increased volume receptor firing rate

‫س‬3. Venous Return:

a) Is increased on standing

b) Decreases by deep inspiration

c) Is decreased by venoconstriction

d) When increased, activates Bainbridge reflex

132 ‫صفحة‬ Dr. MOHAMED YAHIA 0900987639


QUESTION BANK OF Dr. MOHAMED YAHIA

e) When increased, increases end-systolic volume

4. The nerve supply to the heart is as follows:

a) SA Node is innervated by sympathetic only

b) Atrial muscle is innervated by both sympathetic & parasympathetic

c) Right vagus supplies SA node & AV node

d) Left vagus supplies SA node

e) Parasympathetic supplies ventricular muscle

5. Which of the following is not a characteristic of cardiac muscle?

a) It's a syncytium of muscle fibers

b) There are intercalated discs

c) Gap junctions cause spread of depolarization

d) Has myosin and actin muscles

e) There are paracellular spaces

6. The pacemaker prepotential:

a) Is due to a slow decrease in K+ influx

b) Is a slow increase in Resting Membrane Potential

c) Maintained by opening of long acting Ca2+ channels

d) Is augmented by opening of transient Ca2+ channels

e) Occurs only in the SA node

7. Parasympathetic stimulation results in:

a) Decreases K+ efflux in cardiac muscle

b) Increases Ca2+ influx in SA node

133 ‫صفحة‬ Dr. MOHAMED YAHIA 0900987639


QUESTION BANK OF Dr. MOHAMED YAHIA

c) Bradycardia

d) Increases Na+ influx

e) Increasing slope of prepotential

8. Which of the following is true of the electrical activity of cardiac muscle?


a) Increased extracellular K+ causes depolarization

b) Repolarization is due to Na+ current

c) Extracellular Na+ affects the pacemaker potential

d) Plateu of action potential is due to Ca2+ influx

e) Initial repolarization is due to delayed K+ efflux

9. The conductive system of the heart does not include:

a) Internodal pathways

b) Bundle of His & its branches

c) Purkinje system of fibers

d) Interventricular septum

e) AV node

10. Which of the following is not true of the ECG?

a) Needs six pairs of electrodes

b) Is useful in detection of arrhythmias

c) It is recorded from limb leads

d) Its amplitude indicates the mass of cardiac muscle

e) The PR interval indicates strength of contraction

11. In a normal ECG:

a) The P wave indicates the condition of the conductive system

134 ‫صفحة‬ Dr. MOHAMED YAHIA 0900987639


QUESTION BANK OF Dr. MOHAMED YAHIA

b) The P wave represents depolarization of atrial myocardium

c) QRS complex is mainly negative due to spread of depolarization down

d) QRS is due to depolarization in the ventricular septum

e) The T wave is a positive wave in all chest leads

12. Which of the following is true of the cardiac cycle?

a) Ventricular diastole follows atrial systole

b) Ventricular systole causes an immediate rise in aortic pressure

c) The beginning of systole causes closure of AV valves

d) Closure of semi-lunar valves gives rise to first heart sound

e) Isovolumetric relaxation occurs at the end of diastole

‫س‬13. In the atrial pressure wave:

a) The maximum pressure is about 12 mmHg

b) Pressure remains constant while AV valves are closed

c) The C wave is due to bulging of AV valve during ventricular filling

d) The V wave is due to filling of the ventricles

e) The A wave is due to atrial systole

14. Protodiastole:

a) Is characterized by a rapid rise in aortic pressure

b) Occurs at the onset of diastole

c) Is a period in the cardiac cycle with constant blood pressure

d) Is the last one-third of diastole

e) Is a period of slow ejection from the ventricles

135 ‫صفحة‬ Dr. MOHAMED YAHIA 0900987639


QUESTION BANK OF Dr. MOHAMED YAHIA

15. Which of the following statements is true of the heart sounds?

a) The first sound is of high pitch

b) The first sound is of longer duration than the second

c) They are produced by abnormal valves

d) The second sound is heard best at the apex

e) The second sound is due to the closure of AV valves

16. Pressure in the pulmonary artery is:

a) Higher than that in the right atrium

b) About 25 mmHg during systole

c) About 80 mmHg during diastole

d) Is lower than that in the right ventricle

e) Is regulated by parasympathetic innervation

17. The following is not correct about baroreceptors:

a) Found in the wall of carotid sinuses

b) Found in the aortic arch

d) Send impulses through the glosso-pharyngeal nerve

e) Normally fires during late systole

‫س‬18. Chemoreceptors are:

a) Found in aortic and carotid sinuses

b) Stimulated best by hypercapnoea

c) Send impulses to vasomotor center through vagus nerve

d) Have a high blood flow rate

e) Are not sensitive to hydrogen ion concentration

136 ‫صفحة‬ Dr. MOHAMED YAHIA 0900987639


QUESTION BANK OF Dr. MOHAMED YAHIA

19. At rest the LV end-systolic volume is:

a) 10 mL

b) 30 mL

c) 50 mL

d) 120 mL

e) 140 mL

20. The best site to measure mixed venous PO2 is:

a) Superior Vena Cava

b) Right Atrium

c) Pulmonary Artery

d) Pulmonary Vein

e) Left Ventricle

21. The atrial contraction component of ventricular filling is:

a) 5%

b) 10%

c) 30%

d) 50%

e) 80%

22. The lowest intrinsic discharge activity resides in the:

a) SA Node

b) AV Node

c) Bundle of His

137 ‫صفحة‬ Dr. MOHAMED YAHIA 0900987639


QUESTION BANK OF Dr. MOHAMED YAHIA

d) Bundle of branches

e) Purkinje fibers

23. The highest oxygen extraction is found in the:

a) Resting skeletal muscles

b) Heart

c) Kidney

d) Brain

e) Skin

24. With a mixed venous oxygen content of 110 ml/L and an arterial oxygen
content of 150 ml/L an oxygen uptake of 280 ml/min, the cardiac output is:

a) 5 liters/ min

b) 6 liters/ min

c) 7 liters/ min

d) 8 liters/ min

e) 9 liters/ min

25. The pulmonary valve closes when the pressure in the right ventricle is about:

a) 0 mmHg

b) 15 mmHg

c) 30 mmHg

d) 50 mmHg

e) 120 mmHg

26. The velocity of blood flow is the slowest in:

a) Capillaries

138 ‫صفحة‬ Dr. MOHAMED YAHIA 0900987639


QUESTION BANK OF Dr. MOHAMED YAHIA

b) Pulmonary vein

c) Small arteries

d) Inferior Vena Cava

e) Arterioles

27. The volume of blood is greatest in:

a) Systemic capillaries

b) Veins

c) Arteries

d) The spleen

e) The heart

28. Which of the following results in increased stroke volume?

a) An increase in end-diastolic volume

b) An increased after-load

c) Parasympathetic stimulation

d) An increase in the heart rate

e) Venodilation

29. During the cardiac cycle, closure of the aortic valve occurs at:

a) The end of isovolumetric contraction

b) The beginning of rapid ejection phase

c) The beginning of isometric relaxation

d) The end of systole

e) The end of rapid filling phase

30. SA node is the pacemaker of the heart because:

a) Location in the right atrium

139 ‫صفحة‬ Dr. MOHAMED YAHIA 0900987639


QUESTION BANK OF Dr. MOHAMED YAHIA

b) Neural control

c) Natural leakiness to Cl-

d) Natural leakiness to K+

e) Fastest rate of discharge

31. Stroke Volume:

a) Is the volume of blood pumped by the heart per minute

b) Decreases by increased end-diastolic volume

c) Is decreased by increased afterload

d) From the left ventricle is more than from the right ventricle

e) Multiplied by heart rate gives cardiac index

32. In the ECG, the T wave denotes:

a) Atrial contraction

b) Atrial depolarization

c) Ventricular repolarization

d) SA node depolarization

e) Ventricular contraction

33. Cardiac Output (in liters per minute) divided by the heart rate (in beats per
minute) equals to:

a) Cardiac Index

b) Cardiac Efficiency

c) Mean Arterial Pressure

d) Stroke Volume

e) Blood Velocity

140 ‫صفحة‬ Dr. MOHAMED YAHIA 0900987639


QUESTION BANK OF Dr. MOHAMED YAHIA

34. The segment of the vascular bed responsible for local regulation of blood flow
in most tissues is:

a) Distributing arteries

b) Large veins

c) Capillaries

d) Venules

e) Arterioles

35. Absolute refractory period of the heart:

a) Corresponds to the duration of relaxation

b) Lasts till half of cardiac contraction

c) Shorter than refractory period in skeletal muscle

d) Lasts till cardiac contraction

e) The heart can be stimulated by very strong stimulus

36. First heart sound occurs at the beginning of:

a) Isometric relaxation

b) Isotonic relaxation

c) Isovolumetric contraction

d) Isovolumetric relaxation

e) Atrial contraction

37. In the heart, within physiological limits the force of contraction is directly
proportional to the:

a) Pacemaker activity

b) AV nodal delay

c) Initial length of cardiac muscle

d) Respiratory rate

141 ‫صفحة‬ Dr. MOHAMED YAHIA 0900987639


QUESTION BANK OF Dr. MOHAMED YAHIA

e) Vagal stimulation

38. Cardiac muscle has a long refractory period because:

a) The impulse takes about 0.2 s to travel from SA to AV node

b) It obeys the all-or-none law

c) Of the delay in Na+ influx

d) Of Ca2+ influx

e) Is more permeable to K+

39. Stimulation of baroreceptors leads to:

a) Tachycardia

b) Increased stroke volume

c) Stimulation of vasomotor center

d) Vasoconstriction

e) Decreased arterial blood pressure

40. The fourth heart sound is due to:

a) Closure of mitral and tricuspid valve

b) Iso-volumetric contraction

c) Iso-volumetric relaxation

d) Ventricular filling

e) Atrial systole

41. Frank Straling's law of the heart:

a) Explains the tachycardia caused by increased venouse return

b) Explains the tachycardia of exercise

c) Does not operate when the person is at rest

142 ‫صفحة‬ Dr. MOHAMED YAHIA 0900987639


QUESTION BANK OF Dr. MOHAMED YAHIA

d) Explains the increased venous return when end-diastolic volume is

increased

e) Explains the increased stroke volume when end-diastolic volume is

increased

42. The work done by the right ventricle is much less than that done by the left
ventricle because its:

a) Wall is thinner

b) Stroke volume is less

c) Preload is less

d) Afterload is less

e) Systolic pressure is less

43. The following are not signs of pure right ventricular failure:

a) Increased central venous pressure

b) Central venous engorgement and pulsations

c) Pulmonary oedema

d) Hepatomegally

e) Sacral oedema

44. The blood vessel responsible for local regulation of blood flow in most tissues
is:

a) Distributing arteries

b) Large veins

c) Capillaries

d) Venules

e) Arterioles

143 ‫صفحة‬ Dr. MOHAMED YAHIA 0900987639


QUESTION BANK OF Dr. MOHAMED YAHIA

45. The heart rate is increased by all the following except:

a) Fever

b) Increased venous return

c) Atropine

d) Hypothyroidism

e) Adrenaline

46. Stroke Volume:

a) Increases as a result of increased afterload

b) Equals end-diastolic volume minus end-systolic volume

c) Increases as heart rate is increased by electrical pacing

d) Is increased by parasympathetic stimulation

e) Is increased by Ca2+ channel blocks

47. A drug that increases the heart rate from 70 to 100 beats per minute could
be:

a) Stimulation of B-1 adrenergic receptors

b) Inhibitor of alpha adrenergic receptors

c) Stimulation of muscarinic cholinergic receptors

d) Inhibitor B-2 adrenergic receptors

e) Digitalis

48. A high preload is indicated by:

a) Blood volume

b) Pulmonary pressure

c) End-diastolic volume

144 ‫صفحة‬ Dr. MOHAMED YAHIA 0900987639


QUESTION BANK OF Dr. MOHAMED YAHIA

d) Systolic blood pressure

e) Diastolic blood pressure

49. Regurgitation of aortic valve leads to:

a) A decrease in diastolic pressure

b) A decrease in oxygen-consumption by the ventricles

c) A decrease in heart rate

d) A systolic murmur

e) A decreased end-systolic volume

50. Which of the following results in a decreased stroke volume?

a) An increase in end-diastolic volume

b) An increased afterload

c) Parasympathetic stimulation

d) A decrease in total peripheral resistance

e) Increased residual volume

51. An increase in pulse pressure can be caused by:

a) Parasympathetic stimulation

b) Generalized vasodilation

c) Severe hemorrhage

d) Aortic stenosis

e) Heart failure

52. On assuming the upright posture, one would expect:

a) Stimulation of Renin-angiotensin system

b) Increased pulmonary artery pressure

145 ‫صفحة‬ Dr. MOHAMED YAHIA 0900987639


QUESTION BANK OF Dr. MOHAMED YAHIA

c) A decrease in pulse rate

d) Venodilation

e) An increase in renal blood flow

53. Central venous pressure increases:

a) In hypovolaemia

b) By sympathetic stimulation

c) When total peripheral resistance increases

d) When the cardiac output decreases

e) With increased sodium loss

54. Infusion of nor-adrenaline is expected to produce:

a) A decrease in firing-rate of baroreceptors

b) A reflex brachycardia

c) A decrease in total peripheral resistance

d) Increased myocardial contractility due to B2 receptors

e) An increase in cardiac output

55. Stroke volume is decreased when:

a) The sympathetic nerves are stimulated

b) The arterial blood pressure falls

c) Vagal centers are stimulated

d) The end-diastolic volume is increased

e) A patient stands up

56. Generalized vasoconstriction may be produced by:

a) Exposure to heat

146 ‫صفحة‬ Dr. MOHAMED YAHIA 0900987639


QUESTION BANK OF Dr. MOHAMED YAHIA

b) Parasympathetic stimulation

c) An increase in vasomotor tone

d) When the depressor center is stimulated

e) An exposure to a large dose of histamine

57. Arterial vasoconstriction in a limb leads to:

a) An increase in capillary pressure

b) A decrease in arteriovenous oxygen difference

c) Diminished filteration in capillary bed

d) An increase in blood pH leaving the limb

e) Venoconstriction

58. Which of the following is the most important in determining the total
peripheral resistance?

a)Blood viscosity

b) Concentration of plasma protein

c) Arteriolar diameter

d) Cardiac Output

e) Metabolic autoregulation

59. Venodilation at rest leads to:

a) A decrease in venous return

b) A drop in arterial blood pressure

c) Flushing of the skin

d) A decrease in peripheral resistance

e) Increased capillary fluid exchange

147 ‫صفحة‬ Dr. MOHAMED YAHIA 0900987639


QUESTION BANK OF Dr. MOHAMED YAHIA

60. Stroke Volume:

a) Is normally about 90mL

b) Depends on Bainbridge reflex

c) Decreases when the heart rate increases

d) Increases when the end-diastolic volume increases

e) Is mainly controlled by vagal fibers

61. Starling's Law of the heart describes the relationship between:

a) The heart rate and stroke volume

b) The end-diastolic volume cardiac output

c) The blood pressure and heart rate

d) The initial length of cardiac muscle fibers and force of contraction

e) Sympathetic stimulation stroke volume

62. Increased pressure in the carotid sinus leads to:

a) An increase in vasomotor tone

b) Increase in vagal tone

c) Reflex vasoconstriction

d) Venoconstriction

e) Tachycardia

63. In the jugular pulse wave the wave "C":

a) Occurs before the "a" wave

b) Is due to atrial systole

c) Closure of the atrioventricular valve

d) Co-asides with the atrial pulse wave

e) Is due to bulging of tricuspid valves

148 ‫صفحة‬ Dr. MOHAMED YAHIA 0900987639


QUESTION BANK OF Dr. MOHAMED YAHIA

64. The first heart sound:

a) Is due to vibrations in the aorta on closure of the semilunar valves

b) Is caused by closure of tricuspid valve

c) Occurs at the end of isometric contraction of the ventricle

d) Is caused by closure of both atrioventricular valves

e) Is short of high pitch

65. The Cardiac output is:

a) The volume of blood pumped by the heart in one minute

b) Equal to the heart rate multiplied by the stroke volume

c) Measured by dilution method

d) Increased on standing up

e) Greater in left ventricle than the right

66. The chemoreceptors are found in:

a) The lungs

b) Glomus bodies

c) Aortic arch

d) Carotid sinus

e) Medulla

67. Local vasodilation can be produced by:

a) Neural reflexes

b) Myogenic autoregulation

c) An increase in PO2

d) A decrease in H+ concentration

e) Synthesis of ATP

149 ‫صفحة‬ Dr. MOHAMED YAHIA 0900987639


QUESTION BANK OF Dr. MOHAMED YAHIA

68. The following are expected to increase the cardiac output except:

a) Eating

b) Adrenaline Infusion

c) Moderate rise in environmental temperature

d) A 10 degree rise in body temperature

e) Pregnancy

69. Regarding the cardiac cycle which of the following is incorrect?

a) The first heart sound occurs at the start of isovolumetric contraction

b) The "v" wave of atrial pressure occurs during diastole

c) The third heart sound occurs during ventricular diastole

d) The second heart sound occurs in late systole

e) Ventriculor systole continues after the end of the ECG wave

70. The Frank-Starling law explains all the following except:

a) Increase in cardiac with increase in venous return

b) Maintenance of stroke volume in the phase of increased afterload

c) Matching left ventricular output with right ventricular output

d) The increased contractility induced by sympathetic stimulation

e) The normal cardiac output in hypertensive patients

71. A drug that increases the heart rate from 70 to 100 beats per minute could
be:

a) A B1-adrenergic receptor antagonist

b) A cholinergic receptor antagonist

150 ‫صفحة‬ Dr. MOHAMED YAHIA 0900987639


QUESTION BANK OF Dr. MOHAMED YAHIA

c) A cholinergic receptor agonist

d) A B2 adrenergic receptor agonist

e) A non-cholinergic, non-adrenergic agonist

72. Excitation of the ventricles:

a) Proceeds from the subendocardium to the subpericardium

b) Leads to excitation of the atria

c) Results from the action of norepinephrine on ventricular muscle

d) Occurs 2.2 seconds following atrial excitation

e) Results from pacemaker potentials in ventricular cells

73. AV nodal cells:

a) Exhibit action potentials characterized by rapid depolarization

b) Conduct impulses more slowly than either atrial or ventricular cells

c) Are capable of pacemaker activity at an intrinsic rate of 100 beat per minute

d) Exhibit increased permeability to Na+ during the prepotntial

e) Show a steep pre-potential when exposed to acetylcholine

74. Stroke Volume:

a) Increases as a result of increased afterload

b) Equals end-diastolic volume minus end-systolic volume

c) Increases as the heart rate is increased by electrical pacing

d) Is increased by parasympathetic stimulation

e) Is increased by Ca-channel blockers

75. Coronary blood flow of the left ventricle:

a) Is mainly regulated by sympathetic supply to coronary arterioles

151 ‫صفحة‬ Dr. MOHAMED YAHIA 0900987639


QUESTION BANK OF Dr. MOHAMED YAHIA

b) Increases when sympathetic nerves to the heart are blocked

c) Is highest during systole because of myocardial activity

d) Increases when myocardial metabolism increases

e) All of the above are correct

76. The strength of myocardial contraction:

a) Increases as a result of increased afterload

b) Equals end-diastolic volume minus end-systolic volume

c) Increases as heart rate increases by electrical pacing

d) Is increased by parasympathetic stimulation

e) Is increased by calcium channel blockers

77. Increased arterial blood pressure leads to:

a) Decreased firing of carotid sinus baroreceptors

b) Increased sympathetic activity to the ventricles

c) Increased parasympathetic activity to the sinoatrial node

d) Increased parasympathetic activity to arterioles of skeletal muscles and skin

e) Increased parasympathetic stimulation to the ventricles

78. Which of the following is likely to cause postural hypotension?

a) Drugs that block muscarinic cholinergic receptors

b) Decreased firing rate of baroreceptors while standing

c) Exposure to cold environment

d) Drugs that block cholinergic receptors in skeletal muscle vessels

e) Drugs that activates cholinergic receptors in autonomic ganglia

79. Stimulation of parasympathetic nerves to the heart:

152 ‫صفحة‬ Dr. MOHAMED YAHIA 0900987639


QUESTION BANK OF Dr. MOHAMED YAHIA

a) Causes tachycardia

b) Makes the prepotential more horizontal

c) Decreases the rate of potassium efflux in the sino-atrial node

d) Prolongs the refractory period

e) Shortens the duration of the cardiac cycle

80. The prepotential of the pacemaker is mainly due to:

a) Calcium influx in the early phase

b) Increased K+ efflux

c) Increased chloride influx

d) Decreased K+ efflux

e) Decreased Na+ influx

81. The vessels responsible for regulation of blood pressure:

a) Medium-size arteries

b) Small veins

c) Capillaries

d) Venules

e) Arterioles

82. The advantage of the Starling mechanism in the heart is to:

a) Decrease fluid loss from the cardiac capillaries

b) Ensure that the ventricles operate at an optimum length

c) Couple the efficiency of muscle contraction to the heart rate

d) Match the output of one ventricle to that of the other

e) Ensure that the right and left arterial pressure are equal

153 ‫صفحة‬ Dr. MOHAMED YAHIA 0900987639


QUESTION BANK OF Dr. MOHAMED YAHIA

83. An increase in arterial pulse pressure is observed in association with:

a) Exposure to cold

b) An increased peripheral resistance

c) Hypothyroidism

d) An increased vagal activity

e) Muscular exercise

84. A decrease in carotid sinus pressure would lead to a decrease in:

a) Heart rate

b) Myocardial contractility

c)Total peripheral resistance

d) Capacity of venous system

e) Cardiac output

85. In the ECG, the QRS wave is due to:

a) Atrial contraction

b) Atrial depolarization

c) Ventricular depolarization

d) SA node depolarization

e) Ventricular contraction

86. In the ECG, the P wave denotes:

a) Atrial contraction

b) Atrial depolarization

c) Ventricular repolarization

d) SA node depolarization

154 ‫صفحة‬ Dr. MOHAMED YAHIA 0900987639


QUESTION BANK OF Dr. MOHAMED YAHIA

e) Ventricular contraction

87. Stimulation of sympathetic nerves to the heart:

a) Causes tachycardia via alpha-adrenergic receptors

b) Makes the prepotential more horizontal

c) Decreases the rate of potassium efflux in the SA node

d) Prolongs the nodal delay

e) Has no effect on the duration of the cardiac cycle

88. Concerning the heart sounds:

a) The first heart sound is due to closure of the atrioventricular valves

b) The second heart sound is due to opening of the aortic and pulmonary

valves

c) The third heart sound is due to atrial systole

d) The first heart sound occurs at the beginning of the isovolumetric relaxation
phase

e) The second heart sound is followed by isovolumetric contraction phase

89. In the normal ECG the duration of PQ interval is usually:

a) 0.10 sec

b) 0.15 sec

c) 0.30 sec

d) 0.45 sec

e) 0.60 sec

90. Starling's Law of the heart:

a) States that at a given end-diastolic pressure, norepinephrine increases the

volume

155 ‫صفحة‬ Dr. MOHAMED YAHIA 0900987639


QUESTION BANK OF Dr. MOHAMED YAHIA

b) States that increased end-diastolic volume leads to an increased stroke

volume

c) Is primarily the result of changes in the firing rate of sympathetic nerves to

the ventricles

d) Is independent of the venous return

e) Describes the myocardial response to an increased heart rate

91. In the heart, transmission of impulses is fastest in:

a) Atrial muscles

b) Bundle of His

c) Ventricular muscles

d) Purkinje fibers

e) AV nodal fibers

92. The force of contraction of cardiac muscle is a function of:

a) Initial muscle length

b) Body temperature

c) Duration of the action potential

d) End-systolic volume

e) Number of impulses that reach the muscle cell per unit time

93. The second heart sound occurs:

a) During protodiastole

b) During isovolumetric relaxation

c) During isovolumetric contraction

d) 0.5 seconds after the first heart sound

e) At the peak of ejection period

156 ‫صفحة‬ Dr. MOHAMED YAHIA 0900987639


QUESTION BANK OF Dr. MOHAMED YAHIA

94. The P wave of the ECG:

a) Occurs during rapid atrial inflow

b) Follows the "a" wave of atrial pressure

c) Is essential for development of normal QRS complex

d) Occurs during ventricular diastole

e) Is prominent in atrial fibrillaton

95. Which is true of cardiac muscle:

a) All cardiac muscle cells have pacemaker potentials

b) The rate of cardiac muscle contraction is always set by nerves to the heart

c) Impulses spread easily from one muscle cell to another

d) Cardiac muscles look smooth under the microscope

e) Cardiac muscles have relatively short refractory periods compared to

skeletal muscles

96. Stimulation of the right vagus nerve:

a) Increases the heart rate

b) Increases arterial blood pressure

c) Has a reduced effect after administration of physostigmine

d) Has a reduced effect after administration of atropine

e) Has a reduced effect after administration of prornaolol

97. Which of the laws explain the relationship between vessel wall tension and
vessel radius:

a) Poiseuille's Law

b) Ohm's Law

157 ‫صفحة‬ Dr. MOHAMED YAHIA 0900987639


QUESTION BANK OF Dr. MOHAMED YAHIA

c) Starling's Law

d) LaPlace's Law

e) Reynold's Law

98. Which of the following are not innervated by sympathetic nerves?

a) Large arteries

b) Arterioles

c) Venules

d) Capillaries

e) Large veins

99. Which of the following substances will be most likely to dilate systemic
arterioles:

a) Endothelin

b) ADH

c) Histamine

d) Noreadrenaline

e) Aldosterone

100. Resistance to venous return:

a) Is higher during exercise than at rest

b) Equals to the slope of the venous return curve

c) Is reduced by sympathetic stimulation

d) Increases by increase in circulatory filling pressure

e) Increased by constriction of arterioles

101. Closure of the aortic valve occurs at:

158 ‫صفحة‬ Dr. MOHAMED YAHIA 0900987639


QUESTION BANK OF Dr. MOHAMED YAHIA

a) The end of isovolumetric contraction

b) The beginning of isometric relaxation

c) The end of systole

d) The end of rapid filling phase

e) The beginning of rapid ejection phase

102. An increase in the pulse pressure can be caused by:

a) Parasympathetic stimulation

b) Generalized vasodilation

c) Sympathetic stimulation

d) Venodilation

e) Heart failure

103. The contribution of atrial contraction to ventricular filling is greatest:

a) When sympathetic stimulation is pronounced

b) At rapid heart rates

c) When atria and ventricles contract simultaneously

d) During atrial fibrillation

e) During bradycardia

104. The baroreceptor reflex mainly regulates:

a) Stroke volume

b) Heart rate

c) Mean arterial blood pressure

d) Systolic blood pressure

e) Diastolic blood pressure

159 ‫صفحة‬ Dr. MOHAMED YAHIA 0900987639


QUESTION BANK OF Dr. MOHAMED YAHIA

105. Which of the following is a characteristic of cardiac muscle?

a) It has a long refractory period compared to skeletal muscle

b) There are electrical synapses

c) Gap junctions cause limitation of depolarization

d) Has myosin but no actin molecules

e) There are para cellular spaces

106. Factors affecting the strength of ventricular muscle include:

a) Parasympathetic inhibition

b) The length of muscle fibers

c) The length of plateau phase

d) Sympathetic stimulation

e) End-systolic volume

107. Stimulation of parasympathetic supply to the heart results in:

a) Decrease K+ efflux in cardiac muscle

b) Increases Ca2+ influx in SA node

c) Increased delay in AV node

d) An increase in Cl- influx

e) Opening of sodium channel

108. The following increase the cardiac output except:

a) Sympathetic stimulation

b) Venodilation

c) Fever

d) Anemia

160 ‫صفحة‬ Dr. MOHAMED YAHIA 0900987639


QUESTION BANK OF Dr. MOHAMED YAHIA

e) Exercise

109. The cardiac output is increased:

a) By stimulation of sympathetic beta-1 receptors

b) Increased impedance in the aortic arch

c) By high pressure in the arterial system

d) By impulses through the glosso-pharygeal nerve (ix)

e) In hypothyroidisim

110. Ventricular Depolarization:

a) Follows immediately after the closure of atrioventricular valves

b) Occurs during mid systole

c) Depends on sympathetic stimulation

d) Occurs 0.2 sec following atrial excitation

e) Results from pacemaker cells in ventricular cells

111. The isovolumetric relaxation phase of the ventricles:

a) Is associated with rapid filling

b) Occurs during late diastole

c) Ends by opening of the atrioventricular valves

d) Results in falling of ventricular pressure by about 10 mmHg

e) Is marked by the first heart sound

112. Factors that increase the stroke volume include:

a) An increased afterload

b) A high end-diastolic volume

c) An increased heart rate

161 ‫صفحة‬ Dr. MOHAMED YAHIA 0900987639


QUESTION BANK OF Dr. MOHAMED YAHIA

d) Parasympathetic stimulation

e) Ca2+ channel blockers

113. After a loss of one liter of blood:

a) The heart rate decreases slightly

b) Baroreceptors increase their discharge

c) The chemoreceptors have no role

d) The end diastolic volume increases

e) There is increased vasomotor tone

114. A large infarct affecting the left ventricle causes:

a) Pulmonary capillary hydrostatic pressure to decrease

b) Left ventricular end systolic pressure to increase

c) Heart rate to decrease

d) The systolic blood pressure to decrease

e) Congestion of the liver

115. The blood vessels responsible for exchange between plasma & interstitial
fluid are:

a) Distributing arteries

b) Large veins

c) The capillaries

d) The venules

e) Arterioles

116. Conduction velocity of cardiac impulse is slowest in:

162 ‫صفحة‬ Dr. MOHAMED YAHIA 0900987639


QUESTION BANK OF Dr. MOHAMED YAHIA

a) Atrial myocardial fibers

b) AV nodal fibers

c) Purkinje fibers

d) Ventricular myocardial fibers

e) His bundle fibers

117. The contribution of atrial contraction to ventricular filling is greatest:

a) When vagal activity is pronounced

b) At rapid heart rates

c) When atria and ventricles contract simultaneously

d) During atrial fibrillation

e) Early during ventricular diastole

118. The most important function of the Starling mechanism in the heart is:

a) To decrease fluid loss from the cardiac capillaries

b) To ensure that the ventricles operate at an optimum length

c) To couple the efficiency of muscle contraction to the heart rate

d) To match the output of one ventricle to that of the other

e) To ensure that right and left atrial pressures are equal

119. The highest coronary blood flow occurs:

a) Early during systole

b) When the left ventricular pressure is high

c) At the beginning of isovolumetric contraction

d) Towards the end of ventricular diastole

e) At the beginning of diastole

163 ‫صفحة‬ Dr. MOHAMED YAHIA 0900987639


QUESTION BANK OF Dr. MOHAMED YAHIA

120. Which of the following results in an increased stroke volume?

a) An increase in end-diastolic volume

b) An increased afterload

c) Parasympathetic stimulation

d) An increase in the heart rate

e) Venodilation

RESPIRATORY SYSTEM
1. The physiological dead space:

a) Is exactly equal to the anatomic dead space in normal people

b) Increases during exercise

c) Is measured by single breath nitrogen analysis

d) Is higher in elderly than young adults

e) Is the same as the physiological shunt

2. A high alveolo-arterial PO2 difference most likely results from:

a) Low fractional concentration of O2 in the inspired air

b) Hypoventilation

c) Arteriovenous shunt

d) Pulmonary edema

e) Polycythemia

3. At 33 feet under water the ambient pressure is:

a) Two atmospheres

b) One and a half atmosphere

c) 33 pounds per square inch

d) Same as the pressure in a tunnel 33 feet below sea level

e) Half the pressure 33 feet above sea level

164 ‫صفحة‬ Dr. MOHAMED YAHIA 0900987639


QUESTION BANK OF Dr. MOHAMED YAHIA

4. Which of the following combinations are typical findings in compensated


respiratory alkalosis:

a) Low PaCO2, low bicarbonate level and normal pH

b) Low PaCO2, normal bicarbonate level and high pH

c) Low PaCO2, low bicarbonate level and high pH

d) Low PaCO2, normal bicarbonate level and normal pH

e) Low PaCO2, high bicarbonate level and high pH

5. The partial pressure of oxygen in the alveoli is expected to be least affected by:

a) Fractional concentration of O2 in ispired air

b) Respiratory quotient

c) PCO2 in alveoli

d) Hemoglobin concentration

e) Barometric pressure

6. A veno-arterial shunt is likely to:

a) Decrease the cardiac output

b) Increase the pulse pressure

c) Increase venous return

d) Decrease PaO2

e) Increase the peripheral vascular resistance

7. The arterio-venous difference of O2 is highest across:

d)Liver

b) Kidney

c) Cardiac muscle

165 ‫صفحة‬ Dr. MOHAMED YAHIA 0900987639


QUESTION BANK OF Dr. MOHAMED YAHIA

d) Skin

e) Brain

8. In a patient, hypoventilation is most likely if:

a) PaO2 is low

b) pH of arterial blood is low

c) PaCO2 is high

d) O2 in arterial blood is low

e) PCO2 of expired air is high

9. A decrease in the recoil force of the lung is likely to:

a) Decrease total lung capacity

b) Increase vital capacity

c) Increase functional residual capacity

d) Decrease the residual volume

e) Decrease compliance of the lung

10. Central chemoreceptors differ from chemoreceptors in that they:

a) Respond to changes in PaCO2

b) Respond to changes in pH

c) Do not respond to changes in PO2

d) Become more sensitive to CO2 with chronic exposure

e) They are less sensitive to changes in PaCO2 than pH

11. Surfactant:

a) Helps to equalize pressure within interconnected alveoli

166 ‫صفحة‬ Dr. MOHAMED YAHIA 0900987639


QUESTION BANK OF Dr. MOHAMED YAHIA

b) Reduces surface tension more effectively as alveolus

becomes smaller

c)Increases lung compliance

d) Increases the slope of the pressure-volume curve

e) All of the above

12. One month of an exposure to an inspired CO2 of 5% would result in:

a) Increased ventilatory sensitivity to CO2

b) Reduced ventilatory sensitivity to hypoxia

c) Increased plasma bicarbonate

d) Alkalosis in arterial plasma

e) Increased alveolar PO2

13. In a normal adult man, breathing 100% oxygen:

a) Is likely to double O2 in arterial blood

b) Is likely to reduce alveolar PCO2

c) Is likely to raise PO2 in venous blood to about 200 mmHg

d) Is likely to raise arterial PO2 to about 660 mmHg

e) Increases the affinity of hemoglobin to oxygen

14. At the end of maximum inspiration:

a) Intra-alveolar pressure is zero

b) Intra-pleural pressure is zero

c) Recoil force of the chest is greater than the recoil force of the lung

d) Surface tension is lowest

e) Average alveolar PO2 is lower than at the end of maximum expiration

167 ‫صفحة‬ Dr. MOHAMED YAHIA 0900987639


QUESTION BANK OF Dr. MOHAMED YAHIA

15. On ascent to a high altitude:

a) The concentration of oxygen in the air decreases

b) The number of oxygen molecules per liter of air decreases

c) Alveolar PCO2 is kept constant despite the high ventilation

d) Density of air increases

e) The PCO2 in air is the same as at sea level

16. Compliance of the lung:

a) Is higher in small children than in adults

b) Is about 500 ml/cm H2O

c) Is normally twice the compliance of the chest

d) Is higher in elderly than young adults

e) Is decreased in emphysematous changes

17. Dennervation of carotid and aortic bodies leads to all the following except:

a) Decreased sensitivity of ventilation to change in PCO2

b) Decreased sensitivity of ventilation to change in pH

c) Complete absence of response of ventilation to change in PO2

d) Absence response of ventilation to exercise

e) Decreased sensitivity of ventilaton to stagnant hypoxia

18. Exchange of which of the following gases across the respiratory membrane is
normally diffusion limited:

a) Oxygen

168 ‫صفحة‬ Dr. MOHAMED YAHIA 0900987639


QUESTION BANK OF Dr. MOHAMED YAHIA

b) Carbon dioxide

c) Carbon monoxide

d) Nitrous oxide

e) Nitrogen

19. Minute ventilation is:

a) The volume of air moved in or out of the alveoli per minute

b) The volume of air moved in or out of the lung per breath

c) Alveolar ventilation plus dead space ventilation

d) Invariably increases if the respiratory rate increases

e) Is measured using Bohr's equation (CO2 analysis in expired air)

20. The ventilatory response to hypoxia:

a) Is independent of arterial PCO2

b) Is mediated by the central chemoreceptors

c) Decreases gradually with time

d) Is largely mediated by the aortic bodies in humans

e) None of the above

21. With respect to oxygen and carbon dioxide transport in the blood:

a) For the same partial pressure a unit of blood carries the same amount of O2

and CO2

b) High PCO2 favors O2 binding to hemoglobin

c) Metabolic acidosis reduces PCO2

d) Oxygenation of hemoglobin increases the affinity of hemoglobin to CO2

e) Per unit volume of arterial blood there is more O2 than CO2

169 ‫صفحة‬ Dr. MOHAMED YAHIA 0900987639


QUESTION BANK OF Dr. MOHAMED YAHIA

22. PCO2 in arterial blood:

a) Invariably increases with hypoxia

b) Stimulates ventilation mainly through peripheral

chemoreceptors

c) Is the major controller of ventilation

d) Increases early in exercise

e) Increases on ascent to high altitude

23. The functional residual capacity:

a) Is the volume of air in the lung at the end of normal inspiration

b) Increases with age

c) Is measured by simple spirometry

d) Is about 3 liters in an average adult male

e) Is about 1 liter in average adult male

24. At the end of inspiration at sea level the PO2 in the anatomic dead space is
approximately:

a) 150 mmHg

b) 100 mmHg

c) 160 mmHg

d) 95 mmHg

e) Above a hundred but less than 150 mmHg

25. Which is true concerning control of ventilation:

a) Ventilation is more sensitive to changes in PO2 than changes in PCO2 in

arterial blood

b) Carbon dioxide stimulates ventilation only through the central

170 ‫صفحة‬ Dr. MOHAMED YAHIA 0900987639


QUESTION BANK OF Dr. MOHAMED YAHIA

chemoreceptors

c) At high altitude the primary stimulus to ventilation is high PCO2

d) The response of ventilation to a sustained rise in PCO2 increases with time

e) Cutting the vagus nerves decreases the depth of breathing

26. Which of the following is likely to decrease airway resistance?

a) Beta-adrenergic blockers

b) Muscarinic cholinergic agonists

c) Breathing at higher lung volumes

d) Leukotrienes

e) Histamine

27. Compared to normal arterial blood, normal mixed venous blood has:

a) Higher PO2, lower PCO2 and higher pH

b) Lower PO2, higher PCO2 and higher pH

c) Lower PO2, higher PCO2 and lower pH

d) Lower PO2, lower PCO2 and higher pH

e) Higher PO2, lower PCO2 and lower pH

28. Deficiency of surfactant is likely to result in:

a) Increased compliance

b) Decreased work of breathing

c) Reduced surface tension of fluid in the alveoli

d) Decreased compliance

e) Asthma

171 ‫صفحة‬ Dr. MOHAMED YAHIA 0900987639


QUESTION BANK OF Dr. MOHAMED YAHIA

29. For a normal Hb-O2 dissociation curve, the most correct relationship is:

a) PO2 is 40 mmHg, percent saturation 50

b) PO2 is 95 mmHg, percent saturation 97

c) PO2 is 30 mmHg, percent saturation 50

d) PO2 is 60 mmHg, percent saturation 60

e) PO2 is 50 mmHg, percent saturation 50

30. In a normal subject, blood at the end of the pulmonary capillary:

a) Has a PO2 similar to blood in the left ventricle

b) Has a PO2 similar to that in the left atrium

c) Has a PO2 higher than blood from the aorta

d) Has a percent saturation of 95

e) Contains 20 mL of CO2 per 100 mL

31. PCO2 in arterial blood:

a) Invariably increases with hypoxia

b) Stimulates ventilation mainly through peripheral

chemoreceptors

c) Is lower than normal in compensated metabolic acidosis

d) Increases early in exercise

e) Increases on ascent to high altitude

32. In which of the following conditions is the percent saturation of hemoglobin


in arterial blood likely to be normal :

a) Venoarterial shunts

b) Pulmonary edema

c) Carbon monoxide poisoning

172 ‫صفحة‬ Dr. MOHAMED YAHIA 0900987639


QUESTION BANK OF Dr. MOHAMED YAHIA

d) Anemia

e) Methemoglobinemia

33. At which of the following points is the intrapleural pressure closest to zero:

a) End of normal expiration

b) End of normal inspiration

c) End of maximum expiration

d) End of maximum inspiration

e) Mid-normal expiration

34. Regarding the vital capacity, all of the following are correct except:

a) It is higher in young adults than small children

b) It is higher in males than females

c) It is normal in pure restrictive lung disease

d) It can be measured by simple spirometry

e) It is related to body size

35. Immediate acclimatization to high altitude is by:

a) Polycythemia

b) Hyperventilation

c) Increased cardiac output

d) Shift of hemoglobin dissociation curve to the left

e) Increase formation of fetal hemoglobin

36. At the end of maximal expiration the volume of air in the lungs is:

a) FRC - RV

173 ‫صفحة‬ Dr. MOHAMED YAHIA 0900987639


QUESTION BANK OF Dr. MOHAMED YAHIA

b) IRV + RV

c) FRC - TV

d) TLC - VC

e) VC – ERV - IRV

37. At the end of expiration at sea level the PO2 in the anatomic dead space is
approximately:

a) 150 mmHg

b) 100 mmHg

c) 160 mmHg

d) 40 mmHg

e) 130 mmHg

38. Which is true regarding lung and chest wall mechanics:

a) At maximum expiration the lungs have no tendency to collapse but the

chest has a great tendency to expand

b) At end of normal inspiration the recoil tendency of the lung

is greater than that of the chest

c) During inspiration the recoil tendencies of both the lungs and chest

increases

d) At the end of normal expiration the recoil tendency of the chest is slightly

greater than that of the lung

e) The combined compliance is greater than the individual compliances of the

lung and chest

39. Which is true concerning the diffusing capacity of the lungs:

a) It increases whenever the partial pressure gradient increases

174 ‫صفحة‬ Dr. MOHAMED YAHIA 0900987639


QUESTION BANK OF Dr. MOHAMED YAHIA

b) It increases as the alveolar capillary surface area rises

c) It increases as the thickness of the respiratory membrane area rises

d) It is usually measured using carbon dioxide

e) It is usually measured using oxygen

40. A person whose anatomic dead space is 100 mL breathes 12 times per minute
with a tidal volume of 400 mL. His pulmonary ventilation is:

a) 1.2 liters

b) 2.4 liters

c) 3.6 liters

d) 4.8 liters

e) 6.0 liters

41. While repaying the O2 dept:

a) The [H+] is increasing

b) PCO2 is decreasing

c) Muscle blood flow is increasing

d) Respiratory rate is increasing

e) Minute volume is decreasing

42. An average adult male has a vital capacity of:

a) 3 liters

b) 4 liters

175 ‫صفحة‬ Dr. MOHAMED YAHIA 0900987639


QUESTION BANK OF Dr. MOHAMED YAHIA

c) 5 liters

d) 6 liters

e) 7 liters

43. The residual volume can be calculated by subtracting the expiratory reserve
volume from:

a) Vital Capacity

b) Inspiratory capacity

c) Functional residual capacity

d) Total lung capacity

e) Alveolar ventilation

44. The volume of air that describes the lung's ability to eliminate carbon dioxide
is:

a) Tidal volume

b) Pulmonary ventilation

c) Vital capacity

d) Alveolar ventilation

e) Functional residual capacity

45. A decrease in the recoil force of the lung is expected to:

a) Increase vital capacity

b) Decrease residual volume

c) Decrease compliance of the lung

d) Increase functional residual capacity

e) Decrease total lung capacity

46. Deficiency of surfactant may lead to:

176 ‫صفحة‬ Dr. MOHAMED YAHIA 0900987639


QUESTION BANK OF Dr. MOHAMED YAHIA

a) Increased work of breathing

b) Increased compliance of the lung

c) Decreased recoil force of the lung

d) Bronchial asthma

e) An increase in functional residual capacity

47. Continuous measurement of the concentration of nitrogen in the expired air


can be used to measure:

a) Physiological dead space

b) Alveolar ventilation

c) Anatomic dead space

d) Functional residual capacity

e) Expiratory reserve volume

48. Oxygen consumption of an average adult man per minute is about:

a) 250 ml

b) 350 ml

c) 400 ml

d) 500 ml

e) 550 ml

49. Which of the following is expected to increase function residual capacity:

a) Surfactant deficiency

b) Lung fibrosis

c) Degeneration of lung elastic tissue

d) Pulmonary edema

177 ‫صفحة‬ Dr. MOHAMED YAHIA 0900987639


QUESTION BANK OF Dr. MOHAMED YAHIA

e) Pulmonary congestion

50. The most likely cause of hypoxic hypoxia together with hypercapnia is:

a) Ascent to a high altitude

b) Mild pulmonary edema

c) Veno-arterial shunt

d) Hypoventilation

e) Slight ventilation perfusion mismatch

51. Which of the following is correct regarding blood in the pulmonary trunk:

a) PO2 of 60 mmHg

b) 50% saturation of hemoglobin

c) PCO2 of 40 mmHg

d) Bicarbonate concentration lower than in the aorta

e) PN2 higher than in aorta

52. Ventilation is most sensitive to changes in:

a) PO2

b) pH of arterial blood

c) PCO2

d) Body temperature

e) % saturation of hemoglobin

53. Inspiration is initiated by discharge from:

a) The dorsal inspiratory group of neurons

b) The ventral group of neurons

c) The preBottzinger complex

178 ‫صفحة‬ Dr. MOHAMED YAHIA 0900987639


QUESTION BANK OF Dr. MOHAMED YAHIA

d) Apneustic center

e) Pneumotaxic center

54. Voluntary hyperventilation by a normal person is expected to lead to all the


following except:

a) Alkalosis

b) Hypocapnia

c) Low blood bicarbonate concentration in arterial blood

d) A great increase in oxygen concentration in arterial blood

e) Low ionized plasma calcium

55. The normal diffusing capacity of the respiratory membrane of an average


adult male for oxygen is about:

a) 10 ml

b) 25 ml

c) 50 ml

d) 4 ml

e) 15 ml

56. Cyanosis is unlikely to occur in:

a) Pulmonary edema

b) Hypoventilation

c) Severe anemia

d) Radiation

e) Convection

179 ‫صفحة‬ Dr. MOHAMED YAHIA 0900987639


QUESTION BANK OF Dr. MOHAMED YAHIA

57. Which is true concerning the diffusing capacity of the lungs:

a) Rises as the partial pressure gradient rises

b) Rises as the effective surface area of the respiratory membrane rises

c) Is not affected by the thickness of the respiratory membrane

d) Is measured using radioactive CO2

e) Is not affected by the diffusion coefficient of the membrane to gases

58. Which type of hypoxia in which there is a greater than normal atrial-mixed
venous partial pressure difference and relatively normal arterial oxygen
concentration:

a) Hypoxic hypoxia

b) Anemic hypoxia

c) Stagnant hypoxia

d) Histotoxic hypoxia

e) B and C are correct

59. Administration of a gas mixture with higher than normal oxygen


concentration at atmospheric pressure would elevate hypoxia due to:

a) Veno-arterial shunt

b) Diffusion impairment

c) Anemia

d) Cyanide poisoning

e) Reduced blood flow to tissues

60. The PCO2 is an alveolus which is ventilated but not perfused is about:

a) 0.3 mmHg

b) 46 mmHg

c) 40 mmHg

180 ‫صفحة‬ Dr. MOHAMED YAHIA 0900987639


QUESTION BANK OF Dr. MOHAMED YAHIA

d) 5 mmHg

e) 7 mmHg

61. Which is true concerning surface tension in lungs?

a) The normal amount of surfactant reduces surface tension in healthy lungs to

a neglible level

b) In the absence of surfavtant alveoli with small radii collapse into alveoli

with big radii

c) Surfactant is maximally concentrated at liquid-air interface at the end of

inspiration

d) In normal lungs surface tension is responsible for 20% of the recoil force of

the lung

e) Surface tension remains the same throughout the breathing cycle

62. Which is true concerning oxygen transport in the blood?

a) If PO2 is 60 mmHg oxygen concentration is about 18 ml/100 ml blood

b) The P50 is the percent saturation of hemoglobin with oxygen when PO2 is

50 mmHg

c) The normal P50 of arterial blood is higher than that of venous blood

d) Increase in pH increases the P50

e) P50 reflects the capacity of blood to carry oxygen

63. The physiological shunt:

181 ‫صفحة‬ Dr. MOHAMED YAHIA 0900987639


QUESTION BANK OF Dr. MOHAMED YAHIA

a) Represents the wasted perfusion

b) Is equal to the physiological dead space

c) Is measured by single breath nitrogen analysis

d) Occurs only in people with respiratory diseases

e) Is about 5% of the cardiac output

64. At the summit of a mountain the barometric pressure was found to be

300 mmHg. The partial pressure of O2 in the air is saturated with water vapour
at 37oC at this altitude is expected to be about:

a) 45 mmHg

b) 55 mmHg

c) 60 mmHg

d) 65 mmHg

e) 62 mmHg

65. Which of the following parameters is lower in the elderly than in young
adults?

a) Residual volume

b) Vital capacity

c) Lung compliance

d) Functional residual capacity

e) Physiological dead space

66. Surfactant:

a) Is produced by type I pneumocytes

b) Deficiency decreases lung compliance

c) Deficiency decreases work of breathing

d) Production starts only after birth in full term babies

182 ‫صفحة‬ Dr. MOHAMED YAHIA 0900987639


QUESTION BANK OF Dr. MOHAMED YAHIA

e) Production is inhibited by glucocorticoids

67. At the end of normal exporation all the following are correct except:

a) Recoil tendency of the lung as acting inward to collapse the lung

b) Chest recoil is acting outward to expand the lung

c) Alveolar pressure is greater than mouth pressure

d) The average intrapleural pressure is zero

e) Surface tension is less than at the end of normal inspiration

68. Obstructive lung disease is typically characterized by:

a) Reduced total lung capacity

b) Reduced FEV1

c) Reduced functional residual capacity

d) Slightly increased FEV1/FVC ratio

e) Reduced residual volume

69. The following parameters are higher in the elderly than young adults:

a) Vital capacity

b) Basal oxygen consumption per kg body weight

c) Functional residual capacity

d) Anatomical dead space

e) Peak expiratory flow

70. The approximate dead space of a normal adult man breathing through a tube
that is 50 cm long and 6mm in diameter is:

a) 200 ml

b) 300 ml

183 ‫صفحة‬ Dr. MOHAMED YAHIA 0900987639


QUESTION BANK OF Dr. MOHAMED YAHIA

c) 150 ml

d) 250 ml

e) 350 ml

71. Spontaneous respiration ceases after:

a) Transection of the spinal cord below C5

b) Transection of the brain stem between the pons and medulla combined with

bilateral vagotomy

c) Transection of the spinal cord

d) Complete destruction of the peripheral chemoreceptors

e) Transection of the spinal cord at the first cervical segment

72. All the following are expected to decrease airway resistance except:

a) Atropine

b) Adrenaline

c) Parasympathetic stimulation

d) Sympathetic stimulation

e) Shifting from nose to mouth breathing

GASTROINTESTINAL SYSTEM

1. The physiological actions of CCK include:

a) Increased oesophageal motility

b) Closing the sphincter of oddi

c) Increased bile synthesis

d) Contraction of pancreatic acini

e) Increased enzyme secretion from the pancreas

184 ‫صفحة‬ Dr. MOHAMED YAHIA 0900987639


QUESTION BANK OF Dr. MOHAMED YAHIA

2. The component parts of the myenteric plexus of the enteric nervous system
include the following except:

a) Sensory neurons

b) Interneurons

c) Motor neurons

d) Synapses

e) Control centers in the medulla oblongata

3. Saliva does not contain:

a) Blood group antigens

b) Lysozymes

c) Immunoglobulins

d) Platelets

e) Chloride

4. The functions of saliva include the following except:

a) Cleaning the mouth

b) Killing bacteria

c) Digestion of proteins

d) Facilitation of speech

e) Digestion of lipids

5. The pharyngeal phase of swallowing:

a) Is partially voluntary

b) Includes opening of the upper oesophageal sphincter

c) Includes the peristaltic activity of the upper third of the oesophagus

185 ‫صفحة‬ Dr. MOHAMED YAHIA 0900987639


QUESTION BANK OF Dr. MOHAMED YAHIA

d) Is associated with the opening of the glottis

e) Is preceded by deep inspiration

6. The enzymes produced by the exocrine pancreas include the following except:

a) Proelastase

b) DNase

c) Amylase

d) Enteropeptidase

e) Lipase

7. The secretion of the exocrine pancreas is increased by the following except:

a) Vagal stimulation

b) Acetylcholine

c) Secretin

d) CCK

e) Atropine

8. Bile production by the liver cells is increased by:

a) Sympathetic activation

b) Secretin

c) CCK

d) Bile salts

e) Atropine

9. The jejunum is the main site for absorption of the following except:

a) Glucose

b) Amino acids

c) Fatty acids

186 ‫صفحة‬ Dr. MOHAMED YAHIA 0900987639


QUESTION BANK OF Dr. MOHAMED YAHIA

d) Bile salts

e) Water

10. The functions of the colon include the following except:

a) Absorption of water

b) Absorption of sodium

c) Absorption of chloride

d) Absorption of calcium

e) Absorption of potassium

11. The intestinal flora is:

a) Mainly viruses

b) Mainly found in the duodenum

c) Needed for normal digestion

d) Mainly found in the colon

e) Mainly pathogenic bacteria

12. The following is true about the small intestine:

a) Stores food for several hours

b) Is the main source of digestive enzymes

c) Has haustrae

d) Has segmentation contractions

e) Secretes the intrinsic factors

13. The defaecation reflex:

a) Is initiated by stretching the anal canal

b) Is integrated in a control center in the lumbar spinal cord

187 ‫صفحة‬ Dr. MOHAMED YAHIA 0900987639


QUESTION BANK OF Dr. MOHAMED YAHIA

c) Is integrated in a control center in the sacral spinal cord

d) Is controlled by the vagus nerve

e) Involves contraction of the sigmoid colon

14. A tumor producing large amounts of gastrin DOES NOT include:

a) Hyperplasia of gastric epithelium

b) Hyperplasia of gastric mucosal lymphocytes

c) Excessive gastric acid secretion

d) Histamine secretion in the gastric mucosa

e) Recurrent peptic ulcers

15. Which of the following is likely to fail to reduce secretion of hydrochloric acid
in the stomach:

a) A H2 receptor blocker

b) A nicotinic cholinergic receptor blocker

c) A muscarinic cholinergic receptor blocker

d) An inhibitor of the hydrogen-potassium pump

e) A gastrin receptor blocker

16. A tumor producing large amounts of the intestinal hormone GIP may result
in:

a) Hyperglycemia

b) Hypoglycemia

c) Diarrhea

d) Intestinal colic

188 ‫صفحة‬ Dr. MOHAMED YAHIA 0900987639


QUESTION BANK OF Dr. MOHAMED YAHIA

e) Hyperacidity in the stomach

17. The following statements are true about bile except:

a) 25% is produced by bile duct cells

b) The bile stored in the gall bladder contains secondary bile acids

c) Its secretion is increased by secretin

d) Bile pigments are needed for micelle formation

e) Some lipids will be digested and absorbed in the absence of bile

18. A plain X-Ray film of the abdomen of a normal healthy man is expected to
show some gas in all the following except:

a) Stomach

b) Common bile duct

c) Small intestine

d) Transverse colon

e) Rectum

19. The component parts of the myenteric plexus of the enteric nervous system
includes the following except:

a) Sensory neurons

b) Interneurons

c) Motor neurons

d) Synapses

e) Control centers in the medulla oblongata

20. The secretion of the exocrine pancreas is increased by the following except:

a) Vagal stimulation

b) Acetylcholine

189 ‫صفحة‬ Dr. MOHAMED YAHIA 0900987639


QUESTION BANK OF Dr. MOHAMED YAHIA

c) Secretin

d) CCK

e) Noradrenaline

21. The jejunum is the main site for absorption of except:

a) Glucose

b) Amino acids

c) Fatty acids

d) Vitamin B12

e) Water

22. Functions of the human stomach include:

a) Absorption of iron

b) Absorption of Vitamin B12

c) Absorption of 50% of water intake

d) Storage of food for 24 hours

e) Limited digestion of proteins

23. The presence of bilirubin in bile is important because:

a) It gives bile its color

b) It leads to production of stercobilinogen in the intestine

c) It facilitates absorption of lipids

d) It gives faces their color

e) It means a waste product is being excreted

24. Relating to carbohydrate digestion:

a) Cellulose can be digested by human amylases

190 ‫صفحة‬ Dr. MOHAMED YAHIA 0900987639


QUESTION BANK OF Dr. MOHAMED YAHIA

b) About one-third of starch digestion occurs in the mouth and two-thirds

occurs in the small gut

c) It can only occur in the complete absence of protein digestion

d) It occurs only if bile salts are present

e) Sucrose is split by amylase action

25. Micelles:

a) Are emulsified fat globules in the duodenum

b) Are packets of pro-enzymes found in pancreatic acinar cells

c) Are dead enterocytes leaving the tips of villi

d) Are "flickering clusters" of cholesterol and fatty acids coated with bile salts

and 2 mono-glycerides

e) Are the triglyceride particles formed by enterocytes and transported in the

lymphatic system

26. The neurons of the intrinsic enteric nervous system do NOT perform the
following functions:

a) Detect distension of the stomach

b) Control intestinal mucosal cell secretions

c) Coordinate muscle contraction in the large intestine

d) Control lower oesophageal sphincter

e) Contract the gall bladder

27. The part of the gastrointestinal tract that is NOT affected by disorders of the
myenteric plexus of the intrinsic nervous system:

a) Anal canal

b) Rectum

191 ‫صفحة‬ Dr. MOHAMED YAHIA 0900987639


QUESTION BANK OF Dr. MOHAMED YAHIA

c) Caecum

d) Oesophagus

e) Pharynx

28. A patient who undergoes the following operation will NOT survive without
parenteral feeding:

a) Total oesophagectomy

b) Total gastrectomy

c) Right hepatic lobectomy

d) Total oroclocolectomy

e) Subtotal small intestinal resection

29. After a cholecystoctomy, patients may have problems with fat-rich foods
becuase:

a) The liver stops producing enough bile

b) Bile pigments are reduced in amount

c) The enterohepatic circulation has stopped

d) Bile is not well concentrated

e) The sphincter of Oddi fails to open in response to a fatty meal

30. Diseases that reduce pancreatic enzyme secretions do NOT usually decrease
the digestion and absorption of:

a) Animal proteins

b) Plant proteins

192 ‫صفحة‬ Dr. MOHAMED YAHIA 0900987639


QUESTION BANK OF Dr. MOHAMED YAHIA

c) Plant lipids

d) Sucrose

e) Starch

31. The colonic mucosal cells are very efficient in:

a) Converting primary bile acids into secondary bile acids

b) Converting bilirubin into stercobilinogen

c) Absorption of aromatic amino acids

d) Absorption of sodium

e) Absorption of phosphates

32. Contraction of the gallbladder may be due to:

a) Secretin

b) VIP

c) Bile salts

d) Atropine

e) Paraympathetic stimulation

33. Saliva does not contain:

a) Blood group antigens

b) Lysozymes

c) Immunoglobulins

d) Pepsin

e) Chloride

34. The actions of secretin include:

a) Increased salivary secretion

193 ‫صفحة‬ Dr. MOHAMED YAHIA 0900987639


QUESTION BANK OF Dr. MOHAMED YAHIA

b) Relaxation of the oesophageal sphincters

c) Stimulation of gastric secretion

d) Stimulation of secretion of bicarbonate by pancreatic duct cells

e) Stimulation of secretion of bicarbonate by pancreatic acinar cells

35. Saliva is needed for:

a) Digestion of sucrose

b) Digestion of phospholipids

c) Ability to speak

d) Breaking food down into small pieces

e) Absorption of chloride

36. The pharyngeal phase of swallowing includes:

a) A deep breath

b) Elevation of the tongue against the hard palate

c) Contraction of the upper oesophageal sphincter

d) Contraction of the pharyngeal constrictors

e) Elevation of the epiglottis

37. If the stomach is removed (gastrectomy), the patient is unlikely to suffer


from:

a) Iron deficiency anemia

b) Ca2+ deficiency

c) Macrocytic anemia

d) Malnutrition

e) Inability to absorb water

194 ‫صفحة‬ Dr. MOHAMED YAHIA 0900987639


QUESTION BANK OF Dr. MOHAMED YAHIA

38. Functions in the liver include production of:

a) Digestive enzymes

b) Secondary bile acids

c) Stercobilinogen

d) CCK

e) Taurocholic acid

39. The small intestine:

a) Is shorter than the large intestine

b) Its internal surface area is smaller than the large intestine

c) It absorbs less water than the large intestine

d) It contains bacteria as found in the large intestine

e) It produces the enzymes needed for lipid digestion

40. Bile contains all of the following except:

a) Alkaline phosphatase

b) K+

c) Cholesterol

d) CCK

e) Water

41. The functions of gastrin include the following except:

a) Increased production of histamine in the gastric mucosa

b) Stimulation of the growth of intestinal mucosa

c) Increased secretion of pepsinogen

d) Contraction of the gastro-oesophageal junction

195 ‫صفحة‬ Dr. MOHAMED YAHIA 0900987639


QUESTION BANK OF Dr. MOHAMED YAHIA

e) Increased secretion of HCO3- by biliary duct cells

42. Major constituents of feces include all of the following except:

a) Water

b) Cellulose

c) Bacteria

d) Calcium salts

e) Glycogen

43. The exocrine pancreas secretes the following enzymes except:

a) Chymotrypsinogen

b) Prolestase

c) Lactase

d) Alpha-amylase

e) Phospholipase

44. GIT hormones with a major effect on the stomach include the following
except:

a) Gastrin

b) Secretin

c) GIP

d) Neurotensin

e) CCK

45. If the submandibular salivary gland stops functioning:

196 ‫صفحة‬ Dr. MOHAMED YAHIA 0900987639


QUESTION BANK OF Dr. MOHAMED YAHIA

a) The volume of salivary secretion will be reduced by 20%

b) There would be severe deficiency of alpha-amylase

c) There would be no mucins in saliva

d) There would be difficulty in swallowing

e) Parasympathetic stimulation would not increase salivary secretion

46. If the oesophageal muscles are not working:

a) Swallowing in the upright position is not affected because of gravity

b) Water swallowing is not affected

c) There will be dysphagia to solids and liquids

d) There will be frequent attacks of vomiting

e) Gastro-oesophageal reflex becomes less common

47. Atrophy of the gastric mucosal glands is likely to result in:

a) Diminished digestion of starch

b) Failure of digestion and absorption of proteins

c) High gastrin levels in the blood

d) High secretin levels in the blood

e) Normochromic normocytic anemia

48. The brush border enzymes of the small intestine digest:

a) Starch

b) Cholesterol esters

c) Albumin

d) Galactose

e) Lactose

197 ‫صفحة‬ Dr. MOHAMED YAHIA 0900987639


QUESTION BANK OF Dr. MOHAMED YAHIA

49. The average daily amount of fluid that passes through the pylorus in a
healthy adult is about:

a) 2 liters

b) 9 liters

c) 6 liters

d) 8 liters

e) 12 liters

50. The stomach has the ability to absorb:

a) Ca2+

b) Alcohol

c) Glucose

d) Aromatic amino acids

e) Fe2+

51. The effects of parasympathetic stimulation on the GIT include:

a) Constriction of the pylorus

b) Contraction of the internal anal sphincter

c) Contraction of the external anal sphincter

d) Contraction of the gall bladder

e) Decreased mucus secretion

52. Pancreatic exocrine secretion is stimulated by the following except:

a) CCK

b) Secretin

198 ‫صفحة‬ Dr. MOHAMED YAHIA 0900987639


QUESTION BANK OF Dr. MOHAMED YAHIA

c) Vagal stimulation

d) Presence of food in stomach

e) Entropeptidase

53. Absorption of iron:

a) The stomach absorbs 55% of dietary iron due to the presence of HCl

b) The small intestine absorbs most of the iron in the diet

c) Iron absorption is mainly at the terminal ileum

d) The amount of iron absorbed changes according to body stores and rate of

erythropoiesis

e) The amount of iron absorbed is adjusted to balance iron excretion by the

kidney

54. Saliva does not normally contain:

a) Isozyme

b) IgA

c) Lactoferrin

d) Intrinsic factor

e) Hydrogen ions

55. The oesophageal phase of swallowing:

a) Cannot proceed when the mouth is open

b) Is primarily a reflex controlled by the myenteric plexus

c) Is primarily controlled by the swallowing center in the brain

d) Starts when a bolus enters the oropharynx

e) Takes less time in patients with achalasia of the cardia

199 ‫صفحة‬ Dr. MOHAMED YAHIA 0900987639


QUESTION BANK OF Dr. MOHAMED YAHIA

56. Vomitting is not usually caused by:

a) Irritation if the duodenal mucosa

b) Diseases of the inner ear

c) Microbial toxins in the blood

d) Severe exercise

e) Bad smell

57. The function of the migrating motor complex (MMC) is to:

a) Empty non-digestable material from stomach and small intestine during

fasting

b) Initiate mass movements in the large intestine

c) Increase intestinal peristalsis in response to overfeeding

d) Increase ileal motility in response to entry of chyme into the stomach

e) Overcome intestinal obstruction

58. The duodenum and jejunum are the main site for absorption of the following
except:

a) Glucose

b) Vitamin C

c) Fat-soluble vitamins

d) Bile salts

e) Iron

59. The mucosal cells of the human colon perform the following functions:

a) Converting primary bile acids into secondary bile acids

b) Converting bilirubin into stercobilinogen

c) Absorption of Na+

200 ‫صفحة‬ Dr. MOHAMED YAHIA 0900987639


QUESTION BANK OF Dr. MOHAMED YAHIA

d) Absorption of aromatic amino acids

e) Digestion of cellulose

60. It is true to say the following about the motility of the gastrointestinal system:

a) It is regulated neurally but not by hormones

b) Emptying of the stomach is mainly by segmentation contractions

c) Movement of chyme along the small intestine is mainly by the migrating

motor complex

d) Peristalsis is seen in most parts of the GIT

e) The gallbladder is contracted by sympathetic stimulation

61. Secretion of the acinar cells within the exocrine pancreas is not increased by:

a) Secretin

b) CCK

c) Parasympathetic stimulation

d) Bicarbonate ions

e) Acetylcholine

RENAL SYSTEM
1. Which of the following is true about renal blood flow?

a) The kidneys receive 12-15% of cardiac output

b) Is kept constant within a small range by sympathetic

vasomotor tone

c) Measured by renal clearance of inulin

d) The dilution method is used for its measurement

e) Goes mainly to the renal cortex

2. The characteristics of the Loop of Henle include:

201 ‫صفحة‬ Dr. MOHAMED YAHIA 0900987639


QUESTION BANK OF Dr. MOHAMED YAHIA

a) Descending limb is not permeable to water

b) Urine becomes more dilute as it reaches the tip of the loop

c) Ascending limb is permeable to water

d) In thin ascending limb urea is actively transported to interstitial fluid

e) Na is reabsorbed by Na-K ATPase in ascending limb

3. The following is true about the Vasta Recta:

a) Descending vasta recta reabsorbs water from the interstitial fluid

b) The descending capillaries loose Na and Urea

c) Plasma becomes hyperosmotic as it descends in vasta recta

d) Ascending capillaries loose water to interstitial fluid

e) They are responsible for the countercurrent multiplier mechanism

4. About Bicarbonate Reabsorption:

a) Represents a high capacity low gradient system

b) Filtered bicarbonate/ day= 400 – 500 mmol/day

c) More HCO3 is absorbed in distal than proximal tubules

d) PCT apical membrane is permeable to HCO3-

e) H2CO3 is PCT lumen is catalyzed at brush border by carbonic anhydrase to

give CO2 and HCO3-

5. All of the following are true of ammonium production except:

a) It is mainly a function of PCT

b) The source is mainly glutamine

c) It represents the main mechanism in renal excretion of H+

d) It is a low capacity high gradient system

202 ‫صفحة‬ Dr. MOHAMED YAHIA 0900987639


QUESTION BANK OF Dr. MOHAMED YAHIA

e) It is increased when the urine pH is low

6. The functions of the renal proximal convoluted tubes include:

a) Reabsorption of all bicarbonate filtered at the glomerulus

b) Production of renin

c) Reaborption of 80% of filtered bicarbonate

d) Reabsorption of bicarbonate resulting in fixed acid secretion

e) Hydrogen ion secretion by facilitated diffusion

7. Titrable Acidity (TA):

a) Can be measured by titration against sodium hydroxide to pH 7.4

b) Is a measure of H+ mainly buffered by ammonium ions in the distal tubules

c) Is the same as the amion gap

d) Creatinine contributes to significant amounts to TA

e) Ketoacids in diabetic acidosis do not increase TA

8. For those substances that are actively reabsorbed or secreted, the highest limit
of the amount that can be transported per unit time by the kidney tubules:

a) Depends on the rate at which the transport mechanism operates

b) Is directly related to the plasma concentration of the substance

c) Depends on the tubular transport maximum

d) Is dependent upon tubular load

e) Depends on the GFR

9. If the clearance of Substance X is greater than the clearance on inulin, it is


most likely that substance X is:

a) Bound to tubular proteins

b) Bound to plasma proteins

203 ‫صفحة‬ Dr. MOHAMED YAHIA 0900987639


QUESTION BANK OF Dr. MOHAMED YAHIA

c) Secreted

d) Reabsorbed

e) Neither secreted nor reabsorbed

10. Renin is produced and secreted from granules located in the:

a) Macula densa

b) Mesangial cells

c) Intercalated cells

d) Afferent arteriole

e) Interstitial cells

11. The tonicity of urine as it enters the renal collecting duct is:

a) Isotonic

b) Hypotonic or isotonic, but never hypertonic

c) Hypotonic

d) Hypertonic

e) Hypertonic or isotonic, but never hypotonic

12. On assuming the upright posture, one would expect:

a) Stimulation of the baroreceptore reflex

b) Increased pulmonary artery pressure

c) A decrease in pulse rate

d) Venodilation

e) An increase in renal blood flow

13. In a healthy adult man the glomerular filteration rate is expected to be:

a) 225 ml/min

204 ‫صفحة‬ Dr. MOHAMED YAHIA 0900987639


QUESTION BANK OF Dr. MOHAMED YAHIA

b) 325 ml/min

c) 125 ml/min

d) 25 ml/min

e) 180 ml/min

14. The following factors when increased, they will increase the glomerular
filteration rate except:

a) Renal blood flow

b) Afferent arteriolar diameter

c) Efferent arteriolar diameter

d) Permeability of glomerular membrane

e) Capillary surface area

15. A substance that is used to measure renal plasma flow should be:

a) Bound to plasma proteins

b) Easily reabsorbed

c) Highly secreted

d) Metabolized by the body

e) Stored only in the kidney

16. Renal blood flow is:

a) Increased when renal sympathetic nerves are stimulated

b) Autoregulated when mean arterial blood pressure is between 60-180 mmHg

c) Measured by urea clearance

205 ‫صفحة‬ Dr. MOHAMED YAHIA 0900987639


QUESTION BANK OF Dr. MOHAMED YAHIA

d) About 2 liters in a normal man

e) Decreased when the fluid flow at the site of the macula densa is decreased

17. The following hormones can be formed by the kidney except:

a) Erythropoietin

b) Angiotensin II

c) Prostaglandins

d) ADH

e) Calcitrol

18. Sodium reabsorption in the proximal tubules from the luminal surface is
marked by all of the following except:

a) Sodium glucose co-transporters

b) Sodium-hydrogen exchanger

c) Na-K pump

d) Solvent drag

e) Sodium amino acid transporters

19. Glomerular filteration rate would be increased by:

a) A decrease in afferent arteriolar pressure

b) A decrease in renal blood flow

c) A decrease in efferent arteriolar resistance

d) Decrease in the concentration of plasma proteins

e) Obstruction of ureters

20. Potassium secretion in the distal tubule will be increased if there is an


increase in:

a) Plasma (H+)

206 ‫صفحة‬ Dr. MOHAMED YAHIA 0900987639


QUESTION BANK OF Dr. MOHAMED YAHIA

b) Plasma volume

c) Plasma sodium concentration

d) Plasma potassium concentration

e) Plasma calcium concentration

21. The following is true about glucose reabsorption in the renal tubules:

a) Transport maximum is 80 mg/min

b) The transport maximum is dependent on the amount of

glucose filtered

c) It is mainly a symport with K+

d) It is reduced with metabolic inhibitors that reduce the activity Na/K ATPase

e) About 50% of filtered glucose is reabsorbed

22. Water reabsorption in the collecting duct:

a) Is dependent on aldosterone

b) Is increased by ADH

c) Has the largest percentage of total water reabsorbed

d) Is facilitated by sodium reabsorption

e) Independent of osmotic difference across the tubules

23. The Glomerular Filteration Rate (GFR) can be calculated by all of the
following except:

a) PAH clearance

b) Creatinine clearance

207 ‫صفحة‬ Dr. MOHAMED YAHIA 0900987639


QUESTION BANK OF Dr. MOHAMED YAHIA

c) Urea clearance

d) Inulin clearance

e) Formula using plasma creatinine concentration

24. Ammonia produced by the kidneys comes mainly from:

) a Glutamine

b) Glycine

c) Leucine

d) Alanine

e) B and D are correct

25. Which of the following is NOT a function of the kidneys?

a) Regulation of extracellular volume

b) Regulation of plasma glucose concentration

c) Regulation of arterial blood pressure

d) Excretion of end products of protein metabolism

e) Excretion of foreign chemicals (eg. Pharamaceuticals, food additives, etc)

26. The transport of glucose across the luminal membrane in the proximal part
of the renal tubule is:

a) Through insulin dependent glucose transporters

b) Cannot take place against the concentration gradient

c) Is not affected by Na+/K+ pump inhibitors

d) Is a co-transport with sodium

e) Is decreased in diabetes mellitus

27. In a 70kg normal male, the GFR is:

208 ‫صفحة‬ Dr. MOHAMED YAHIA 0900987639


QUESTION BANK OF Dr. MOHAMED YAHIA

a) 25 ml/minute

b) 50 ml/minute

c) 125 ml/minute

d) 225 ml/minute

e) 250 ml/minute

28. The following contributes to the high osmolarity of the renal medulla:

a) Low blood flow in the medulla

b) The very low rate of tubular flow in the medulla

c) The increase in the permeability of the collecting ducts caused by ADH

d) Aldosterone

e) The sodium dependent organic substance transporter

29. The restriction of protein to pass through the filtration membrane is due to:

a) The size of the pores in the capillary wall

b) The size of the spaces between the cells of the epithelial layer

c) The basement membrane

d) The positive charge on the membrane

e) The presence of the phagocytic mesangial cells

30. The following hormones act on the proximal tubules of the kidney:

a) ADH

b) Aldosterone

209 ‫صفحة‬ Dr. MOHAMED YAHIA 0900987639


QUESTION BANK OF Dr. MOHAMED YAHIA

c) 25 hydroxy cholecalciferol

d) Angiotensin II

e) Dihydrotestosterone

31. The following decreases the GFR:

a) Increased constriction of the afferent arteriole

b) Decrease in the pressure of the afferent arteriole

c) Decrease in concentration of plasma protein

d) Decreased hydrostatic pressure in Bowman's capsule

e) Decrease in the radius of efferent arterioles

32. The tubular fluid as it leaves the proximal tubules is:

a) Hypotonic

b) Hypertonic

c) Has the same sodium concentration as the filtrate

d) Has a volume that is 20% of the filtrate

e) Has very low inulin concentration following inulin administration

33. The clearance of a substance was found to be less than that of inulin. This
substance is:

a) Not bound to plasma proteins

b) Actively reabsorbed

c) Secreted

d) Neither reabsorbed nor secreted

e) Secreted at a rate greater than reabsorbed

34. High osmalarity of the extracellular fluid results in:

210 ‫صفحة‬ Dr. MOHAMED YAHIA 0900987639


QUESTION BANK OF Dr. MOHAMED YAHIA

a) Decreased thirst stimulation

b) Increased rennin secretion

c) Increased ADH secretion

d) Increased urine formation

e) Increased stimulation of macula densa cells

35. The micturition reflex:

a) Is integrated in the medulla

b) Efferent limb is parasympathetic

c) Can take place if the sacral spinal segments were damaged

d) Is absent in children

e) Threshold is higher in males than females

36. In the proximal tubules:

a) 20% of filtered amino acid is reabsorbed

b) Sodium reabsorption is aldosterone dependent

c) Equal percentage of sodium and water is reabsorbed

d) Potassium is mainly secreted

e) Calcium cannot be reabsorbed

37. Renin secretion by the kidney is increased by increasing:

a) Tubular fluid flow rate

211 ‫صفحة‬ Dr. MOHAMED YAHIA 0900987639


QUESTION BANK OF Dr. MOHAMED YAHIA

b) Mean systemic blood pressure

c) Atrial natriuretic peptide

d) Renal sympathetic nerve activity

e) Discharge from baroreceptors

38. The segment of the nephron that contributes most to potassium excretion
when dietary potassium is altered is:

a) Proximal convulated tube

b) Descending limb of Henle's loop

c) Proximal straight tubule

d) Distal tubule

e) Thick ascending limb of Henle's loop

39. The ability of the kidney to excrete a concentrated urine will increase if:

a) The permeability of the proximal tubules decreases

b) The rate of blood flow through the medulla decreases

c) The rate of flow through the loop of Henle increases

d) The activity of the Na-K pump in the loop of Henle decreases

e) The permeability of the collecting duct to water decreases

40. Potassium excretion is enhanced by:

a) An osmotic diuresis

b) Acute metabolic acidosis

c) A low aldosterone level

d) Decreased tubular flow rate

e) Ingestion of a low-potassium diet

212 ‫صفحة‬ Dr. MOHAMED YAHIA 0900987639


QUESTION BANK OF Dr. MOHAMED YAHIA

41. Which of the following decreases the glomerular filteration rate:

a) An increase in Bowman's capsule pressure

b) A decrease in glomerular capillary hydrostatic pressure

c) An increase in tubular fluid hydrostatic pressure

d) A decrease in efferent arteriolar resistance

e) A decrease in plasma colloid pressure

42. In renal hemodynamics:

a) Renal plasma flow is measured by clearance of creatinine

b) Urea clearance provides a close estimation of GFR

c) If the renal clearance of a substance exceeds that of inulin it must be

secreted into the tubules

d) A substance that has a clearance less than that of inulin must be reabsorbed

by the tubules

e) Inulin clearance rises with an increase in inulin plasma level

43. Which of the following substances is more concentrated at the end of the
proximal tubule than at the beginning:

a) Glucose

b) Potassium

c) Bicarbonate

d) Creatinine

e) Sodium

44. The bicarbonate: carbonic acid ration of the plasma that corresponds to pH
7.4 equals:

a) 1:1

213 ‫صفحة‬ Dr. MOHAMED YAHIA 0900987639


QUESTION BANK OF Dr. MOHAMED YAHIA

b) 5:1

c) 10:1

d) 15:1

e) 20:1

45. The macula densa in the juxtaglomerular apparatus regulate:

a) Plasma pH

b) Glomerular fileration rate

c) Reabsorption of potassium by the proximal tubules

d) Red blood cell production

e) Phosphate excretion

ENDOCRINE SYSTEM
1. A tumor that produces large amounts of catecholamines can be detected by
examining the urine for:

a) K+

b) Uric acid

c) Choline

d) VMA

e) Calcium

2. The following hormone is secreted by the anterior pituitary gland:

a) Cortisol

b) TRH

c) CRH

d) Vasopressin

e) TSH

214 ‫صفحة‬ Dr. MOHAMED YAHIA 0900987639


QUESTION BANK OF Dr. MOHAMED YAHIA

3. A ten year old boy has high levels of the growth hormone. He is likely to
develop:

a) Acromegaly

b) Hyperglycemia

c) Mental retardation

d) Muscle atrophy

e) Hyperkalaemia

4. Stimuli for aldosterone regulation includes:

a) High sodium in the distal renal tubule

b) High serum potassium

c) Hypervolaemia

d) ACTH as a potent stimulator

e) Increased renal blood flow

5. Thyroid hormone actions include:

a) Augmentation of sympathetic effect on GI motility

b) Release of insulin

c) Lipogenesis

d) Increasing the numbers of Beta adrenergic receptors

e) Increasing diastolic arterial blood pressure

6. Release of oxytocin by the posterior pituitary gland is controlled by:

a) Oxytocin releasing hormone

b) Neural signals from hypothalamus

c) Neural signals from the thalamus

215 ‫صفحة‬ Dr. MOHAMED YAHIA 0900987639


QUESTION BANK OF Dr. MOHAMED YAHIA

d) Changes in body temperature

e) Changes in arterial blood pressure

7. The following can be observed in a patient who has Grave's disease


(hyperthyroidism):

a) Drop of the upper eyelid

b) High level of TSH

c) Intolerance to cold

d) High systolic blood pressure

e) Constipation

8. The thyrotropin releasing hormone, stimulates the secretion of:

a) Growth hormone

b) Prolactin

c) Adrenocorticotrophic hormone

d) Melanocyte stimulating hormone

e) Luteinizing hormone

9. The following is true about estrogen:

a) It is essential for development of female external genitalia

b) Is released only during the follicular phase of the ovarian cycle

c) Is released from granulosa cells

d) Its secretion is decreased by inhibin

e) Its receptors are located on cell membranes

10. The most sensitive regulatory mechanism of ADH secretion is dependent on:

a) The volume receptors

216 ‫صفحة‬ Dr. MOHAMED YAHIA 0900987639


QUESTION BANK OF Dr. MOHAMED YAHIA

b) Renin-angiotensin system

c) Hypothalamic osmoreceptors

d) Renal glomerulotubular feedback

e) Renal principal cells

11. Clinical uses of Oxytocin include:

a) Stimulation of milk ejection in breastfeeding women

b) Stimulation of ejaculation in infertile men

c) Stimulation of uterine contraction in pregnant women

d) Induction of breast development in adolescent girls

e) Induction of vasoconstriction in hypotension

12. All the following is true about control of secretion of TSH except:

a) It is inhibited by T4

b) It is increased in cold weather

c) It is increased in Grave's disease

d) It is increased in TRH

e) Has no marked diurnal rhythm

13. The actions of ACTH include the following except:

a) Stimulation of melanocytes

b) Increased secretion of aldosterone

c) Induction of growth of the adrenal gland

d) Induction of secretion of adrenomedullins

e) Feedback inhibition of CRH

14. A most effective method of treating type I diabetes mellitus is:

217 ‫صفحة‬ Dr. MOHAMED YAHIA 0900987639


QUESTION BANK OF Dr. MOHAMED YAHIA

a) The patient should not eat any carbohydrates

b) Exercise

c) Reduce weight

d) Drugs to stimulate the B cells of the pancreas

e) Daily injections of insulin

15. The following are actions of insulin, except:

a) Inhibition of protein degradation

b) Inhibition of gluconeogenesis

c) Inhibition of ketogenesis

d) Reduction of extracellular K+

e) Increased renal tubular absorption of glucose

16. The following factors have a direct marked effect on the adrenal cortex to
stimulate the secretion of aldosterone except:

a) Angiotensin III

b) Angiotensin II

c) Hyperkalemia

d) Hypernatremia

e) High levels of ACTH

17. A major regulator of bone growth is:

a) Parathyroid hormone

218 ‫صفحة‬ Dr. MOHAMED YAHIA 0900987639


QUESTION BANK OF Dr. MOHAMED YAHIA

b) Calcitonin

c) Growth hormone

d) Prolactin

e) Active vitamin D

18. If a patient dies of hypercalcemia, the most likely cause would be:

a) Excessive bleeding due to failure of clotting

b) Paralysis of skeletal muscles due to failure of interaction between actin and

myosin

c) Uncontrolled contraction of skeletal muscles due to hyperexcitability of

nerves and muscles

d) Failure of the SA node to generate impulses

e) Hypo-osmolality

19. Primary hyperaldosteronism leads to:

a) Moderate hyperglycemia

b) Sodium natriuresis

c) Oedema

d) Alkalosis

e) Increased rennin secretion

20. Which of the following is a manifestation of Addison's disease:

a) Hypopigmentation

b) Good appetite

c) Hypoglycemia

d) Obesity

e) Hypertension

219 ‫صفحة‬ Dr. MOHAMED YAHIA 0900987639


QUESTION BANK OF Dr. MOHAMED YAHIA

21. Excess cortisol in Cushing's syndrome is associated with:

a) Hypotension

b) Protein depletion

c) Hypoglycemia

d) Dry thick skin

e) Increased body hair

22. Insulin secretion is inhibited by:

a) Alpha adrenergic receptor stimulation

b) Vagal stimulation

c) Amino acids

d) GIP

e) Gastrin

23. Calcitonin:

a) Increases serum calcium by stimulation of osteoclasts

b) Prevents postprandial hypercalcemia

c) Is important in long-term regulation of serum ionized calcium

d) Is stimulated by a hypothalamic releasing hormone

e) Is released during stress

24. Hypomagnesemia is a recognized cause of:

a) Diabetes mellitus

b) Diabetes insipidus

220 ‫صفحة‬ Dr. MOHAMED YAHIA 0900987639


QUESTION BANK OF Dr. MOHAMED YAHIA

c) Hypervitaminosis D

d) Tetany

e) Hypothyroidism

25. A ten year old boy has high levels of growth hormone. He is likely to develop:

a) Acromegaly

b) Hyperglycemia

c) Mental retardation

d) Muscle atrophy

e) Hyperkalemia

26. Products of the anterior pituitary gland include:

a) Melatonin

b) Acetylcholine

c) Beta endorphin

d) Inhibin

e) Somatomedins

27. Secretion of gonadotropins is ordinarily inhibited by which of the following

a) Estradiol

b) Testosterone

c) Calcium

d) Oxytocin

e) A or B

28. Testosterone action on target cells below the neck leads to increases on which
of the following?

221 ‫صفحة‬ Dr. MOHAMED YAHIA 0900987639


QUESTION BANK OF Dr. MOHAMED YAHIA

a) Spermatogenesis

b) Prostate size

c) Penile size

d) None of the above

e) A, B and C are correct

29. In humans, each of the following facilitates ovulation except:

a) Pulsatile secretion of gonadotropin-releasing hormone

b) Positive feedback effect of estradiol on LH secretion

c) Negative feedback effect of estradiol on LH secretion

d) Action of FSH on granulose cells

e) Secretion of LH in a surge pattern

30. Ionized calcium:

a) Forms 60% of total blood calcium

b) Cannot be filtered by the kidneys

c) Low levels stimulate calcitonin secretion

d) Levels in blood are increased by increase in H+ concentration

e) Concentration inside the cell is the same as the interstitial fluid

31. Insulin can be described as:

a) Gluconeogenic

b) Ketogenic

c) Protein anabolic

d) Diuretic

e) Vasoconstrictive

222 ‫صفحة‬ Dr. MOHAMED YAHIA 0900987639


QUESTION BANK OF Dr. MOHAMED YAHIA

32. The main effect of somatostatin secreted by the D cells of the islets of
Langerhans appears to be:

a) Inhibition of the secretion of growth hormone

b) Stimulation of glucagon secretion

c) Inhibition of gastric acid secretion

d) Inhibition of insulin secretion

e) Relaxation of the sphincter of Oddi

33. Factors that act on the cells of the adrenal gland to cause secretion of
aldosterone include:

a) Renin

b) Angiotensinogen

c) Angiotensin I

d) Atrial natriuretic peptide (ANP)

e) Potassium ions

34. The following effects can be caused by cortisol except:

a) Excessive loss of K+ in urine

b) Hypertension

c) Hyperglycemia

d) Increased production of CRH by the hypothalamus in cases of Cushing's

syndrome

e) Immunosuppression

35. T3 is different from T4 in that:

a) It binds to receptors located on cell membranes

b) Its concentration increases in starvation

c) Its volume of distribution is high

223 ‫صفحة‬ Dr. MOHAMED YAHIA 0900987639


QUESTION BANK OF Dr. MOHAMED YAHIA

d) It increases the efficiency of energy utilization

e) Most of it in plasma is bound to TBG

36. Known effects of ACTH include all of the following except:

a) Increased secretion of adrenal androgens

b) Increased secretion of aldosterone transiently

c) Increased pigmentation of body tissues

d) Stimulation of growth of zona reticularis of the adrenal cortex

e) Direct stimulation of surfactant secretion

37. Prolactin secretion:

a) Has diurnal rhythm

b) Can be inhibited by L-dopa

c) Is inhibited by chlorpromazine

d) Is stimulated by bromocriptine

e) Is continuously stimulated by prolactin releasing hormone

‫شيت المجلس‬
224 ‫صفحة‬ Dr. MOHAMED YAHIA 0900987639
QUESTION BANK OF Dr. MOHAMED YAHIA

1. The speed of transmission of an impulse along an axon is faster when:


a. the axon is myelinated
b. the diameter of the axon is larger but un myelinated
c. the number of Na+ channels is doubled
d. the number of K+ channels is reduced
e. the number of mitochondria is increased

2. Red muscle fibers:


a. are suitable for quick movements
b. fatigue easily
c. can store oxygen
d. are rich in creatinine
e. have no troponin

3. Transmitters of the autonomic nervous system include:


a. acetylcholine at all post ganglionic parasympathetic terminals
b. acetylcholine in all autonomic ganglia
c. adrenalin in most sympathetic postganglionic neurons to the adrenal medulla
d. adrenalin and nor-adrenalin in sympathetic postganglionic neurons
e. dopamine in pre-ganglionic neurons to the adrenal medulla

4. Sympathetic B1 receptors are found in:


a. the sino-atrial node
b. smooth muscle of the constrictor pupillae
c. smooth muscle of the bronchioles
d. muscle of the upper eye lid
e. smooth muscle of skin vessels

5. A drug that increases the heart rate from 70 to 100 beats per minute could be:
a. a B1 adrenergic receptor antagonist
b. a cholinergic receptor antagonist
c. a cholinergic receptor agonist
d. a B2 adrenergic receptor agonist
e. a non-cholinergic, non-adrenergic agonist

6. Body water
a. is about 60 liters
b. can be measured by inulin dilution
c. is found mainly intra-cellularly
d. has solutes osmotic pressure of about 400 mosm/l
e. loss of 5% causes symptoms & signs of dehydration

7. The main extra-cellular cation is


a. Chloride
b. Potassium
c. sodium

225 ‫صفحة‬ Dr. MOHAMED YAHIA 0900987639


QUESTION BANK OF Dr. MOHAMED YAHIA

d. protein
e. calcium

8. Extra-cellular fluid volume in a 70 Kg male is about


a. 10 liters
b. 12 liters
c. 15 liters
d. 25 liters
e. 40 liters

9. Localizededema in a limb is mainly due to


a. increased capillary permeability
b. low plasma albumin
c. low interstitial fluid hydrostatic pressure
d. high capillary hydrostatic pressure
e. blockage of lymph flow

10. Generalized edema can be caused by


a. Hypothyroidism
b. hypoalbuminaemia
c. lymphadenopathy
d. cancer
e. deep venous thrombosis

11. The effects of parasympathetic stimulation on the GIT include:


a. constriction of the pylorus
b. contraction of the internal anal sphincter
c. contraction of the external anal sphincter
d. contraction of the gall bladder
e. decreased mucus secretion

12. Stroke volume:


a. increases as a result of increased after load
b. equals end-diastolic volume minus end-systolic volume
c. increases as heart rate is increased by electrical pacing
d. is increased by parasympathetic stimulation
e. is increased by Ca channel blockers

13. Which of the following results in an increase in systolic blood pressure?


a. increased vasomotor tone
b. decreased sympathetic activity to the ventricles
c. an increased rate of the sinoatrial node pacemaker potential
d. increased sympathetic activity to arterioles of skeletal muscle & skin
e. increased parasympathetic stimulation to the ventricles

14. The most sensitive regulatory mechanism of ADH secretion is dependent on:
a. The volume receptors
b. Rennin- angiotensin system
c. Hypothalamic osmoreceptors
d. Renal glomerulotubular feedback

226 ‫صفحة‬ Dr. MOHAMED YAHIA 0900987639


QUESTION BANK OF Dr. MOHAMED YAHIA

e. Renal principal cells

15. The actions of ACTH include


a. Stimulation of release of lymphocytes
b. Increased secretion of aldosterone
c. Induction of growth of the adrenal gland
d. Induction of secretion of adrenomedullins
e. Feedback inhibition of TSH

16. The following are actions of insulin, except:


a. stimulation of protein synthesis
b. inhibition of gluconeogenesis
c. inhibition of ketogenesis
d. reduction of extracellular K+
e. increased lipolysis

17. Which of the following is true of cortisol excess:


a. a significant increase in BMR
b. marked peripheral vasoconstriction
c. anxiety neurosis
d. it stops the circadian rhythm of melatonin
e. loss of bone matrix

18. A major regulator of bone growth is:


a. parathyroid hormone
b. Calcitonin
c. growth hormone
d. Prolactin
e. active vitamin D

19. If a patient dies of hypocalcaemia, the most likely cause would be:
a. excessive bleeding
b. loss of normal smooth muscles contractility
c. uncontrolled contraction of skeletal muscles
d. failure of the SA node to generate impulses
e. hypo-osmolality

20. Which of the following is true about the glomerular filtration (GFR):
a. the rate of filtration is about 180 liters /day
b. GFR= about 250 ml/min
c. its composition is similar to that of plasma
d. water, electrolytes pass by diffusion
e. organic molecules up to 80 D can pass in the glomerular filtrate

21. the functions of the renal proximalconvoluted tubules include:


a. Reabsorption of all bicarbonate filtered at glomerulus
b. Production of ammonium (NH4)
c. reabsorption of about 70% of filtered bicarbonate
d. reabsorbed bicarbonate results in fixed acid secretion
e. Hydrogen ion secretion is facilitated by ammonium

227 ‫صفحة‬ Dr. MOHAMED YAHIA 0900987639


QUESTION BANK OF Dr. MOHAMED YAHIA

22. Fine touch sensation is transmitted through:


a. spino-thalamic tracts only
b. gracile&cuneate tracts only
c. dorsal column &spinothalamic tracts
d. olivospinal tracts
e. corticospinal tracts

23. The spinal cord has the center/s for:


a. abdominal reflexes
b. Stretch reflex
c. touch & pressure discrimination
d. visceral reflexes
e. plantar reflexes

24. The dorsal column tracts of the spinal cord serve the following modalities
a. Pain
b. crude touch
c. ticle& itch
d. sexual sensations
e. proprioception

25. The stretch reflex is exaggerated in


a. lower motor neuron lesions
b. immediately after injury to spinal cord
c. medullary reticular formation
d. lesions of primary motor cortex
e. pontine reticular formation

26. Peripheral neuropathy


a. causes dissociated sensory loss
b. affects distal cutanous sensation
c. is caused by cerebrovascular thrombosis
d. causes muscle weakness of limb girdles
e. can cause a form of upper motor neuron lesion

27. Hemiplegia is caused by


a. transection of the spinal cord
b. abnormalities of the utricle &sacule
c. upper motor neuron disease
d. polio myelitis
e. a lesion in the internal capsule

28. Brown-Seqard syndrome is charachterised by


a. transaction of the spinal cord
b. loss of all sensations below the level of the lesion
c. lower motor neuron lesion below the level of the lesion
d. dissociated loss of proprioception
e. loss of pain sensation on the opposite side below the lesion

228 ‫صفحة‬ Dr. MOHAMED YAHIA 0900987639


QUESTION BANK OF Dr. MOHAMED YAHIA

29. The concentration of the following is higher in the extracellular fluid than in the
intracellular fluid:
a. Ca++
b. H+
c. K+
d. Mg++
e. Protein

30. Extracellular fluid volume is measured by the following:


a. Inulin
b. Mannitol
c. Albumin
d. Tritium oxide
e. C1-51

31. Regarding the role of the kidney in regulation of acid base balance. The following is
true:
a. The kidney maintains the urine pH at constant levels
b. Increased titrable acidity increases sodium re-absorption
c. Increased excretion of NH4+ increases Hco3- loss in urine
d. Increased urine pH increases secretion of NH3
e. Titrable acidity increases five folds in metabolic acidosis

32. One of the following does not cause metabolic alkalosis:


a. Dehydration
b. Vomiting
c. Increased glomerular filtration
d. Increased aldosterone secretion
e. Ingestion of ammonium chloride

33. One of the following statements is not true about platelets:


a. Are derived from the megakaryocytes
b. Secrete thromboxane A2
c. Count is increased by splenectomy
d. Aggregation is inhibited by aspirin
e. Has α dense granules

34. Dorsal column tracts: One statement is false:


a. Carry fine touch sensation
b. Carry vibration sensation
c. Are ascending tracts
d. Go directly to the thalamus without decussating
e. Relay on gracile and cunate nuclei before going to the thalamus
35. Regarding pain: Only one statement is true:
a. Fast pain is carried by type C fibers
b. Visceral pain is associated with sweating
c. Referred pain is only felt on the skin
d. Pain fibers ascend on the dorsal column tracts
e. Pain fibers relay on the midline nuclei of the thalamus convey sharp pain

229 ‫صفحة‬ Dr. MOHAMED YAHIA 0900987639


QUESTION BANK OF Dr. MOHAMED YAHIA

36. What is the chief factor in producing sensation of heat on palpation of an acute
abscess?
a. Increased metabolism in the tissues around the abscess
b. Increased rate of destruction of micro-organisms
c. Increased blood flow through the skin
d. Increased metabolism of the epidermis
e. Increased body temperature

37. In a polymorph white blood cell what is the cytoplasmic organelle that destroys
the bacterium:
a. Lysosome
b. Mitochondrion
c. Phagocytic vacuole
d. Phagosome
e. Pinocytic vacuole

38. Which one of the following cells is not a tissue macrophage?


a. Osteoblasts
b. Microglial cells
c. Kupffer cells
d. Mesangial cells
e. Endothelial cells

39. The determinants of the mean arterial pressure include:


a. cardiac output and peripheral resistance.
b. arterial and venous volume.
c. cardiac output and arterial volume.
d. peripheral resistance and blood volume.
e. cardiac output and venous volume.

40. A prolonged decrease in arterial pressure observed in an elderly person is


produced mainly by:
a. decreased stroke volume.
b. decreased heart rate.
c. a decreased sympathetic activity.
d. an increase in vagal tone.
e. decreased arterial elasticity.

41. Vitamin B12 :


a. is a fat soluble vitamin
b. is found only in vegetables
c. is needed for DNA formation
d. deficiency causes microcytic anemia
e. deficiency causes a defect in sensation

230 ‫صفحة‬ Dr. MOHAMED YAHIA 0900987639


QUESTION BANK OF Dr. MOHAMED YAHIA

42. The distal ileum is the main site for absorption of:
a. Glucose
b. vitamin C
c. fat soluble vitamins
d. bile salts
e. iron

43. The secretion of saliva:


a. shows circadian rhythm
b. is due to adrenergic stimulation
c. is modified by aldosterone
d. is stimulated by VIP
e. stimulated before vomiting

44. Which receptors are most effective in mediation of secretion of HCL in the
stomach?
a. PG2 receptors
b. H2 receptors
c. adrenergic B2 receptors
d. cholinergic muscarinic receptors
e. gastrin receptors

45. Micelle formation is facilitated by:


a. glycerol
b. galactose
c. phospholipids
d. bile salts
e. cholesterol

46. Sodium re-absorption in the proximal renal tubules:


a. is the largest fraction of Na+ reabsorbed in the tubules
b. is dependent on aldosterone
c. is increased on inhibition of Na+ - K+ ATPase activity
d. is utilized for potassium reabsorption
e. has a transport maximum of 375 mg/minute

47. On measuring the osmolarity of urine in a normal subject, it was found to be 1000
mosm/L. in this subject we expect the following:
a. ADH is secreted
b. 30% of water is reabsorbed in the proximal tubules
c. 20% of filtered fluid is reabsorbed in the ascending limb of the loop of Henle
d. the thick part of the loop of Henle became permeable to water
e. glucose reabsorption is inhibited

231 ‫صفحة‬ Dr. MOHAMED YAHIA 0900987639


QUESTION BANK OF Dr. MOHAMED YAHIA

48. The renal blood flow:


a. about 15% of cardiac output
b. measured by PAH (para-aminohipurate)
c. is sensitive to changes in systemic blood pressure
d. increases on increasing the activity of the sympathetic nerves to the kidney
e. mainly distributed to renal medulla

49. Depletion of body potassium:


a. is best detected by chemical analysis of plasma
b. is associated with metabolic alkalosis
c. leads to increased intestinal motility
d. is seen in uncontrolled diabetes mellitus
e. is a feature of Cushing's syndrome

50. The movement of fluid from the capillaries to the interstitial space is increased by
the following EXCEPT:
a. lymphatic obstruction
b. arteriolar constriction
c. blockage of veins
d. increased proteins in the interstitial fluid
e. histamine

51. The osmolality of plasma is:


a. lower than the osmolality of the interstitial proteins
b. determined mainly by plasma proteins
c. determined mainly by Na+
d. do not affected by the level of blood glucose
e. normally between 300-310 mosm/L

52. Active transport is responsible for the:


a. initiation of the action potential
b. movement of fatty acids across the cell membrane
c. movement of Na+ to the outside of the cell
d. movement of Na+ to the inside of the cell
e. movement of the water due to ADH

53. Pulmonary edema may result from the following:


a. expansion of the extracellular fluid volume
b. mitral stenosis
c. decreased capillary permeability
d. decreased pulmonary venous pressure
e. raised systolic arterial pressure

54. In chronic compensated metabolic acidosis:


a. the plasma bicarbonate concentration is above normal
b. the pH of the plasma is increased

232 ‫صفحة‬ Dr. MOHAMED YAHIA 0900987639


QUESTION BANK OF Dr. MOHAMED YAHIA

c. the partial pressure of carbon dioxide in the blood is decreased


d. ammonia production by the kidney is decreased
e. potassium secretion by the kidney is increased

55. Growth hormone:


a. increases somatomedin formation in the liver
b. stimulates glucose entry into cells
c. inhibits protein synthesis in muscles
d. decreases the level of ketone bodies in the blood
e. decreases mobilization of FFAs

56. Signs of hypothyroidism include:


a. loss of weight
b. good appetite
c. coarse tremors
d. decreased reflex time
e. bradycardia

57. A patient with D.U is treated successfully with the drug omeprazol, the basis for
cimetidine inhibition of gastric H+ secretion is that it:
a. blocks muscarinic receptors on parietal cells
b. blocks H2 receptors at parietal cells
c. increases intracellular cyclic adenosine monophosphate (CAMP) levels
d. blocks H+, K+ adenosine triphosphate (ATPase)
e. enhances the action of acetylecholine (ACh) on parietal cells

58. When parietal cells are stimulated, they secrete:


a. HCL and intrinsic factor
b. HCL and pepsinogen
c. HCL and HCo3
d. HCo3 and intrinsic factor
e. mucous and pepsinogen

59. Which of the following is the site of gastrin secretion?


a. gastric antrum
b. gastric fundus
c. duodenum
d. ileum
e. colon

60. Which of the following inhibits the secretion of growth hormone by the anterior
pituitary?
a. Hypoglycemia
b. Starvation
c. Sleep
d. Stress
e. Somatomedins

61. Which of the following causes increased aldsterone secretion?


a. decreased blood volume

233 ‫صفحة‬ Dr. MOHAMED YAHIA 0900987639


QUESTION BANK OF Dr. MOHAMED YAHIA

b. hypokalaemia
c. hyperosmolerity
d. hypernatraemia
e. administration of an inhibitor of angiotensin converter enzyme (ACE)

62. Which of the following will be expected in a patient with hypothyroidism?


a. increased oxygen consumption
b. decreased thyroid stimulating hormone
c. heat sensitivity
d. weight loss
e. decreased triiodothyronine T3 level

63. Which of the following results from the action of parathyroid hormone (PTH) on
the renal tubule?
a. inhibition of 1 alpha hydroxylase
b. stimulation of Ca+2reabsorption in the distal tubule
c. decreased urinary excretion of cyclic adenosine monophosphate (AMP)
d. stimulation of phosphate reabsorption in the proximal tubules
e. interaction with receptors on the luminal membrane of the proximal tubular
cells

64. Which of the following causes hyperkalemia?


a. insulin injection
b. decreased serum osmolarity
c. alkalosis
d. treatment with β-agonists
e. metabolic acidosis

65. Which of the following is a cause of metabolic alkalosis


a. chronic renal failure
b. ethyl glycol injection
c. treatment with acetazolamide
d. diarrhea
e. hyperaldosteronism

66. Which of the following is the highest airway resistance?


a. trachea
b. largest bronchi
c. medium sized bronchi
d. smallest bronchi
e. alveoli

67. Which vascular bed does hypoxia cause vasoconstriction?


a. skin
b. muscle
c. cerebral

234 ‫صفحة‬ Dr. MOHAMED YAHIA 0900987639


QUESTION BANK OF Dr. MOHAMED YAHIA

d. pulmonary
e. coronary

68. Which of the following responses is mediated by parasympathetic muscarinic


receptors?
a. dilatation of bronchiolar smooth muscle
b. erection
c. ejaculation
d. constriction of gastro-intestinal sphincters
e. increased cardiac contractility

69. A lesion of the chorda tympani nerve would most likely result in:
a. impaired olfactory function
b. impaired vestibular function
c. impaired auditory function
d. impaired taste function
e. nerve deafness

70. Which autonomic receptor is activated by low concentration of epinephrine


released from adrenal medulla and causes vasodilatation?
a. adrenergic alpha receptors
b. adrenergic β1 receptors
c. cholinergic muscurinic receptors
d. adrenergic β2 receptors
e. cholinergic nicotinic receptors

71. At which site is systolic blood pressure the highest?


a. aorta
b. central vein
c. pulmonary artery
d. right atrium
e. renal artery

72. A patient experienced orthostatic hypotension after sympathectomy. The


explanation for this occurrence:
a. an exaggerated response of the rennin-angiotensin-aldosterone system
b. a suppressed response of the rennin-angiotensin-aldosterone system
c. an exaggerated response of baroreceptors
d. a suppressed response to baroreceptors
e. an increase in parasympathetic outflow

73. In which of the following is pulmonary blood flow greater than aortic blood flow?
a. normal adult
b. fetus
c. left-to-right ventricular shunt

235 ‫صفحة‬ Dr. MOHAMED YAHIA 0900987639


QUESTION BANK OF Dr. MOHAMED YAHIA

d. right-to-left ventricular shunt


e. right ventricular failure

74. Carbon dioxide (Co2) regulates blood to which of the following organs?
a. heart
b. skin
c. brain
d. skeletal muscle at rest
e. skeletal muscle during exercise

75. Propanolol has which of the following effects?


a. decreases heart rate
b. increases heart ventricular ejection fracture
c. increases stroke volume
d. decreases splanchnic vascular resistance
e. decreases cutaneous vascular resistance

76. Cardiac output of the right side of the heart is what percentage of the cardiac
output of the left heart?
a. 25%
b. 50%
c. 75%
d. 100%
e. 125%

77. A shift of oxygen dissociation curve to the right:


a. indicates higher affinity of haemoglobin to oxygen
b. facilitates the release of oxygen to active tissues
c. indicates increase oxygen content at a given PO2
d. is normal in fetal haemoglobin
e. is found in stored blood

78. Characteristics of restrictive lung disease include:


a. TLC is increased
b. FEV1 is reduced
c. FVC is reduced
d. FEV1/FVC ratio is increased
e. Both C & D are correct

79. In a normal standing person:


a. ventilation/perfusion ratio (V/Q) is greater at the lung bases
b. V/Q in the apex is less than 1
c. pulmonary blood flow is equal in the different parts of the lung
d. bronchial blood flow is equal to the cardiac output of the left ventricle
e. wasted perfusion represents arteriovenous shunt and decreases arterial
Po2

236 ‫صفحة‬ Dr. MOHAMED YAHIA 0900987639


QUESTION BANK OF Dr. MOHAMED YAHIA

80. Which of the following is correct:


a. hypoxia causes pulmonary vasoconstriction
b. anemic hypoxia is associated with cyanosis
c. hypoxic hypoxia results from heart failure
d. stagnant hypoxia is corrected by oxygen therapy
e. hypoxic hypoxia may results from morphine poisoning

81. Complete transection of the spinal cord at T1 would most likely result in:
a. loss of stretch reflexes below the lesion
b. temporary loss of coordination proprioception below the lesion
c. permanent loss of voluntary control micturition
d. permanent loss of consciousness above the lesion
e. non of the above

82. Repolarization:
a. is due to active transport K+ out of the cell
b. is partly due to influx of Cl-
c. is usually much quicker than depolarization
d. coincides with the relaxation phase of the muscle twitch in skeletal muscles
e. is due to Ca2+ influx in cardiac muscle

83. The excitatory postsynaptic potential:


a. may be due to efflux of Cl-
b. can be summated to generate an action potential in the dendrites
c. is usually due to Na+ influx
d. necessarily leads to generation of an action potential
e. is also known as the prepotential

84. A skeletal muscle cell requires ATP for the following except:
a. to provide energy for the power stroke of myosin
b. the release of myosin head from actin
c. the reuptake of Ca2+ by the sarcoplasmic reticulum
d. the transport of Na+ to the ECF
e. the release of Ca2+ from the sarcoplasmic reticulum

85. The interaction of the parasympathetic and sympathetic nervous systems in the
control of heart rate:
a. is an example of reciprocal innervations
b. determines the range of control of the heart rate
c. works predominantly through muscarinic receptors
d. enables dual control of ventricular muscle contraction
e. all of the above are correct

237 ‫صفحة‬ Dr. MOHAMED YAHIA 0900987639


QUESTION BANK OF Dr. MOHAMED YAHIA

86. postganglionic parasympathetic and postganglionic sympathetic neurons are


different in that:
a. They can be excited by dopamine
b. Postganglionic parasympathetic nerve bodies are located in ganglia close to
the spinal cord
c. Most postganglionic parasympathetic nerves release the neurotransmitter
acetylcholine.
d. Some postganglionic parasympathetic nerves release the neurotransmitter
norepinephrine.
e. They are most active under conditions of stress, such as the fight/flight

87. Acetylcholine:
a. Is blocked by propanolol.
b. Is blocked by curare in the autonomic ganglion.
c. Is the neurotransmitter at all sites that are blocked by atropine.
d. Is secreted by postganglionic sympathetic to sweat glands.
e. Is degraded by the enzyme COMT.

88. Measurement of intravascular fluid


a. is measured by the indirect dilution method
b. can be done using thiosulphate
c. depends on ability of the substance to pass through the capillary membrane
d. can be achieved by using labeled albumin
e. gives a good indication of water balance

89. Atrophy of the gastric mucosal glands is likely to result in:


a. diminished digestion of starch
b. failure of digestion and absorption of proteins
c. high gastrin levels in the blood
d. high secretin levels in the blood
e. normochromic normocytic anaemia

90. The brush border enzymes of the small intestine digest:


a. Starch
b. cholesterol esters
c. albumin
d. galactose
e. lactose

91. The average daily amount of fluid that passes through the pylorus in a healthy
adult is about:
a. 2 -3litres
b. 9-10 litres
c. 5-6 litres
d. 8-10 litres
e. 10-12 litres

92. The stomach has the ability to absorb:

238 ‫صفحة‬ Dr. MOHAMED YAHIA 0900987639


QUESTION BANK OF Dr. MOHAMED YAHIA

a. Ca2+
b. Alcohol
c. Glucose
d. aromatic aminoacids
e. Fe2+

93. Stimulation of parasympathetic nerves to the heart:


a. Causes tachycardia.
b. Makes the prepotential more horizontal.
c. Decreases the rate of potassium efflux in the sinu-atrial node.
d. Prolongs the refractory period.
e. Shortens the duration of the cardiac cycle.

94. AV nodal cells:


a. exhibit action potentials characterized by rapid depolarization (phase 0)
b. conduct impulses more slowly than either atrial or ventricular cells
c. are capable of pacemaker activity at an intrinsic rate of 100 beats per
min.
d. exhibit increased permeability to Na during the pre-potential
e. show a steep pre-potential when exposed to acetylcholine

95. Starlings Law of the Heart (heterometricautoregulation):


a. states that, at a given end-diastolic pressure, norepinephrine increases volume
b. states that increased end-systolic volume leads to increased stroke volume (up
to a point)
c. is primarily the result of changes in the firing rate of sympathetic nerves to
the ventricles
d. is the mechanism that keeps left and right cardiac output equal in the
steady state
e. describes the myocardial response to an increased heart rate

96. Which of the following is most likely to cause postural hypotension?


a. a drug that blocks muscarinic cholinergic receptors
b. decreased firing rate of baroreceptors while standing
c. exposure to cold environment
d. a drug that blocks cholinergic receptors in skeletal muscle vessels
e. a drug that blocks cholinergic receptors in autonomic ganglia

97. Coronary blood flow to the right ventricle:


a. is mainly regulated by sympathetic supply to the coronaryanterioles
b. increases when sympathetic nerves to the heart are blocked
c. occurs mainly during diasystole
d. increases when myocardial metabolism increases
e. depends on myogenic autoregulation

98. Concerning the heart sounds:


a. The first heart sound is due to closure of the semilunar valves.
b. The second heart sound is due to opening of the aortic and pulmonary valves.
c. The third heart sound is due rapid ejection phase of systole.

239 ‫صفحة‬ Dr. MOHAMED YAHIA 0900987639


QUESTION BANK OF Dr. MOHAMED YAHIA

d. The first heart sound occurs at the beginning of the isovolumic


contraction phase
e. The second heart sound follows the isovolumic relaxation phase.

99. Tripling the resting heart rate of a healthy young adult during heavy exercise:
a. is achieved by activation of the sympathetic nerves
b. a decrease in cholinergic discharge at the sinoatrial node
c. occurs as a result of withdrawal of normal parasympathetic activity
d. occurs as a result of the Bainbridge reflex
e. is primarily due to an increase in venous returns

100. Correct sequences of steps in the short-term compensation for hemorrhage


include:
a. decreased arterial pressure → increased baroreceptor firing rate
b. increased formation of angiotensin II → increased renin release from kidneys
c. decreased excretion of Na+ and water → increased aldosterone formation
d. decreased firing of baroreceptors → increased sympathetic activity
e. decreased atrial volume → ANP release

101. All the following is true about control of secretion of TSH, except:
a. It is inhibited by T4
b. It is increased in cold weather
c. It is increased in Grave’s disease
d. It is increased by TRH
e. Has no marked diurnal rhythm

102. A most effective method of treating type I diabetes mellitus is:


a. To stop consuming carbohydrates
b. Regular muscular exercise
c. Reduction of body weight
d. Drugs to stimulate the B cells of the pancreas
e. Daily injections of insulin

103. Which of the following factors has a directly stimulates the secretion of
aldosterone from adrenal corex:
a. angiotensin I
b. hypotension
c. hyperkalemia
d. hyponatraemia
e. ACTH

104. The onset of puberty in females


a. depends on regular menstrual cycles
b. is triggered by Melatonin
c. is a not related hypothalamic functions
d. results in regular ovulation
e. is associated with an increased release of adrenal androgen

105. The duration of the menstrual cycle

240 ‫صفحة‬ Dr. MOHAMED YAHIA 0900987639


QUESTION BANK OF Dr. MOHAMED YAHIA

a. is determined by LH
b. is longer in young women
c. is dependent on the duration of the proliferative phase
d. depends on the life span of the corpus luteum
e. decreases with age

106. Hormonal changes during pregnancy include


a. an increase in oestradiol more than oestrone
b. an increase in progesterone which continues throughout pregnancy
c. inhibition of pituitary growth hormone
d. a late increase in HCG
e. a surge of placental lactogen just before term

107. The actions of estrogens include


a. growth of duct system of breast
b. decreased motility of uterine tubes
c. increased secreations of the endometrium
d. decreased excitability of myometrium during pregnancy
e. decreased sodium retention

108. Which of the following is true about renal blood flow


a. The kidneys receive 10- 15% of cardiac output
b. Is markedly decreased during muscular exercise
c. Measured by renal clearance of inulin
d. The dilution method is used for its measurement
e. Most of it goes to renal cortex

109. The forces responsible for increasing glomerular filtration include


a. Capillary Colloid Osmotic Pessure
b. hydrostatic pressure in glomerular capsule
c. hydrostatic pressure in glomerular capillaries
d. Increased intracapsular hydrostatic pressure
e. increased blood volume

110. GFR is measured by clearance of a substance which


a. is not metabolized
b. must be secreted by renal tubules
c. Rapidly metabolized
d. Has molecular size less than 12 nm
e. Is filtered & not secreted or reabsorbed

111. Facilitated diffusion


a. requires energy expenditure
b. is a form of active transport
c. transport by a protein carrier down concentration gradient
d. diffusion down concentration gradient
e. faster than primary active transport

112. Active reabsorption of Na in proximal tubules results in


a. paracellular diffusion of potassium ions

241 ‫صفحة‬ Dr. MOHAMED YAHIA 0900987639


QUESTION BANK OF Dr. MOHAMED YAHIA

b. removal of most of the filtered Na


c. passive trans-cellular diffusion of water
d. secondary active transport of potassium
e. lower concentration of Cl in lumen than interstitial fluid

113. the characteristics of the loop of Henle include


a. Descending limb is impermeable to water
b. Urine becomes more dilute as it reaches the tip of loop
c. Ascending limb is permeable to water
d. Thin descending limb is permeable to urea which diffuses to ISF
e. Na Cl diffuses passively out of thin part of ascending limb

114. The following is true about the vasa recta


a. Descending vasa recta reabsorb water from ISF
b. the ascending capillaries gain Na & urea
c. plasma becomes hyperosmotic as it descends in vasa recta
d. ascending capillaries reabsorb & remove excess sodium
e. they are responsible for countercurrent multiplier system

115. About Bicarbonate reabsorption


a. represents the high capacity high gradient system
b. filtered bicarbonate/day = 2000 – 3000 mmol/day
c. more HCO3 is absorbed in distal than proximal tubules
d. Proximal tubules apical membrane impermeable to HCO ³¯
e. H2CO3 in pCT lumen is catalyzed at brush border by carbonic anhydrase
to give CO2 & HCO ³¯

116. Ammonium production


a. is mainly a function of PCT
b. source is mainly glycine
c. represents the least important mechanism in renal excretion of H+
d. is a high capacity low gradient system
e. is increased when the urine pH is high

117. The following is true about the generator potential of a sensory receptor
a. it is immediately propagated
b. cannot be summated
c. is a form of local potential
d. obeys the all or non law
e. is transmitted to the axon

118. The dorsal column tracts are also called


a. lemniscal tracts
b. anterolateralpathway
c. spino-olivary tracts
d. reticulospinal tracts
e. cerebellospinal tracts

119. Adaptation to sensory modalities


a. can be explained by fatigue

242 ‫صفحة‬ Dr. MOHAMED YAHIA 0900987639


QUESTION BANK OF Dr. MOHAMED YAHIA

b. occurs at the same rate in different receptors


c. is due to inactivation of Na channels
d. causes release of inhibitory transmitter
e. is defined as decreased firing of receptors when a stimulus is increased

120. The localization of sensation is a function of


a. adequate stimulus
b. specific receptors
c. medullary centres
d. primary sensory cortex
e. hypothalamus

121. The intensity (strength) of a stimulus is coded by


a. the type of receptor
b. the voltage of the generator potentials
c. the amplitude if the action potential
d. columns of cells in sensory cortex where it arrives
e. the frequency of impulses

122. The static stretch reflex


a. can be elicited by by tapping the muscle tendon
b. is stimulated by gravity
c. is mainly due to vestibular stimuli
d. causes muscle contraction
e. causes a form of isotonic contraction

123. Lower motor neuron lesions


a. result in exaggerated superficial reflexes
b. are not accompanied by sensory loss
c. are associated with minimal muscle wasting
d. are complications of diabetes mellitus
e. of the facial nerve lead to exaggerated labial fold on the affected side

124. Apraxia (loss of skilled movement) may be caused by


a. hemisection of the spinal cord
b. lesion of sensory cortex
c. upper motor neuron disease
d. a lesion of primary motor cortex
e. disease of the inner ear

125. One of the following does no result from diarrhea:


a. Metabolic alkalosis
b. Increased [H+] in arterial plasma
c. Increased re-absorption of HCO3
d. Increased titrable acidity

243 ‫صفحة‬ Dr. MOHAMED YAHIA 0900987639


QUESTION BANK OF Dr. MOHAMED YAHIA

e. Decreased PCO2 in arterial plasma

126. Regarding erythropoiesis: One of the following is incorrect:


a. Is formation of red blood cells
b. Is stimulated by erythropoiten
c. Is stimulated by hypoxia
d. Occurs outside the bone narrow
e. Is enhanced by haemolysis

127. Alveolar air: One of the following statements is true:


a. Is rich in O2 compared with atmospheric air
b. Is similar in all alveoli
c. Exchange with atmospheric air is more with shallow breathing
d. O2 decrease leads to hypoxic hypoxia
e. PCO2 is increased by progesterone

128. In the physiology of the heart: One of the following is not true:
a. Total de-enervation of the heart result in heart rate of 105 beats/min.
b. Vagal stimulation decreases the heart rate
c. The A.V. node is innervated by the Rt.vagus nerve
d. Stimulation of B-adrenergic receptors has a positive inotropic effect.
e. The resting membrane potential of the S.A node is –50 nv

129. Regarding the anti-diuretic hormone ADH: One of the following is not true:
a. Is stored in the anterior pituitary
b. Action is on the distal and collecting tubules
c. Deficiency causes diabetes insipidus
d. Is produced by the posterior pituitary
e. Has a vasopressor effect

130. In isotonic muscle contraction a muscle:


a. does not shorten but develops tension
b. shortens and performs work
c. lengthens as it contacts under load
d. does not shorten perform no work
e. lengthens and spends energy

131. The following hormones are secreted by the adenohypophysis except:


a. prolactin
b. leuitinizing releasing hormone
c. gonadotrophin releasing hormone
d. thyroid stimulating hormone
e. growth hormone

132. Prolactin:
a. is a glycoprotein
b. causes milk ejection
c. is mainly controlled by inhibitory hormone from hypothalamus
d. causes milk formation during pregnancy
e. estimulate secretion of oestrogen from the ovaries

244 ‫صفحة‬ Dr. MOHAMED YAHIA 0900987639


QUESTION BANK OF Dr. MOHAMED YAHIA

133. Growth hormone:


a. secretion is increased after meals
b. effects on bone is brought about by somatomedins
c. deficiency in adults results in obesity
d. blood level is decreased during exercise
e. secretion is stimulated during REM sleep

134. The increased excitability of nerve and muscle associated with plasma calcium
is due to:
a. increased entry of calcium into synaptic knob at neuromuscular junction
b. increased release of acetylcholine into synaptic junction
c. decreased potassium efflux
d. increased entry of sodium through voltage gated sodium channels
e. opening of chemically gated channels for sodium

135. Parathyroid hormone:


a. is secreted by para-follicular cells of thyroid gland
b. is controlled by a hypothalamic hormone
c. secretion is stimulated by 1.25 dihydroxycholecalciferol
d. increases activity of osteoclasts
e. stimulates calcium reabsorption in proximal convoluted tubule of the kidney

136. The most important function of 1,25 dihydroxycholecalciferol is:


a. inhibition of parathyroid hormone secretion
b. absorption of calcium from small intestine
c. bone resorption
d. reabsorption of phosphate in renal tubule
e. inhibition of calcitonin secretion

137. The hypoglycemic action of insulin is mainly due to:


a. inhibition of gluconeogenesis
b. inhibition of glycogenolysis
c. inhibition of growth hormone secretion
d. stimulation of glucose transporters
e. inhibition of cortisol secretion

138. Insulin stimulates:


a. hormone sensitive lipase
b. lipoprotein lipase
c. gluconeogenesis
d. potassium exit from cells
e. erythropoiesis

139. In the absence of insulin patients suffer from:


a. hypoglycemia

245 ‫صفحة‬ Dr. MOHAMED YAHIA 0900987639


QUESTION BANK OF Dr. MOHAMED YAHIA

b. hypertension
c. hypokalaemia
d. metabolic acidosis
e. respiratory alkalosis

140. The myenteric plexus of the enteric nervous system includes:


a. Somatic sensory neurons
b. Inhibitory interneurons
c. Presynaptic motor neurons
d. excitatory sympathetic neurons
e. sympathetic postganglionic neurons

141. The pharyngeal phase of swallowing:


a. is partially voluntary
b. includes opening of the upper esophageal sphincter
c. includes the peristaltic activity of the upper third of the oesophagus
d. is associated with opening of the glottis
e. is preceded by deep inspiration

142. The secretion of enzyme by the exocrine pancreas is increased by:


a. inhibitors of gastrin hormone
b. sympathetic stimulation
c. secretin
d. CCK
e. noradrenaline

143. Contraction of the gallbladder is mainly mediated by:


a. secretin
b. VIP
c. bile salts
d. atropine
e. CCK

144. The jejunum is the main site for absorption of the following except:
a. glucose
b. amino acids
c. fatty acids
d. bile salts
e. water

145. The movements of the colon include:


a. migrating motor complex slow wave movement
b. pendular movement
c. enterocolic reflux
d. segmentation

146. Central stimuli to vomiting include:


a. irritation of the colon
b. diseases of the inner ear
c. microbial toxins in the blood

246 ‫صفحة‬ Dr. MOHAMED YAHIA 0900987639


QUESTION BANK OF Dr. MOHAMED YAHIA

d. severe exercise
e. cerebellar lesions

147. The functions of human stomach include:


a. absorption of iron
b. absorption of vitamin B12
c. absorption of 50% of water intake
d. storage of food for 4 hours
e. limited digestion of proteins

148. Pancreatic proenzymes activation is stimulated by:


a. CCK
b. secretin
c. trypsinogen
d. gastrin
e. enteropeptidase

149. Micelles:
a. are emulsified fat globules in the duodenum
b. are packets of pro-enzymes found in pancreatic acinar cells.
c. are dead enterocytes leaving the tips of villi.
d. are micro structures of cholesterol and fatty acids coated with bile salts
e. are the triglyceride particles formed by enterocytes and transported in the
lymphatic system.

150. The following cells produce antibodies:


a. helper T lymphocytes
b. cytotoxic T lymphocytes
c. monocytes
d. plasma cells
e. eosinophils

151. Platelets:
a. normal count is about 4000/mm3
b. have a life span of 30 days
c. contain clotting factors
d. secret prostacyclin
e. has receptors for protein C

152. Concerning plasma:


a. forms 40% of the blood volume
b. forms 8% of the body volume
c. has the same composition as serum
d. fibrinogen is the main plasma protein
e. 90% of the weight is water

153. Neutrophils:
a. contain heparin

247 ‫صفحة‬ Dr. MOHAMED YAHIA 0900987639


QUESTION BANK OF Dr. MOHAMED YAHIA

b. have kidney shape nucleus


c. have the granules
d. are transformed into macrophages in the tissues
e. circulate in the blood for few hours

154. All of the following may be a complication of blood transfusion EXCEPT:


a. sodium overload
b. potassium overload
c. iron overload
d. volume overload
e. allergic reactions

155. Iron is transported in the blood bound to:


a. transferrin
b. ferritin
c. hemosiderin
d. intrinsic factors
e. all of the above

156. reticulocyte count in normal blood is:


a. 10%
b. 7%
c. 5%
d. 3%
e. 1%

157. The following cell produce heparin:


a. neutrophil
b. eosinophil
c. basophil
d. lymphocyte
e. monocyte

158. causes of hypochromic anemia include:


a. removal of the ileum
b. iron deficiency
c. folic acid deficiency
d. sickle cell anemia
e. B12 deficiency

159. Hemoglobin A1C:


a. has a half life of fewer days
b. has iron in the ferric form
c. is low in folic acid deficiency
d. is an indicator of diabetes mellitus control

248 ‫صفحة‬ Dr. MOHAMED YAHIA 0900987639


QUESTION BANK OF Dr. MOHAMED YAHIA

e. is not found in normal subjects

160. Saltatory conduction in nerve fibers:


a. is fast in type B compared to type A fibers
b. occurs in non myelinated fibers
c. is inversely related to the diameter of the nerve fiber
d. is slow in demyelinating neuropathies
e. is a measure of axonal damage

161. Excitatory postsynaptic potential:


a. is a form of action potential
b. is all or none
c. is subject to summation
d. its value reaches – 80mV
e. is propagated

162. The sliding theory of skeletal muscle contraction:


a. myosin slides on actin
b. energy is only required for detachment of the head of the cross bridge
c. tropomyosin functions to relax the muscle
d. myosin binds calcium ions
e. the cross bridge stems from the body of actin

163. All of the following are monosynaptic reflexes EXCEPT the:


a. knee jerk
b. ankle jerk
c. biceps jerk
d. supinator jerk
e. abdominal reflex

164. Signs of cerebellar dysfunction include:


a. dysmeteria
b. clonus
c. hypertonia
d. shuffling gait
e. muscle weakness

165. Alpha waves in the EEG:


a. ranges between 8-14Hz
b. are best seen over the parietal cortex
c. decrease on opening the eyes
d. are epileptiform
e. arise from the hypothalamus

166. Homonymous hemianopia:


a. is caused by pituitary tumours
b. follows optic nerve lesion
c. might follow temporal lobe lesion
d. is caused by visual cortical damage

249 ‫صفحة‬ Dr. MOHAMED YAHIA 0900987639


QUESTION BANK OF Dr. MOHAMED YAHIA

e. is best tested by confrontational method

167. Conductive deafness:


a. is caused by 8th cranial nerve lesion
b. results in a positive Rennie's test
c. follows otosclerosis
d. is caused by a lesion in the medulla
e. is diagnosed by brain stem evoked potential studies

168. Sharp pain:


a. is transmitted by type B fibers
b. usually occurs in visceral organs
c. is localized
d. is transmitted by the posterior column tract
e. does not respond to analgesic medications

169. Vibration sense:


a. is carried by the anterolateral spinothalamic tract
b. might be lost in vitamin B12 deficiency
c. its sensory perception occurs in the thalamus
d. is transmitted by type C fibers
e. its perception is not affected by age

170. Primary memory:


a. its recall last for minutes or hours
b. the neurophysiology behind it is explained on reverberation circuit
c. its loss is delayed in head injuries
d. its centre lies in the parietal lobe
e. is lost early in Parkinson's disease

171. Botulinum toxin:


a. is a competitive blocking agent at the neuromuscular junction
b. stimulates muscle contraction
c. prevents acetylcholine release from the presynaptic endings
d. is useful in treating myasthenia gravis
e. depress the respiratory centre

172. Stroke volume is decreased when:


a. the sympathetic nerves are stimulated
b. the arterial blood pressure falls
c. vagal centres are stimulated
d. the end diastolic volume is increased
e. a patient stands up

173. Stroke volume:


a. normal about 90 ml
b. depends on Bainbridge reflex
c. decreases when the heart rate increase
d. increases when the end diastolic volume increase

250 ‫صفحة‬ Dr. MOHAMED YAHIA 0900987639


QUESTION BANK OF Dr. MOHAMED YAHIA

e. is mainly controlled by vagal fibers

174. Starlings law of the heart describes the relationship between:


a. the heart rate and stroke volume
b. the and diastolic volume and cardiac output
c. the blood pressure and heart rate
d. the initial length of cardiac muscle fibers and force of contraction
e. sympathetic stimulation and stroke volume

175. The first heart sound:


a. is due to vibration in the aortic on closure of semi lunar valves
b. is caused by closure of tricuspid valve
c. occurs at the end of isometric contraction of the ventricles
d. is caused by closure of both atrioventricular valves
e. is short of high pitch

176. The chemoreceptors are found in:


a. the lungs
b. carotid sinus
c. aortic arch
d. coronary sinus
e. medulla

177. During the cardiac cycle, closure of the aortic valve occurs at:
a. the end of the isovolumetric contraction
b. the beginning of rapid ejection phase
c. the beginning of isometric relaxation
d. the end of systole
e. the end of rapid filling phase

178. Cardiac muscle leas a long refractory period because:


a. it has more concentration of protein
b. of CI- influx
c. of the delay in Na+ influx
d. of Ca+2influx
e. it is more permeable to K+

179. In the ECG the p wave denotes:


a. atrial contraction
b. atrial depolarization
c. ventricular repolarization
d. SA node depolarization
e. ventricular contraction

180. Regurgitation of the aortic valve leads to:


a. a decrease in diastolic pressure
b. a decrease in O2 consumption
c. a decrease in heart rate
d. a systolic murmur
e. decreased end-systolic volume

251 ‫صفحة‬ Dr. MOHAMED YAHIA 0900987639


QUESTION BANK OF Dr. MOHAMED YAHIA

181. Cardiac output is affected by:


a. stroke volume only
b. heart rate and stroke volume
c. blood volume
d. venous return
e. increased Na+ efflux

182. Barorceptors are located in:


a. coronary sinuses
b. carotid bodies
c. aortic arch
d. A and B
e. B and C

183. A patient in the A& E department with profuse hemorrhage from a severed
limb artery will have increased:
a. sodium excretion
b. sympathetic nerve activity
c. vagal activity
d. arteriolar diameter in skin
e. water excretion

184. Hemodynamic changes in response to obstruction of venous return to the right


side of heart include:
a. cardiac output will fall and systemic arterial blood pressure will fall
b. cardiac output will rise and systemic arterial blood pressure will rise
c. cardiac output will fall and systemic arterial blood pressure will rise
d. cardiac output will rise and systemic arterial blood pressure will fall
e. cardiac output will remain unchanged and systemic arterial blood pressure will
fall

185. Which of the following statements about stroke volume is true?


a. it is the difference between the ventricular end –systolic volume and
ventricular end diastolic volume
b. it is 120 ml in an adult weighting 70kg
c. it is decreased by sympathetic activation of the heart
d. it is increased by increased venous return
e. it is increased by systemic hypertension

186. In a resting healthy individual the ejection fraction is:


a. 20%
b. 30%
c. 45%
d. >60%
e. 90%

187. The venous return:


a. if increased will increase end systolic volume
b. activates Bainbridge reflex when venous return is increased

252 ‫صفحة‬ Dr. MOHAMED YAHIA 0900987639


QUESTION BANK OF Dr. MOHAMED YAHIA

c. decrease with deep inspiration


d. increase on standing
e. decrease by venoconstriction

188. Moderate exercise will decrease the following:


a. heart rate
b. cardiac output
c. pulse pressure
d. total peripheral resistance (TPR)
e. arterio venous O2 difference

189. Blood pressure is affected by:


a. heart rate and stroke volume
b. blood vessels
c. psychological state
d. venous return
e. all of the above

190. Inferior wall MI will be picked by which of the following ECG leads:
a. V1 – V3
b. II, III, avf
c. RV4, RV5
d. I, AVL, V4, V6
e. V1 – V6

191. The central venous pressure (CVP) increases:


a. in hypovollemia
b. during hot climate
c. when total peripheral resistance increases
d. when cardiac output decreases
e. with increased sodium loss

192. Glomerular filtration rate is increased by:


a. Relaxation of mesangial cells
b. angiotensin II
c. vasopressin
d. sympathetic stimulation
e. decreased renal blood flow

193. Renin secretion by the kidney is increased by increasing:


a. tubular fluid flow rate
b. mean systemic blood pressure
c. atrial natriuretic peptide
d. renal sympathetic nerve activity
e. discharge of baroreceptors

253 ‫صفحة‬ Dr. MOHAMED YAHIA 0900987639


QUESTION BANK OF Dr. MOHAMED YAHIA

194. The transport of glucose across the luminal membrane in the proximal part of
the renal tubules:
a. is through insulin dependent glucose transporters
b. cannot take place against concentration gradient
c. is not affected by Na+ -K+ pump inhibitors
d. is a co-transport with sodium
e. is decreased in diabetes mellitus

195. In the proximal tubules:


a. 20% of filtered amino acid is reabsorbed
b. sodium reabsorption is aldosterone dependant
c. equal percentage of sodium and water is reabsorbed
d. potassium is mainly secreted
e. calcium cannot be reabsorbed

196. Concerning acid-based balance:


a. the plasma proteins and hemoglobin represent the major extracellular buffer
b. the normal range for plasma pH is 7.26 – 7.74
c. the normal range for plasma bicarbonate is 30-34 mmol/L
d. most of the hydrogen secreted is buffered by bicarbonate
e. most of the hydrogen ions that are excreted in the urine are secreted by the
proximal tubule

197. With regard to the glomerular capillaries:


a. they are more permeable to albumin in comparison with other capillaries in the
body
b. the oncotic pressure of the plasma leaving them is lower than the general
systemic oncotic pressure
c. afferent arteriole constriction leads to a decrease in the capillary blood
pressure
d. the plasma glucose concentration in them is more than in the glomerular
filtrate
e. as blood flows through them the hydrostatic pressure drops sharply

198. Sodium re-absorption in renal tubules:


a. is a passive process in all parts of the tubule
b. is increased when angiotensin II is decreased
c. is stimulated by atrial natriuretic peptide (ANP)
d. depends on K+ level in tubular fluid
e. is coupled to glucose reabsorption in proximal tubule

199. Which of the following is true about micturition:


a. it is controlled by a spinal reflex
b. it is initiated by impulses from the motor cortex
c. parasympathetic stimulation to the bladder leads to relaxation of detrusor
muscle
d. the external urethral sphincter is under sympathetic control
e. the micturition reflex is inhibitied by increased sympathetic stimulation

254 ‫صفحة‬ Dr. MOHAMED YAHIA 0900987639


QUESTION BANK OF Dr. MOHAMED YAHIA

200. The plasma concentration at which a particular substance begins to appear in


the urine is the:
a. transport maximum (Tm)
b. fractional excretion
c. filtered load
d. renal threshold
e. filtration point
201. The plasma glucose concentration in a fasting patient is found to be 400 mg/dl.
If the GFR is 100 ml/min and the renal transport maximum for glucose in this
patient is 300 mg/min, at what rate is glucose being excreted in the urine?
a. 0 mg/min
b. 100 mg/min
c. 200mg/min
d. 300 mg/min
e. 450 mg/min

202. When the kidney concentrates the urine:


a. the fluid leaving the proximal tubule is hypo-osmotic with plasma
b. the fluid leaving the loop of Henle is hyperosmotic
c. the fluid leaving the cortical portion of the collecting duct is hypo-osmotic
d. the osmolarity of the renal medulla is high
e. the urea concentration of the fluid increases as it passes through the cortical
section of the collecting duct

203. The principal cells of the collecting duct:


a. passively secretes ammonia
b. is the primary site for the re-absorption of water
c. becomes more permeable to water in the presence of ADH
d. passively reabsorbs potassium ions
e. the activity of the sodium/potassium pump in the epithelium is inhibited by
aldosterone

204. Increased sympathetic discharge in the renal nerves will result in:
a. dilatation of the afferent and constriction of the efferent arterioles
b. decreased secretion of renin
c. decreased renal blood flow (RBF)
d. an increase in the GFR
e. renal excretion of sodium and water

205. The actions of angiotensin II include all the following except:


a. peripheral vasodilatation
b. stimulation of thirst
c. inhibition of aldosterone secretion
d. stimulation of ADH secretion
e. stimulation of renin secretion

206. The effects of aldosterone on the kidney include:


a. renal retention of sodium
b. renal retention of potassium
c. decreased excretion of hydrogen ions in the urine

255 ‫صفحة‬ Dr. MOHAMED YAHIA 0900987639


QUESTION BANK OF Dr. MOHAMED YAHIA

d. a decrease in the plasma bicarbonate concentration


e. an increase in the secretion of renin

207. Concerning the pituitary-hypothalamic relationship:


a. trophic hormones reach the pituitary through axons
b. antidiuretic hormone is synthesized in nerve ending of posterior pituitary
c. corticotrophin hormone reaches the pituitary through portal circulation
d. supraoptic nucleus of the hypothalamus forms mainly oxytocin
e. vasopressin is secreted by stimulating posterior pituitary

208. The following hormones bind to receptors on cell membrane:


a. glucagon
b. cortisol
c. 1.25 dihydroxycholecalciferol
d. T3
e. testosterone

209. The receptors for the following hormones are present in the cell membrane
except:
a. growth hormone
b. aldosterone
c. insulin
d. thyroid stimulating hormone
e. catecholamines

210. Acromegaly:
a. is due to excess growth hormone secretion in children
b. may result in diabetes mellitus
c. is characterized by hypertension
d. results in taller individuals
e. may result from increased number of growth hormone receptors

211. The most important function of ADH is:


a. vasoconstriction
b. decrease in renal medulla blood flow
c. reabsorption of urea in collecting ducts
d. water reabsorption in collecting ducts
e. neurotransmission in the brain

212. Absorption of calcium from GIT:


a. occurs mainly in ileum
b. is significantly greater in adults than in children
c. is significantly decreased by calcitonin
d. is mainly controlled by 1.25 dihydroycholecalciferol
e. requires bile acids

213. Plasma calcium:


a. Varies directly with plasma phosphate

256 ‫صفحة‬ Dr. MOHAMED YAHIA 0900987639


QUESTION BANK OF Dr. MOHAMED YAHIA

b. 50% of it is present in the ionized form


c. 40% of it is bound to plasma globulin
d. Is a major contributor to plasma osmolality
e. Increase results in Tetany

214. The physiological significance of amino acids stimulating both insulin and
glucagon secretion is:
a. entry of amino acids into cells
b. protein synthesis
c. avoiding hypokalaemia
d. avoiding hypoglycemia
e. gluconeogenesis in liver

215. Insulin secretion is best stimulated by:


a. gastric inhibitory peptide
b. growth hormone
c. glucagon like peptide
d. glucagon
e. thyroid hormone

216. Insulin secretion is inhibited by:


a. alpha adrenergic receptor stimulation
b. vagal stimulation
c. amino acids
d. GIP
e. parathyroid hormone

217. Partial pressure of oxygen is less in alveoli compared to atmosphere due to:
a. high partial pressure of nitrogen.
b. low partial pressure of CO2.
c. presence of carbon monoxide.
d. the dead space.
e. water vapor.

218. In a patient with shortness of breath after a chest stab wound the following can
be expected
a. FEV1 is high.
b. FVC is low.
c. PEFR is normal.
d. Total lung capacity is high.
e. Anatomical dead space is low.

219. Physiological dead space is


a. more than the anatomical one.
b. 150 ml in adults
c. involved in gas exchange
d. not related to lung perfusion
e. measured by the computerized spirometer.

257 ‫صفحة‬ Dr. MOHAMED YAHIA 0900987639


QUESTION BANK OF Dr. MOHAMED YAHIA

220. Compliance of the lung is increased in


a. Hemothorax
b. pulmonary embolism.
c. pulmonary edema.
d. lung fibrosis
e. emphysema.

221. In high altitude


a. Alveolar PO2 is high.
b. Atmospheric O2% is low.
c. Arterial PCO2 is low.
d. Arterial PO2 is normal.
e. Alveolar PCO2 is high.

222. In a diabetic patient the pH was found to be 7.3, PaCO2 was found to be 46
mmHg and his HCO3- normal, fasting blood sugernorma; he is suffering from
a. respiratory alkalosis.
b. metabolic alkalosis.
c. metabolic acidosis.
d. diabetic ketoacidosis.
e. respiratory acidosis.

223. Pneumonectomy is expected to reduce the vital capacity by


a. 15%
b. 20%
c. 33%
d. 40%
e. 50%

224. Hb affinity to oxygen is decreased in


a. high pH
b. high PCO2
c. low temperature.
d. low 2,3 DPG
e. fetalHb.

225. The Dorsal Respiratory Group


a. has mainly expiratory cells.
b. Is the rythmicity center
c. Is located in the pons.
d. Produces inspiration
e. Is part of the vasomotor center.

226. Lung stretch receptors are involved in


a. Proprioception.
b. Hering Breuer reflex.
c. Cough reflex.

258 ‫صفحة‬ Dr. MOHAMED YAHIA 0900987639


QUESTION BANK OF Dr. MOHAMED YAHIA

d. Stimulation of chemoreceptors.
e. Voluntary respiration.

227. In left sided pleural effusion the intrapleural pressure in that side will be
a. -2.5 mmHg at quiet expiration.
b. More than that in the right side.
c. -6 mmHg in quiet inspiration.
d. Can reach -30 in forced inspiration.
e. More effective in lung expansion.

228. Respiratory muscles do not fatigue due to alternation of contraction between:


a. inspiratory and expiratory muscles.
b. diaphragm and internal intercostals.
c. internal and external intercostals.
d. diaphragm and external intercostals.
e. the fibers of inspiratory muscles.

229. Bronchodilation can be produced in response to


a. leukotrienes.
b. VIP.
c. Cortisol.
d. Cool air.
e. Exercise.

230. The base of the lung compared to the apex


a. has higher ventilation perfusion ratio
b. has more perfusion but less ventilation.
c. has more ventilation but less perfusion.
d. has more perfusion and ventilation.
e. has similar ventilation and perfusion.

231. 2,3 DFG (diphosphglycerate)


a. increases in high altitude.
b. increases in stored blood.
c. decreases in fetalHb.
d. decreases in alkalosis.
e. decreases in exercise.

232. A patient showed the following findings in pulmonary function tests:


FEV1=2L, FVC=4L, FEV1 after salbutamol inhaler=2.2L, his diagnosis could be:
a. Chronic bronchitis.
b. Lung fibrosis.
c. Pulmonary embolism.
d. Asthma.
e. Restrictive lung disease.

259 ‫صفحة‬ Dr. MOHAMED YAHIA 0900987639


QUESTION BANK OF Dr. MOHAMED YAHIA

233. Surfactant is decreased


a. By cortisol.
b. in term babies.
c. during 24% oxygen therapy.
d. in smokers.
e. in heart failure.

234. In hypoxia the glomus cells of the carotid bodies are stimulated due to:
a. increased calcium.
b. increased dopamine.
c. closure of potassium channels
d. exocytosis of granules.
e. depolarization of nerve fibres.

235. In bone:
a. Osteoclasts are thought to be responsible for bone resorption.
b. A normal calcium content depends on mechanical stress being applied to the
bone
c. The width of the epighyseal plate is an indication of the rate of growth.
d. Strontium ions can replace some of the calcium ions
e. All of the above are true.

236. Non-myelinated fiber differ from myelinated in that


a. Lack of node of Ranvier
b. Are more excitable
c. Have higher conduction velocity
d. Are not capable of regeneration
e. Are not associated with Schwann cells

237. A skeletal muscle cell:


a. Obey all or none phenomenon
b. Becomes more excitable when its membrane becomes hypopolarised
c. Has a resting membrane potential which is negative inside compared to
outside
d. Contain intracellular store of Ca+ +
e. All of the above are true.

238. Which of the following is formed by osteoblast prior to formation of bone


osteoid
a. Mineralization inducing enzymes
b. Mucopolysaccharides
c. Collagen
d. Alkaline phosphates
e. Non of the above

260 ‫صفحة‬ Dr. MOHAMED YAHIA 0900987639


QUESTION BANK OF Dr. MOHAMED YAHIA

239. Growth maturation is dependent upon the balance between:


a. Thyroid and anterior pituitary hormones
b. Estrogen and testosterone
c. Corticotrophins and testosterone
d. Thyroid and posterior pituitary hormones
e. All of the above

240. During spinal shock:


a. All motor reflexes disappear
b. All motor and autonomic reflexes disappear
c. The motor reflexes disappear but the autonomic enhanced
d. The extensor reflexes disappear , flexors are enhanced
e. All reflexes are unchanged only BP fall

241. A patient shows involuntary athetoid movement, he probably has:


a. lesion in the cerebral cortex
b. Hypothalamic lesion
c. Cerebellar lesion
d. lesion in the basal ganglia
e. Thalamic lesion

242. The most important factor regulating blood flow through exercising muscles
is:
a. Systemic blood pressure
b. Venous tone
c. Vasodilator metabolites
d. Sympathetic over activity
e. Parasympathetic control

243. The morphological layer of the articular cartilage most responsible for the
resistance to shear stress is:
a. Superficial zone
b. middle(transitional) zone
c. Radial zone
d. Tide mark
e. Zone of calcified cartilage

244. Muscle tone is:


a. Reduced by curare like drugs
b. Is increased in decerebrate man
c. Decreased in cerebellar lesion
d. Is redistributed when the function of semicircular canals one one side is
disturbed by disease
e. All of the above

261 ‫صفحة‬ Dr. MOHAMED YAHIA 0900987639


QUESTION BANK OF Dr. MOHAMED YAHIA

245. Folic acid:


a. is a fat soluble vitamin
b. is found only in meats
c. is needed for DNA formation
d. deficiency causes microcytic anaemia
e. deficiency causes defect in myelin formation

246. Micelle formation is necessary for the intestinal absorption of:


a. Glycerol
b. Galactose
c. Leucine
d. Bile acid
e. Vit D

247. Which one of the following is the site of Na+-bile cotransport:


a. Gastric antrum
b. Gastric fundus
c. Duodenum
d. Ileum
e. Colon

248. Which one of the following substances inhibits gastric empting:


a. Secretin
b. Gastrin
c. CCK
d. Vasoactive intestinal peptide (VIP)
e. Gastric inhibitory peptide (GIP)

249. Which one of the following substances is secreted in response to an oral


glucose load:
a. Secretin
b. Gastrin
c. CCK
d. Vasoactive intestinal peptide (VIP)
e. Gastric inhibitory peptide (GIP)

250. Elastic work of breathing:


a. Is due to airway resistance
b. Is less than the non-elastic work
c. Represents about 65% of the total work
d. Is dependent on the viscosity
e. Occurs due to the negative intra-pleural pressure

CHAPTER 2
262 ‫صفحة‬ Dr. MOHAMED YAHIA 0900987639
QUESTION BANK OF Dr. MOHAMED YAHIA

285 Question

PRETEST MICROBIOLOGY

263 ‫صفحة‬ Dr. MOHAMED YAHIA 0900987639


QUESTION BANK OF Dr. MOHAMED YAHIA

VIROLOGY
1. An HIV-positive patient asks you if you can tell him the chances of him
progressing to symptomatic AIDS. Which one of the following tests would be
most useful?

a. CD4 lymphocyte count


b. HIV antibody test
c. HIV RT PCR
d. Neopterin
e. HIV p24 antigen

The answer is: C

2. Which of the following viruses causes an acute febrile rash and produces
disease in immunocompetent children but has been associated with transient
aplastic crises in persons with sickle cell disease?

a. Rubeola
b. Varicella-zoster
c. Parvovirus
d. Rubella
e. Herpes simplex

The answer is: C

3. Infection with herpes simplex virus, a common human pathogen, is best


described by which of the following statements?

a. The CNS and visceral organs are usually involved


b. It rarely recurs in a host who has a high antibody titer
c. It can be reactivated by emotional disturbances or prolonged exposure to sun
light
d. Initial infection usually occurs by intestinal absorption of the virus
e. Infection with type 1 virus is most common

The answer is: C

3. The latest and most effective therapy for AIDS patients includes
azidothymidine (AZT), dideoxyinosine (DDI), and saquinavir or similar agents.
Use of these three drugs would inhibit which of the following viral processes?

a. RNase, DNase
b. gp120 formation
c. p24 antibody expression
d. All membrane synthesis
e. Reverse transcriptase, protease

264 ‫صفحة‬ Dr. MOHAMED YAHIA 0900987639


QUESTION BANK OF Dr. MOHAMED YAHIA

The answer is: E

4. An HIV-positive patient prior to being treated with AZT, DDI, and saquinavir
has a CD4 lymphocyte count and an HIV RNA viral load test done. Results are
as follows:

CD4: 50 CD4 lymphocytes per microliter

HIV RNA: 750,000 copies per ml

Which of the following statements best describes the above patient?

a. This patient is no longer in danger of opportunistic infection


b. The 5-year prognosis is excellent
c. The patient’s HIV screening test is most likely negative
d. The patient is not infectious
e. The viral load of 750,000 copies per ml suggests that the patient will respond
to triple therapy

The answer is: E

5. This HIV-positive patient with a viral load of 750,000 copies of HIV RNA/ml
and a total CD4 count of 50 is at an increased risk for a number of infectious
diseases. For which of the following diseases is the patient at no more added risk
than an immunocompetent host?

a. Pneumocystic pneumonia
b. Mycobacterial disease
c. Kaposi’s sarcoma
d. Pneumococcal pneumonia
e. Herpes simplex virus

The answer is: A

6. Infectious mononucleosis, a viral disorder that can be debilitating, is


characterized by which of the following statements?

a. It is most prevalent in children less than 14 years old


b. It is caused by a rhabdovirus
c. The causative pathogen is an Epstein-Barr virus
d. Affected persons respond to treatment with the production of heterophil
antibodies
e. Ribavirin is the treatment of choice

The answer is: C

265 ‫صفحة‬ Dr. MOHAMED YAHIA 0900987639


QUESTION BANK OF Dr. MOHAMED YAHIA

7. Coronaviruses are recognized by club-shaped surface projections that are 20


nm long and resemble solar coronas. These viruses are characterized by their
ability to

a. Infect infants more frequently than adults


b. Cause the common cold
c. Grow well in the usual cultured cell lines
d. Grow profusely at 50°C
e. Agglutinate human red blood cells

The answer is: B

8. Delta hepatitis only occurs in patients who also have either acute or chronic
infection with hepatitis B virus. The delta agent is

a. An incomplete hepatitis B virus


b. Related to hepatitis A virus
c. A hepatitis B mutant
d. An incomplete RNA virus
e. Hepatitis C

The answer is: D

9. Which of the following antiviral agents is a purine nucleoside analogue that


has shown promise with Lassa fever, influenza A and B, and respiratory
syncytial virus (RSV)?

a. Amantadine
b. Rimantadine
c. Vidarabine
d. Ribavirin
e. Acyclovir

The answer is: D

10. Echoviruses are cytopathogenic human viruses that mainly infect the

a. Respiratory system
b. Central nervous system
c. Blood and lymphatic systems
d. Intestinal tract
e. Bladder and urinary tract

The answer is: D

266 ‫صفحة‬ Dr. MOHAMED YAHIA 0900987639


QUESTION BANK OF Dr. MOHAMED YAHIA

11. The most sensitive test for the diagnosis of herpes simplex (HSV) meningitis
in a newborn infant is

a. HSV IgG antibody


b. HSV polymerase chain reaction (PCR)
c. HSV culture
d. Tzanck smear
e. Cerebrospinal fluid (CSF) protein analysis

The answer is: B

12. Acute hemorrhagic conjunctivitis (AHC) is a contagious ocular infection


characterized by pain, swelling of the eyelids, and subconjunctival hemorrhages.
AHC has been reported to be caused by which of the following viruses?

a. Coronavirus
b. Reovirus
c. Rhinovirus
d. Enterovirus
e. Respiratory syncytial virus

The answer is: D

13. Mumps virus accounts for 10 to 15% of all cases of aseptic meningitis in the
United States. Infection with mumps virus

a. Is apt to recur periodically in many affected persons


b. Will usually cause mumps orchitis in postpubertal males
c. Is maintained in a large canine reservoir
d. Usually produces severe systemic manifestations
e. Is preventable by immunization

The answer is: B

14. The serum of a newborn infant reveals a 1:32 cytomegalovirus (CMV) titer.
The child is clinically asymptomatic. Which of the following courses of action
would be advisable?

a. Repeat the CMV titer immediately


b. Wait 6 months and obtain another titer on the baby
c. Obtain a CMV titer from all siblings
d. Obtain an anti-CMV IgM titer from the mother
e. Obtain an anti-CMV IgM titer from the baby

The answer is: E

267 ‫صفحة‬ Dr. MOHAMED YAHIA 0900987639


QUESTION BANK OF Dr. MOHAMED YAHIA

15. A 3-year-old child presents at the physician’s office with symptoms of coryza,
conjunctivitis, low-grade fever, and Koplik’s spots. The causative agent of this
disease belongs to which group of viruses?

a. Adenovirus
b. Herpesvirus
c. Picornavirus
d. Orthomyxovirus
e. Paramyxovirus

The answer is: E

16. One of the most common sexually transmitted diseases that may lead to
cervical carcinoma is caused by which of the following viruses?

a. Cytomegalovirus
b. Papillomavirus
c. Epstein-Barr virus
d. Herpes simplex virus
e. Adenovirus

The answer is: B

17. Which virus is the leading cause of the croup syndrome in young children
and, when infecting mammalian cells in culture, will hemabsorb red blood cells?

a. Group B coxsackievirus
b. Rotavirus
c. Parainfluenza virus
d. Adenovirus
e. Rhinovirus

The answer is: C

18. Hepatitis E, a recently characterized hepatitis virus, is best described by


which of the following statements?

a. It is not a threat to the blood supply


b. It is a major cause of blood-borne hepatitis
c. It is prevalent in North America
d. It is a single-stranded DNA virus
e. The disease resembles hepatitis C

The answer is: A

19. Kuru is a fatal disease of certain New Guinea natives and is characterized by
tremors and ataxia; Creutzfeldt-Jakob disease (CJD) is characterized by both
ataxia and dementia. These diseases are thought to be caused by

268 ‫صفحة‬ Dr. MOHAMED YAHIA 0900987639


QUESTION BANK OF Dr. MOHAMED YAHIA

a. Slow viruses
b. Cell wall–deficient bacteria
c. Environmental toxins
d. Prions
e. Flagellates

The answer is: D

20. According to recommendations issued by the U.S. Public Health Service,


which of the following statements regarding vaccination against smallpox is
true?

a. Pregnant women should be vaccinated in the first trimester


b. Persons who have eczema should be vaccinated soon after diagnosis
c. Persons who have immune deficiencies should be vaccinated every 5 years
d. Persons traveling abroad need not be vaccinated
e. Children should be vaccinated before they begin school

The answer is: D

21. Meningitis is characterized by the acute onset of fever and stiff neck. Aseptic
meningitis may be caused by a variety of microbial agents. During the initial 24 h
of the course of aseptic meningitis, an affected person’s cerebrospinal fluid is
characterized by

a. Decreased protein content


b. Elevated glucose concentration
c. Lymphocytosis
d. Polymorphonuclear leukocytosis
e. Eosinophilia

The answer is: D

22. What serologic test can be used to determine whether a patient with HDV is
an HBV carrier?

a. HBsAg
b. HBc IgM
c. HBeAg
d. HBs IgM
e. HBs IgG

The answer is: B

23. A nurse develops clinical symptoms consistent with hepatitis. She recalls
sticking herself with a needle approximately 4 months before after drawing
blood from a patient. Serologic tests for HBsAg, antibodies to HBsAg, and

269 ‫صفحة‬ Dr. MOHAMED YAHIA 0900987639


QUESTION BANK OF Dr. MOHAMED YAHIA

hepatitis A virus (HAV) are all negative; however, she is positive for IgM core
antibody. The nurse

a. Does not have hepatitis B


b. Has hepatitis A
c. Is in the late stages of hepatitis B infection
d. Is in the “window” (after the disappearance of HBsAg and before the
appearance of anti-HBsAg)
e. Has hepatitis C

The answer is: D

24. German measles virus (rubella), a common cause of exanthems in children, is


best described by which of the following statements?

a. Measles (rubeola) and German measles (rubella) are caused by the same virus
b. Incubation time is approximately 3 to 4 weeks
c. Vesicular rashes are characteristic
d. Onset is abrupt with cough, coryza, and fever
e. Specific antibody in the serum does not prevent disease

The answer is: D

25. The presence of Negri inclusion bodies in host cells is characteristic of

a. Mumps
b. Infectious mononucleosis
c. Congenital rubella
d. Aseptic meningitis
e. Rabies

The answer is: E

26. Hepatitis D virus (delta agent) is a defective virus that can replicate only in
cells already infected with which of the following viruses?

a. Hepatitis A virus
b. Epstein-Barr virus
c. Hepatitis G virus
d. Hepatitis B virus
e. HIV

The answer is: D

27. A patient presents with keratoconjunctivitis. The differential diagnosis


should include infection with which of the following viruses?

a. Parvovirus

270 ‫صفحة‬ Dr. MOHAMED YAHIA 0900987639


QUESTION BANK OF Dr. MOHAMED YAHIA

b. Adenovirus
c. Epstein-Barr virus
d. Respiratory syncytial virus
e. Varicella-zoster virus

The answer is: B

28. A hospital worker is found to have hepatitis B surface antigen. Subsequent


tests reveal the presence of e antigen as well. The worker most likely

a. Is infective and has active hepatitis


b. Is infective but does not have active hepatitis
c. Is not infective
d. Is evincing a biologic false-positive test for hepatitis
e. Has both hepatitis B and C

The answer is: A

29. An immunocompromised person with history of seizures had an MRI that


revealed a temporal lobe lesion. Brain biopsy results showed multinucleated
giant cells with intranuclear inclusions. The most probable cause of the lesion is

a. Hepatitis C virus
b. Herpes simplex virus
c. Listeria monocytogenes
d. Coxsackievirus
e. Parvovirus

The answer is: B

30. Which of the following procedures or clinical signs is most specific for the
diagnosis of infectious mononucleosis caused by the Epstein-Barr virus?

a. Laboratory diagnosis is based on the presence of “atypical lymphocytes” and


EBV-specific antibody
b. Growth in tissue culture cells
c. Heterophile antibodies in serum
d. Lymphadenopathy and splenomegaly on physical examination
e. B-cell lymphocyte proliferation

The answer is: A

31. An infant, seen in the ER, presents with a fever and persistent cough.
Physical examination and a chest x-ray suggest pneumonia. Which of the
following is most likely the cause of this infection?

a. Rotavirus
b. Adenovirus

271 ‫صفحة‬ Dr. MOHAMED YAHIA 0900987639


QUESTION BANK OF Dr. MOHAMED YAHIA

c. Coxsackievirus
d. Respiratory syncytial virus
e. Rhinovirus

The answer is: D

32. An obstetrician sees a pregnant patient who was exposed to rubella virus in
the eighteenth week of pregnancy. She does not remember getting a rubella
vaccination. The best immediate course of action is to

a. Terminate the pregnancy


b. Order a rubella antibody titer to determine immune status
c. Reassure the patient because rubella is not a problem until after the thirtieth
week
d. Administer rubella immune globulin
e. Administer rubella vaccine

The answer is: D

33. Mad Cow Disease has been highly publicized in Great Britain. This disease,
which is similar to scrapie, is caused by

a. A prion
b. A virus
c. Rickettsiae
d. An autoimmune reaction
e. A bacterium with a defective cell wall

The answer is: A

34. A patient has all the gastrointestinal symptoms of infection with hepatitis A
virus (HAV), yet all the tests for HAV-IgG and HAV-IgM are nonreactive. A
possible cause of this infection is

a. Hepatitis B surface antigen


b. Hepatitis C
c. Hepatitis D
d. Hepatitis E
e. Rotavirus

The answer is: D

35. A 70-year-old nursing home patient refused the influenza vaccine and
subsequently developed influenza. She died of acute pneumonia 1 week after
contracting the “flu.” The most common cause of acute postinfluenzal
pneumonia is

a. Legionella

272 ‫صفحة‬ Dr. MOHAMED YAHIA 0900987639


QUESTION BANK OF Dr. MOHAMED YAHIA

b. Listeria
c. Staphylococcus aureus
d. Klebsiella
e. Escherichia coli

The answer is: C

36. Erythema infectiosum (fifth disease), a self-limited disease of children, is


caused by

a. Measles
b. Parvovirus
c. Rubella
d. Human herpesvirus type 6
e. Norwalk virus

The answer is: B

37. Which one of the following viruses may be human tumor virus?

a. Epstein-Barr virus (EBV)


b. HIV
c. Papillomavirus
d. Varicella-zoster virus (VZV)
e. Herpes simplex virus, type 2 (HSV)

The answer is: C

38. Parvovirus infection, the cause of a mild exanthem in children, is


characterized by

a. Epidemic acute respiratory disease


b. Gastroenteritis
c. Whooping cough–like disease
d. Keratoconjunctivitis
e. Acute hemolytic anemia

The answer is: D

39. Subacute sclerosing panencephalitis virus (SSPE) is best described by which


of the following statements?

a. It is a progressive disease involving both white and gray matter


b. It is a late CNS manifestation of mumps
c. It is a common event occurring in 1 of 300,000 cases of mumps
d. Viral DNA can be demonstrated in brain cells
e. Demyelination is characteristic

273 ‫صفحة‬ Dr. MOHAMED YAHIA 0900987639


QUESTION BANK OF Dr. MOHAMED YAHIA

The answer is: A

40. Cytomegalovirus (CMV) infection is common. Which one of the following


statements best characterizes CMV?

a. It can be transmitted across the placental barrier


b. While a common infection, CMV is almost always symptomatic
c. The CMV can be cultured from red blood cells of infected patients
d. Unlike other viral infections, CMV is not activated by immunosuppressive
therapy
e. There is no specific therapy for CMV

The answer is: A

41. Human rotaviruses are characterized by which of the following statements?

a. They produce an infection that is primarily seen in adults


b. They produce cytopathic effects in many conventional tissue culture systems
c. They are lipid-containing RNA viruses possessing a double-shelled capsid
d. They can be sensitively and rapidly detected in stools by the enzyme-linked
immunosorbent assay (ELISA) technique
e. They have been implicated as a major etiologic agent of infantile respiratory
disease

The answer is: C

42. Rotavirus is a double-stranded RNA virus with a double-walled capsid.


Which one of the following statements best describes rotavirus?

a. There are no related animal viruses


b. It is a major cause of neonatal diarrhea
c. It is readily cultured from the stool of infected persons
d. Maternal antibody does not appear to be protective
e. Early breast-feeding offers no protection to neonates against it

The answer is: B

43. Paramyxoviruses are most commonly associated with which of the following
diseases?

a. Fifth disease
b. Rubella
c. Croup
d. Tonsillitis
e. Otitis media

The answer is: C

274 ‫صفحة‬ Dr. MOHAMED YAHIA 0900987639


QUESTION BANK OF Dr. MOHAMED YAHIA

44. Human papillomavirus is most commonly associated with

a. Rectal polyps
b. Prostate cancer
c. Condyloma acuminatum
d. Hepatic carcinoma
e. Carcinoma of the lung

The answer is: C

45. Reverse transcriptase is an enzyme unique to the retroviruses. Which one of


the following is a function of the enzyme reverse transcriptase?

a. DNase activity
b. RNA-dependent RNA polymerase activity
c. RNA isomerase activity
d. RNA-dependent DNA polymerase activity
e. Integration activity

The answer is: D

46. A 17-year-old girl presents with cervical lymphadenopathy, fever, and


pharyngitis. Infectious mononucleosis is suspected. The most rapid and clinically
useful test to make this diagnosis is

a. IgM antibody to viral core antigen (VCA)


b. IgG antibody to VCA
c. Antibody to Epstein-Barr nuclear antigen (EBNA)
d. Culture
e. C reactive protein (CRP)

The answer is: A

47. Which one of the following viruses would be most likely to establish a latent
infection?

a. Adenovirus
b. Measles virus
c. Influenza virus
d. Parvovirus
e. Coxsackievirus group B

The answer is: A

48. A regimen that includes appropriately administered gamma globulin may be


contraindicated in which one of the following diseases?

a. Hepatitis A

275 ‫صفحة‬ Dr. MOHAMED YAHIA 0900987639


QUESTION BANK OF Dr. MOHAMED YAHIA

b. Hepatitis B
c. Rabies
d. Poliomyelitis
e. Infectious mononucleosis

The answer is: E

49. Atypical lymphocytosis is most likely to be found in which one of the


following diseases?

a. Encephalitis caused by herpes simplex virus (HSV)


b. Mononucleosis induced by Epstein-Barr virus
c. Parvovirus infection
d. Chronic hepatitis C
e. Rotavirus gastroenteritis

The answer is: B

50. A patient has arthralgia, a rash, lymphadenopathy, pneumonia but no fever.


Which of the following diseases is most likely based on these symptoms?

a. Dengue fever
b. St. Louis encephalitis
c. Infectious mononucleosis
d. Hepatitis
e. HIV infection

The answer is: A

51. Hepatitis C (HCV) is usually a clinically mild disease, with only minimal
elevation of liver enzymes. Hospitalization is unusual. Which one of the following
statements best characterizes HCV?

a. Few cases progress to chronic liver disease


b. It often occurs in posttransfusion patients
c. HBV but not HCV infections occur in IV drug abusers
d. It is a DNA virus
e. Blood products are not tested for antibody to HCV

The answer is: B

52. Which of the following markers is usually the first viral marker detected
after hepatitis B infection?

a. HBeAg
b. HBsAg
c. HBcAg
d. Anti-HBc

276 ‫صفحة‬ Dr. MOHAMED YAHIA 0900987639


QUESTION BANK OF Dr. MOHAMED YAHIA

e. HbeAb

The answer is: B

53. Which of the following may be the only detectable serological marker during
the early convalescent phase of HBV infection (window phase)?

a. HBeAg
b. HBsAg
c. HBcAg
d. Anti-HBc
e. HbeAb

The answer is: D

54. Which one of the following markers is closely associated with HBV infectivity
and DNA polymerase activity?

a. HBeAg
b. HBsAg
c. HBcAg
d. Anti-HBc
e. HBeAb

The answer is: A

55. Which of the following is found within the nuclei of infected hepatocytes and
not usually in the peripheral circulation?

a. HBeAg
b. HBsAg
c. HBcAg
d. Anti-HBc
e. HbeAb

The answer is: C

56. Which one of the following viruses is the leading cause of congenital
malformations?

a. Rabies
b. Rhinovirus
c. Cytomegalovirus
d. Respiratory syncytial virus
e. Mumps

The answer is: C

277 ‫صفحة‬ Dr. MOHAMED YAHIA 0900987639


QUESTION BANK OF Dr. MOHAMED YAHIA

57. Orchitis, which may cause sterility, is a possible manifestation of which of the
following?

a. Rabies
b. Rhinovirus
c. Cytomegalovirus
d. Respiratory syncytial virus
e. Mumps

The answer is: E

58. Which of the following is a leading cause of pneumonia primarily in infants?

a. Rabies
b. Rhinovirus
c. Cytomegalovirus
d. Respiratory syncytial virus
e. Mumps

The answer is: D

59. Which of the following causes a fatal encephalitis for which a vaccine is
available?

a. Rabies
b. Rhinovirus
c. Cytomegalovirus
d. Respiratory syncytial virus
e. Mumps

The answer is: A

60. Traditional vaccination for the common cold is virtually impossible because
there are multiple serotypes of which one of the following viruses?

a. Rabies
b. Rhinovirus
c. Cytomegalovirus
d. Respiratory syncytial virus
e. Mumps

The answer is: B

61. Chicken pox is a common disease of childhood. It is caused by which of the


following viruses?

a. Cytomegalovirus
b. Rotavirus

278 ‫صفحة‬ Dr. MOHAMED YAHIA 0900987639


QUESTION BANK OF Dr. MOHAMED YAHIA

c. Varicella-zoster virus
d. Adenovirus
e. Papillomavirus

The answer is: C

62. Excluding influenza, which one of the following viruses is a common cause of
acute respiratory disease?

a. Cytomegalovirus
b. Rotavirus
c. Varicella-zoster virus
d. Adenovirus
e. Papillomavirus

The answer is: D

63. Human warts are not only cosmetically unsightly but may lead to cancer of
the cervix. They are caused by which one of the following viruses?

a. Cytomegalovirus
b. Rotavirus
c. Varicella-zoster virus
d. Adenovirus
e. Papillomavirus

The answer is: E

64. A vaccine is available for one of the most common causes of infantile
gastroenteritis. However, it has recently been recalled. The virus is

a. Cytomegalovirus
b. Rotavirus
c. Varicella-zoster virus
d. Adenovirus
e. Papillomavirus

The answer is: B

65. A child has mononucleosis-like symptoms yet the test for mononucleosis and
the EBV titers are negative. One of the causes of heterophile negative
mononucleosis is

a. Cytomegalovirus
b. Herpes simplex virus
c. Varicella-zoster virus
d. Adenovirus
e. Coxsackievirus

279 ‫صفحة‬ Dr. MOHAMED YAHIA 0900987639


QUESTION BANK OF Dr. MOHAMED YAHIA

The answer is: A

66. Malaise and fatigue with increased “atypical” lymphocytes and a reactive
heterophil antibody test is most commonly caused by

a. Toxoplasma
b. Borrelia burgdorferi
c. Epstein-Barr virus
d. Parvovirus
e. Rubella virus

The answer is: C

67. Lethargy, malaise, and fatigue are observed in a patient 2 weeks after eating
raw hamburger at a restaurant. The most likely infectious cause is

a. Toxoplasma
b. Cytomegalovirus
c. E. coli
d. Salmonella
e. Clostridium

The answer is: A

68. Burkitt’s lymphoma is characterized by elevated “early antigen” tests with a


restricted pattern of fluorescence. This disease is caused by

a. Cytomegalovirus
b. B. burgdorferi
c. Epstein-Barr virus
d. Lymphogranuloma venereum
e. Herpes simplex virus

The answer is: C

69. This virus may be detected by the polymerase chain reaction (PCR) in a
variety of cells of patients with nasopharyngeal carcinoma.

a. Measles
b. Mumps
c. Rubella
d. Parvovirus
e. Epstein-Barr virus

The answer is: E

Questions 70–74

280 ‫صفحة‬ Dr. MOHAMED YAHIA 0900987639


QUESTION BANK OF Dr. MOHAMED YAHIA

Assume you are asked by a resident what the most appropriate specimen is for
the detection of a particular virus.

70. Human papillomavirus

a. Cervical tissue
b. Synovial fluid
c. Blood
d. Skin

The answer is: A

71. Cytomegalovirus

a. Cervical tissue
b. Synovial fluid
c. Blood
d. Skin
e. Cerebrospinal fluid

The answer is: C

72. Enterovirus

a. Cervical tissue
b. Synovial fluid
c. Blood
d. Skin
e. Cerebrospinal fluid

The answer is: E

73. Varicella-zoster virus (VZV)

a. Cervical tissue
b. Synovial fluid
c. Blood
d. Skin
e. Cerebrospinal fluid

The answer is: D

74. Adenovirus 40/41

a. Cervical tissue
b. Synovial fluid
c. Blood
d. Stool
e. Cerebrospinal fluid

281 ‫صفحة‬ Dr. MOHAMED YAHIA 0900987639


QUESTION BANK OF Dr. MOHAMED YAHIA

The answer is: D

75. Which of the following is transmitted by the fecal-oral route; can be acquired
from shellfish; and often causes acute jaundice, diarrhea, and liver function
abnormalities?

a. Rotavirus
b. Adenovirus 40/41
c. Norwalk virus
d. Astrovirus
e. Hepatitis A virus

The answer is: E

76. Which of the following is the second most common cause of pediatric
gastroenteritis? Unlike other similar viruses, this virus causes only
gastroenteritis.

a. Rotavirus
b. Adenovirus 40/41
c. Norwalk virus
d. Astrovirus
e. Hepatitis A virus

The answer is: B

77. Which of the following is the most common cause of pediatric gastroenteritis?
It is difficult to grow in cell culture but can be detected easily by immunologic
methods (ELISA).

a. Rotavirus
b. Adenovirus 40/41
c. Norwalk virus
d. Astrovirus
e. Hepatitis A virus

The answer is: A

78. Which of the following is a common cause of epidemic gastroenteritis,


particularly aboard cruise ships and in summer camps? It may be detected by
ELISA methods or electron microscopy.

a. Rotavirus
b. Adenovirus 40/41
c. Norwalk virus
d. Astrovirus
e. Hepatitis A virus

282 ‫صفحة‬ Dr. MOHAMED YAHIA 0900987639


QUESTION BANK OF Dr. MOHAMED YAHIA

The answer is: C

79. Which of the following is a cause of mild gastroenteritis? It can be


transmitted by the fecal-oral route but not by food consumption.

a. Rotavirus
b. Adenovirus 40/41
c. Norwalk virus
d. Astrovirus
e. Hepatitis A virus

The answer is: D

80. IgM antibody to the viral particle is the method of choice for laboratory
diagnosis of which one of the following hepatitis viruses?

a. Hepatitis A
b. Hepatitis B
c. Hepatitis C
d. Hepatitis D
e. Hepatitis E

The answer is: A

81. This virus belongs to the family of flaviviruses and its reservoir is strictly
human. Transmission is blood-borne so the blood supply is routinely screened
for this virus.

a. Hepatitis A
b. Hepatitis B
c. Hepatitis C
d. Hepatitis D
e. Hepatitis E

The answer is: C

82. Vaccination for this hepatic disease is with viral surface antigen and usually
provides immunity.

a. Hepatitis A
b. Hepatitis B
c. Hepatitis C
d. Hepatitis D
e. Hepatitis E

The answer is: B

283 ‫صفحة‬ Dr. MOHAMED YAHIA 0900987639


QUESTION BANK OF Dr. MOHAMED YAHIA

83. This hepatitis virus is a calicivirus. The reservoir is in pigs, and humans
acquire it via the fecal-oral route.

a. Hepatitis A
b. Hepatitis B
c. Hepatitis C
d. Hepatitis D
e. Hepatitis E

The answer is: E

84. This hepatitis virus is a defective virus in that it cannot replicate


independently without the presence of hepatitis B virus.

a. Hepatitis A
b. Hepatitis B
c. Hepatitis C
d. Hepatitis D
e. Hepatitis E

The answer is: D

85. This virus is a single-stranded RNA orthomyxovirus. Annual vaccination is


necessary because of antigenic drift and shift.

a. Measles virus
b. Influenza virus
c. Respiratory syncytial virus
d. Parainfluenza virus
e. Adenovirus

The answer is: B

86. This is a double-stranded DNA virus. It is responsible for 15% of pediatric


respiratory infections and 10 to 15% of acute diarrhea in children.

a. Measles virus
b. Influenza virus
c. Respiratory syncytial virus
d. Parainfluenza virus
e. Adenovirus

The answer is: E

87. This virus causes a mononucleosis-like syndrome caused by a latent


herpesvirus; it is often a congenital infection. Large amounts of the virus are
excreted in the urine; thus, urine becomes the fluid of choice for diagnosis of this
disease.

284 ‫صفحة‬ Dr. MOHAMED YAHIA 0900987639


QUESTION BANK OF Dr. MOHAMED YAHIA

a. Epstein-Barr virus
b. Cytomegalovirus
c. HHV-6
d. Parvovirus
e. Norwalk virus

The answer is: B

BACTERIOLOGY
1. A patient with a peptic ulcer was admitted to the hospital and a gastric biopsy
was performed. The tissue was cultured on chocolate agar incubated in a
microaerophilic environment at 37°C for 5 to 7 days. At 5 days of incubation,
colonies appeared on the plate and were curved, Gram negative rods, oxidase-
positive. The most likely identity of this organism is

a. Campylobacter jejuni
b. Vibrio parahaemolyticus
c. Haemophilus influenzae
d. Helicobacter pylori
e. Campylobacter fetus

The answer is: D

2. An inhibitor was designed to block a biologic function in H. influenzae. If the


goal of the experiment was to reduce the virulence of H. influenzae, the most
likely target would be

a. Exotoxin liberator
b. Endotoxin assembly
c. Flagella synthesis
d. Capsule formation
e. IgA protease synthesis

The answer is: D

3. An experimental compound is discovered that prevents the activation of


adenyl cyclase and the resulting increase in cyclic AMP. The toxic effects of
which of the following bacteria might be prevented with the use of this
experimental compound?

a. Vibrio cholerae
b. Corynebacterium diphtheriae
c. Pseudomonas
d. Listeria monocytogenes
e. Brucella

285 ‫صفحة‬ Dr. MOHAMED YAHIA 0900987639


QUESTION BANK OF Dr. MOHAMED YAHIA

The answer is: A

Questions 4–6: A 21-year-old college student complained of malaise, low-grade


fever, and a harsh cough, but not of muscle aches and pains. An x-ray revealed a
diffuse interstitial pneumonia in the left lobes of the lung. The WBC count was
normal. The student has been ill for a week.

4. Based on the information given, the most likely diagnosis is

a. Mycoplasma pneumonia
b. Pneumococcal pneumonia
c. Staphylococcal pneumonia
d. Influenza
e. Legionellosis

The answer is: A

5. Based on the information given, which of the following laboratory tests would
most rapidly assist you in making the diagnosis?

a. Cold agglutinins
b. Viral culture
c. Complement fixation (CF) test
d. Gram stain of sputum
e. Culture of sputum

The answer is: A

6. The following laboratory data were available within 2 days: cold agglutinins—
negative; complement fixation (M. pneumoniae)—1:64; viral culture—pending,
but negative to date; bacterial culture of sputum on blood agar and
MacConkey’s agar—normal oral flora. In order to confirm the diagnosis, which
of the following procedures could be ordered to achieve a specific and sensitive
diagnosis?

a. Culture of the sputum on charcoal yeast extract


b. A repeat cold agglutinin test
c. A DNA probe to the 16S ribosomal RNA of an organism lacking a cell wall
d. A repeat CF test in 5 days
e. Another viral culture in 1 week

The answer is: C

7. Pathogenic mechanisms involved in tuberculosis can be primarily attributed to


which of the following?

286 ‫صفحة‬ Dr. MOHAMED YAHIA 0900987639


QUESTION BANK OF Dr. MOHAMED YAHIA

a. Toxin production by the mycobacteria


b. Specific cell adhesion sites
c. Cell-mediated hypersensitivity
d. Humoral immunity
e. Clogging of alveoli by large numbers of acid-fast mycobacteria

The answer is: C

8. The class of antibiotics known as the quinolones are bactericidal. Their mode
of action on growing bacteria is thought to be

a. Inhibition of DNA gyrase


b. Inactivation of penicillin-binding protein II
c. Inhibition of β -lactamase
d. Prevention of the cross-linking of glycine
e. Inhibition of reverse transcriptase

The answer is: A

9. Which one of the statements concerning Vancomycin-indeterminate S.


aureus(VISA) is the most correct?

a. Minimum inhibitory concentration (MIC) for vancomycin is at least 1.0


mcg/mL
b. VISA isolates are usually methicillin susceptible (methicillin-resistant S.
aureus, MRSA)
c. VISAs have emerged because of the extended use of vancomycin for MRSAs
d. Patients with VISA isolates need not be isolated
e. VISA isolates are infrequent, so surveillance at the present time is not
warranted

The answer is: C

10. A sputum sample was brought to the laboratory for analysis. Gram stain
revealed the following: rare epithelial cells, 8 to 10 polymorphonuclear
leukocytes per high-power field, and pleomorphic Gram-negative rods. As the
laboratory consultant, which of the following interpretations should you make?

a. The sputum specimen is too contaminated by saliva to be useful


b. There is no evidence of an inflammatory response
c. The patient has pneumococcal pneumonia
d. The patient has Vincent’s disease
e. The appearance of the sputum is suggestive of Haemophilus pneumonia

The answer is: E

11. An isolate from a wound culture is a Gram-negative rod identified as


Bacteroides fragilis. Anaerobic infection with B. fragilis is characterized by

287 ‫صفحة‬ Dr. MOHAMED YAHIA 0900987639


QUESTION BANK OF Dr. MOHAMED YAHIA

a. A foul-smelling discharge
b. A black exudate in the wound
c. An exquisite susceptibility to penicillin
d. A heme-pigmented colony formation
e. Severe neurologic symptoms

The answer is: A

12. L. monocytogenes causes a variety of diseases, including food poisoning.


Listeria are small, Gram-positive, motile rod-shaped bacteria. Which of the
following best describes these microorganisms?

a. Listeria are facultative intracellular pathogens


b. Once infected, the immune system cannot destroy Listeria
c. Listeria cannot be cultivated on artificial media
d. Flagella are produced both at room temperature and at 37°C
e. There is no relationship between Listeria serovars and human infection

The answer is: A

13. Virtually all prokaryotic cells (bacteria, both Gram-positive and Gram
negative) contain peptidoglycan as well as specific enzymes for its synthesis. All
of the following statements concerning Gram-positive and Gram-negative
bacteria are true except

a. The extent of cross-linking of peptidoglycan is a function of different species


of bacteria
b. The peptidoglycan-synthesizing enzymes can be antibiotic targets
c. Both Gram-positive and Gram-negative bacteria contain significant amounts
of teichoic acid
d. With the exception of the structures that are cross-linked, peptidoglycan
structure is common to most bacteria
e. The physical shape of bacteria is a function of peptidoglycan

The answer is: C

14. A 30-year-old male patient was seen by the emergency service and reported a
2-week history of a penile ulcer. He noted that this ulcer did not hurt. Which one
of the following conclusions/actions is most valid?

a. Draw blood for a herpes antibody test


b. Perform a dark-field examination of the lesion
c. Prescribe acyclovir for primary genital herpes
d. Even if treated, the lesion will remain for months
e. Failure to treat the patient will have no untoward effect, as this is a self-
limiting infection

288 ‫صفحة‬ Dr. MOHAMED YAHIA 0900987639


QUESTION BANK OF Dr. MOHAMED YAHIA

The answer is: B

15. The laboratory reports that the Venereal Disease Research Laboratory
(VDRL) test performed on the above patient is reactive at a dilution of 1:4 (4
dils). The patient also reports to you that he has recently been diagnosed with
hepatitis A. Which one of the following actions would be most appropriate?

a. Report this patient to the health department, as he has syphilis


b. Order a confirmatory test such as the fluorescent treponemal antibody test
(FTA)
c. Repeat the VDRL test
d. Order a rapid reagin test (RPR)
e. Perform a spinal tap to rule out central nervous system syphilis

The answer is: B

16. In the above patient, which one of the following test combinations for syphilis
is most appropriate?

a. FTA-Abs (IgG)/FTA-Abs (IgM)


b. RPR/FTA-Abs
c. RPR/culture of the lesion
d. VDRL/RPR
e. Treponema pallidumhemagglutination (TPHA) / microhemagglutination -
Treponema pallidum (MHTP) tests

The answer is: B

17. Assume that the patient absolutely denied any contact, sexual or otherwise,
with a person who had syphilis. Assume also that both the RPR and the FTA Abs
were positive on this patient. Which one of the following tests could be used to
show that this patient probably does not have syphilis?

a. VDRL
b. Quantitative RPR
c. Treponema pallidum immobilization (TPI) test
d. Frei test
e. MHTP test

The answer is: C

18. A 55-year-old man who is being treated for adenocarcinoma of the lung is
admitted to a hospital because of a temperature of 38.9°C (102°F), chest pain,
and a dry cough. Sputum is collected. Gram’s stain of the sputum is
unremarkable and culture reveals many small Gram-negative rods able to grow
only on a charcoal yeast extract agar. This organism most likely is

a. Klebsiella pneumoniae

289 ‫صفحة‬ Dr. MOHAMED YAHIA 0900987639


QUESTION BANK OF Dr. MOHAMED YAHIA

b. Mycoplasma pneumoniae
c. Legionella pneumophila
d. Chlamydia trachomatis
e. S. aureus

The answer is: C

19. A patient was hospitalized after an automobile accident. The wounds became
infected and the patient was treated with tobramycin, carbenicillin, and
clindamycin. Five days after antibiotic therapy was initiated, the patient
developed severe diarrhea and pseudomembranous enterocolitis. Antibiotic-
associated diarrhea and the more serious pseudomembranous enterocolitis can
be caused by

a. Clostridium sordellii
b. Clostridium perfringens
c. Clostridium difficile
d. S. aureus
e. B. fragilis

The answer is: C

20. A child comes to an emergency room because of an infected dog bite. The
wound is found to contain small Gram-negative rods. The most likely cause of
infection is

a. E. coli
b. H. influenzae
c. Pasteurella multocida
d. Brucella canis
e. Klebsiella rhinoscleromatis

The answer is: C

21. Group B streptococcus sepsis in an infant is preventable. Which one of the


following procedures is most likely to reduce the incidence of group B
streptococcal disease?

a. Intrapartum antibiotic treatment


b. Use of a polysaccharide vaccine
c. Screening of pregnant females in the last trimester
d. Identification of possible high-risk births
e. Screening of pregnant females at the first office visit, usually during the first
trimester

The answer is: A

290 ‫صفحة‬ Dr. MOHAMED YAHIA 0900987639


QUESTION BANK OF Dr. MOHAMED YAHIA

22. There has been much speculation on the pathogenesis of group B


streptococcal disease in the neonate. One of the most likely pathogenic
mechanisms is

a. Complement C5a, a potent chemoattractant, activates PMNs


b. The streptococci are resistant to penicillin
c. The alternative complement pathway is activated
d. In the absence of specific antibody, opsonization, phagocyte recognition, and
killing do not proceed normally

The answer is: D

23. A patient complained to his dentist about a draining lesion in his mouth. A
Gram’s stain of the pus showed a few Gram-positive cocci, leukocytes, and many
branched Gram-positive rods. The most likely cause of the disease is

a. Actinomyces israelii
b. Actinomyces viscosus
c. C. diphtheriae
d. Propionibacterium acnes
e. S. aureus

The answer is: A

24. A man who has a penile chancre appears in a hospital’s emergency service.
The VDRL test is negative. The most appropriate course of action for the
physician in charge would be to

a. Send the patient home untreated


b. Repeat the VDRL test in 10 days
c. Perform dark-field microscopy for treponemes
d. Swab the chancre and culture on Thayer-Martin agar
e. Perform a Gram stain on the chancre fluid

The answer is: C

25. Fever of unknown origin in a farmer who raises goats would most likely be
caused by which of the following organisms?

a. Brucella melitensis
b. Clostridium novyi
c. T. pallidum
d. Histoplasma capsulatum
e. Mycobacterium tuberculosis

The answer is: A

291 ‫صفحة‬ Dr. MOHAMED YAHIA 0900987639


QUESTION BANK OF Dr. MOHAMED YAHIA

26. Cholera is a toxicogenic dysenteric disease common in many parts of the


world. In the treatment of patients who have cholera, the use of a drug that
inhibits adenyl cyclase would be expected to

a. Kill the patient immediately


b. Eradicate the organism
c. Increase fluid secretion
d. Reduce intestinal motility
e. Block the action of cholera toxin

The answer is: E

27. S. aureus causes a wide variety of infections, ranging from wound infection to
pneumonia. Treatment of S. aureus infection with penicillin is often complicated
by the

a. Inability of penicillin to penetrate the membrane of S. aureus


b. Production of penicillinase by S. aureus
c. Production of penicillin acetylase by S. aureus
d. Lack of penicillin binding sites on S. aureus
e. Allergic reaction caused by staphylococcal protein

The answer is: B

28. Symptoms of C. botulinum food poisoning include double vision, in- ability to
speak, and respiratory paralysis. These symptoms are consistent with

a. Invasion of the gut epithelium by C. botulinum


b. Secretion of an enterotoxin
c. Endotoxin shock
d. Ingestion of a neurotoxin
e. Activation of cyclic AMP

The answer is: D

29. In people who have sickle cell anemia, osteomyelitis usually is associated with
which of the following organisms?

a. Micrococcus
b. Escherichia
c. Pseudomonas
d. Salmonella
e. Streptococcus

The answer is: D

30. The treatment of choice for a patient with C. jejuni enterocolitis is

292 ‫صفحة‬ Dr. MOHAMED YAHIA 0900987639


QUESTION BANK OF Dr. MOHAMED YAHIA

a. Erythromycin
b. Ciprofloxacin
c. Ampicillin
d. Pepto-Bismol
e. Campylobacter antitoxin

The answer is: A

31. A hyperemic edema of the larynx and epiglottis that rapidly leads to
respiratory obstruction in young children is most likely to be caused by

a. K. pneumoniae
b. M. pneumoniae
c. Neisseria meningitidis
d. H. influenzae
e. H. hemolyticus

The answer is: D

32. Acute hematogenous osteomyelitis is often diagnosed by isolation of the


organism from the blood and is caused most often by

a. Proteus mirabilis
b. Streptococcus faecalis
c. Staphylococcus epidermidis
d. S. aureus
e. E. coli

The answer is: D

Questions 33–35

A 70-year-old female patient was readmitted to a local hospital with fever and
chills following cardiac surgery at a major teaching institution. Blood cultures
were taken and a Gram-positive coccus grew from the blood cultures within 24
hours. Initial tests indicated that this isolate was resistant to penicillin.

33. The most likely identification is

a. Streptococcus pneumoniae
b. Neisseria
c. Group A streptococcus
d. Enterococcus
e. Group B streptococcus

The answer is: D

293 ‫صفحة‬ Dr. MOHAMED YAHIA 0900987639


QUESTION BANK OF Dr. MOHAMED YAHIA

34. Further testing revealed that the isolate possessed the group D antigen, was
not β-lactamase-positive, but was resistant to vancomycin. The most likely
identification of this isolate is

a. Enterococcus faecalis
b. Enterococcus durans
c. Enterococcus cassiflavus
d. S. pneumoniae
e. Enterococcus faecium

The answer is: E

35. The treatment of choice for the isolate in question 34 is

a. Gentamicin
b. Gentamicin and ampicillin
c. Ciprofloxacin
d. Rifampin
e. No available treatment

The answer is: E

36. Diphtheria toxin is produced only by those strains of C. diphtheria that are

a. Glucose fermenters
b. Sucrose fermenters
c. Lysogenic for β-prophage
d. Of the mitis strain
e. Encapsulated

The answer is: C

Questions 37–39: A 28-year-old menstruating woman appeared in the emergency


room with the following signs and symptoms: fever, 104°F (40°C); WBC,
16,000/µL; blood pressure, 90/65 mmHg; a scarlatiniform rash on her trunk,
palms, and soles; extreme fatigue; vomiting; and diarrhea.

37. The patient described in the case above most likely has

a. Scalded skin syndrome


b. Toxic shock syndrome
c. Guillain-Barré syndrome
d. Chickenpox
e. Staphylococcal food poisoning

The answer is: B

294 ‫صفحة‬ Dr. MOHAMED YAHIA 0900987639


QUESTION BANK OF Dr. MOHAMED YAHIA

38. Culture of the menstrual fluid in the case cited would most likely reveal a
predominance of

a. S. aureus
b. S. epidermidis
c. C. perfringens
d. C. difficile
e. Gardnerella vaginalis

The answer is: A

39. The most characteristic finding not yet revealed in the case just presented
would be

a. Travel to Vermont
b. Recent exposure to rubella
c. A retained tampon
d. Heavy menstrual flow
e. A meal of chicken in a fast-food restaurant

The answer is: C

40. A 2-year-old infant is brought to the emergency room with hemolytic uremic
syndrome and thrombocytopenia. Which one of the following bacteria would
most likely be isolated from a stool specimen?

a. Shigella
b. Salmonella
c. Aeromonas
d. E. coli 0157/H7
e. Enterobacter

The answer is: D

41. E. coli causes disease by a variety of different methods. Which one of the
following E. coli types is characterized by the presence of LT (heat-labile) and
ST (heat-stable) proteins?

a. Enteroinvasive (EIEC)
b. Enterotoxigenic (ETEC)
c. Enterohemorrhagic (EHEC)
d. Enteropathogenic (EPEC)
e. Enterohemolytic (EHEEC)

The answer is: B

42. Yersinia pestis, the causative agent of plague, is enzootic in the United States
west of the one-hundredth meridian. Human plague can be bubonic or

295 ‫صفحة‬ Dr. MOHAMED YAHIA 0900987639


QUESTION BANK OF Dr. MOHAMED YAHIA

pneumonic. The primary epidemiologic difference between the two clinical forms
of plague is

a. Season of the year


b. Route of infection
c. Age of the patient
d. Health of the animal vector
e. Geographic location of the animal vector

The answer is: B

43. Recently, there have been sensational media reports of patients infected with
invasive, “flesh-eating” bacteria that spread rapidly through the tissues. This
necrotizing fasciitis is usually caused by

a. S. aureus
b. Group A streptococci
c. Micrococcus
d. Bacillus cereus
e. Clostridium tetani

The answer is: B

44. The most effective noninvasive test for the diagnosis of Helicobacter
associated gastric ulcers is

a. Detection of H. pylori antigen in stool


b. Growth of H. pylori from a stomach biopsy
c. Growth of H. pylori in the stool
d. IgM antibodies to H. pylori
e. Culture of stomach contents for H. pylori

The answer is: A

45. A 9-year-old child is brought to the emergency room with the chief complaint
of enlarged, painful axillary lymph nodes. The resident physician also notes a
small, inflamed, dime-sized lesion surrounding what appears to be a small
scratch on the forearm. The lymph node is aspirated and some pus is sent to the
laboratory for examination. A Warthin-Starry silver impregnation stain reveals
many highly pleomorphic, rod-shaped bacteria. The most likely cause of this
infection is

a. Y. pestis
b. Yersinia enterocolitica
c. Mycobacterium scrofulaceum
d. B. canis
e. Bartonella henselae

296 ‫صفحة‬ Dr. MOHAMED YAHIA 0900987639


QUESTION BANK OF Dr. MOHAMED YAHIA

The answer is: E

46. If a quellung test was done on the following bacterial isolates, which one
would you expect to be positive?

a. S. pneumoniae
b. Enterobacter
c. Haemophilus parainfluenzae
d. C. diphtheriae
e. N. gonorrhoeae

The answer is: A

47. Bacteria cause disease in a number of ways. One mechanism of pathogenesis


is the secretion of potent protein toxins. All the following diseases are caused by
microbial protein toxins, but one toxin has been used for a variety of maladies. It
is

a. Tetanus
b. Botulism
c. Bacillary (Shigella) dysentery
d. Diphtheria
e. Disseminated intravascular coagulation

The answer is: B

48. A 2-year-old child was admitted to the hospital with acute meningitis. The
Gram stain revealed Gram-positive short rods, and the mother indicated that the
child had received “all” of the meningitis vaccinations. What is the most likely
cause of the disease?

a. N. meningitidis, group A
b. N. meningitidis, group C
c. Listeria
d. S. pneumoniae
e. H. influenza

The answer is: C

49. A patient appeared in the emergency room with a submandibular mass. A


smear was made of the drainage and a bewildering variety of bacteria were seen,
including branched, Gram-positive rods. The most clinically appropriate action
is

a. Do no further clinical workup


b. Suggest to the laboratory that low colony counts may reflect infection
c. Determine if fluorescent microscopy is available for the diagnosis of
actinomycosis

297 ‫صفحة‬ Dr. MOHAMED YAHIA 0900987639


QUESTION BANK OF Dr. MOHAMED YAHIA

d. Consider vancomycin as an alternative drug


e. Suggest a repeat antibiotic susceptibility test

The answer is: C

50. Rheumatic fever (RF) is a disease seen in children and young adults. Which
one of the following statements best typifies the disease?

a. It is characterized by inflammatory lesions that may involve the heart, joints,


subcutaneous tissues, and the central nervous system
b. The pathogenesis is related to the similarity between a staphylococcal antigen
and a human cardiac antigen
c. Prophylaxis with benzathine penicillin is of little value
d. It is a complication of group A streptococcal skin disease but usually not of
pharyngitis
e. It is very common in developing countries but extremely rare and decreasing
in incidence in the United States

The answer is: A

51. The fermentation patterns for four strains of Gram-negative cocci are given
below (strains C and D grow on plain nutrient agar). Which of these strains is
likely to cause venereal disease in humans?

Acid Produced From

Maltose Dextrose Sucrose

a. a. Strain A + + −
b. b. Strain B − + −
c. c. Strain C − − −
d. d. Strain D + + +

The answer is: B

52. The antibiotic therapy of choice for legionellosis is

a. Penicillin
b. Ampicillin
c. Erythromycin
d. Vancomycin
e. Ceftriaxone

The answer is: C

53. The antibiotic of choice for pneumococcal pneumonia is

a. Penicillin
b. Ampicillin

298 ‫صفحة‬ Dr. MOHAMED YAHIA 0900987639


QUESTION BANK OF Dr. MOHAMED YAHIA

c. Erythromycin
d. Vancomycin
e. Ceftriaxone

The answer is: A

54. The antibiotic of choice for Lyme disease is

a. Penicillin
b. Ampicillin
c. Erythromycin
d. Vancomycin
e. Ceftriaxone

The answer is: E

55. The antibiotic of choice for streptococcal pharyngitis is

a. Penicillin
b. Ampicillin
c. Erythromycin
d. Vancomycin
e. Ceftriaxone

The answer is: A

56. The therapy of choice for pseudomembranous enterocolitis is

a. Penicillin
b. Ampicillin
c. Erythromycin
d. Vancomycin
e. Ceftriaxone

The answer is: D

57. N. gonorrhoeae is a fastidious pathogen and found in sites often


contaminated with normal flora. The best medium for isolation is

a. Sheep blood agar


b. Löffler’s medium
c. Thayer-Martin agar
d. Thiosulfate citrate bile salts sucrose medium
e. Löwenstein-Jensen medium

The answer is: C

299 ‫صفحة‬ Dr. MOHAMED YAHIA 0900987639


QUESTION BANK OF Dr. MOHAMED YAHIA

58. V. cholerae, the causative agent of cholera, is best isolated using

a. Sheep blood agar


b. Löffler’s medium
c. Thayer-Martin agar
d. Thiosulfate citrate bile salts sucrose medium
e. Löwenstein-Jensen medium

The answer is: D

59. M. tuberculosis can be found in the sputum of patients with tuberculosis.


After digestion of the sputum, isolation is best accomplished using

a. Sheep blood agar


b. Löffler’s medium
c. Thayer-Martin agar
d. Thiosulfate citrate bile salts sucrose medium
e. Löwenstein-Jensen medium

The answer is: E

60. C. diphtheriae may be difficult to isolate from the nasopharynx without the
use of special media. The medium of choice is

a. Sheep blood agar


b. Löffler’s medium
c. Thayer-Martin agar
d. Thiosulfate citrate bile salts sucrose medium
e. Löwenstein-Jensen medium

The answer is: B

61. S. aureus has a distinctive appearance on which one of the following media?

a. Sheep blood agar


b. Löffler’s medium
c. Thayer-Martin agar
d. Thiosulfate citrate bile salts sucrose medium
e. Löwenstein-Jensen medium

The answer is: A

62. Streptococcus pyogenes is a toxigenic bacterium causing a variety of diseases.


Which of the following statements best characterizes this organism?

a. It secretes erythrogenic toxin that causes the characteristic signs of scarlet


fever

300 ‫صفحة‬ Dr. MOHAMED YAHIA 0900987639


QUESTION BANK OF Dr. MOHAMED YAHIA

b. It produces toxin that blocks protein synthesis in an infected cell and carries a
lytic bacteriophage that produces the genetic information for toxin production
c. It produces at least one protein toxin consisting of two subunits, A and B, that
cause severe spasmodic cough usually in children
d. It has capsules of polyglutamic acid, which is toxic when injected into rabbits
e. It secretes exotoxin that has been called “verotoxin” and “Shiga-like toxin”;
infection is mediated by specific attachment to mucosal membranes

The answer is: A

63. N. meningitidis causes meningitis in all age groups. A characteristic


physiological trait is that it

a. Possesses N-acetylneuraminic acid capsule and adheres to specific tissues by


pili found on the bacterial cell surface
b. Has capsule of polyglutamic acid, which is toxic when injected into rabbits
c. Synthesizes protein toxin as a result of colonization of vaginal tampons
d. Causes spontaneous abortion and has tropism for placental tissue due to the
presence of erythritol in allantoic and amniotic fluid
e. Secretes two toxins, A and B, in large bowel during antibiotic therapy

The answer is: A

64. Brucella is pathogenic for humans and animals. Which one of the following
statements best characterizes this organism?

a. It has capsule of polyglutamic acid, which is toxic when injected into rabbits
b. It synthesizes protein toxin as a result of colonization of vaginal tampons
c. It causes spontaneous abortion and has tropism for placental tissue due to the
presence of erythritol in allantoic and amniotic fluid
d. It secretes two toxins, A and B, in large bowel during antibiotic therapy
e. It has 82 polysaccharide capsular types; capsule is antiphagocytic; type 3
capsule (b-d-glucuronic acid polymer) most commonly seen in infected adults

The answer is: C

Questions 65–69

An 18-year-old male patient appeared at the emergency room with a 3-day


history of fever, dry cough, difficulty breathing, and muscle aches and pains. His
chest x-ray showed a diffuse left upper lobe infiltrate. The following 5 questions
focus on the etiology of “atypical” or community- acquired pneumonia:

65. Mycoplasma pneumoniae pneumonia (walking pneumonia) may be rapidly


identified by which of the following procedures?

a. Detection of specific antigen in urine


b. Cold agglutinin test

301 ‫صفحة‬ Dr. MOHAMED YAHIA 0900987639


QUESTION BANK OF Dr. MOHAMED YAHIA

c. Electron microscopy of sputum


d. Culture of respiratory secretions in HeLa cells after centrifugation of the
inoculated tubes
e. Culture of respiratory secretions on monkey kidney cells

The answer is: B

66. Influenza can be treated; therefore, specific detection of the virus becomes
much more important. Which of the following would be best for detection of
influenza?

a. Detection of specific antigen in urine


b. Cold agglutinin test
c. Electron microscopy of sputum
d. Detection of antigen in respiratory secretions
e. Culture of respiratory secretions on monkey kidney cells

The answer is: A

67. Legionnaires’ disease is most rapidly diagnosed by which one of the following
procedures?

a. Detection of specific antigen in urine


b. Cold agglutinin test
c. Electron microscopy of sputum
d. Detection of antigen in respiratory secretions
e. Culture of respiratory secretions on a charcoal-based nutrient agar.

The answer is: D

68. Chlamydia pneumoniae has recently been implicated in respiratory disease


primarily in children. Which of the following would best isolate this fastidious
bacterium?

a. Detection of specific antigen in urine


b. Cold agglutinin test
c. Electron microscopy of sputum
d. Culture of respiratory secretions in HeLa cells after centrifugation of the
inoculated tubes
e. Culture of respiratory secretions on monkey kidney cells

The answer is: D

69. Bordetella bronchitis, sometimes called whooping cough, can best be detected
by which of the following procedures?

a. Fluorescent antibody detection of the organism in sputum


b. Cold agglutinin test

302 ‫صفحة‬ Dr. MOHAMED YAHIA 0900987639


QUESTION BANK OF Dr. MOHAMED YAHIA

c. Direct microscopy of sputum by Gram stain


d. Culture of respiratory secretions in HeLa cells after centrifugation of the
inoculated tubes
e. Culture of respiratory secretions on Regan-Lowe agar

The answer is: E

70. Which of the following is the predominant organism on skin commonly seen
as a blood culture contaminant?

a. α-hemolytic streptococci
b. Lactobacillus
c. S. epidermidis
d. Escherichia coli
e. B. fragilis

The answer is: C

71. Which of the following is the predominant flora of the mouth that is the
major cause of dental caries?

a. α-hemolytic streptococci
b. Lactobacillus
c. S. epidermidis
d. E. coli
e. B. fragilis

The answer is: A

72. The bowel contains many microorganisms but the most prevalent bacterium
is

a. α-hemolytic streptococci
b. Lactobacillus
c. S. epidermidis
d. E. coli
e. B. fragilis

The answer is: E

73. Which of the following is the most prevalent microorganism in the vagina
that may also be protective?

a. α-hemolytic streptococci
b. Lactobacillus
c. S. epidermidis
d. E. coli
e. B. fragilis

303 ‫صفحة‬ Dr. MOHAMED YAHIA 0900987639


QUESTION BANK OF Dr. MOHAMED YAHIA

The answer is: B

MYCOLOGY AND PARASITOLOGY:


1. A patient with AIDS has a persistent cough and has shown progressive
behavioral changes in the past few weeks after eating an undercooked
hamburger. A cerebrospinal fluid (CSF) sample is collected and an encapsulated,
yeast-like organism is observed. Based only on these observations, what is the
most likely organism?

a. Toxoplasma
b. Cryptosporidium
c. Candida
d. Cryptococcus
e. Pneumocystis

The answer is: D

2. A clinical diagnosis of meningitis is confirmed with a latex agglutination test


on CSF for the capsular polysaccharide of the organism. The most likely
causative agent is

a. Candida albicans
b. Cryptococcus
c. Paracoccidioides brasiliensis
d. Histoplasma capsulatum
e. Aspergillus fumigates

The answer is: B

3. A section of tissue from the foot of a person assumed to have eumycotic


mycetoma shows a white, lobulated granule composed of fungal hyphae. In the
United States, the most common etiologic agent of this condition is a species of

a. Acremonium
b. Nocardia
c. Actinomyces
d. Pseudallescheria (Petriellidium)
e. Madurella

The answer is: D

4. The formation of granulomas is seen in major systemic fungal infections.


Which of the following groups of fungi is most likely to cause granulomas?

a. Aspergillus, Coccidioides, Cryptococcus


b. Mucor, Candida, Malassezia
c. Cladosporium, Aspergillus, Microsporum

304 ‫صفحة‬ Dr. MOHAMED YAHIA 0900987639


QUESTION BANK OF Dr. MOHAMED YAHIA

d. Coccidioides, Blastomyces, Histoplasma


e. Epidermophyton, Blastomyces, Trichophyton

The answer is: D

5. C. albicans is recognized in microscopic examination of infected tissues by the


presence of

a. Spherules containing endospores


b. Metachromatic granules
c. Yeasts and pseudohyphae
d. Asci containing 2–8 ascospores
e. Abundance of septate rhizoids

The answer is: C

6. The mechanism of mucosal invasion by C. albicans is at least partially


understood. Which one of the following modifications in the structure or
function of this yeast would be most likely to affect its invasive ability?

a. Loss of ability to produce ethanol from glucose


b. Loss of ability to produce germ tubes or hyphae
c. Reduced ability to grow at 37°C
d. Loss of ability to produce a polysaccharide capsule
e. Replacement of mannans in the cell wall with glucan

The answer is: B

7. You have been designated as coordinator of construction of a bone marrow


transplant unit (BMTU). There will be extensive removal of walls and floors in
order to install the laminar flow rooms required for a BMTU. From the
standpoint of frequency and lethality, which one of the following fungi should be
your biggest concern?

a. Aspergillus
b. Candida
c. Wangiella
d. Cryptococcus
e. Blastomyces

The answer is: A

8. An immunocompromised patient is suspected of having an infection with A.


fumigatus. Which of the clinical conditions is most likely to occur?

a. Wound infection
b. Urinary tract infection
c. Invasive aspergillosis causing thrombosis and infarction

305 ‫صفحة‬ Dr. MOHAMED YAHIA 0900987639


QUESTION BANK OF Dr. MOHAMED YAHIA

d. Thrush
e. Superficial rash

The answer is: C

9. Human infection with the beef tapeworm, Taenia saginata, usually is less
serious than infection with the pork tapeworm, T. solium, because

a. Acute intestinal stoppage is less common in beef tapeworm infection


b. Larval invasion does not occur in beef tapeworm infection
c. Toxic by-products are not given off by the adult beef tapeworm
d. The adult beef tapeworms are smaller
e. Beef tapeworm eggs cause less irritation of the mucosa of the digestive tract

The answer is: B

10. Trypanosoma cruzi initially penetrates through the mucous membranes on


the skin and then multiplies in a lesion known as a chagoma. In the chronic stage
of the disease, the main lesions are often observed in the

a. Spleen and pancreas


b. Heart and digestive tract
c. Liver and spleen
d. Digestive tract and respiratory tract
e. Heart and liver

The answer is: B

11. A woman, recently returned from Africa, complains of having paroxysmal


attacks of chills, fever, and sweating; these attacks last a day or two at a time and
recur every 36 to 48 h. Examination of a stained blood specimen reveals ringlike
and crescent-like forms within red blood cells. The infecting organism most
likely is

a. Plasmodium falciparum
b. Plasmodium vivax
c. Trypanosoma gambiense
d. Wuchereria bancrofti
e. Schistosoma mansoni

The answer is: A

12. A “parasite” that may be a fungus is the initial clinical manifestation in up to


60% of patients with AIDS. This organism is

a. Microsporidium
b. Cryptosporidium
c. Pneumocystis

306 ‫صفحة‬ Dr. MOHAMED YAHIA 0900987639


QUESTION BANK OF Dr. MOHAMED YAHIA

d. Blastocystis
e. Blastomyces

The answer is: C

13. Schistosomiasis is a disease characterized by granulomatous reactions to the


ova or to products of the parasite at the place of oviposition. Clinical
manifestations include which one of the following?

a. Bladder wall hyperplasia


b. Pulmonary embolism
c. Splenomegaly
d. Cardiac abnormalities
e. Arthropathies

The answer is: C

307 ‫صفحة‬ Dr. MOHAMED YAHIA 0900987639


QUESTION BANK OF Dr. MOHAMED YAHIA

BRS MICROBIOLOGY

BACTERIOLOGY:

1. A bacterial toxin with superantigen activity is produced by

(A) Clostridium tetani


(B) Bordetella pertussis
(C) Escherichia coli
(D) Staphylococcus aureus
(E) Vibrio cholera

The answer is: D

2. Sugar transport into bacteria is frequently facilitated by

(A) Acetyltransferase
(B) Neuraminidases
(C) Oxidases
(D) Penicillin-binding proteins
(E) Phosphotransferases

The answer is: E

3. Thayer-Martin and Martin-Lewis media are used to isolate and identify

(A) E. coli
(B) Mycobacteria
(C) Neisseria
(D) Salmonella
(E) Shigella

The answer is: C

4. β-lactamases confer antibiotic resistance by

(A) Altering antibiotic permeability


(B) Altering penicillin-binding proteins
(C) Altering 70S ribosome structure
(D) Modifying cellular RNA polymerase
(E) Modifying antibiotic structure

The answer is: E

308 ‫صفحة‬ Dr. MOHAMED YAHIA 0900987639


QUESTION BANK OF Dr. MOHAMED YAHIA

5. Polymers of N-acetylglucosamine and N-acetylmuramic acid are found in


which of the following structures?

(A) Teichoic acid


(B) Cell wall
(C) Glycocalyx
(D) Lipopolysaccharide

The answer is: B

6. A phage that is not inactivated by proteases is called a

(A) Prophage
(B) Virulent phage
(C) Temperate phage
(D) Filamentous phage

The answer is: A

7. Bacteria that synthesize organic compounds from inorganic compounds are

(A) Heterotrophs
(B) Obligate anaerobes
(C) Aerobes
(D) Facultative anaerobes
(E) Autotrophs

The answer is: E

8. A bacterial structure involved in adherence is

(A) Capsule
(B) Lipopolysaccharide
(C) Common pili
(D) O-specific side chain
(E) Teichoic acid

The answer is: C

9. Aminoglycoside antibiotics are

(A) Bactericidal for Gram-positive bacteria


(B) Inactivated by R-factor phosphotransferases
(C) Mycolic acid synthesis inhibitors
(D) Peptidoglycan synthesis inhibitors
(E) Items that require bacterial growth for the effect

The answer is: E

309 ‫صفحة‬ Dr. MOHAMED YAHIA 0900987639


QUESTION BANK OF Dr. MOHAMED YAHIA

10. A-B subunit structure as it relates to bacterial pathogenesis refers to the


structure of

(A) Bacterial exotoxins


(B) Gram-negative bacteria endotoxin
(C) Nucleic acid inhibitor antibiotics
(D) Penicillin-binding proteins
(E) Resistance transfer factors

The answer is: A

11. Which of the following displays the Pasteur effect?

(A) Heterotrophs
(B) Obligate anaerobes
(C) Aerobes
(D) Facultative anaerobes
(E) Autotrophs

The answer is: D

12. Which of the following toxins acts on synaptosomes?

(A) E. coli heat-labile toxin


(B) Clostridium tetani exotoxin
(C) Corynebacterium diphtheriae exotoxin
(D) Pseudomonas aeruginosa exotoxin
(E) Clostridium perfringens alpha-toxin

The answer is: B

13. Superoxide dismutase-containing bacteria

(A) Need superoxide to grow


(B) Are frequently obligate anaerobes
(C) Grow slowly in the presence of CO 2
(D) Produce hydrogen peroxide from hydrogen ion and the superoxide free
radical (O 2 •2 )

The answer is: D

14. Lysogenic phage conversion involves

(A) The transformation of a virulent phage to a lysogenic phage


(B) A change in bacterial phenotype due to the presence of a prophage
(C) The conversion of a prophage to a temperate phage
(D) The incorporation of a prophage into the bacterial chromosome

The answer is: B

310 ‫صفحة‬ Dr. MOHAMED YAHIA 0900987639


QUESTION BANK OF Dr. MOHAMED YAHIA

15. Bacteria capable of growth in a high salt concentration are best isolated in
which of the following media?

(A) Minimal growth media


(B) Complex growth media
(C) Differential growth media
(D) Selective growth media

The answer is: D

16. Bacteria lacking superoxide dismutase are

(A) Heterotrophs
(B) Obligate anaerobes
(C) Aerobes
(D) Facultative anaerobes
(E) Autotrophs

The answer is: B

17. The regulation of enzyme activity in bacterial cells can

(A) Be coupled to the binding of effector molecules


(B) Be controlled by a catabolite activator protein (CAP)
(C) Occur via attenuation sequences
(D) Involve inducer molecules

The answer is: A

18. The plasma membrane

(A) Contains matrix porins


(B) Includes endotoxin
(C) Contains glycocalyx
(D) Contains the enzymes involved in bacterial oxidative phosphorylation

The answer is: D

19. Bacterial antibiotic resistance is frequently conveyed by

(A) A temperate bacteriophage


(B) An R-factor plasmid
(C) A replicon
(D) A lytic bacteriophage
(E) An intron

The answer is: B

20. The expression of the lac operon

311 ‫صفحة‬ Dr. MOHAMED YAHIA 0900987639


QUESTION BANK OF Dr. MOHAMED YAHIA

(A) Must be initiated by the binding of an inducer protein


(B) Involves the release of allolactose from a repressor protein
(C) Does not involve the expression of structural genes
(D) Necessitates the finding of RNA polymerase followed by transcription

The answer is: A

21. Bacteriophage containing host-cell DNA is involved in which of the following


processes?

(A) Transformation
(B) Conjugation
(C) Transduction
(D) Transcription
(E) Recombination

The answer is: C

22. The exchange of allelic forms of genes is involved in which of the following
processes?

(A) Transformation
(B) Conjugation
(C) Transduction
(D) Transcription
(E) Recombination

The answer is: E

23. Which of the following processes creates high-frequency recombination


donors?

(A) Transformation
(B) Conjugation
(C) Transduction
(D) Transcription
(E) Recombination
(F) Translation

The answer is: E

24. Toxins of enterotoxic Gram-negative bacteria are transferred outside of the


cell by

(A) ATP-activated pores


(B) GTP-coupled transporters

312 ‫صفحة‬ Dr. MOHAMED YAHIA 0900987639


QUESTION BANK OF Dr. MOHAMED YAHIA

(C) PBPs
(D) Pili
(E) Protein secretion systems

The answer is: E

25. A mutation which rarely disrupts gene product function is a

(A) Deletion
(B) Frameshift
(C) Insertion
(D) Nonsense
(E) Nucleotide substitution

The answer is: E

26. A 21-year-old male college student who had complained of headache and
feeling feverish the night before is brought this morning to the emergency
department (ED) when his roommate was unable to rouse him. He had been well
until yesterday. Vital signs include fever (39.8°C/103.1°F), tachycardia, and
hypotension (BP 70/55). Remarkable on physical examination is petechial rash
(purpuric in areas) and nuchal rigidity with positive Kernig and Brudzinski
signs. CSF is cloudy with high protein and low glucose. Intracellular, red
diplococci are seen on Gram stain. What is the most likely genus?

(A) Staphylococcus
(B) Streptococcus
(C) Chlamydia
(D) Mycobacterium
(E) Neisseria

The answer is: E

27. A 24-year-old female presents with dysuria, as well as urinary urgency and
frequency. A urine dipstick test is positive for both leukocyte esterase and
nitrites. What genus or family is noted for the production of nitrites?

(A) Escherichia
(B) Staphylococcus
(C) Streptococcus
(D) Vibrio

The answer is: A

28. What rapid test commonly used on Gram- negative rods rules out
Enterobacteriaceae if positive?

(A) Catalase

313 ‫صفحة‬ Dr. MOHAMED YAHIA 0900987639


QUESTION BANK OF Dr. MOHAMED YAHIA

(B) Coagulase
(C) Oxidase
(D) Chitinase
(E) Urease

The answer is: C

29. A patient presents with rapid onset severe respiratory symptoms. Chest
radiographs show a hemorrhagic lymphadenitis. The isolation of chains of fairly
large, aerobic Gram-positive rods, some of which have started to sporulate from
a patient with this presentation, should raise a major concern of which
organism? (You should be able to answer this question from the genus alone,
although a question might also mention that it was nonmotile.)

(A) Actinomyces israelii


(B) Bacillus anthracis
(C) Campylobacter jejuni
(D) Clostridium perfringens
(E) Haemophilus influenza

The answer is: B

30. A patient undergoing chemotherapy develops a cough. Acid-fast stain of his


sputum shows rods and slightly longer forms, with some branching; they vary in
their acid- fast reaction from one area of the slide to the next. The acid-fast stain
was performed by an experienced medical technologist and, when redone,
showed the same variation. The growth was done aerobically. What is the most
likely agent?

(A) Actinomyces
(B) Chlamydophila
(C) Mycobacterium avian-intracellulare (MAI or MAC)
(D) Nocardia

The answer is: D

31. A female patient with a new genital lesion presents to your sexually
transmitted disease clinic. She is homeless, has no health insurance, and is an
intravenous drug user. You suspect syphilis. Which of these techniques would be
most appropriate to demonstrate treponemes?

(A) Immunological test such as the VDRL


(B) Dark-field microscopy
(C) Acid-fast stain
(D) Gram stain

314 ‫صفحة‬ Dr. MOHAMED YAHIA 0900987639


QUESTION BANK OF Dr. MOHAMED YAHIA

(E) Electrophoresis

The answer is: B

32. The CSF from a 2-week-old infant with meningitis shows rods with tumbling
motility. These bacteria are found to be Gram-positive and do not form spores.
What is the most likely agent?

(A) Actinomyces
(B) Bacillus
(C) Clostridium
(D) Corynebacterium
(E) Listeria

The answer is: E

33. Both a 53-year-old farmer and his 21-year old son present in August with
fever, myalgia, and malaise, which they came down with within a few hours of
each other. The son had been home in southern Minnesota for only 3 weeks to
help field train two new hunting dogs. You ask about potential tick bites, and the
son did have one on him, which was quite engorged. Platelets and granulocytes
are low in each man’s blood. You ask one of your experienced techs to do a
Giemsa stain on a thick blood smear. He calls, reporting clusters of cells
resembling raspberries in granulocytes, even though nothing grows in any of the
blood cultures. You realize that the blood cultures you set up will not grow and
that you have two patients who have infections with a tick-borne obligate
intracellular parasite of granulocytes. What genus does the organism belong to?

(A) Anaplasma (formerly Ehrlichia)


(B) Borrelia
(C) Chlamydia
(D) Haemophilus
(E) Mycoplasma

The answer is: A

34. A full-term 6-day-old neonate is brought in with a purulent conjunctivitis


which the parents noticed earlier today. On Gram stain of the purulent exudate,
no bacteria are seen. Which of the following bacteria is most likely the cause of
the conjunctivitis?

(A) Chlamydia trachomatis


(B) Escherichia coli
(C) Listeria monocytogenes
(D) Neisseria gonorrhoeae
(E) Streptococcus pneumonia

315 ‫صفحة‬ Dr. MOHAMED YAHIA 0900987639


QUESTION BANK OF Dr. MOHAMED YAHIA

The answer is: A

35. An 83-year-old who still lives in her own home has developed pneumonia
following influenza. The Gram stain of her sputa is shown. What is the most
likely agent?

(A) Chlamydophila pneumoniae


(B) Influenza virus
(C) Klebsiella pneumoniae
(D) Mycoplasma pneumoniae
(E) Staphylococcus aureus
(F) Streptococcus pneumonia

The answer is: E

36. The reagent used to distinguish staphylococci from streptococci is

(A) Hydrogen peroxide


(B) Fibronectin
(C) Fibrinogen
(D) Oxidase

The answer is: A

37. A 14-month-old boy is brought in by his parents with fever, fussiness and
lethargy, and apparent headache. On examination, the neck is stiff. His parents
have not allowed his routine childhood vaccines. Very short Gram negative rods
are seen in the CSF, so antibiotics are immediately started. The organism grows
on chocolate agar but not blood agar. No one else in the family is ill. What is the
most likely causative agent?

(A) Escherichia coli


(B) Haemophilus influenzae type b
(C) Klebsiella pneumoniae
(D) Neisseria meningitidis
(E) Streptococcus pneumonia

The answer is: B

38. A healthy 7-year-old boy who has not traveled outside the United States is
brought in by his parents in June with signs of meningitis. No bacteria are seen
in the Gram stain of the CSF, and no bacterial capsule material is present as
determined by a series of latex particle agglutination tests standard to the
diagnosis of meningitis. The CSF glucose level is slightly low, protein is near
normal, and white cell count is less than 500 cells/microL, mainly lymphocytes.
What is the most likely causative agent?

(A) Chlamydophila pneumoniae

316 ‫صفحة‬ Dr. MOHAMED YAHIA 0900987639


QUESTION BANK OF Dr. MOHAMED YAHIA

(B) Enterovirus
(C) Mycoplasma pneumoniae
(D) Mycobacterium tuberculosis
(E) Treponema pallidum

The answer is: B

39. In any case, a clue indicating that the causative organism is an obligate
intracellular pathogen transmitted by an arthropod bite should lead you to
which of the following groups of organisms?

(A) Chlamydiae
(B) Enterobacteriaceae
(C) Rickettsias including Anaplasma and Ehrlichia
(D) Spirochetes including Borrelia burgdorferi

The answer is: C

40. A 36-year-old man presents with focal central nervous system signs. Imaging
shows a brain abscess. The dominant organism is an anaerobe normally found as
part of the oral flora. Which of the following best fits that description?

(A) Nocardia
(B) Actinomyces
(C) Mycobacterium
(D) Pseudomonas aeruginosa

The answer is: B

41. A 23-year-old man who has recently started working on a sheep farm in Nova
Scotia develops pneumonia shortly after helping with lambing. His cough
produces little sputum, and a saline-induced sputum sample shows no
predominant organism either with Gram stain or with acid-fast stain. It is
established that he acquired the pneumonia from parturition products from the
sheep. Which agent is most likely to be the cause of his pneumonia?

(A) Rickettsia akari


(B) Rickettsia typhi
(C) Rickettsia rickettsii
(D) Coxiella burnetii
(E) Anaplasma phagocytophilum

The answer is: D

42. A 3-year-old girl presents with difficulty breathing and will not lie down to be
examined. You suspect acute bacterial epiglottitis and examine the child’s
epiglottis, which is highly inflamed. Which vaccine are you most likely to find
that the child is missing?

317 ‫صفحة‬ Dr. MOHAMED YAHIA 0900987639


QUESTION BANK OF Dr. MOHAMED YAHIA

(A) Diphtheria
(B) Neisseria meningitidis
(C) Polio
(D) Streptococcus pneumoniae (conjugate vaccine)
(E) Haemophilus influenza

The answer is: E

43. A 22-year-old man with cystic fibrosis presents with fever and increasing
dyspnea. A Gram-negative organism is found in unusually high numbers in the
pulmonary mucus. Which virulence factor is most important in colonization and
maintenance of the organism in the lungs?

(A) Exotoxin A
(B) Pyocyanin (blue-green pigment)
(C) Polysaccharide slime
(D) Endotoxin

The answer is: C

44. From the above case, exotoxin A of the causative agent most closely
resembles the action of which other microbial toxin?

(A) Heat-labile toxin (LT) of Escherichia coli


(B) Shiga toxin
(C) Diphtheria toxin
(D) Vibrio cholerae toxin
(E) Verotoxin

The answer is: C

45. A 36-year-old man who immigrated to the United States 15 years ago and
lived in a crowded resettlement camp before coming to the United States
presents with a cough that has been bothering him for several weeks. He has also
lost 10 pounds. A gamma interferon release blood test is positive. Which of the
following factors is known to be most important in triggering the granulomatous
reaction to wall off and contain the infection?

(A) Cord factor


(B) Mycolic acid
(C) Purified protein derivative (PPD)
(D) Sulfatides
(E) Wax D

The answer is: A

318 ‫صفحة‬ Dr. MOHAMED YAHIA 0900987639


QUESTION BANK OF Dr. MOHAMED YAHIA

46. A 75-year-old patient develops diarrhea 5 days after starting antibiotic


treatment for a serious staphylococcal infection. What is the most likely
causative agent?

(A) Clostridium perfringens


(B) Clostridium difficile
(C) Pseudomonas aeruginosa
(D) Shigella sonnei

The answer is: B

47. A 23-year-old woman presents with mild gastroenteritis a few days after
having a variety of sushi at a party. There is no blood or pus in the stool. Which
causative agent is most likely to have caused this illness?

(A) Vibrio cholerae


(B) Vibrio parahaemolyticus
(C) Salmonella typhi
(D) Shigella sonnei

The answer is: B

48. Yersinia pestis may be transferred by

(A) Dermacentor tick bite


(B) Human body louse bite
(C) Ixodes tick bite
(D) Respiratory droplets

The answer is: D

49. A patient who had surgery to put in a pace- maker and who states he felt fine
for the first 2 months now presents 3 months postoperatively with complaints of
malaise and increasing fatigue. He is running a low-grade fever, tires easily, and
has worsening heart murmurs. Which of the following staphylococcal organisms
causes subacute bacterial endocarditis that generally occurs 2 months or more
after heart surgery?

(A) Staphylococcus aureus


(B) Staphylococcus epidermidis
(C) Staphylococcus haemolyticus
(D) Staphylococcus saprophyticus

The answer is: B

50. A previously healthy 6-month-old boy presents with upper body weakness.
He cannot hold his eyes open, pupils do not react, and he cannot hold his head
up. What is the proper treatment?

319 ‫صفحة‬ Dr. MOHAMED YAHIA 0900987639


QUESTION BANK OF Dr. MOHAMED YAHIA

(A) Send him home on amoxicillin and clindamycin (to stop the toxin production
quickly)
(B) Give him a dose of equine anti-botulinum immunoglobulin
(C) Offer monitored supportive care with antibiotics and human anti-botulinum
immunoglobulin
(D) Offer monitored supportive care with human anti-botulinum
immunoglobulin
(E) Offer monitored supportive care with no antibiotics and no antitoxin

The answer is: D

51. A 78-year-old man presents with a high fever, cough producing a blood-
tinged sputum, and difficulty breathing. Sputum shows an organism consistent
with Streptococcus pneumoniae. What is the most important virulence factor?

(A) Endotoxin
(B) A phospholipase allowing Streptococcus pneumoniae to escape the
phagosome quickly
(C) Polypeptide capsule
(D) Polysaccharide capsule

The answer is: D

52. Which of the following organisms grows in 40% bile?

(A) Enterococcus faecalis


(B) Streptococcus pneumoniae
(C) Group B streptococci
(D) Viridans streptococci

The answer is: A

53. A 45-year-old man who recently returned from Africa has been febrile for
several days and now presents with abdominal pain. His blood cultures grow out
Salmonella typhi. What was the most likely source of his infection?

(A) Raw chicken


(B) Undercooked hamburger
(C) Contact with baby goats on a farm and then eating without washing hands
(D) A food preparer with bad personal hygiene
(E) Undercooked pork

The answer is: D

54. A 4-day-old infant girl now showing signs of sepsis is brought to the
emergency department. She was preterm (33 weeks) and born at home to her 16-
year-old mom after 22 hours of labor following the rupture of the membranes. A
friend helped the mother deliver the baby. What is the best description for the

320 ‫صفحة‬ Dr. MOHAMED YAHIA 0900987639


QUESTION BANK OF Dr. MOHAMED YAHIA

agent most likely causing the sepsis if it was acquired during labor but prior to
delivery? All organisms in the answer choices are Gram-positive, catalase
negative cocci found in pairs or short chains.

(A) Nonhemolytic organisms found as part of the normal fecal flora; resistant to
bile and optochin; carries a high level of drug resistance
(B) Alpha-hemolytic diplococci sensitive to both bile and optochin
(C) Beta-hemolytic cocci in chains and carrying Lancefield’s Group B antigen
(D) Alpha-hemolytic cocci in chains; resistant to bile and optochin

The answer is: C

55. A 62-year-old woman presents with signs of a gastric ulcer. She does not
regularly take nonsteroidal anti-inflammatory agents. Which characteristic
appears to play a central role in the organism’s ability to survive transit of the
lumen to colonize the stomach?

(A) Phospholipase-C production


(B) Urease production
(C) Microaerophilic lifestyle
(D) O antigens

The answer is: B

56. A 54-year-old man develops a pyogenic infection along the suture line after
knee surgery. The laboratory gives a preliminary report of a beta-hemolytic,
catalase- positive, coagulase-positive, Gram- positive coccus. The most likely
causative agent is

(A) Moraxella catarrhalis


(B) Staphylococcus aureus
(C) Staphylococcus epidermidis
(D) Streptococcus agalactiae

The answer is: B

VIROLOGY:
1. Clinical viral disease

(A) Is most frequently due to toxin production


(B) Usually follows virus infection
(C) Can result without infection of host cells
(D) Is associated with target organs in most disseminated viral infections

The answer is: D

2. Linear, single-stranded DNA is the genetic material of

321 ‫صفحة‬ Dr. MOHAMED YAHIA 0900987639


QUESTION BANK OF Dr. MOHAMED YAHIA

(A) Caliciviruses
(B) Flaviviruses
(C) Papillomaviruses
(D) Parvoviruses

The answer is: D

3. Amantadine inhibits

(A) Influenza A and B virus hemagglutinin binding activity


(B) Influenza A virus M2 protein activity
(C) Influenza A and B virus neuraminidase activity
(D) Influenza B virus RNA-dependent RNA polymerase activity

The answer is: B

4. Viruses whose genomes have a messenger (positive-sense) polarity are

(A) Adenoviruses
(B) Papovaviruses
(C) Paramyxoviruses
(D) Polioviruses

The answer is: D

5. Dane particles are associated with

(A) Hepatitis A virus


(B) Hepatitis B virus
(C) Hepatitis C virus
(D) Hepatitis E virus

The answer is: B

6. The Monospot test is based on

(A) Destruction of Downey cells


(B) Heterophile antibodies
(C) Syncytia inhibition
(D) Viral capsid antigen antibodies

The answer is: B

7. An 8-year-old boy is brought to your office by his mother. He has had a slight
fever and a sore throat for the past 2 days. He has eight ulcerative lesions in his
mouth, three vesicular lesions on his left hand, and five similar lesions on his
right foot. The most probable cause of his disease is

322 ‫صفحة‬ Dr. MOHAMED YAHIA 0900987639


QUESTION BANK OF Dr. MOHAMED YAHIA

(A) Coxsackie A virus


(B) Human herpes virus 6
(C) HSV
(D) HPV

The answer is: A

8. A 16-year-old boy presents at your office with a sore throat, fever, and
enlarged lymph nodes. His tonsils are enlarged, the pharynx is inflamed, and
splenomegaly is observed. He complains of severe fatigue. Confirmation of the
causative agent is best done by observing

(A) A positive Tzanck smear


(B) IgM heterophile antibodies
(C) Koilocytotic cells
(D) RT-PCR for enterovirus

The answer is: B

9. The most common cause of congenital infections is

(A) CMV
(B) HSV
(C) Parvovirus
(D) Rubella virus

The answer is: A

10. A 23-year-old medical student on the Caribbean island of Dominica presents


at the Student Health Clinic complaining of an increasingly severe headache and
back and bone pain. Yesterday she was nauseated and vomited several times
during the night. She has a 39.5°C/103 ° F fever, which appeared suddenly, and a
generalized rash that blanches under pressure. She had been hiking in the
rainforest 1 week earlier and was particularly bothered by mosquitoes at that
time. The most likely infectious agent causing her symptoms is

(A) Dengue virus


(B) LCM virus
(C) West Nile virus
(D) Yellow fever virus

The answer is: A

11. A 4-year-old girl is brought to your rural clinic office by her mother who
states the child has a runny nose, barking cough, and a sore throat. Your
examination indicates respiration is labored. None of her three siblings is sick.
The most probable viral cause of her symptoms is

323 ‫صفحة‬ Dr. MOHAMED YAHIA 0900987639


QUESTION BANK OF Dr. MOHAMED YAHIA

(A) Adenovirus
(B) Influenza virus
(C) Parainfluenza virus
(D) RSV

The answer is: C

12. The individual most likely to develop chronic liver disease is a

(A) 1-month-old infant infected with HBV


(B) 22-year-old coinfected with HBV and HDV
(C) 26-year-old alcoholic intravenous drug abuser infected with HBV
(D) 30-year-old infected with yellow fever virus

The answer is: A

13. A 64-year-old man living on a farm in southern Minnesota is brought on July


15 to the emergency room by his brother. The brother said the man had a 2-day
history of fever, headache, and some vomiting, but today he appeared confused.
He is confused by some of the simple questions you ask him. His spinal tap is
clear with 75% PMNs and a head CT is normal. The most likely cause of his
symptoms is

(A) California encephalitis virus


(B) Enterovirus
(C) HSV
(D) West Nile virus

The answer is: D

14. A mother brings her 18-month-old son to your office. She was called by her
day care center who reported he had vomited twice during the morning and had
diarrhea as well. She noted he had a slight fever the past 2 days and had not been
very hungry. The most likely cause of his illness is

(A) Adenovirus
(B) Astrovirus
(C) Norovirus
(D) Rotavirus

The answer is: D

15. A vaccine is available for protection against the disease observed in review
question 8. The immunizing agent(s) for this vaccine is a(n)

(A) Attenuated virus


(B) Formalin-inactivated virus
(C) Preparation of reassortment viruses

324 ‫صفحة‬ Dr. MOHAMED YAHIA 0900987639


QUESTION BANK OF Dr. MOHAMED YAHIA

(D) Viral attachment protein preparation

The answer is: C

16. Donors for liver transplants must be monitored and determined to be


serologically negative for previous

(A) CMV infections


(B) EBV infections
(C) HBV infections
(D) HEV infections

The answer is: C

17. On September 17, a 22-year-old male college student appears at the Student
Health Clinic complaining of moderate headache, nausea, and vomiting. His
temperature is 38.5°C/101°F and his physical examination shows stiffness in the
neck. What is the most likely viral cause of the symptoms?

(A) CMV
(B) Enterovirus
(C) EBV
(D) HSV type 1

The answer is: B

18. Knowing the genotype of the causative virus is important for determining the
treatment of chronic

(A) CMV infections


(B) HCV infections
(C) IC virus infections
(D) VZV infections

The answer is: B

MYCOLOGY
1. A florist presents with a subcutaneous lesion on the hand, which she thinks
resulted from a jab wound she received while she was making a sphagnum moss-
wire frame for a floral wreath. The nodule has ulcerated and not healed despite
use of antibacterial cream, and a new nodule is forming above the original lesion.
What is most likely to be an appropriate treatment for this infection?

(A) Oral itraconazole or potassium iodide


(B) Miconazole cream

325 ‫صفحة‬ Dr. MOHAMED YAHIA 0900987639


QUESTION BANK OF Dr. MOHAMED YAHIA

(C) Cortisone cream


(D) Oral griseofulvin
(E) Penicillin

The answer is: A

2. Although hard to find in the above mentioned nodule, what form would be
present in the tissue?

(A) Lots of hyphae


(B) Long, branching hyphae with acute angles
(C) Yeasts with broad-based buds
(D) Cigar-shaped to oval yeasts
(E) Yeast with multiple buds (mariner’s wheel)

The answer is: D

3. A patient presents with paranasal swelling and bloody exudate from both his
eyes and nares, and he is nearly comatose. Necrotic tissue in the nasal turbinates
show nonseptate hyphae consistent with Rhizopus, Mucor, or Absidia (phylum
Zygomycota, class Phycomycetes). What is the most likely compromising
condition underlying this infection?

(A) AIDS
(B) Ketoacidotic diabetes
(C) Neutropenia
(D) B-cell defects
(E) Chronic sinusitis

The answer is: B

4. A patient presents with a circular, itchy, inflamed skin lesion that is slightly
raised; it is on his left side where his dog sleeps next to him. His dog has had
some localized areas of hair loss. The patient has no systemic symptoms. What
would you expect to find in a KOH of skin scrapings?

(A) Clusters of yeastlike cells and short curved septate hyphae


(B) Hyphae with little branching but possibly with some hyphae breaking up into
arthroconidia
(C) Filariform larvae
(D) Budding yeasts with some pseudohyphae and true hyphae
(E) Large budding yeast cells with broad bases on the buds and thick cell walls

The answer is: B

5. A severely neutropenic patient presents with pneumonia. Bronchial alveolar


fluid shows dichotomously branching (generally with acute angles), septate
hyphae. What is the most likely causative agent?

326 ‫صفحة‬ Dr. MOHAMED YAHIA 0900987639


QUESTION BANK OF Dr. MOHAMED YAHIA

(A) Aspergillus
(B) Cryptococcus
(C) Candida
(D) Malassezia
(E) Rhizopus

The answer is: A

6. What is a mass of fungal filaments called?

(A) Pseudohyphae
(B) Hyphae
(C) Mycelium
(D) Septum
(E) Yeast

The answer is: C

7. A premature infant on intravenous nutrients and high-lipid fluids has


developed septicemia that cultures out on blood agar only when over- laid with
sterile olive oil. What is the most likely causative agent?

(A) Aspergillus
(B) Candida
(C) Cryptococcus
(D) Malassezia
(E) Sporothrix

The answer is: D

8. A filamentous fungus subunit is a

(A) Coenocyte
(B) Hypha
(C) Mycelium
(D) Septum
(E) Yeast

The answer is: B

9. To treat a patient with a life-threatening fungal infection, you choose an


antifungal drug that causes pore formation in the fungal membrane and actually
kills the cells. Which drug would this be?

(A) Amphotericin B
(B) Griseofulvin
(C) Ketoconazole
(D) Miconazole

327 ‫صفحة‬ Dr. MOHAMED YAHIA 0900987639


QUESTION BANK OF Dr. MOHAMED YAHIA

(E) Nystatin

The answer is: A

10. A 15-year-old dirt-bike rider visiting southern California the first time has
developed pneumonia. The causative organism has environmental form that
consists of hyphae that break up into arthroconidia, which become airborne.
What is the agent?

(A) Aspergillus fumigatus


(B) Blastomyces dermatitidis
(C) Coccidioides immitis
(D) Histoplasma capsulatum
(E) Sporothrix schenckii

The answer is: C

11. Which of the following drugs inhibits ergosterol synthesis, is important in


treating Candida fungemias, and is used orally to suppress relapses of
cryptococcal meningitis in AIDS patients?

(A) Amphotericin B
(B) Fluconazole
(C) Griseofulvin
(D) Echinocandins
(E) Nystatin

The answer is: B

12. A patient has splotchy hypopigmentation on the chest and back with only
slight itchiness. What is most likely to be seen on a KOH mount of the skin
scraping?

(A) Yeasts, pseudohyphae, and true hyphae


(B) Filaments with lots of arthroconidia
(C) Clusters of round fungal cells with short, curved, septate hyphae
(D) Darkly pigmented, round cells with sharp interior septations
(E) Cigar-shaped yeasts

The answer is: C

13. A patient has a dry, scaly, erythematous penis. Skin scales stained with
calcofluor white show fluorescent blue-white yeasts and a few pseudohyphae.
What is the causative agent of this dermatophytic look-alike?

(A) Candida
(B) Trichosporon
(C) Trichophyton

328 ‫صفحة‬ Dr. MOHAMED YAHIA 0900987639


QUESTION BANK OF Dr. MOHAMED YAHIA

(D) Malassezia
(E) Microsporum

The answer is: A

14. A recent immigrant from rural Brazil presents with a swollen face and
extremely poor dental hygiene, including loss of an adult tooth, which appears to
be the focus of the current infection. There are two open ulcers on the outside of
the swollen cheek. Small yellow “grains” are seen in one of the ulcers. Gram
stain shows purple- staining fine filaments. What is the most likely disease?

(A) Actinomycotic mycetoma


(B) Chromomycosis
(C) Eumycotic mycetoma
(D) Sporotrichosis
(E) Paracoccidioidomycosis

The answer is: A

15. A patient who is a recent immigrant from a tropical, remote, rural area with
no medical care is now working with a group of migrant crop harvesters. He has
a large, raised, colored, cauliflower-like ankle lesion. Darkly pigmented, yeastlike
sclerotic bodies are seen in the tissue biopsy. Which of the following is the most
likely diagnosis?

(A) Actinomycotic mycetoma


(B) Chromoblastomycosis
(C) Eumycotic mycetoma
(D) Sporotrichosis
(E) Tinea nigra

The answer is: B

16. A premature baby, now 4 days old, has developed a white coating on her
buccal mucosa extending onto her lips. It appears to be painful. What is the most
likely causative agent?

(A) Actinomyces
(B) Aspergillus
(C) Candida
(D) Fusobacterium
(E) Microsporum

The answer is: C

17. Which of the following stains allows differentiation of fungus from human
tissue by staining the fungus a pink-red color?

329 ‫صفحة‬ Dr. MOHAMED YAHIA 0900987639


QUESTION BANK OF Dr. MOHAMED YAHIA

(A) Calcofluor white stain


(B) Gomori methenamine-silver stain
(C) Periodic acid-Schiff stain
(D) Hematoxylin and eosin stain

The answer is: C

18. A normally healthy 8-year-old boy from Florida is visiting friends on a farm
in Iowa during the month of July. He presents on July 28 with a fever, cough,
and lower respiratory symptoms (no upper respiratory tract symptoms). He has
been ill for 4 days. His chest sounds are consistent with pneumonia, so a chest
radiograph is obtained. The radiograph shows small, patchy infiltrates with hilar
adenopathy. His blood smear shows small, nondescript yeast forms inside
monocytic cells. What is the most likely causative agent?

(A) Aspergillus fumigatus


(B) Blastomyces dermatitidis
(C) Coccidioides immitis
(D) Histoplasma capsulatum
(E) Pneumocystis jiroveci

The answer is: D

19. Which of the following is a polyene antifungal agent used for many life-
threatening fungal infections?

(A) Amphotericin B
(B) Griseofulvin
(C) Itraconazole
(D) Miconazole
(E) Nystatin

The answer is: A

20. A logger undergoing chemotherapy for cancer has developed pneumonia and
skin lesions. Biopsy of the skin lesions demonstrates the presence of large yeasts
with thick cell walls and broad-based buds. What is the most likely causative
agent?

(A) Aspergillus fumigatus


(B) Blastomyces dermatitidis
(C) Coccidioides immitis
(D) Histoplasma capsulatum
(E) Sporothrix schenckii

The answer is: B

21. What is the scientific name for a fungal cross wall?

330 ‫صفحة‬ Dr. MOHAMED YAHIA 0900987639


QUESTION BANK OF Dr. MOHAMED YAHIA

(A) Coenocyte
(B) Hypha
(C) Mycelium
(D) Septum
(E) Yeast

The answer is: B

22. A noncompliant, human immunodeficiency virus (HIV)-positive patient has


been complaining of a stiff neck and a severe headache. The headache was
initially lessened by analgesics, but the analgesics are no longer effective. His
current CD4 + count is 180/mm 3 . He is not on any prophylactic drugs. What is
the most likely causative agent?

(A) Aspergillus
(B) Cryptococcus
(C) Candida
(D) Malassezia
(E) Sporothrix

The answer is: B

23. Which of the following features differentiates fungal cells from human cells?

(A) 80S ribosomes


(B) Presence of an endoplasmic reticulum
(C) Ergosterol as the major membrane sterol
(D) Enzymes that allow them to use carbon dioxide as their sole carbon source
(E) Presence of chloroplasts

The answer is: C

PARASITOLOGY
1. A biology graduate student who recently visited a tropical region of Africa
presents with new visual impairment and the sensation that something is moving
in her eye. She tells you that she is concerned because she had been warned
about eye disease transmitted by black flies. When in Africa, she was in a river
area, and despite her best efforts she received a lot of black fly bites. She also has
some subcutaneous nodules. If her infection was acquired by black fly bite, what
is the most likely causative agent?

(A) Ancylostoma braziliense


(B) Dracunculus medinensis
(C) Loa loa

331 ‫صفحة‬ Dr. MOHAMED YAHIA 0900987639


QUESTION BANK OF Dr. MOHAMED YAHIA

(D) Onchocerca volvulus


(E) Wuchereria bancrofti

The answer is: D

2. A woman who imports food from Mexico and spends several months per year
in rural Mexico had to have a compound leg fracture pinned and set in Mexico
and has returned 3 days later. She now has signs of acute appendicitis and is
taken to surgery in Houston. When her appendix is removed, it is found to
contain a light-colored, 20.5-cm-long roundworm as well as bile-stained, knobby
eggs consistent with Ascaris. How did she acquire this infection?

(A) Ingestion of water containing filariform larvae


(B) Skin penetration by filariform larvae
(C) Skin penetration by rhabditiform larvae
(D) Ingestion of food contaminated with the eggs
(E) Inhalation of dust carrying the cysts

The answer is: D

3. A patient whose major source of protein is smoked and cooked fish develops
what appears to be pernicious anemia. What parasite is noted for causing a look-
alike vitamin B 12 anemia in certain genetically predisposed infected
individuals?

(A) Echinococcus granulosus


(B) Diphyllobothrium latum
(C) Hymenolepis nana
(D) Dipylidium caninum
(E) Taenia solium

The answer is: B

4. Which of the following protozoans is free living and is such that acquisition
does not generally indicate fecal contamination?

(A) Acanthamoeba
(B) Dientamoeba fragilis
(C) Entamoeba histolytica
(D) Entamoeba coli
(E) Giardia

The answer is: A

5. A 26-year-old woman with uncomplicated malaria who was treated initially


with chloroquine now has relapsed. What is the reason for a chloroquine-treated
case of Plasmodium vivax relapsing?

332 ‫صفحة‬ Dr. MOHAMED YAHIA 0900987639


QUESTION BANK OF Dr. MOHAMED YAHIA

(A) P. vivax has a significant level of chloroquine resistance.


(B) P. vivax has a persistent erythrocytic stage.
(C) P. vivax has a persistent exoerythrocytic stage (hypnozoite).
(D) Chloroquine is not one of the drugs of choice.

The answer is: C

6. How is Leishmania donovani transmitted?

(A) Anopheles mosquito bite


(B) Black fly bite
(C) Culex mosquito bite
(D) Sandfly bite
(E) Skin penetration by trauma

The answer is: D

7. How is Schistosoma haematobium transmitted?

(A) Ingestion of raw or undercooked snail, frog, or snake


(B) Invasion of filariform larvae from soil
(C) Handling aquatic birds
(D) Standing or swimming in contaminated water
(E) Tsetse fly bite

The answer is: D

8. An untreated AIDS patient (CD4 1 count of 180 cells/mm 3 ) from southern


California has developed a progressively severe headache and mental confusion,
along with ataxia and retinochoroiditis. Focal lesions are present on a computed
tomography scan of his brain. No mucocutaneous lesions are found. He has been
living under a bridge for the past 2 years. His level of immunoglobulin G (IgG)
to the infectious agent is high. What is the most likely explanation for how this
current infection started?

(A) Earlier exposure to pigeons


(B) Earlier exposure to desert sand
(C) Reactivation of bradyzoites in cysts from an earlier infection
(D) Recent exposure to cat feces
(E) Recent exposure to bats

The answer is: C

9. Which of the following is the tapeworm acquired from eating undercooked


pork?

(A) Dipylidium spp.


(B) Echinococcus granulosus

333 ‫صفحة‬ Dr. MOHAMED YAHIA 0900987639


QUESTION BANK OF Dr. MOHAMED YAHIA

(C) Taenia saginata


(D) Taenia solium
(E) Trichinella spiralis

The answer is: D

10. What roundworm is most likely to be transmitted by ingestion of food or


water contaminated with feces?

(A) Ascaris lumbricoides


(B) Enterobius vermicularis
(C) Necator americanus
(D) Taenia saginata
(E) Toxocara canis

The answer is: A

11. What roundworm is transmitted by filariform larvae that are found in the
soil and penetrate the skin?

(A) Dracunculus medinensis


(B) Enterobius vermicularis
(C) Strongyloides stercoralis
(D) Taenia saginata
(E) Toxocara canis

The answer is: C

12. How is Clonorchis sinensis (Chinese liver fluke) most likely transmitted to
humans?

(A) Fish ingestion


(B) Mosquito bite
(C) Swimming or water contact
(D) Rare beef ingestion
(E) Mango fly (Chrysops)

The answer is: A

13. A 48-year-old subsistence farmer from rural Brazil dies of heart failure. His
autopsy shows a greatly enlarged heart. What was the vector for the most likely
infectious agent that may have been responsible for his death?

(A) Ixodes tick


(B) Mosquito
(C) Reduviid bug
(D) Sandfly
(E) Tsetse fly

334 ‫صفحة‬ Dr. MOHAMED YAHIA 0900987639


QUESTION BANK OF Dr. MOHAMED YAHIA

The answer is: C

14. A 16-year-old man who recently returned from camping in Canada presents
with fatty diarrhea and acute abdominal pain following many meals. How does
the most likely agent cause the diarrhea?

(A) Coinfection with bacteria


(B) Enterotoxin production
(C) Suction disk attachment
(D) Tissue invasion leading to an inflammatory response and prostaglandin
production

The answer is: C

15. Which of the following protozoans is transmitted primarily by the motile


trophozoite form?

(A) Balantidium coli


(B) Entamoeba histolytica
(C) Giardia lamblia
(D) Taenia solium
(E) Trichomonas vaginalis

The answer is: E

335 ‫صفحة‬ Dr. MOHAMED YAHIA 0900987639


QUESTION BANK OF Dr. MOHAMED YAHIA

CHAPTER 3

241 Question

336 ‫صفحة‬ Dr. MOHAMED YAHIA 0900987639


QUESTION BANK OF Dr. MOHAMED YAHIA

PRETEST BIOCHEMISTRY

METABOLISM
1. A child develops chronic diarrhea and liver inflammation in early infancy
when the mother begins using formula that includes corn syrup. Evaluation of
the child demonstrates sensitivity to fructose in the diet. Which of the following
glycosides contains fructose and therefore should be avoided when feeding or
treating this infant?

a. Sucrose
b. Oaubain
c. Lactose
d. Maltose
e. Streptomycin

The answer is: A

2. Which of the following carbohydrates would be most abundant in the diet of


strict vegetarians?

a. Amylose
b. Lactose
c. Cellulose
d. Maltose
e. Glycogen

The answer is: C

3. The major metabolic product produced under normal circumstances by


erythrocytes and by muscle cells during intense exercise is recycled through the
liver in the Cori cycle. The metabolite is

a. Oxaloacetate
b. Glycerol
c. Alanine
d. Pyruvate
e. Lactate

The answer is: E

4. Chronic alcoholics require more ethanol than do nondrinkers to become


intoxicated because of a higher level of specific enzyme. However, independent of

337 ‫صفحة‬ Dr. MOHAMED YAHIA 0900987639


QUESTION BANK OF Dr. MOHAMED YAHIA

specific enzyme levels, the availability of what other substance is rate-limiting in


the clearance of ethanol?

a. NADH
b. NAD +
c. FADH
d. FAD +
e. NADPH

The answer is: B

5. In lung diseases such as emphysema or chronic bronchitis, there is chronic


hypoxia that is particularly obvious in vascular tissues such as the lips or nail
beds (cyanosis). Poorly perfused areas exposed to chronic hypoxia have
decreased metabolic energy for tissue maintenance and repair. An important
reason for this is

a. Increased hexokinase activity owing to increased oxidative phosphorylation


b. Increased ethanol formation from pyruvate on changing from anaerobic to
aerobic metabolism
c. Increased glucose utilization via the pentose phosphate pathway on changing
from anaerobic to aerobic metabolism
d. Decreased ATP generation and increased glucose utilization on changing
from aerobic to anaerobic metabolism
e. Decreased respiratory quotient on changing from carbohydrate to fat as the
major metabolic fuel

The answer is: D

6. Following a fad diet meal of skim milk and yogurt, an adult female patient
experiences abdominal distention, nausea, cramping, and pain followed by a
watery diarrhea. This set of symptoms is observed each time the meal is
consumed. A likely diagnosis is

a. Steatorrhea
b. Lactase deficiency
c. Maltose deficiency
d. Sialidase deficiency
e. Lipoprotein lipase deficiency

The answer is: B

7. Asians and Native Americans may flush and feel ill after drinking small
amounts of ethanol in alcoholic beverages. This reaction is due to genetic
variation in an enzyme that metabolizes the liver metabolite of alcohol, which is

a. Methanol

338 ‫صفحة‬ Dr. MOHAMED YAHIA 0900987639


QUESTION BANK OF Dr. MOHAMED YAHIA

b. Acetone
c. Acetaldehyde
d. Hydrogen peroxide
e. Glycerol

The answer is: C

8. Which one of the following enzymes catalyzes high-energy phosphorylation of


substrates during glycolysis?

a) Pyruvate kinase
b) Phosphoglycerate kinase
c) Triose phosphate isomerase
d) Aldolase
e) Glyceraldehyde-3-phosphate dehydrogenase

The answer is: E

9. Which one of the following enzymes is common to both glycolysis and


gluconeogenesis?

a. Pyruvate kinase
b. Pyruvate carboxylase
c. Hexokinase
d. Phosphoglycerate kinase
e. Fructose-1,6-bisphosphatase

The answer is: D

10. During the first week of a diet of 1500 calories per day, the oxidation of
glucose via glycolysis in the liver of a normal 59-kg (130-lb) woman is inhibited
by the lowering of which one of the following?

a. Citrate
b. ATP
c. Fatty acyl CoA
d. Ketone bodies
e. Fructose-2,6-bisphosphate

The answer is: E

11. Familial fructokinase deficiency causes no symptoms because

a. Hexokinase can phosphorylate fructose


b. Most tissues utilize fructose
c. Liver fructose-1-P aldolase is still active
d. Excess fructose does not escape into the urine
e. Excess fructose spills into the bowel and is eliminated in feces

339 ‫صفحة‬ Dr. MOHAMED YAHIA 0900987639


QUESTION BANK OF Dr. MOHAMED YAHIA

The answer is: A

12. A newborn begins vomiting after feeding, becomes severely jaundiced, and
has liver disease. Treatment for possible sepsis is initiated, and the urine is found
to have reducing substances. A blood screen for galactosemia is positive, and
lactose-containing substances are removed from the diet. Lactose is toxic in this
case because

a. Excess glucose accumulates in the blood


b. Galactose is converted to the toxic substance galactitol (dulcitol)
c. Galactose competes for glucose during hepatic glycogen synthesis
d. Galactose is itself toxic in even small amounts
e. Glucose metabolism is shut down by excess galactose

The answer is: B

13. Which one of the following enzymes catalyzes phosphorylation with the use of
inorganic phosphate?

a. Hexokinase
b. Phosphofructokinase
c. Glyceraldehyde-3-phosphate dehydrogenase
d. Phosphoglycerate kinase
e. Pyruvate kinase

The answer is: C

14. After a well-rounded breakfast, which of the following would be expected to


occur?

a. Increased activity of pyruvate carboxylase


b. Decreased activity of acetyl CoA carboxylase
c. Decreased rate of glycogenolysis
d. Decreased rate of protein synthesis
e. Increased activity of phosphoenolpyruvate carboxykinase

The answer is: C

15. Which of the following metabolites is involved in glycogenolysis, glycolysis,


and gluconeogenesis?

a. Galactose-1-phosphate
b. Glucose-6-phosphate
c. Uridine diphosphoglucose
d. Fructose-6-phosphate
e. Uridine diphosphogalactose

The answer is: B

340 ‫صفحة‬ Dr. MOHAMED YAHIA 0900987639


QUESTION BANK OF Dr. MOHAMED YAHIA

16. Which of the following is an allosteric effector that enhances activity of


phosphofructokinase of the glycolytic pathway?

a. Adenosine monophosphate (AMP)


b. Citric acid
c. Adenosine triphosphate (ATP)
d. Glucose-6-phosphate
e. Glucose

The answer is: A

17. Which of the following hormones stimulates gluconeogenesis?

a. Progesterone
b. Glucagon
c. Aldosterone
d. Epinephrine
e. Thyroxine
f. Growth hormone
g. Insulin
h. Glucocorticoids

The answer is: B

18. The key regulatory enzyme of the pentose phosphate pathway is positively
regulated by

a. Reduced nicotinamide dinucleotide (NADH)


b. Adenosine diphosphate (ADP)
c. Guanosine triphosphate (GTP)
d. Nicotinamide dinucleotide phosphate (NADP + )
e. Reduced flavine adenine dinucleotide (FADH)

The answer is: D

19. The activity of pyruvate carboxylase is dependent upon the positive allosteric
effector

a. Succinate
b. AMP
c. Isocitrate
d. Citrate
e. Acetyl CoA

The answer is: E

341 ‫صفحة‬ Dr. MOHAMED YAHIA 0900987639


QUESTION BANK OF Dr. MOHAMED YAHIA

20. Which of the following explains why individuals with hyperlipidemia and/or
gout should minimize their intake of sucrose and high fructose syrups?

a. Fructose is initially phosphorylated by liver fructokinase


b. After initial modification, fructose is cleaved by a specific enolase
c. Fructose is converted to UDP-fructose
d. Fructose is ultimately converted to galactose
e. Fructose can be phosphorylated by hexokinase in adipose cells

The answer is: A

21. Glycogen synthetase, the enzyme involved in the biosynthesis of glycogen,


may

a. Be activated by the phosphorylation of a specific serine residue


b. Be activated by increased calcium levels
c. Be more specifically defined as UDP-glucose-glycogen glucosyl transferase
d. Synthesize glycogen without a polymer primer
e. Employ UDP- D -glucose as a glucosyl donor in both plants and animals

The answer is: C

22. Which one of the following activities is simultaneously stimulated by


epinephrine in muscle and inhibited by epinephrine in the liver?

a. Fatty acid oxidation


b. Glycogenolysis
c. Cyclic AMP synthesis
d. Glycolysis
e. Activation of phosphorylase

The answer is: D

23. Which one of the following compounds is common to both the oxidative
branch and the nonoxidative branch of the pentose phosphate pathway?

a. Xylulose-5-phosphate
b. Glucose-6-phosphate
c. Glyceraldehyde-3-phosphate
d. Fructose-6-phosphate
e. Ribulose-5-phosphate

The answer is: E

24. A Nigerian medical student studying in the United States develops hemolytic
anemia after taking the oxidizing antimalarial drug pamaquine. This severe
reaction is most likely due to

342 ‫صفحة‬ Dr. MOHAMED YAHIA 0900987639


QUESTION BANK OF Dr. MOHAMED YAHIA

a. Glucose-6-phosphate dehydrogenase deficiency


b. Concomitant scurvy
c. Vitamin C deficiency
d. Diabetes
e. Glycogen phosphorylase deficiency

The answer is: A

25. Which of the following events occurs during formation of phospho-


enolpyruvate from pyruvate during gluconeogenesis?

a. CO 2 is consumed
b. Inorganic phosphate is consumed
c. Acetyl CoA is utilized
d. ATP is generated
e. GTP is generated

The answer is: A

26. Among the many molecules of high-energy phosphate compounds formed as


a result of the functioning of the citric acid cycle, one molecule is synthesized at
the substrate level. In which of the following reactions does this occur?

a. Citrate → α-ketoglutarate
b. α-ketoglutarate → succinate
c. Succinate → fumarate
d. Fumarate → malate
e. Malate → oxaloacetate

The answer is: B

27. After alcohol ingestion, which of the following intermediates accumulates in


liver that is not typical of glycolysis or the citric acid cycle?

a. Acetyl CoA
b. Lactate
c. Acetaldehyde
d. Citrate
e. Oxaloacetate

The answer is: C

28. Citrate has a positive allosteric effect on which one of the following enzymes?

a. Pyruvate kinase
b. Acetyl CoA carboxylase
c. Phosphofructokinase
d. Fatty acid synthetase

343 ‫صفحة‬ Dr. MOHAMED YAHIA 0900987639


QUESTION BANK OF Dr. MOHAMED YAHIA

e. Enolase

The answer is: B

29. Reduction of which one of the following substrates leads to a reducing


equivalent in a step of the citric acid cycle?

a. Succinyl CoA
b. Malate
c. Fumarate
d. Oxaloacetate
e. Citrate

The answer is: B

30. The entry point into the citric acid cycle for isoleucine, valine, and the
product of odd-chain fatty acids is

a. Fumarate
b. Pyruvate
c. Oxaloacetate
d. Citrate
e. Succinyl CoA

The answer is: E

31. A child has ingested cyanide from her parents’ garage and is rushed to the
emergency room. Which of the following components of the citric acid cycle will
be depleted first in this child?

a. NAD + cofactor
b. Citrate synthase
c. Aconitase
d. Citrate production
e. Acetyl coenzyme A (CoA) production

The answer is: A

32. Which of the following statements correctly describes ketone bodies?

a. They accumulate in children with fatty acid oxidation disorders


b. They accumulate in diabetes mellitus after insulin therapy
c. They are produced by muscle but not by liver
d. They include β-hydroxybutyrate and acetone
e. They are found in blood but not in urine

The answer is: D

344 ‫صفحة‬ Dr. MOHAMED YAHIA 0900987639


QUESTION BANK OF Dr. MOHAMED YAHIA

33. Oxidative degradation of acetyl coenzyme A (CoA) in the citric acid cycle
gives a net yield of which of the following chemicals?

a. Flavin adenine dinucleotide (FAD + )


b. Nicotinamide adenine dinucleotide (NAD + )
c. Adenosine triphosphate (ATP)
d. Guanosine diphosphate (GDP)
e. Carbon dioxide (CO 2 )

The answer is: E

34. The citric acid cycle is inhibited by which of the following?

a. Fluoroacetate
b. Fluorouracil
c. Aerobic conditions
d. Arsenic
e. Malic acid

The answer is: B

35. In the pathway leading to biosynthesis of acetoacetate from acetyl CoA in the
liver, the immediate precursor of acetoacetate is which of the following
substances?

a. 3-hydroxybutyrate
b. Acetoacetyl CoA
c. 3-hydroxybutyryl CoA
d. Mevalonic acid
e. 3-hydroxy-3-methylglutaryl CoA

The answer is: E

36. A child presents with low blood glucose (hypoglycemia), enlarged liver
(hepatomegaly), and excess fat deposition in the cheeks (cherubic facies). A liver
biopsy reveals excess glycogen in hepatocytes. Deficiency of which of the
following enzymes might explain this phenotype?

a. α-1,1-glucosidase
b. α-1,1-galactosidase
c. α-1,4-glucosidase
d. α-1,4-galactosidase
e. α-1,6-galactosidase

The answer is: C

345 ‫صفحة‬ Dr. MOHAMED YAHIA 0900987639


QUESTION BANK OF Dr. MOHAMED YAHIA

37. Which of the following statements about glycogen metabolism is true?

a. Cyclic AMP–activated protein kinase stimulates glycogen synthase


b. Phosphorylase kinase is activated by phosphorylation
c. Phosphorylase b is inactivated by phosphorylation
d. Cyclic AMP levels are lowered by epinephrine and glucagon stimulation of
adenylate cyclase
e. Glycogen synthesis is stimulated by glucagon

The answer is: B

38. A man goes on a hunger strike and confines himself to a liquid diet with
minimal calories. Which of the following would occur after 4 to 5 h?

a. Decreased cyclic AMP and increased liver glycogen synthesis


b. Increased cyclic AMP and increased liver glycogenolysis
c. Decreased epinephrine levels and increased liver glycogenolysis
d. Increased Ca ++ in muscle and decreased glycogenolysis
e. Decreased Ca ++ in muscle and decreased glycogenolysis

The answer is: B

39. After a meal, blood glucose enters cells and is stored as glycogen, particularly
in the liver. Which of the following is the donor of new glucose molecules in
glycogen?

a. UDP-glucose-1-phosphate
b. UDP-glucose
c. UDP-glucose-6-phosphate
d. glucose-6-phosphate
e. glucose-1-phosphate

The answer is: B

40. Which of the following statements about the structure of glycogen is true?

a. Glycogen is a copolymer of glucose and galactose


b. There are more branch residues than residues in straight chains
c. Branch points contain α-1,4 glycosidic linkages
d. New glucose molecules are added to the C1 aldehyde group of chain termini,
forming a hemiacetal
e. The monosaccharide residues alternate between D - and L –glucose

The answer is: D

346 ‫صفحة‬ Dr. MOHAMED YAHIA 0900987639


QUESTION BANK OF Dr. MOHAMED YAHIA

41. Which of the following steps is involved in the generation of glucose from
lipolysis?

a. Glycerol from lipolysis is converted to triglycerides


b. Fatty acids from lipolysis are oxidized, producing NADH and stimulating
gluconeogenesis
c. Glycerol from lipolysis is phosphorylated, converted to fructose-1,6-
bisphosphate, and eventually converted to glucose
d. Fatty acids from lipolysis stimulate the citric acid cycle
e. Glycerol from lipolysis is taken up by liver cells and dimerized to fructose

The answer is: C

42. McArdle’s disease causes muscle cramps and muscle fatigue with increased
muscle glycogen. Which of the following enzymes is deficient?

a. Hepatic hexokinase
b. Muscle glycogen synthetase
c. Muscle phosphorylase
d. Muscle hexokinase
e. Muscle debranching enzyme

The answer is: C

43. Which of the following enzymes is associated with glycogen synthesis?

a. Amylo-(1,4 → 1,6)-transglycosylase
b. Phosphorylase
c. Phosphorylase kinase
d. Amylo-1,6-glucosidase
e. Glucose-6-phosphatase

The answer is: A

44. A child with a large head, multiple fractures, and blue scleras (whites of the
eyes) is evaluated for osteogenesis imperfecta (166200). One study involves
labeling of collagen chains in tissue culture to assess their mobility by gel
electrophoresis. Amino acids labeled with radioactive carbon 14 are added to the
culture dishes in order to label the collagen. Which of the following amino acids
would not result in labeled collagen?

a. Serine
b. Glycine
c. Aspartate
d. Glutamate
e. Hydroxylserine

The answer is: B

347 ‫صفحة‬ Dr. MOHAMED YAHIA 0900987639


QUESTION BANK OF Dr. MOHAMED YAHIA

45. Liver aminotransferases, which are also called transaminases, catalyze the
transfer of α-amino groups from many different amino acids to α-ketoglutarate.
The intermediate produced is deaminated back to α-ketoglutarate with the
formation of ammonium ion. The structure of α-ketoglutarate is shown below.
What is the intermediate produced?

a. Aspartate
b. Alanine
c. Oxaloacetate
d. Glutamate
e. Pyruvate

The answer is: D

46. The reactions of the urea cycle occur

a. In the cytosol
b. In the mitochondrial matrix
c. In the mitochondrial matrix and the cytosol
d. Only in lysosomes
e. In peroxisomes

The answer is: C

47. A newborn becomes progressively lethargic after feeding and increases his
respiratory rate. He becomes virtually comatose, responding only to painful
stimuli, and exhibits mild respiratory alkalosis. Suspicion of a urea cycle
disorder is aroused and evaluation of serum amino acid levels is initiated. In the
presence of hyperammonemia, production of which of the following amino acids
is always increased?

a. Glycine
b. Arginine
c. Proline
d. Histidine
e. Glutamine

The answer is: E

48. The thyroid hormone thyroxine (T 4 ) is derived from

a. Threonine
b. Tyrosine
c. Thiamine
d. Tryptophan
e. Tyramine

The answer is: B

348 ‫صفحة‬ Dr. MOHAMED YAHIA 0900987639


QUESTION BANK OF Dr. MOHAMED YAHIA

49. Which of the metabolites below is a precursor of tyrosine?

a. L -dihydroxyphenylalanine (dopa)
b. Dopamine
c. Norepinephrine
d. Epinephrine
e. Phenylalanine

The answer is: E

50. Which of the following amino acids is a precursor to cysteine?

a. Threonine
b. Methionine
c. Glutamine
d. Lysine
e. Alanine

The answer is: B

51. An adolescent female develops hemiballismus (repetitive throwing motions of


the arms) after anesthesia for a routine operation. She is tall and lanky, and it is
noted that she and her sister both had previous operations for dislocated lenses
of the eyes. The symptoms are suspicious for the disease homocystinuria
(236300). Which of the statements below is descriptive of this disease?

a. Patients may be treated with dietary supplements of vitamin B 12


b. Patients may be treated with dietary supplements of vitamin C
c. There is deficient excretion of homocysteine
d. There is increased excretion of cysteine
e. There is a defect in the ability to form homocysteine from methionine by
methylation

The answer is: A

52. Which clinical laboratory observation below is suggestive of Hartnup disease


(neutral amino acid transport deficiency)?

a. Burnt-sugar smell in urine


b. High plasma phenylalanine levels
c. Extremely high levels of citrulline in urine
d. Elevation of glutamine in blood and urine
e. Elevated plasma tyrosine and methionine levels
f. Dark urine
g. High fecal levels of tryptophan and indole derivatives

The answer is: G

349 ‫صفحة‬ Dr. MOHAMED YAHIA 0900987639


QUESTION BANK OF Dr. MOHAMED YAHIA

53. The important reactive group of glutathione in its role as an antioxidant is

a. Serine
b. Sulfhydryl
c. Tyrosine
d. Acetyl coenzyme A (CoA)
e. Carboxyl

The answer is: B

54. Which one of the following hormones is derived most completely from
tyrosine?

a. Glucagon
b. Thyroxine
c. Insulin
d. Prostaglandins
e. Endorphins

The answer is: B

55. A newborn develops jaundice (yellow skin and yellow scleras) that requires
laboratory evaluation. Which of the following porphyrin derivatives is
conjugated, reacts directly, and is a major component of bile?

a. Bilirubin diglucuronide
b. Stercobilin
c. Biliverdin
d. Urobilinogen
e. Heme
f. Bilirubin
g. Urobilin

The answer is: A

56. Which of the following porphyrins gives stools their characteristic brown
color?

a. Biliverdin
b. Urobilinogen
c. Heme
d. Stercobilin
e. Urobilin

The answer is: D

57. Chylomicrons, intermediate-density lipoproteins (IDLs), low-density


lipoproteins (LDLs), and very-low-density lipoproteins (VLDLs) are all serum

350 ‫صفحة‬ Dr. MOHAMED YAHIA 0900987639


QUESTION BANK OF Dr. MOHAMED YAHIA

lipoproteins. What is the correct ordering of these particles from the lowest to
the highest density?

a. LDLs, IDLs, VLDLs, chylomicrons


b. Chylomicrons, VLDLs, IDLs, LDLs
c. VLDLs, IDLs, LDLs, chylomicrons
d. Chylomicrons, IDLs, VLDLs, LDLs
e. LDLs, VLDLs, IDLs, chylomicrons

The answer is: B

58. A 3-year-old child is brought into the ER while you are on duty. She is cold
and clammy and is breathing rapidly. She is obviously confused and lethargic.
Her mother indicates she has accidentally ingested automobile antifreeze while
playing in the garage. Following gastrointestinal lavage and activated charcoal
administration, one of the treatments you immediately initiate involves

a. Intravenous infusion of oxalic acid


b. Nasogastric tube for ethanol administration
c. Flushing out the bladder via a catheter
d. Intramuscular injection of epinephrine
e. Simply waiting and measuring vital signs

The answer is: B

59. Ceramide is a precursor to which of the following compounds?

a. Phosphatidyl serine
b. Sphingomyelin
c. Phosphatidyl glycerol
d. Phosphatidyl choline
e. Phosphatidyl ethanolamine

The answer is: B

60. Which of the following steps in the biosynthesis of cholesterol is thought to be


rate-controlling and the locus of metabolic regulation?

a. Geranyl pyrophosphate → farnesyl pyrophosphate


b. Squalene → lanosterol
c. Lanosterol → cholesterol
d. 3-hydroxy-3-methylglutaryl CoA → mevalonic acid
e. Mevalonic acid → geranyl pyrophosphate

The answer is: D

351 ‫صفحة‬ Dr. MOHAMED YAHIA 0900987639


QUESTION BANK OF Dr. MOHAMED YAHIA

61. Multiple sclerosis is a disease characterized by chronic inflammation. There


are significant data to indicate that susceptibility to multiple sclerosis is inherited
and causes a primary change in

a. Membrane lipids
b. Anti-inflammatory steroids
c. Blood proteins
d. Stored carbohydrates
e. Nucleotide metabolism

The answer is: A

62. Which of the following is most characteristic of a sphingolipidosis?

a. Multifactorial inheritance
b. Variable activities of abnormal enzyme in different patient tissues
c. Deficiency of a hydrolytic enzyme
d. Abnormalities of sphingolipid synthesis
e. Accumulation of ceramide-containing lipids

The answer is: E

63. The end product of cytosol fatty acid synthetase in humans is

a. Oleic acid
b. Arachidonic acid
c. Linoleic acid
d. Palmitic acid
e. Palmitoleic acid

The answer is: D

64. It has been noted that infants placed on extremely low-fat diets for a variety
of reasons often develop skin problems and other symptoms. This is most often
due to

a. Lactose intolerance
b. Glycogen storage diseases
c. Antibody abnormalities
d. Deficiency of fatty acid desaturase greater than ∆ 9
e. Deficiency of chylomicron and VLDL production

The answer is: D

65. The fatty acid synthase complex of mammals

a. Is a dimer of unsimilar subunits


b. Is composed of seven different proteins

352 ‫صفحة‬ Dr. MOHAMED YAHIA 0900987639


QUESTION BANK OF Dr. MOHAMED YAHIA

c. Dissociates into eight different proteins


d. Catalyzes eight different enzymatic steps
e. Is composed of covalently linked enzymes

The answer is: E

66. The primary biochemical lesion in homozygotes with familial


hypercholesterolemia (type IIa) is

a. The loss of feedback inhibition of liver hydroxymethylglutaryl CoA reductase


b. The increased production of low-density lipoproteins from very-low-density
lipoproteins
c. The loss of apolipoprotein B
d. The malfunctioning of acyl CoA–cholesterol acyl transferase (ACAT)
e. The functional deficiency of plasma membrane receptors for low-density
lipoproteins

The answer is: E

67. Which one of the following apolipoproteins is synthesized in the liver as part
of the coat of very-low-density lipoproteins (VLDLs)?

a. AI
b. B48
c. CII
d. B100
e. E

The answer is: D

68. Which of the following lipoproteins would contribute to a measurement of


plasma cholesterol in a normal person following a 12-h fast?

a. Very-low-density lipoproteins
b. High-density lipoproteins
c. Chylomicrons
d. Chylomicron remnants
e. Adipocyte lipid droplets

The answer is: B

69. Which of the following statements correctly describes the enzyme thiokinase?

a. It yields acetyl CoA as a product


b. It yields ADP as a product
c. It yields CoA as a product
d. It forms CoA thioesters as a product
e. It requires β-ketoacyl CoA as a substrate

353 ‫صفحة‬ Dr. MOHAMED YAHIA 0900987639


QUESTION BANK OF Dr. MOHAMED YAHIA

The answer is: D

70. A 15-year-old boy has a long history of school problems and is labeled as
hyperactive. His tissues are puffy, giving his face a “coarse” appearance. His IQ
tests have declined recently and are now markedly below normal. Laboratory
studies demonstrate normal amounts of sphingolipids in fibroblast cultures with
increased amounts of glycosaminoglycans in urine. Which of the following
enzyme deficiencies might explain the boy’s phenotype?

a. Hexosaminidase A
b. Glucocerebrosidase
c. α- L -iduronidase
d. α-galactocerebrosidase
e. β-gangliosidase A

The answer is: C

71. Leukocyte samples isolated from the blood of a newborn infant are
homogenized and incubated with ganglioside GM 2 . Approximately 47% of the
expected normal amount of N-acetylgalactosamine is liberated during the
incubation period. These results indicate that the infant

a. Is a heterozygote (carrier) for Tay-Sachs disease


b. Is homozygous for Tay-Sachs disease
c. Has Tay-Sachs syndrome
d. Will most likely have mental deficiency
e. Has relatively normal β-N-acetylhexosaminidase activity

The answer is: A

72. Most of the reducing equivalents utilized for synthesis of fatty acids can be
generated from

a. The pentose phosphate pathway


b. Glycolysis
c. The citric acid cycle
d. Mitochondrial malate dehydrogenase
e. Citrate lyase

The answer is: A

73. During fatty acid metabolism in humans, coenzyme A (CoA) is different from
acyl carrier protein (ACP) in which one of the following ways?

a. Binding of malonic acid with a phosphopantetheine


b. Binding of fatty acids
c. Function in fatty acid oxidation
d. Function in the cytosol

354 ‫صفحة‬ Dr. MOHAMED YAHIA 0900987639


QUESTION BANK OF Dr. MOHAMED YAHIA

e. Function in fatty acid synthesis

The answer is: C

74. For every 2 mol of free glycerol released by lipolysis of triacylglycerides in


adipose tissue

a. 2 mol of triacylglycerides is released


b. 2 mol of free fatty acids is released
c. 1 mol of glucose can be synthesized in gluconeogenesis
d. 1 mol of triacylglyceride is released
e. 3 mol of acyl CoA is produced

The answer is: C

75. Which one of the following compounds is a key intermediate in the synthesis
of both triacylglycerols and phospholipids?

a. CDP-choline
b. Phosphatidate
c. Triacylglyceride
d. Phosphatidylserine
e. CDP-diacylglycerol

The answer is: B

76. Which of the following is not used in the synthesis of fatty acids?

a. Cobalamin (vitamin B 12 )
b. NADPH
c. AMP
d. FADH 2
e. HCO 3

The answer is: D

77. A 45-year-old man has a mild heart attack and is placed on diet and
mevastatin therapy. Which of the following will be a result of this therapy?

a. Low blood glucose


b. Low blood LDLs
c. High blood cholesterol
d. High blood glucose
e. Low oxidation of fatty acids
f. Ketosis
g. Lipolysis

The answer is: B

355 ‫صفحة‬ Dr. MOHAMED YAHIA 0900987639


QUESTION BANK OF Dr. MOHAMED YAHIA

78. The major source of extracellular cholesterol for human tissues is

a. Very-low-density lipoproteins (VLDLs)


b. Low-density lipoproteins (LDLs)
c. High-density lipoproteins (HDLs)
d. Albumin
e. γ-globulin

The answer is: B

79. The synthesis of 3-hydroxy-3-methylglutaryl CoA can occur

a. Only in mitochondria of all mammalian tissues


b. Only in the cytosol of all mammalian tissues
c. Only in the endoplasmic reticulum of all mammalian tissues
d. In both the cytosol and mitochondria
e. In lysosomes

The answer is: D

80. When the liver is actively synthesizing fatty acids, a concomitant decrease in
β oxidation of fatty acids is due to

a. Inhibition of a translocation between cellular compartments


b. Inhibition by an end product
c. Activation of an enzyme
d. Detergent effects
e. Decreases in adipocyte lipolysis

The answer is: A

81. A 4-year-old girl presents in the clinic with megaloblastic anemia and failure
to thrive. Blood chemistries reveal orotic aciduria. Enzyme measurements of
white blood cells reveal a deficiency of the pyrimidine biosynthesis enzyme
orotate phosphoribosyltransferase and abnormally high activity of the enzyme
aspartate transcarbamoylase. Which one of the following treatments will reverse
all symptoms if carried out chronically?

a. Blood transfusion
b. White blood cell transfusion
c. Dietary supplements of phosphoribosylpyrophosphate (PRPP)
d. Oral thymidine
e. Oral uridine

The answer is: E

356 ‫صفحة‬ Dr. MOHAMED YAHIA 0900987639


QUESTION BANK OF Dr. MOHAMED YAHIA

82. In most patients with gout as well as those with Lesch-Nyhan syndrome,
purines are overproduced and overexcreted. Yet the hypoxanthine analogue
allopurinol, which effectively treats gout, has no effect on the severe neurological
symptoms of Lesch-Nyhan patients because it does not

a. Decrease de novo purine synthesis


b. Decrease de novo pyrimidine synthesis
c. Diminish urate synthesis
d. Increase phosphoribosylpyrophosphate (PRPP) levels
e. Inhibit xanthine oxidase

The answer is: A

83. Which of the following would rule out hyperuricemia in a patient?

a. Lesch-Nyhan syndrome
b. Gout
c. Xanthine oxidase hyperactivity
d. Carbamoyl phosphate synthase deficiency
e. Purine overproduction secondary to Von Gierke’s disease

The answer is: D

84. Which one of the following contributes nitrogen atoms to both purine and
pyrimidine rings?

a. Aspartate
b. Carbamoyl phosphate
c. Carbon dioxide
d. Glutamine
e. Tetrahydrofolate

The answer is: A

85. Which statement best describes xanthine?

a. It is a direct precursor of guanine


b. It covalently binds to allopurinol
c. It is a substrate rather than a product of the enzyme xanthine oxidase
d. It is oxidized to form uric acid
e. It is oxidized to form hypoxanthine

The answer is: D

86. Feedback inhibition of pyrimidine nucleotide synthesis can occur by which of


the following means?

a. Increased activity of carbamoyl phosphate synthetase

357 ‫صفحة‬ Dr. MOHAMED YAHIA 0900987639


QUESTION BANK OF Dr. MOHAMED YAHIA

b. Increased activity of aspartate transcarbamoylase


c. CTP allosteric effects
d. UMP competitive inhibition
e. TTP allosteric effects

The answer is: C

87. Purine nucleotide biosynthesis can be inhibited by which of the following?

a. Guanosine triphosphate (GTP)


b. Uridine monophosphate (UMP)
c. Adenosine monophosphate (AMP)
d. Adenosine triphosphate (ATP)
e. Inosine diphosphate (IDP)

The answer is: C

88. Which base derivative can serve as a precursor for the synthesis of two of the
other base derivatives shown?

a. Cytidine triphosphate (CTP)


b. Uridine monophosphate (UMP)
c. Deoxythymidine monophosphate (dTMP)
d. Adenosine triphosphate (ATP)
e. Deoxyadenosine monophosphate (dAMP)

The answer is: B

89. Which is the rate-controlling step of pyrimidine synthesis that exhibits


allosteric inhibition by cytidine triphosphate (CTP)?

a. Aspartate transcarbamoylase
b. Hypoxanthine-guanine phosphoribosyl transferase (HGPRT) 5-fluorouracil
c. Thymidylate synthase
d. Ribose-phosphate pyrophosphokinase
e. Xanthine oxidase

The answer is: C

90. Which of the following compounds is a required substrate for purine


biosynthesis?

a. 5-methyl thymidine
b. Ara C
c. Ribose phosphate
d. 5-phosphoribosylpyrophosphate (PRPP)
e. 5-fluorouracil

358 ‫صفحة‬ Dr. MOHAMED YAHIA 0900987639


QUESTION BANK OF Dr. MOHAMED YAHIA

The answer is: B

91. Which of the following compounds is an analogue of hypoxanthine?

a. Ara C
b. Allopurinol
c. Ribose phosphate
d. 5-phosphoribosylpyrophosphate (PRPP)
e. 5-FU

The answer is: E

92. A pentose with a 5′-phosphate group, a 2′-hydroxyl group, and a 1′-


pyrimidine group describes which of the following structures?

a. Cytosine
b. Guanosine
c. Thymidine
d. Thymidylate
e. Cytidylate

The answer is: E

NUTRITION
94. A deficiency of vitamin B 12 causes

a. Cheilosis
b. Beriberi
c. Pernicious anemia
d. Scurvy
e. Rickets

The answer is: C

95. In adults, a severe deficiency of vitamin D causes

a. Night blindness
b. Osteomalacia
c. Rickets
d. Osteogenesis imperfecta
e. Osteopetrosis

The answer is: B

96. Which of the following vitamins would most likely become deficient in a
person who develops a completely carnivorous lifestyle?

a. Thiamine

359 ‫صفحة‬ Dr. MOHAMED YAHIA 0900987639


QUESTION BANK OF Dr. MOHAMED YAHIA

b. Niacin
c. Cobalamin
d. Pantothenic acid
e. Vitamin C

The answer is: C

97. Which of the following statements regarding vitamin A is true?

a. It is not an essential vitamin


b. It is related to tocopherol
c. It is a component of rhodopsin
d. It is derived from ethanol
e. It is also known as opsin

The answer is: C

98. Fully activated pyruvate carboxylase depends upon the presence of

a. Malate and niacin


b. Acetyl CoA and biotin
c. Acetyl CoA and thiamine pyrophosphate
d. Oxaloacetate and biotin
e. Oxaloacetate and niacin

The answer is: B

99. Pantothenic acid is a constituent of the coenzyme involved in

a. Decarboxylation
b. Acetylation
c. Dehydrogenation
d. Reduction
e. Oxidation

The answer is: B

100. Studies of the actions of two anticoagulants—dicumarol and warfarin (the


latter also a hemorrhagic rat poison)—have revealed that

a. Vitamin C is necessary for the synthesis of fibrinogen


b. Vitamin C activates fibrinogen
c. Vitamin K is a clotting factor
d. Vitamin K is essential for γ-carboxylation of glutamate
e. The action of vitamin E is antagonized by these compounds

The answer is: D

101. Biotin is involved in which of the following types of reactions?

360 ‫صفحة‬ Dr. MOHAMED YAHIA 0900987639


QUESTION BANK OF Dr. MOHAMED YAHIA

a. Hydroxylations
b. Carboxylations
c. Decarboxylations
d. Dehydrations
e. Deaminations

The answer is: B

102. Which of the following vitamins is the precursor of CoA?

a. Riboflavin
b. Pantothenate
c. Thiamine
d. Cobamide
e. Pyridoxamine

The answer is: B

103. In the Far East, beriberi is a serious health problem. It is characterized by


neurologic and cardiac symptoms. Beriberi is caused by a deficiency of

a. Choline
b. Ethanolamine
c. Thiamine
d. Serine
e. Glycine

The answer is: C

104. Both acyl carrier protein (ACP) of fatty acid synthetase and coenzyme A
(CoA)

a. Contain reactive phosphorylated tyrosine groups


b. Contain thymidine
c. Contain phosphopantetheine-reactive groups
d. Contain cystine-reactive groups
e. Carry folate groups

The answer is: C

105. Which one of the following transfers acyl groups?

a. Thiamine pyrophosphate
b. Lipoamide
c. ATP
d. NADH

361 ‫صفحة‬ Dr. MOHAMED YAHIA 0900987639


QUESTION BANK OF Dr. MOHAMED YAHIA

e. FADH

The answer is: B

106. Which one of the following cofactors must be utilized during the conversion
of acetyl CoA to malonyl CoA?

a. Thiamine pyrophosphate
b. Acyl carrier protein (ACP)
c. NAD 1
d. Biotin
e. FAD

The answer is: D

107. A deficiency of which of the following vitamins is associated with the


occurrence of neural tube defects (anencephaly and spina bifida)?

a. Ascorbic acid (vitamin C)


b. Thiamine (vitamin B 1 )
c. Riboflavin (vitamin B 2 )
d. Niacin (vitamin B 3
e. Biotin
f. Pantothenic acid
g. Folic acid
h. Cobalamin (vitamin B 12 )

The answer is: G

108. An African American infant presents with prominent forehead, bowing of


the limbs, broad and tender wrists, swellings at the costochondral junctions of
the ribs, and irritability. Which of the following treatments are recommended?

a. Lotions containing retinoic acid


b. Diet of baby food containing leafy vegetables
c. Diet of baby food containing liver and ground beef
d. Milk and sunlight exposure
e. Removal of eggs from diet

The answer is: D

109. Which of the following statements describing vitamin K is true?

a. Vitamin K is broken down by intestinal bacteria


b. Vitamin K is obtained by eating citrus fruits, spinach, and cabbage

362 ‫صفحة‬ Dr. MOHAMED YAHIA 0900987639


QUESTION BANK OF Dr. MOHAMED YAHIA

c. Vitamin K is not found in dairy or meat products; it is obtained by eating egg


yolk and liver
d. Vitamin K is required for liver synthesis of prothrombin
e. Vitamin K prevents thrombosis

The answer is: D

110. A term infant is born at home and does well with breast-feeding. Two days
later, the mother calls frantically because the baby is bleeding from the umbilical
cord and nostrils. The most likely cause is

a. Deficiency of vitamin C due to a citrus-poor diet during pregnancy


b. Hypervitaminosis A due to ingestion of beef liver during pregnancy
c. Deficiency of vitamin K because infant intestines are sterile
d. Deficiency of vitamin K because of disseminated intravascular coagulation
(disseminated clotting due to infantile sepsis)
e. Deficiency of vitamin E due to maternal malabsorption during pregnancy

The answer is: C

111. Which of the following statements regarding vitamin A is true?

a. Vitamin A promotes maintenance of epithelial tissue


b. Vitamin A is necessary for hearing but not for vision
c. Vitamin A is synthesized in skin
d. All vitamin A derivatives are safe to use during pregnancy
e. Vitamin A is a form of calciferol

The answer is: A

112. Which of the following conditions most rapidly produces a functional


deficiency of vitamin K?

a. Coumadin therapy to prevent thrombosis in patients prone to clot formation


b. Broad-spectrum antibiotic therapy
c. Lack of red meat in the diet
d. Lack of citrus fruits in the diet
e. Premature birth

The answer is: A

113. A 3-month-old boy presents with poor feeding and growth, low muscle tone
(hypotonia), elevation of blood lactic acid (lactic acidemia), and mild acidosis
(blood pH 7.3 to 7.35). The ratio of pyruvate to lactate in serum is elevated, and
there is decreased conversion of pyruvate to acetyl coenzyme A in fibroblasts.
Which of the following compounds might be considered for therapy?

a. Pyridoxine

363 ‫صفحة‬ Dr. MOHAMED YAHIA 0900987639


QUESTION BANK OF Dr. MOHAMED YAHIA

b. Thiamine
c. Free fatty acids
d. Biotin
e. Ascorbic acid

The answer is: B

114. A homeless person is brought into the emergency room with psychotic
imagery and alcohol on his breath. Which of the following compounds is most
important to administer?

a. Glucose
b. Niacin
c. Nicotinic acid
d. Thiamine
e. Riboflavin

The answer is: D

115. Which of the following vitamins becomes a major electron acceptor, aiding
in the oxidation of numerous substrates?

a. Vitamin B 6
b. Niacin
c. Riboflavin
d. Thiamine
e. Vitamin B 1

The answer is: B

116. Which of the following vitamins can act without phosphorylation?

a. Pyridoxine
b. Lipoamide
c. Niacin
d. Thiamine
e. Riboflavin

The answer is: B

117. A 2-year-old child presents with chronic cough and bronchitis, growth
failure, and chronic diarrhea with light-colored, foul-smelling stools. A
deficiency of which of the following vitamins should be considered?

a. Vitamin A
b. Vitamin C
c. Vitamin B 1
d. Vitamin B 2

364 ‫صفحة‬ Dr. MOHAMED YAHIA 0900987639


QUESTION BANK OF Dr. MOHAMED YAHIA

e. Vitamin B 6

The answer is: A

118. Pantothenic acid is important for which of the following steps or pathways?

a. Pyruvate carboxylase
b. Fatty acid synthesis
c. Pyruvate carboxykinase
d. Gluconeogenesis
e. Glycolysis

The answer is: B

GENETICS:
1. Which of the following statements regarding a double-helical molecule of DNA
is true?

a. All hydroxyl groups of pentoses are involved in linkages


b. Bases are perpendicular to the axis
c. Each strand is identical
d. Each strand has parallel, 5′ to 3′ direction
e. Each strand replicates itself

The answer is: B

2. A sample of human DNA is subjected to increasing temperature until the


major fraction exhibits optical density changes due to disruption of its helix
(melting or denaturation). A smaller fraction is atypical in that it requires a
much higher temperature for melting. This smaller, atypical fraction of DNA
must contain a higher content of

a. Adenine plus cytosine


b. Cytosine plus guanine
c. Adenine plus thymine
d. Cytosine plus thymine
e. Adenine plus guanine

The answer is: B

3. It is well known that DNA polymerases synthesize DNA only in the 5′ to 3′


direction. Yet, at the replication fork, both strands of parental DNA are being
replicated with the synthesis of new DNA. How is it possible that while one
strand is being synthesized in the 5′ to 3′ direction, the other strand appears to
be synthesized in the 3′ to 5′ direction? This apparent paradox is explained by

a. 3′ to 5′ DNA repair enzymes

365 ‫صفحة‬ Dr. MOHAMED YAHIA 0900987639


QUESTION BANK OF Dr. MOHAMED YAHIA

b. 3′ to 5′ DNA polymerase
c. Okazaki fragments
d. Replication and immediate crossover of the leading strand
e. Lack of RNA primer on one of the strands

The answer is: C

4. A child presents with severe growth failure, accelerated aging that causes
adult complications such as diabetes and coronary artery disease, and
microcephaly (small head) due to increased nerve cell death. In vitro assay of
labeled thymidine incorporation reveals decreased levels of DNA synthesis
compared to controls, but normal-sized labeled DNA fragments. The addition of
protein extract from normal cells, gently heated to inactivate DNA polymerase,
restores DNA synthesis in the child’s cell extracts to normal. Which of the
enzymes used in DNA replication is likely to be defective in this child?

a. DNA-directed DNA polymerase


b. Unwinding proteins
c. DNA polymerase I
d. DNA-directed RNA polymerase
e. DNA ligase

The answer is: B

5. Which of the following statements correctly describes eukaryotic nuclear


chromosomal DNA?

a. Each discontinuous piece making up the chromosomes of eukaryotes is about


the same size as each prokaryotic chromosome
b. Unlike bacterial DNA, no histones are associated with it
c. It is not replicated semiconservatively
d. It is a linear and unbranched molecule
e. It is not associated with a specific membranous organelle

The answer is: D

6. Which of the following statements describes both the spiral structure of


double-stranded DNA and the spiral structure found in certain segments of
protein?

a. They are repeating spiral structures with intervals of pleated sheets


b. They have four alternative units arranged in polymeric chains
c. They are held together by hydrogen bonding
d. They are α-helical
e. They have covalently linked backbones

The answer is: E

366 ‫صفحة‬ Dr. MOHAMED YAHIA 0900987639


QUESTION BANK OF Dr. MOHAMED YAHIA

7. Which of the following descriptions of DNA replication is not common to the


synthesis of both leading and lagging strands?

a. RNA primer is synthesized


b. DNA polymerase III synthesizes DNA
c. Helicase (rep protein) continuously unwinds duplex DNA at the replication
fork during synthesis
d. Nucleoside monophosphates are added in a 5′ to 3′ direction along the growing
DNA chain
e. DNA ligase repeatedly joins the ends of DNA along the growing strand

The answer is: E

8. Which of the following statements describing restriction endonucleases is


true?

a. They always yield overhanging single-stranded ends


b. They recognize methylated DNA sequences
c. They recognize triplet repeats
d. They cleave both strands in duplex DNA
e. They always yield blunt ends

The answer is: D

9. The first drug to be effective against AIDS, including the reduction of


maternal-to-child AIDS transmission by 30%, was AIDS drug azidothymidine
(AZT). Which of the following describes its mechanism of action?

a. It inhibits viral protein synthesis


b. It inhibits RNA synthesis
c. It inhibits viral DNA polymerase
d. It stimulates DNA provirus production
e. It inhibits viral reverse transcriptase

The answer is: E

10. Which of the following enzymes can polymerize deoxyribonucleotides into


DNA?

a. Primase
b. DNA ligase
c. DNA gyrase
d. RNA polymerase III
e. Reverse transcriptase

The answer is: E

367 ‫صفحة‬ Dr. MOHAMED YAHIA 0900987639


QUESTION BANK OF Dr. MOHAMED YAHIA

11. Which of the following statements correctly describes the recombinant DNA
tool known as plasmids?

a. They are found more commonly in viruses than in bacteria


b. They are single-stranded circles
c. They sometimes enhance bacterial susceptibility to antibiotics
d. They sometimes enhance bacterial resistance to antibiotics
e. They are too small to be useful as vectors for the cloning of mammalian DNA
segments

The answer is: D

12. Which of the following molecules is found in a nucleoside?

a. A pyrophosphate group
b. A 1′ base linked to a pentose sugar
c. A 5′-phosphate group linked to a pentose sugar
d. A 3′-phosphate group linked to a pentose sugar
e. A terminal triphosphate

The answer is: B

13. Human cells have a finite lifespan and this contributes to the aging process.
Stem cells and neoplastic cells have indefinite life spans. The reason for these
observations is that chromosomes in a cell get progressively shorter with each
cell division because the telomere sequences at the ends of the chromosomes get
shorter with each cell division. The chromosomes in stem cells and neoplastic
cells do not generally shorten with each cell division. The enzyme utilized by
stem cells and neoplastic cells to lengthen the telomeres is which of the following?

a. DNA polymerase delta


b. DNA polymerase alpha
c. DNA ligase
d. topoisomerase
e. telomerase

The answer is: E

14. Some antineoplastic drugs act by inhibiting which of the following?

a. DNA helicase
b. topoisomerase
c. telomerase
d. DNA polymerase delta
e. DNA polymerase alpha

The answer is: C

368 ‫صفحة‬ Dr. MOHAMED YAHIA 0900987639


QUESTION BANK OF Dr. MOHAMED YAHIA

15. Which one of the following is an accurate statement regarding chromosome


replication?

a. It is semiconservative.
b. It occurs during G1 in the cell cycle.
c. Inactive genes are replicated first.
d. It starts with the synthesis of Okazaki fragments.

The answer is: A

16. The leading strand of DNA in the replication fork is synthesized by which one
of the following mechanisms?

a. continuously by DNA polymerase alpha.


b. discontinuously by DNA polymerase delta.
c. continuously by DNA polymerase delta.
d. discontinuously by DNA polymerase alpha.

The answer is: A

17. The autosomal recessive disease Fanconi anemia is characterized by


chromosome breakage and rearrangements and most individuals with the
disease will develop some kind of cancer. Which one of the following is defective
in individuals with Fanconi anemia?

a. DNA polymerase delta


b. DNA repair enzyme
c. DNA ligase
d. DNA primase

The answer is: B

18. Which is the most correct sequence of events in gene repair mechanisms in
patients without a mutated repair process?

a. Nicking, excision, replacement, sealing, recognition


b. Sealing, recognition, nicking, excision, replacement
c. Recognition, nicking, excision, replacement, sealing
d. Nicking, sealing, recognition, excision, replacement
e. Nicking, recognition, excision, sealing, replacement

The answer is: C

19. Which of the following enzymes can be described as a DNA-dependent RNA


polymerase?

a. DNA ligase
b. Primase

369 ‫صفحة‬ Dr. MOHAMED YAHIA 0900987639


QUESTION BANK OF Dr. MOHAMED YAHIA

c. DNA polymerase III


d. DNA polymerase I
e. Reverse transcriptase

The answer is: B

20. Mammalian chromosomes have specialized structures with highly repetitive


DNA at their ends (telomeres). Which aspect of telomeric DNA replication is
different from that of other chromosomal regions?

a. The DNA polymerase uses an RNA primer but does not degrade it
b. The DNA polymerase contains an RNA molecule that serves as template for
DNA synthesis
c. The DNA polymerase must cross-link the 5′ and 3′ termini
d. The DNA polymerase has a σ subunit that facilitates binding to repetitive
DNA
e. The DNA polymerase does not use an RNA template or primer

The answer is: B

21. Which statement about the “genetic code” is most accurate?

a. Information is stored as set of dinucleotide repeats called codons


b. The code is degenerate (i.e., more than one codon may exist for a single amino
acid)
c. Information is stored as set of trinucleotide repeats called codons
d. There are 64 codons, all of which code for amino acids
e. The sequence of codons that make up a gene exhibits an exact linear
correspondence to the sequence of amino acids in the translated protein

The answer is: B

22. In contrast to DNA polymerase, RNA polymerase

a. Fills in the gap between Okazaki fragments


b. Works only in a 5′ to 3′ direction
c. Edits as it synthesizes
d. Synthesizes RNA primer to initiate DNA synthesis
e. Adds nucleoside monophosphates to the growing polynucleotides

The answer is: D

23. The removal of introns and subsequent self-splicing of adjacent exons occurs
in some portions of primary ribosomal RNA transcripts. The splicing of introns
in messenger RNA precursors is

a. RNA-catalyzed in the absence of protein


b. Self-splicing

370 ‫صفحة‬ Dr. MOHAMED YAHIA 0900987639


QUESTION BANK OF Dr. MOHAMED YAHIA

c. Carried out by spliceosomes


d. Controlled by RNA polymerase
e. Regulated by RNA helicase

The answer is: C

24. A promoter site on DNA

a. Transcribes repressor
b. Initiates transcription
c. Codes for RNA polymerase
d. Regulates termination
e. Translates specific proteins

The answer is: B

25. The σ factor found in many bacteria is best described as a

a. Subunit of RNA polymerase responsible for the specificity of the initiation of


transcription of RNA from DNA
b. Subunit of DNA polymerase that allows for synthesis in both 5' to 3' and 3' to
5' directions
c. Subunit of the 50S ribosome that catalyzes peptide bond synthesis
d. Subunit of the 30S ribosome to which mRNA binds
e. Factor that forms the bridge between the 30S and 50S particles constituting the
70S ribosome

The answer is: A

26. The consensus sequence 5′ TATAAAA 3′ found in eukaryotic genes is quite


similar to a consensus sequence observed in prokaryotes. It is important as the

a. Only site of binding of RNA polymerase III


b. Promoter for all RNA polymerases
c. Termination site for RNA polymerase II
d. Major binding site of RNA polymerase I
e. First site of binding of a transcription factor for RNA polymerase II

The answer is: E

27. The so-called caps of RNA molecules

a. Allow tRNA to be processed


b. Occur at the 3′ end of tRNA
c. Are composed of poly A
d. Are unique to eukaryotic mRNA
e. Allow correct translation of prokaryotic Mrna

371 ‫صفحة‬ Dr. MOHAMED YAHIA 0900987639


QUESTION BANK OF Dr. MOHAMED YAHIA

The answer is: D

28. In bacterial RNA synthesis, the function of factor ρ is to

a. Bind catabolite repressor to the promoter region


b. Increase the rate of RNA synthesis
c. Eliminate the binding of RNA polymerase to the promoter
d. Participate in the proper termination of transcription
e. Allow proper initiation of transcription

The answer is: D

29. Which of the following statements correctly describes the nucleolus of a


mammalian cell?

a. It differs from that found in bacterial cells in that histones are present
b. It may contain hundreds of copies of genes for different types of ribosomal
RNAs
c. It synthesizes 5S ribosomal RNA
d. It synthesizes 60S and 40S ribosomal subunits
e. It synthesizes all ribosomal RNA primary transcripts

The answer is: B

30. Which one of the following statements correctly describes the synthesis of
mammalian messenger RNA (mRNA)?

a. Each mRNA often encodes several different proteins


b. Several different genes may produce identical mRNA molecules
c. There is colinearity of the RNA sequence transcribed from a gene and the
amino acid sequence of its encoded protein
d. The RNA sequence transcribed from a gene is virtually identical to the mRNA
that exits from nucleus to cytoplasm
e. Mammalian mRNA undergoes minimal modification during its maturation

The answer is: B

31. Studies of the genetic code in bacteria have revealed that

a. Messenger RNA (mRNA) molecules specify only one polypeptide chain


b. Many triplets can be “nonsense” triplets
c. No signal exists to indicate the end of one codon and the beginning of another
d. The nucleotide on the 5′ end of a triplet has the least specificity for an amino
acid
e. Gene sequence and encoded proteins are not collinear

The answer is: C

372 ‫صفحة‬ Dr. MOHAMED YAHIA 0900987639


QUESTION BANK OF Dr. MOHAMED YAHIA

32. Which one of the following binds to specific nucleotide sequences that are
upstream of the start site of transcription?

a. RNA polymerase
b. Primase
c. Helicase
d. Histone protein
e. Restriction endonuclease

The answer is: A

33. Which of the following most correctly describes mammalian messenger


RNAs?

a. They are usually transcribed from both DNA strands


b. They are normally double-stranded
c. Their content of uridine equals their content of adenine
d. They have an overall negative charge at neutral pH
e. Their ratio of ribose to purine bases equals 1

The answer is: D

34. New proteins destined for secretion are synthesized in the

a. Golgi apparatus
b. Smooth endoplasmic reticulum
c. Free polysomes
d. Nucleus
e. Rough endoplasmic reticulum

The answer is: E

35. How many high-energy phosphate-bond equivalents are utilized in the


process of activation of amino acids for protein synthesis?

a. Zero
b. One
c. Two
d. Three
e. Four

The answer is: C

36. The hydrolytic step leading to the release of a polypeptide chain from a
ribosome is catalyzed by

a. Stop codons
b. Peptidyl transferase

373 ‫صفحة‬ Dr. MOHAMED YAHIA 0900987639


QUESTION BANK OF Dr. MOHAMED YAHIA

c. Release factors
d. Dissociation of ribosomes
e. UAA

The answer is: B

37. The function of signal recognition particles is to

a. Cleave signal sequences


b. Detect cytosolic proteins
c. Direct the signal sequences to ribosomes
d. Bind ribosomes to endoplasmic reticulum
e. Bind mRNA to ribosomes

The answer is: D

38. Which of the following statements about ribosomes is true?

a. They are an integral part of transcription


b. They are found both free in the cytoplasm and bound to membranes
c. They are bound together so tightly they cannot dissociate under physiologic
conditions
d. They are composed of RNA, DNA, and protein
e. They are composed of three subunits of unequal size

The answer is: B

39. Guanosine triphosphate (GTP) is required by which of the following steps in


protein synthesis?

a. Aminoacyl-tRNA synthetase activation of amino acids


b. Attachment of ribosomes to endoplasmic reticulum
c. Translocation of tRNA–nascent protein complex from A to P sites
d. Attachment of mRNA to ribosomes
e. Attachment of signal recognition protein to ribosomes

The answer is: C

40. Which of the following is required for certain types of eukaryotic protein
synthesis but not for prokaryotic protein synthesis?

a. Ribosomal RNA
b. Messenger RNA
c. Signal recognition particle
d. Peptidyl transferase
e. GTP

The answer is: C

374 ‫صفحة‬ Dr. MOHAMED YAHIA 0900987639


QUESTION BANK OF Dr. MOHAMED YAHIA

BRS BIOCHEMISTRY
METABOLISM
1. A 47-year-old obese man complains of having to get out of bed three times a
night to urinate (polyuria), being constantly thirsty (polydipsia), and eating more
often (polyphagia). The patient is diagnosed with insulin resistant diabetes
mellitus (type 2). If the patient’s symptoms are due to a problem at the level of
the glucose transporter, which one of the tissues indicated below will be most
affected?

(A) RBCs
(B) Small intestine
(C) Muscle
(D) Brain
(E) Liver

The answer is: A

2. Which one of the following statements is correct concerning the formation of


muscle lactate during exercise?

(A) Lactate formation occurs when the NADH/ NAD + ratio is high.
(B) The liver preferentially converts lactate into carbon dioxide and water.
(C) The heart preferentially converts lactate into glucose.
(D) Lactate formation is less likely to be found in the eye, testes, and RBCs than
in other tissues.
(E) The intracellular pH is typically increased when lactate is produced

The answer is: A

3. A 36-year-old woman is training for her first marathon, and her coach has her
keeping a pace that allows her to stay below her anaerobic threshold. By
avoiding anaerobic muscle glycolysis, the pyruvate produced in the muscle does
not accumulate because it is converted to which one of the following?

(A) Ethanol
(B) Lactic acid
(C) Acetyl CoA
(D) Alanine
(E) OAA

The answer is: C

4. A patient presents with dizziness, fatigue, and tremors. A fingerstick test


indicates a blood glucose of 36 mmol/L. Of the allosteric activators of glycolysis

375 ‫صفحة‬ Dr. MOHAMED YAHIA 0900987639


QUESTION BANK OF Dr. MOHAMED YAHIA

in the liver, which one of the following is the most important in allowing the liver
to maintain a normal blood glucose level?

(A) Citrate
(B) ATP
(C) Fructose 2,6-bisphosphate
(D) Glucose 6-phosphate
(E) Acetyl CoA

The answer is: C

5. A 24-year-old woman complains of intermittent right upper quadrant pain


that extends to the inferior tip of her scapula. An ultrasound confirms your
suspicion of cholelithiasis, and the patient undergoes cholecystectomy. Analysis
indicates gall stones containing bilirubin. Measurement of metabolic
intermediates, such as 2,3- bisphosphoglycerate and glucose 6-phosphate, are
elevated in her serum. A deficiency of which of the following enzymes most likely
led to her pigmented gallstones and release of these metabolites into the blood?

(A) Glucose 6-phosphate dehydrogenase


(B) PFK-1
(C) Pyruvate kinase
(D) Pyruvate dehydrogenase
(E) Pyruvate carboxylase

The answer is: C

6. Emergency medical services are called to the scene of a diabetic patient who
has collapsed and is in a confused state. The patient uses an insulin pump, which
appears to have malfunctioned. The patient’s blood sugar is found to be 12
mg/dL, and the squad is having difficulty getting intravenous access to
administer intravenous glucose. On the way to the hospital, the squad
administers an intramuscular injection of glucagon. Which one of the following
statements is true regarding the use of glucagon?

(A) It is synthesized in the liver.


(B) It inhibits gluconeogenesis.
(C) It is secreted in the presence of somatostatin.
(D) It is secreted in the presence of insulin.
(E) It inhibits pyruvate formation and is also used in the treatment of b-blocker
overdose.

The answer is: E

7. A 33-year-old triathlete is admitted to the hospital after he spent the whole day
training. He looks ill and complains of diffuse weakness, fatigue, and myalgia.
Laboratory tests are sent for analysis, and his lactate level is elevated, creatinine

376 ‫صفحة‬ Dr. MOHAMED YAHIA 0900987639


QUESTION BANK OF Dr. MOHAMED YAHIA

is elevated (suggesting acute renal failure), creatine kinase is 76,000, and urine
tests positive for myoglobin. You determine he has rhabdomyolysis and treat
him with aggressive intravenous hydration. The basis for the elevated lactate is
which one of the following?

(A) An increase in ATP due to the lack of oxygen for the muscle
(B) An increase in NADH due to the lack of oxygen for the muscle
(C) A defect in the M form of lactate dehydrogenase
(D) A defect in the H form of lactate dehydrogenase
(E) A defect in the B form of muscle aldolase

The answer is: B

8. A pediatric hematologist sees an 18-month- old patient with jaundice,


splenomegaly, and hemolytic anemia. A blood smear indicates RBCs that are
more rigid in appearance than normal, and a diagnosis of pyruvate kinase
deficiency is made. Because pyruvate kinase catalyzes the last step in the
glycolytic pathway, products before this step of the pathway will accumulate.
Which one of the following products associated with the pathway will be made in
abnormal amounts?

(A) Acetyl CoA


(B) Glucose
(C) 2,3-Bisphosphoglycerate
(D) OAA
(E) Pyruvate

The answer is: C

9. A biochemistry graduate student isolates all the enzymes of the TCA cycle and
adds OAA and acetyl CoA, including the appropriate energy precursors,
cofactors, and water. Which of the following will not be a direct product of his
experiment?

(A) ATP
(B) GTP
(C) NADH
(D) CO 2
(E) FADH 2

The answer is: A

10. A 24-year-old woman presents with diarrhea, dysphagia, jaundice, and white
transverse lines on the fingernails (Mee lines). The patient is diagnosed with
arsenic poisoning, which inhib- its which one of the following enzymes?

(A) Citrate synthase

377 ‫صفحة‬ Dr. MOHAMED YAHIA 0900987639


QUESTION BANK OF Dr. MOHAMED YAHIA

(B) Isocitrate dehydrogenase


(C) Pyruvate dehydrogenase
(D) Malate dehydrogenase
(E) Succinate dehydrogenase

The answer is: C

11. A 3-year-old boy presents to the pediatric clinic with the symptoms of
hypotonia, lactic acidosis, and seizures. After an extensive workup, he is
diagnosed with PDHC deficiency, an X-linked recessive disorder. Which one of
the following cofactors is not required by this enzyme to convert pyruvate to
acetyl CoA?

(A) Thiamine
(B) Lipoic acid
(C) Pantothenate
(D) Niacin
(E) Ascorbic acid

The answer is: E

12. A 3-year-old boy presents to the emergency room after having a generalized
tonic-clonic seizure. The child has a history of epilepsy, ataxia, and lactic
acidosis. When questioned, the parents state that their child was born with a rare
metabolic disease, pyruvate carboxylase deficiency. Which one of the following
metabolites is this child unable to produce effectively?

(A) Pyruvate
(B) Alanine
(C) Acetyl CoA
(D) Oxaloacetate
(E) Acetoacetate

The answer is: D

13. MELAS is a mitochondrial disorder characterized by mitochondrial


encephalopathy, lactic acidosis, and strokelike episodes. If a cell were to contain
100% nonfunctional mitochondria, what would be the net ATP yield that would
be produced from 1 mole of glucose?

(A) 1 mole
(B) 2 moles
(C) 4 moles
(D) 8 moles
(E) 0 moles

The answer is: B

378 ‫صفحة‬ Dr. MOHAMED YAHIA 0900987639


QUESTION BANK OF Dr. MOHAMED YAHIA

14. A newborn is found to have fasting hypoglycemia. The nursery staff begins
overnight feeds by nasogastric tube because they find that the child has
consistently low blood sugars. A liver biopsy and molecular studies demonstrate
an absence of glycogen synthase. The normal function of this enzyme is to do
which of the following?

(A) Remove glucose residues one at a time from glycogen in the liver
(B) Remove glucose residues one at a time from glycogen in muscles
(C) Transfer glucose from UDP-glucose to the nonreducing end of a glycogen
primer
(D) Hydrolyze a-1,6 bonds of glycogen
(E) Function as a glucosyl 4:6 transferase

The answer is: C

15. A newborn is experiencing failure to thrive. On physical examination,


organomegaly is appreciated owing to accumulation of glycogen in the lysosomes
of several organs, including the heart, muscle, and liver. You diagnose the
condition as Pompe disease. Which one of the following biochemical deficits is
seen in this disorder?

(A) A deficiency of glycogenin


(B) Loss of a-1,6-glucosidase activity
(C) Loss of glucose 6-phosphatase activity
(D) Loss of muscle glycogen phosphorylase activity
(E) Loss of a lysosomal glucosidase activity

The answer is: E

16. A second-year medical student decides to do research in a nutrition


laboratory that is studying the effects of caffeine on cellular metabolism.
Caffeine inhibits cAMP phosphodiesterase. If caffeine were added to liver cells,
in the presence of glucagon, which of the following enzymes would be
phosphorylated and inactivated?

(A) Phosphorylase kinase


(B) Pyruvate kinase
(C) Phosphorylase
(D) Protein kinase A
(E) Calmodulin

The answer is: B

17. A 28-year-old professional cyclist has been training for an opportunity to


race in the Tour de France. His coach strongly suggests that he consume
carbohydrates after each of his workouts to ensure that his muscle glycogen

379 ‫صفحة‬ Dr. MOHAMED YAHIA 0900987639


QUESTION BANK OF Dr. MOHAMED YAHIA

storage can endure the 28-day race. The activity of muscle glycogen synthase in
resting muscles is increased by the action of which of the following?

(A) Epinephrine
(B) Glucagon
(C) Insulin
(D) Phosphorylation
(E) Fasting and starvation

The answer is: C

18. A patient had large deposits of liver glycogen, which, after an overnight fast,
contained shorter than normal branches. A defective form of which of the
following could cause this abnormality?

(A) Glycogen phosphorylase


(B) Glucagon receptor
(C) Glycogenin
(D) Amylo-1,6-glucosidase (a-glucosidase)
(E) Amylo-4,6-transferase (4:6 transferase)

The answer is: D

19. A sprinter is trying to optimize his performance. He has calculated that, even
under anaerobic conditions, his glycogen stores will supply him with enough
energy to last the race. What would the energy difference be between using
glucose from a dietary source versus relying solely on glucose from glycogen
stores as fuel for his race?

(A) Dietary would give 1 more mole of ATP/ glucose


(B) Dietary would give 2 more moles of ATP/ glucose
(C) Dietary would give the same ATP/glucose
(D) Dietary would give 1 less mole of ATP/ glucose
(E) Dietary would give 2 less moles of ATP/ Glucose

The answer is: D

20. Which one of the following occurs in an individual who is rested and has
fasted for 12 hours?

(A) Gluconeogenesis is the major process by which blood glucose is maintained.


(B) Adenylate cyclase has been inactivated in liver.
(C) Liver glycogen stores have been depleted.
(D) Glycogen phosphorylase, pyruvate kinase, and glycogen synthase are
phosphorylated in the liver.
(E) Glycogen synthase has been activated in liver.

The answer is: D

380 ‫صفحة‬ Dr. MOHAMED YAHIA 0900987639


QUESTION BANK OF Dr. MOHAMED YAHIA

21. A 32-year-old bodybuilder has decided to go on a diet consisting of only egg


whites to ensure optimal protein for muscle growth. After a few weeks, he notices
decreased energy and is found to be hypoglycemic. A nutritionist tells the patient
that he most likely has a functional biotin deficiency. Which of the following
enzymes is unable to catalyze a key step in synthesizing glucose from pyruvate?

(A) Pyruvate carboxylase


(B) Phosphoenolpyruvate carboxykinase
(C) Fructose 1,6-bisphosphatase
(D) Glucose 6-phosphatase
(E) Phosphoglycerate kinase

The answer is: A

22. A 62-year-old, obese man complains of polydipsia (increased drinking),


polyuria (increased urination), and fatigue. A glucose tolerance test confirms the
diagnosis of diabetes. He is placed on metformin, which works by which of the
following mechanisms?

(A) Inhibiting hepatic gluconeogenesis


(B) Increasing glucagon levels
(C) Increasing cellular responsiveness to circulating insulin
(D) Stimulating the release of preformed insulin
(E) Replacing the need for endogenous insulin

The answer is: C

23. A 41-year-old woman presents with severe, sharp epigastric abdominal pain
that radiates to her back and with nausea and vomiting. Laboratory results
indicate a serum triglyceride level of 5000 mg/dL. She is diagnosed with
pancreatitis, in part owing to her elevated serum triacyl- glycerol levels. To form
triacylglycerol from diacylglycerol, which of the following compounds is also
required?

(A) Glycerol
(B) Glycerol 3-phosphate
(C) Fatty acyl CoA
(D) Acetyl CoA
(E) Malonyl CoA

The answer is: C

24. An 18-year-old woman presents with xanthomas on her eyelids and is found
to have a rare genetic deficiency of lipoprotein lipase. She is diagnosed with type
I hyperlipidemia. In this disorder, chylomicrons are abnormally elevated in the
serum. In which cell or tissue does triacyl- glycerol packaging into chylomicrons
occur?

381 ‫صفحة‬ Dr. MOHAMED YAHIA 0900987639


QUESTION BANK OF Dr. MOHAMED YAHIA

(A) Intestinal epithelial cell


(B) Liver cell
(C) Muscle cell
(D) Heart cell
(E) Adipose cell

The answer is: A

25. A 2-week-old child underwent complex con- genital heart malformation


repair. The cardiothoracic surgeon accompanies the patient back from the
operating room and tells the pediatric intensive care unit staff that the ASD
(atrial septal defect) and VSD (ventricular septal defect) were successfully
repaired. However, the thoracic duct was accidentally cut, and daily
echocardiograms will be needed to evaluate for cor pulmonale (alterations in the
right ventricle of the heart). Which one of the following statements is true
concerning fat metabolism in this patient?

(A) The thoracic duct carries a substantial volume of lymph and triglycerides
from the enteric circulation to the venous system.
(B) Triacylglycerol is primarily stored in the liver.
(C) In the intestinal cell, glucose is converted to triacylglycerol by phosphatidic
acid.
(D) In adipose cells, triacylglycerol is converted to VLDL.
(E) Somatostatin has no role in the treatment of chylothorax in children.

The answer is: A

26. An 18-year-old obese woman maintains a sedentary lifestyle and eats a high-
fat, high carbohydrate diet. Maintenance of this diet and lifestyle has led to
lipogenesis and obesity. Which of the following statements correctly describes an
aspect of lipogenesis?

(A) The primary source of carbons for fatty acid synthesis is glycerol.
(B) Fatty acids are synthesized from acetyl CoA in the mitochondria.
(C) Fatty acid synthesis and esterification to glycerol to form triacylglycerols
occurs primarily in muscle cells.
(D) The fatty acyl chain on the fatty acid synthase complex is elongated two
carbons at a time.
(E) NADP + , which is important for fatty acid synthesis, is produced by the
pentose phosphate pathway.

The answer is: D

27. A 45-year-old man presents with multiple gunshot wounds to the abdomen
requiring an emergent laparotomy, jejunectomy, and colectomy. After surgery,
he is placed on intravenous nutrition (i.e., TPN). Which of the following
compounds should be a component of TPN?
382 ‫صفحة‬ Dr. MOHAMED YAHIA 0900987639
QUESTION BANK OF Dr. MOHAMED YAHIA

(A) Palmitate
(B) Linoleate
(C) Phosphatidic acid
(D) Glycerol
(E) Glucose

The answer is: B

28. A 16-year-old girl presents with extreme slenderness. Her body weight is
35% below expected. She feels as though she is obese and severely restricts her
food intake. She is diagnosed with anorexia nervosa. In this patient, breakdown
of fatty acids is required to provide energy. Before being oxidized, fatty acids are
activated in the cytosol to form which of the following?

(A) ATP
(B) CoA
(C) Fatty acyl CoA
(D) Carnitine
(E) Malonyl CoA

The answer is: C

29. After surgical resection of part of her small intestine, a 40-year-old woman
presents with chronic foul-smelling diarrhea and weight loss. She is diagnosed
with short bowel syndrome. In this syndrome, fat cannot be properly absorbed,
so long-chain fatty acids are mobilized from adipose tissue to generate energy for
cell survival. The initiating substrate for fatty acid oxidation is which of the
following?

(A) Long-chain fatty acid


(B) Fatty acyl carnitine
(C) Fatty acyl CoA
(D) b-Hydroxyacyl CoA
(E) Acetyl CoA

The answer is: C

30. An infant is born with a high forehead, abnormal eye folds, and deformed ear
lobes and shows little muscle tone and movement. After multiple tests, he is
diagnosed with Zellweger syndrome, a disorder caused by peroxisome
malformation. What type of fatty acid would you expect to accumulate in
patients with Zellweger syndrome?

(A) Short-chain fatty acids


(B) Acetyl CoA
(C) Dicarboxylic acids
(D) Long-chain fatty acids

383 ‫صفحة‬ Dr. MOHAMED YAHIA 0900987639


QUESTION BANK OF Dr. MOHAMED YAHIA

(E) Very-long-chain fatty acids

The answer is: E

31. A 4-month-old infant presents with a seizure. His mother reports that her
infant has been irritable and lethargic over the past several days. The infant is
found to be profoundly hypoglycemic and have low ketones. Short-chain
dicarboxylic acids are found to be elevated in the serum. The most likely enzyme
deficiency is which of the following?

(A) Medium-chain acyl CoA dehydrogenase (MCAD)


(B) Carnitine acyltransferase I
(C) Hormone-sensitive lipase
(D) Pyruvate carboxylase
(E) Fatty acyl CoA synthetase

The answer is: A

31. A 12-year-old Jamaican boy presents with intractable vomiting, abdominal


pain, and lethargy and is profoundly hypoglycemic. His symptoms are caused by
Jamaican vomiting syndrome, a sickness caused by ingestion of hypoglycin,
which is present in unripe ackee fruit. Hypoglycin is metabolized to a form of
nonmetabolizable carnitine, which interferes with normal fatty acid oxidation.
What is the primary role of carnitine?

(A) Activates long-chain fatty acids in the cytosol


(B) Transport of acyl groups across the inner mitochondrial membrane
(C) Is converted to enoyl CoA
(D) Is converted to b-hydroxyacyl CoA
(E) Is involved in breakdown of even-chain, but not odd-chain, fatty acids

The answer is: B

32. Which of the following apoproteins is an activator of lipoprotein lipase?

(A) Apo A
(B) Apo B
(C) Apo C-II
(D) Apo D
(E) Apo E

The answer is: C

33. The major carriers of triacylglycerols are which of the following?

(A) Chylomicrons and VLDL


(B) IDL and LDL
(C) VLDL and LDL

384 ‫صفحة‬ Dr. MOHAMED YAHIA 0900987639


QUESTION BANK OF Dr. MOHAMED YAHIA

(D) HDL and LDL


(E) Chylomicrons and LDL

The answer is: A

34. A 40-year-old Hispanic woman with a body mass index of 34 presents with
acute right upper quadrant pain, nausea, and vomiting after eating a meal rich
in lipids. She is diagnosed with having cholelithiasis and is placed on a bile salt
analog that is used to inhibit the formation of cholesterol gallstones. Which of the
following is an example of a bile salt?

(A) HMG-CoA
(B) Mevalonate
(C) Squalene
(D) Lanosterol
(E) Chenocholic acid

The answer is: E

35. An 8-year-old boy presents with orange- colored tonsils, a very low HDL
level, and an enlarged liver and spleen and is diagnosed with Tangier disease.
Which of the following statements best describes HDL?

(A) It is produced in skeletal muscle.


(B) It scavenges cholesterol from cell membranes.
(C) Its major protein is apo E.
(D) It is formed when VLDL is digested by lipo- protein lipase.
(E) It activates ACAT.

The answer is: B

36. A 40-year-old man presents with chest pain that radiates to his left jaw and
shoulder. He is diagnosed with a myocardial infarction and is prescribed a statin
medication. Statins are competitive inhibitors of HMG-CoA reductase, which
converts HMG-CoA to which of the following?

(A) Mevalonate
(B) Isopentenyl pyrophosphate
(C) Geranyl pyrophosphate
(D) Farnesyl pyrophosphate
(E) Cholesterol

The answer is: A

37. A 45-year-old woman presents with oily, foul-smelling stool, which appears to
be due to an obstruction of the bile duct. Which of the following statements
correctly describes bile salts?

385 ‫صفحة‬ Dr. MOHAMED YAHIA 0900987639


QUESTION BANK OF Dr. MOHAMED YAHIA

(A) They can act as detergents, aiding in lipid digestion.


(B) They are stored in the intestines.
(C) Ninety-five percent of bile salts are excreted in the feces, and 5% are recycled
back to the liver.
(D) Bile salts are synthesized in the intestines.
(E) Squalene and lanosterol are examples of bile salts.

The answer is: A

38. A 55-year-old woman presents with crushing substernal chest pain and
shortness of breath. A coronary artery is occluded owing to an athero- sclerotic
plaque, and a high myocardial infarct is diagnosed. High serum HDL levels are
protective against the development of atherosclerosis because HDL does which of
the following?

(A) Inhibits cholesterol production by the liver


(B) Inhibits HMG-CoA reductase
(C) Increases VLDL production
(D) Increases LDL production
(E) Brings cholesterol esters back to the liver

The answer is: E

39. A 30-year-old man presents with weakness in his right upper and lower
extremities. He is diagnosed with an acute middle cerebral artery stroke
secondary to atherosclerosis. Genetic studies show that he has familial
hypercholesterolemia, type II, a disorder caused by a deficiency of LDL
receptors. Which of the following statements best describes patients with type II
familial hypercholesterolemia?

(A) After LDL binds to the LDL receptor, the LDL is degraded extracellularly.
(B) The number of LDL receptors on the surface of hepatocytes increases.
(C) Cholesterol synthesis by hepatocytes increases.
(D) Excessive cholesterol is released by LDL.
(E) The cholesterol level in the serum decreases.

The answer is: C

40. A 40-year-old woman presents with an LDL serum level of 400


(recommended level is <130), and a triglyceride level of 170 (recommended level
is <150). She is diagnosed with type II familial hypercholesterolemia. In this
disorder, a mutated LDL receptor is formed, such that it cannot bind to LDL.
Which of the following would result?

(A) Cellular HMG-CoA reductase activity is not inhibited.


(B) The triglycerides in chylomicrons cannot be degraded.
(C) The VLDL level in the serum increases.

386 ‫صفحة‬ Dr. MOHAMED YAHIA 0900987639


QUESTION BANK OF Dr. MOHAMED YAHIA

(D) The HDL level in the serum increases.


(E) The VLDL cannot be converted to IDL.

The answer is: A

41. A 25-year-old woman presents with a low red blood cell count, corneal
opacities, and kidney insufficiency. She is diagnosed with LCAT deficiency.
LCAT is involved in which of the following processes?

(A) Converting cholesterol to cholesterol esters


(B) The transfer of cholesterol esters from HDL to other lipoproteins
(C) Endocytosis of HDL particles into hepatocytes
(D) Hydrolysis of HDL
(E) Decreased uptake of cholesterol by Hepatocytes

The answer is: A

42. A 5-year-old mentally retarded child is seen by an ophthalmologist for


‘‘blurry vision.’’ Ocular examination demonstrates bilateral lens dislocations,
and further workup is significant for osteoporosis and homocystinuria. Serum
analysis would most likely show an elevation of which of the following
substances?

(A) Cystathionine
(B) Valine
(C) Phenylalanine
(D) Tyrosine
(E) Methionine

The answer is: E

43. A 3-month-old child presents with vomiting and convulsions. Notable


findings include hepatomegaly and hyperammonemia. A deficiency in which of
the following enzymes would most likely cause an elevation of blood ammonia
levels?

(A) CPS II
(B) Glutaminase
(C) Argininosuccinate lyase
(D) Asparagine synthetase
(E) Urease

The answer is: A

44. A 55-year-old man suffers from cirrhosis of the liver. He has been admitted
to the hospital several times for hepatic encephalopathy. His damaged liver has
compromised his ability to detoxify ammonia. Which of the following amino

387 ‫صفحة‬ Dr. MOHAMED YAHIA 0900987639


QUESTION BANK OF Dr. MOHAMED YAHIA

acids can be used to fix ammonia and thus transport and store ammonia in a
nontoxic form?

(A) Aspartate
(B) Glutamate
(C) Serine
(D) Cysteine
(E) Histidine

The answer is: B

45. A 27-year-old, semiprofessional tennis player seeks advice from a hospital-


based nutritionist concerning his diet supplements. His coach had given him
amino acid supplements consisting of phenylalanine and tyrosine. The rationale
was that these neurotransmitter precursors would ‘‘help his brain focus’’ on his
game. In reality, excess phenylalanine will be metabolized to provide energy.
Phenylalanine will enter the TCA cycle as which one of the following TCA cycle
intermediates?

(A) Oxaloacetate
(B) Citrate
(C) a-Ketoglutarate
(D) Fumarate
(E) Succinyl CoA

The answer is: D

46. A 2-year-old girl was seen in the emergency room for vomiting and tremors.
Laboratory tests revealed a plasma ammonium ion concentration of 195 mM
(normal, 11- to 50 mM) and serum elevation of arginine. Two days later, after
stabilization, ammonia and arginine levels were normal. You conclude that this
patient may have a defect in which of the following enzymes?

(A) CPS I
(B) CPS II
(C) Ornithine transcarbamoylase
(D) Arginase
(E) Argininosuccinate lyase

The answer is: D

47. A 23-year-old Golden Gloves boxing contender presents with assorted


metabolic disorders, most notably ketosis. During the history and physical
examination, he describes his training regimen, which is modeled after the Rocky
films and involves consuming a dozen raw eggs a day for protein. Raw eggs
contain a 70-kD protein called avidin, with an extremely high affinity for a
cofactor required by propionyl CoA carboxylase, pyruvate carboxylase, and

388 ‫صفحة‬ Dr. MOHAMED YAHIA 0900987639


QUESTION BANK OF Dr. MOHAMED YAHIA

acetyl CoA carboxylase. The patient is functionally deficient in which one of the
following cofactors?

(A) Tetrahydrobiopterin
(B) Tetrahydrofolate
(C) Biotin
(D) Methylcobalamin
(E) Pyridoxal phosphate

The answer is: C

48. A new test is developed that can nonradioactively ‘‘label’’ compounds in the
human body. As a physician with a background in the new field of metabolomics,
you assess a 21-year-old with classic PKU. The patient is fed phenylalanine with
a label in the phenyl ring, and a 24-hour urine sample is collected. Which of the
following compounds would you expect to contain the greatest amount of label in
this urine sample?

(A) Tyrosine
(B) Tryptophan
(C) Epinephrine
(D) Phenylketone
(E) Acetate

The answer is: D

49. During a medical rotation, a medical student volunteered for a respiratory


physiology examination that determines basal metabolic rate and the respiratory
quotient. She followed the protocol for a resting individual in the post absorptive
state. Which of the following amino acids would be found in the highest
concentration in serum?

(A) Alanine and glutamine


(B) Arginine and ornithine
(C) Glutamate and aspartate
(D) Branched chain amino acid
(E) Hydrophobic amino acids

The answer is: A

NUTRITION
50. A 57-year-old alcoholic man with chronic pancreatitis is admitted to the
hospital for treatment. The absorption of which one of the following vitamins
may be affected with pancreatitis?

(A) Vitamin B 12 (cobalamin)

389 ‫صفحة‬ Dr. MOHAMED YAHIA 0900987639


QUESTION BANK OF Dr. MOHAMED YAHIA

(B) Folic acid


(C) Vitamin B 2 (riboflavin)
(D) Vitamin B 6 (pyridoxine)
(E) Vitamin D

The answer is: E

51. A 54-year-old Native American living on an Indian reservation in southwest


Arizona is brought into the clinic by a family member. They are concerned
because of impaired memory, diarrhea, and a rash on the face, neck, and dorsum
of the hands. Which vitamin deficiency do these symptoms represent?

(A) Niacin
(B) Cobalamin
(C) Folic acid
(D) Vitamin C
(E) Vitamin E

The answer is: A

52. A 32-year-old woman presents to the physician with extreme fatigue and
vague neurologic complaints. On examination, it is found that she has decreased
positional and vibrational sense, and her complete blood count reveals a
megaloblastic anemia. She relates a history of gastric resection 4 years ago for
severe stomach ulcers. Which vitamin deficiency does this represent?

(A) Vitamin C
(B) Vitamin D
(C) Vitamin K
(D) Vitamin B 12
(E) Folate

The answer is: D

53. A 75-year-old chronic alcoholic man presents to the emergency room after
being found unconscious on the floor of his home. On examination, he is found to
have a distended abdomen consistent with ascites. Which of the following
functions of the liver has been compromised to lead To the finding of abdominal
ascites?

(A) Lipid metabolism


(B) Albumin synthesis
(C) Estrogen metabolism
(D) Alcohol detoxification
(E) Decreased production ofcoagulation factors

The answer is: B

390 ‫صفحة‬ Dr. MOHAMED YAHIA 0900987639


QUESTION BANK OF Dr. MOHAMED YAHIA

54. A 45-year-old alcoholic man walks into the emergency room with a clumsy,
wide-based gait and appears confused. He has pronounced nystagmus, and
laboratory tests are significant for a metabolic acidosis and a serum blood
alcohol level of 0.13. This patient should most probably be treated with IV fluids
containing which of the following?

(A) Thiamine
(B) Riboflavin
(C) Niacin
(D) Pantothenic acid
(E) Biotin

The answer is: A

55. A fourth-year medical student does an international rotation in Sub-Saharan


Africa. While immunizing children against polio, he sees hundreds of
malnourished children in refugee camps with bloated-appearing abdomens. He
learns that they are severely protein deficient because they are fed a diet of
cornmeal that is provided by international relief agencies. These children likely
suffer from which of the following?

(A) Marasmus
(B) Anorexia nervosa
(C) Bulimia
(D) Kwashiorkor
(E) Cachexia

The answer is: D

GENETICS
56. A 23-year-old man presents to his family physician with a painless swelling of
his testicles. An ultrasound is suspicious for a neoplasm, and a biopsy confirms
the presence of cancer. He is referred to an oncologist, who begins treatment
with the topoisomerase inhibitor etoposide. The normal function of this enzyme
is to do which of the following?

(A) Repair nuclear DNA in the event of DNA damage


(B) Unwind the DNA helix during replication
(C) Break and rejoin the DNA helix during replication
(D) Prevent the single strands of DNA from reannealing during replication
(E) Synthesize RNA primers for DNA polymerase

The answer is: C

391 ‫صفحة‬ Dr. MOHAMED YAHIA 0900987639


QUESTION BANK OF Dr. MOHAMED YAHIA

57. A 33-year-old homosexual man is recently diagnosed with HIV. His CD4 + T-
cell count is dramatically decreased, and he has a high HIV viral load. He is
referred to an infectious disease clinic where they begin him on a nucleoside
analog. Certain nucleoside analogs inhibit DNA synthesis because they lack
which of the following properties required for normal DNA polymerization?

(A) A 5 0 -phosphate
(B) A 3 0 -hydroxyl
(C) A 7-methyl G modification
(D) A poly(A) tail
(E) A consensus sequence

The answer is: B

58. A 53-year-old man is referred to a neurologist because he is beginning to


develop spastic like movements in his lower limbs. Magnetic resonance imaging
of the head shows loss of mass in the caudate nucleus, and a presumptive
diagnosis of Huntington disease is made. The genetic basis of this disease is best
described as which of the following?

(A) Triple repeat expansion


(B) Nucleotide deletion
(C) Point mutation
(D) Transposition of genetic material
(E) DNA methylation

The answer is: A

59. A 37-year-old immigrant from Thailand develops fevers, night sweats, weight
loss, and a blood-tinged cough. He present to the emergency room, where an
infectious disease doctor is consulted and immediately prescribes a multidrug
regimen that includes rifampin. Rifampin inhibits which one of the following
types of enzymes?

(A) DNA-dependent DNA polymerase


(B) DNA-dependent RNA polymerase
(C) RNA-dependent DNA polymerase
(D) RNA-dependent RNA polymerase
(E) Reverse transcriptase

The answer is: B

60. Two couples present to the emergency room with severe nausea, vomiting,
and diarrhea. One of the patients admits that she had a dinner party and served
a salad containing mushrooms she had picked during a hike in the forest earlier
that day. Inhibition of which enzyme or process explains the clinical
manifestations of a-amanitin poisoning seen in these patients?

392 ‫صفحة‬ Dr. MOHAMED YAHIA 0900987639


QUESTION BANK OF Dr. MOHAMED YAHIA

(A) RNA polymerase II


(B) RNA polymerase I
(C) RNA splicing
(D) RNA polyadenylation
(E) RNA polymerase III

The answer is: A

61. A 23-year-old diabetic woman reports having fevers and dysuria. Physical
examination reveals costovertebral tenderness, and her urinary analysis shows
the presence of bacteria in her urine. Her physician suspects a complicated
urinary tract infection and begins a 5-day course of ciprofloxacin. Such
quinolone antibiotics inhibit which one of the following enzymes?

(A) Eukaryotic topoisomerase


(B) Helicase
(C) Primase
(D) Gyrase
(E) Poly(A) polymerase

The answer is: D

62. A 4-year-old child is referred by the pediatrician to a pediatric neurologist


after presenting with myoclonic seizures and lactic acidosis. The neurologist
orders a muscle biopsy, and the pathology returns with the appearance of
‘‘ragged red fibers.’’ The parents are informed that the child has MERFF
syndrome, a mitochondrial DNA (mtDNA) disorder. Which of the following
statements best explains mtDNA?

(A) It is inherited equally from both parents.


(B) It is replicated with increased fidelity with respect to nuclear DNA.
(C) It shares the same genetic code as nuclear DNA.
(D) It is a double-stranded circular DNA.
(E) It encodes all the proteins necessary for the electron transport chain.

The answer is: D

63. A 34-year-old man of Italian descent is seen for a yearly physical


examination. He has no complaints and is in good health. However, he does relay
a family history of anemia, and a complete blood count demonstrates a mild
anemia; the physician suspects thalassemia minor in the patient. Thalassemia is
often due to an alteration in RNA splicing, which is an essential part of mRNA
processing in eukaryotes. Which of the following is a correct statement concern-
ing mRNA processing?

(A) Poly(A) RNA is the initial transcript produced, which is subsequently spliced
to mRNA.

393 ‫صفحة‬ Dr. MOHAMED YAHIA 0900987639


QUESTION BANK OF Dr. MOHAMED YAHIA

(B) The coding region of the gene is found within introns.


(C) The coding regions of the gene are found within exons.
(D) All human genes require splicing of introns.
(E) The 5 0 end of the intron contains the nucleotides AG followed by an
invariant GU.

The answer is: C

64. A 4-year-old child, on a well-child checkup, is found to have a large flank


mass. Computed tomography demonstrates a large mass arising from the
kidney, and a subsequent biopsy reveals a diagnosis of Wilms tumor. A pediatric
oncologist starts chemotherapy including the transcription inhibitor actinomycin
D. Which of the following statements is correct regarding transcription
regulation in bacteria?

(A) All mRNAs are monocistronic.


(B) RNA polymerase requires a primer.
(C) The RNA chain grows in the 3 0 to 5 0 direction.
(D) Rho factor is critical for initiation of RNA synthesis.
(E) The TATA box contains a consensus sequence for the binding of RNA
polymerase.

The answer is: E

65. A second-year medical student is working in a laboratory studying gene


regulation in a mouse model of hepatocellular carcinoma. He isolates nucleic
acids from the cells after expo- sure to a known carcinogen and has the
sequences analyzed. He is surprised to find that some of the nucleotides are
pseudouridine and ribothymidine. Which type of nucleic acid has the student
likely isolated?

(A) tRNA
(B) rRNA
(C) hnRNA
(D) mRNA
(E) snRNP

The answer is: A

66. A 14-year-old African American girl with sickle cell anemia presents with
extreme pain in her chest and legs. A peripheral smear shows sickling of her red
blood cells, and her reticul cyte count is 6.7%. She is diagnosed with an acute
pain crisis secondary to microvascular occlusion from sickled erythrocytes.
Sickle cell anemia is caused by a mutation that results in one amino acid being
replaced by another, which results from which of the following?

(A) A missense point mutation

394 ‫صفحة‬ Dr. MOHAMED YAHIA 0900987639


QUESTION BANK OF Dr. MOHAMED YAHIA

(B) A nonsense point mutation


(C) An insertion
(D) A deletion
(E) A frameshift mutation

The answer is: A

67. A 53-year-old man sees his family physician with concerns that his skin is
‘‘bronzing.’’ He is found to have diabetes as well as an elevated ferritin level (a
sign of iron overload). The physician suspects hemochromatosis and confirms the
diagnosis with genetic testing. It is found that the patient’s DNA carries a
mutation in which tyrosine is substituted for cysteine at position 282 (C282Y)
within the HFE gene. What word below best describes this type of mutation?

(A) Silent
(B) Missense
(C) Nonsense
(D) Frameshift
(E) Deletion

The answer is: B

68. A 43-year-old woman, who has recently had difficulty getting up and out of a
chair and finds going up stairs to be troublesome, is referred to a rheumatologist
for her progressive weakness. A full rheumatologic workup reveals anti–Jo-1
antibodies, indicating that the patient likely has polymyositis. The physician
explains to the patient that she has autoantibodies directed against histidinyl-
tRNA synthetase (the Jo-1 antigen). Which of the following best describes a
property of this protein?

(A) To form an aminoacyl-tRNA synthetase complex in the absence of energy


(B) To initiate transcription by interacting with the 30S subunit of the ribosome
(C) To recognize and covalently link a particular amino acid and a particular
tRNA for that amino acid
(D) To bind puromycin, which terminates protein synthesis
(E) To covalently link amino acids to the 5 0 end of a corresponding Trna

The answer is: C

69. A 54-year-old man presents to his family physician with a 3- to 4-week


history of a non- productive cough and a low-grade fever. The physician suspects
an atypical pneumonia from Mycoplasma pneumoniae, and decides to treat the
patient empirically with erythromycin. Which of the following best describes the
mechanism of action of erythromycin?

(A) Binds to the 5 0 cap of mRNA and prevents translation


(B) Binds to the 30S ribosomal subunit and blocks peptidyl transferase activity

395 ‫صفحة‬ Dr. MOHAMED YAHIA 0900987639


QUESTION BANK OF Dr. MOHAMED YAHIA

(C) Binds to the Shine-Dalgarno sequence of mRNA and prevents translation


(D) Binds to and inhibits initiation factor IF-1 from binding to the initiation
complex
(E) Binds to the 50S ribosomal subunit and blocks translocation during
translation

The answer is: E

70. A 27-year-old man is seen by his physician for a week-long cough, sore
throat, and difficulty swallowing. He is diagnosed with diphtheria, which has
reactivated because of waning immunity. One way in which diphtheria toxin
leads to cell death is through the inhibition of eEF-2. Which statement best
explains the function of eEF-2?

(A) It is required for the translocation of peptidyl-tRNA during translation.


(B) It is required for the initiation of protein synthesis.
(C) It is the agent that binds to, and is inactivated by, chloramphenicol.
(D) It functions as a peptidyl transferase.
(E) It is analogous to the prokaryotic factor eIF-1.

The answer is: A

71. A third-year medical student joins a laboratory that studies gene regulation.
The laboratory uses bacteria to study gene expression and metabolic regulation
after exposure to toxic com- pounds. The goal is to generalize the observations
seen in prokaryotic cells in response to the toxins and to compare the response to
eukaryotic cells. Which of the following statements is true regarding prokaryotic
and eukaryotic genomes?

(A) They are both diploid.


(B) They both organize and compact their DNA with histones.
(C) They both have short repetitive DNA sequences throughout their genome.
(D) They both organize their genes into operons.
(E) They both use the same genetic code to convert codons to amino acids.

The answer is: E

72. A 23-year-old man is seen in the gastroenterologist’s office for a referral


concerning a family history of a 1 -antitrypsin deficiency. The physician
arranges for the interventional radiologist to perform a liver biopsy, which will
allow a determination of the extent of accumulation of non secreted protein in
the hepatocyte. Which of the following statements is true concerning proteins
like a 1 -antitrypsin that are normally secreted from the cell?

(A) They are synthesized on ribosomes attached to the smooth endoplasmic


reticulum.
(B) They contain a hydrophilic signal sequence.

396 ‫صفحة‬ Dr. MOHAMED YAHIA 0900987639


QUESTION BANK OF Dr. MOHAMED YAHIA

(C) The signal sequence is found on the C terminus of the protein.


(D) Glycosylation takes place only in the endoplasmic reticulum.
(E) Proteins travel from the endoplasmic reticulum to the Golgi apparatus and
are ultimately secreted by exocytosis.

The answer is: E

73. One colony of bacteria is split into two Petri plates: one plate with growth
medium containing glucose and all 20 amino acids and one medium with one
sugar (lactose) and one nitrogen source (NH 4 + ). Which of the following
statements is correct concerning the cells growing in the second medium?

(A) cAMP levels will be lower than cells growing in the presence of glucose.
(B) CAP protein (cAMP-binding protein) will be bound to the lac promoter.
(C) The lac repressor will be bound to the lac operator.
(D) RNA polymerase will not bind to the trp promoter.
(E) Attenuation of transcription of the trp operon will increase.

The answer is: B

74. Gene transcription rates and mRNA levels were determined for an enzyme
that is induced by glucocorticoids. Compared with untreated levels,
glucocorticoid treatment caused a 10-fold increase in the gene transcription rate
and a 20-fold increase in both mRNA levels and enzyme activity. These data
indicate that a primary effect of glucocorticoid in this assay is to decrease which
of the following?

(A) The activity of RNA polymerase II


(B) The rate of mRNA translation
(C) The ability of nucleases to act on mRNA
(D) The rate of binding of ribosomes to mRNA
(E) The activity of RNA polymerase III

The answer is: C

75. An 8-year-old boy of Ashkenazi Jewish descent presents with bone pain and
easy bruising. His parents have no known history of serious medical ailments. He
is found to have hepatosplenomegaly (enlarged liver and spleen), anemia, and an
Erlenmeyer flask deformity of his distal femur. Which of the following is true for
this patient?

(A) The likelihood of another, to be born, sibling being affected with the disease
is 50%.
(B) At least one person in every generation of his pedigree is likely to be affected.
(C) His disease will only affect males.
(D) The probability that the patient’s unaffected siblings will be heterozygous
carriers is 67%.

397 ‫صفحة‬ Dr. MOHAMED YAHIA 0900987639


QUESTION BANK OF Dr. MOHAMED YAHIA

(E) His parents will become affected with the disease.

The answer is: D

76. A 2-year-old boy with Down syndrome requires intubation in the intensive
care unit due to difficulty breathing. He is afflicted with congenital heart disease
associated with the disease, and he dies shortly after admission. What is the most
common genetic cause of Down syndrome?

(A) Meiotic nondisjunction


(B) Autosomal dominant inheritance
(C) X-linked recessive inheritance
(D) Decreased maternal age
(E) Monosomy 21

The answer is: A

77. A 30-year-old woman presents to her obstetrician for her 6-month pregnancy
examination. The patient has already had one son born with Edwards syndrome,
who died after 1 week of life. She is concerned about the fate of the current
pregnancy because after her first child was born she was seen by a clinical
geneticist who told her that she is a mosaic for trisomy 18. Which of the
following statements is true for this patient?

(A) She will develop severe trisomy 18 later in her life.


(B) Her baby will definitely have Edwards syndrome if the child is a boy.
(C) Her baby will not have trisomy if the child is a girl.
(D) Trisomy 18 in her son was likely due to some of her primary oocytes being
45,XX,?18
(E) Some of her primary oocytes are 47,XX,+18.

The answer is: E

78. A 50-year-old woman presents with spleen enlargement and a decreased red
blood cell count. She is diagnosed with chronic myelogenous leukemia (CML), a
blood cell cancer in which a portion of chromosome 9 is aberrantly attached to
chromosome 22. CML thus serves as a paradigm for other blood cell cancers, as
an example of which of the following?

(A) Transgenic expression


(B) Reciprocal gene translocation
(C) Robertsonian translocation
(D) Gene knockout
(E) Single gene defect disease

The answer is: B

398 ‫صفحة‬ Dr. MOHAMED YAHIA 0900987639


QUESTION BANK OF Dr. MOHAMED YAHIA

79. A 50-year-old man presents with personality changes and an uncontrolled


choreic (dancing) movement of one arm. His mother has Huntington disease,
which is an autosomal dominant disease characterized by an abnormally long
trinucleotide repeat sequence in a specific area on one chromosome. He would
like to know if he also has Huntington disease. Which of the following techniques
would be most precise in detecting this genetic abnormality?

(A) Karyotyping
(B) Phenotype characterization
(C) DNA sequencing
(D) Amniocentesis
(E) Chorionic villus sampling

The answer is: C

80. A woman is pregnant with her fourth child. She has had an uneventful
pregnancy and is offered amniocentesis. Cytogenetic analysis performed on
amniotic fluid reveals 47,XY,+21.Which of the following statements is true of
amniocentesis?

(A) The procedure can be performed at 4 to 10 weeks


(B) The procedure is unlikely to reveal a cyto- genetic abnormality
(C) Risk includes miscarriage and spontaneous abortion
(D) A biopsy of chorionic villus undergoes DNA analysis
(E) The procedure is offered to women younger than 35 years.

The answer is: C

81. A17-year-old boy receives a pediatric consult for an upper respiratory


infection with jaundice. History reveals he frequently becomes jaundiced when
he has minor infections. The patient’s history contains no evidence of
consanguinity. His father had a few mild cases of hyperbilirubinemia, and his
paternal grandmother had episodic fatigue with mild sclera licterus. Based on
the family history, what is the most plausible inheritance pattern of this
hyperbilirubinemia?

(A) X-linked dominant


(B) X-linked recessive
(C) Autosomal dominant
(D) Autosomal recessive
(E) Mitochondrial

The answer is: C

82. A 50-year-old man undergoes genetic testing for hemochromatosis, an


autosomal recessive disease characterized by abnormally elevated serum iron
levels leading to organ toxicity. He is positive for a genetic mutation and is

399 ‫صفحة‬ Dr. MOHAMED YAHIA 0900987639


QUESTION BANK OF Dr. MOHAMED YAHIA

diagnosed with the disease. However, he never develops signs of elevated serum
iron levels or organ toxicity. Which of the following terms best describes this
patient’s disease?

(A) Low penetrance


(B) Low expressivity
(C) Low mosaicism
(D) Low lyonization
(E) Low mendelian inheritance

The answer is: A

83. A 30-year-old man is a heterozygote for a disease that is prevalent in the


population. If the population is in Hardy-Weinberg equilibrium, then p 2 + 2pq
+ q 2 ¼ 1 and p + q ¼ 1. Which of the following in this equation indicates the
prevalence of heterozygotes?

(A) p 2
(B) q 2
(C) 2pq
(D) p
(E) q

The answer is: C

400 ‫صفحة‬ Dr. MOHAMED YAHIA 0900987639


QUESTION BANK OF Dr. MOHAMED YAHIA

CHAPTER 4

905 Question

401 ‫صفحة‬ Dr. MOHAMED YAHIA 0900987639


QUESTION BANK OF Dr. MOHAMED YAHIA

BRS PATHOLOGY

1. A 16-year-old girl undergoes radiologic imaging of her abdomen and is found


to have only one kidney. She had been entirelyunaware of this problem. Which
of the following terms is most descriptive of this finding?
(A) Agenesis
(B) Atrophy
(C) Hyperplasia
(D) Hypoplasia
(E) Metaplasia
The answer is: A
2. An impending myocardial infarction was successfully averted by thrombolytic
(clot-dissolving) therapy in a 55-year-old man. Which of the following
biochemical events most likely occurred during the period of hypoxia?
(A) Decreased hydrogen ion concentration
(B) Increase in oxidative phosphorylation
(C) Loss of intracellular Na + and water
(D) Stimulation of ATP synthesis
(E) Stimulation of anaerobic glycolysis and glycogenolysis
The answer is: E
3. A 45-year-old man with a long history of alcoholism presents with severe
epigastric pain, nausea, vomiting, fever, and an increase in serum amylase.
During a previous hospitalization for a similar episode, computed tomography
scanning demonstrated calcifications in the pancreas. A diagnosis of acute
pancreatitis superimposed on chronic pancreatitis was made. In this condition,
which of the following types of necrosis is most characteristic?
(A) Caseous
(B) Coagulative
(C) Enzymatic
(D) Fibrinoid
(E) Liquefactive
The answer is: C
4. A 29-year-old man hospitalized for acquired immunodeficiency syndrome
(AIDS) is found to have pulmonary tuberculosis. Which type of necrosis is found
in the granulomatous lesions (clusters of modified macrophages) characteristic of
this increasingly frequent complication of AIDS?
(A) Caseous
(B) Coagulative
(C) Enzymatic

402 ‫صفحة‬ Dr. MOHAMED YAHIA 0900987639


QUESTION BANK OF Dr. MOHAMED YAHIA

(D) Fibrinoid
(E) Liquefactive
The answer is: A
5. A 45-year-old woman is investigated for hypertension and is found to have
enlargement of the left kidney. The right kidney is smaller than normal.
Contrast studies reveal stenosis of the right renal artery. The size change in the
right kidney is an example of which of the following adaptive changes?
(A) Aplasia
(B) Atrophy
(C) Hyperplasia
(D) Hypertrophy
(E) Metaplasia
The answer is: B
6. A 56-year-old man recovered from a myocardial infarction after his
myocardium was entirely “saved” by immediate thrombolytic therapy. If it had
been possible to examine microscopic sections of his heart during his ischemic
episode, which of the following would be the most likely cellular change to be
found?
(A) Karyolysis
(B) Karyorrhexis
(C) Pyknosis
(D) Swelling of the endoplasmic reticulum
The answer is: D
7. A 64-year-old woman presents with fever, chills, headache, neck stiffness,
vomiting, and confusion. The Kernig sign (passive knee extension eliciting neck
pain) and Brudzinski sign (passive neck flexion eliciting bilateral hip flexion) are
both positive. Examination of the cerebrospinal fluid reveals changes consistent
with bacterial meningitis, and brain imaging demonstrates a localized abscess.
Which of the following types of necrosis is most characteristic of abscess
formation?
(A) Caseous
(B) Coagulative
(C) Enzymatic
(D) Fibrinoid
(E) Liquefactive
The answer is: E
8. A 20-year-old man presents with yellowing of the sclerae, skin, and oral
mucosa. Which of the following accumulations underlies these findings?
(A) Bilirubin
(B) Hemosiderin
(C) Lead

403 ‫صفحة‬ Dr. MOHAMED YAHIA 0900987639


QUESTION BANK OF Dr. MOHAMED YAHIA

(D) Melanin
(E) Silver
The answer is: A
9. A 45-year-old man is referred because of a recent diagnosis of hereditary
hemochromatosis. Which of the following is a correct statement about this
disorder?
(A) Damage to organs results from abnormal deposition of lead
(B) It can progress to liver cirrhosis, diabetes mellitus, and skin pigmentation
(C) Most cases are due to spontaneous mutations
(D) Skin hyperpigmentation is due to bilirubin accumulation
(E) The TIBC is characteristically increased
The answer is: B
10. A 60-year-old woman with breast cancer and widespread bony metastases is
found to have calcification of multiple organs. The calcifications are best
described as
(A) dystrophic with decreased serum calcium.
(B) dystrophic with increased serum calcium.
(C) metastatic with decreased serum calcium.
(D) metastatic with increased serum calcium.
The answer is: D
11. A 56-year-old man dies 24 hours after the onset of substernal chest pain
radiating down his left arm to the ulnar aspect of his fingertips. Which of the
following morphologic myocardial findings is an indicator of irreversible injury?
(A) Cell blebs
(B) Depletion of glycogen
(C) Mitochondrial swelling
(D) Myelin figures
(E) Pyknotic nuclei
The answer is: E
12. A 72-year-old man presents with a 3-day history of progressively worsening
productive cough, fever, chills, and signs of toxicity. Prominent physical findings
include signs of consolidation and rales over the right lung base. Sputum culture
is positive for Streptococcus pneumoniae. An intra-alveolar exudate filling the
alveoli of the involved portion of the lung is present. Which of the following types
of inflammatory cells is most likely a prominent feature of this exudate?
(A) Basophils
(B) Eosinophils
(C) Lymphocytes
(D) Monocytes–macrophages
(E) Neutrophils
The answer is: E

404 ‫صفحة‬ Dr. MOHAMED YAHIA 0900987639


QUESTION BANK OF Dr. MOHAMED YAHIA

13. A routine complete blood count per- formed on a 22-year-old medical


student reveals an abnormality in the differential leukocyte count. She has been
complaining of frequent sneezing and “watery” eyes during the past several
weeks and reports that she frequently had such episodes in the spring and
summer. Which of the following cell types is most likely to be increased?
(A) Basophils
(B) Eosinophils
(A) Lymphocytes
(B) Monocytes
(C) Neutrophils
The answer is: B
14. A 16-year-old boy presents with a 24-hour history of severe abdominal pain,
nausea, vomiting, and low-grade fever. The pain is initially periumbilical in
location but has migrated to the right lower quadrant of the abdomen, with
maximal tenderness elicited at a site one-third of the way between the crest of the
ileum and the umbilicus (McBurney point). The leukocyte count is 14,000/mm 3 ,
with 74% segmented neutrophils and 12% bands. Surgery is performed. Which
of the following describes the expected findings at the affected site?
(A) Fistula (abnormal duct or passage) connecting to the abdominal wall
(B) Granulation tissue (new vessels and young fibroblasts) with a prominent
infiltrate of eosinophils
(A) Granulomatous inflammation with prominent aggregates of epithelioid cells
and multinucleated giant cells
(B) Massive infiltration of lymphocytes and plasma cells
(C) Prominent areas of edema, congestion, and a purulent reaction with localized
areas of abscess formation
The answer is: C
15. A 2-year-old boy presents with recurrent infections involving multiple organ
systems. Extensive investigation results in a diagnosis of chronic granulomatous
disease of childhood. Which of the following most closely characterizes the
abnormality in this patient’s phagocytic cells?
(A) Decreased killing of microorganisms because of enhanced production of
hydrogen peroxide
(B) Deficiency of NADPH oxidase activity
(C) Impaired chemotaxis and migration caused by abnormal microtubule
formation
(D) Inability to kill streptococci
(E) Increased myeloperoxidase–halide- mediated killing of catalase-positive
organisms when compared with catalase-negative organisms
The answer is: B
16. A laboratory experiment is performed to evaluate the chemotactic potential
of a group of potential mediators. Which of the following substances most likely
has the greatest affinity for neutrophils?

405 ‫صفحة‬ Dr. MOHAMED YAHIA 0900987639


QUESTION BANK OF Dr. MOHAMED YAHIA

(A) C5a
(B) Fucosyl transferase
(C) β 2 -Integrin
(D) P-selectin
(E) TNF-α
The answer is: A
17. A 26-year-old African-American woman has bilateral hilar adenopathy, and
radiography reveals multiple reticular densities in both lung fields. A
bronchoscopic biopsy reveals granulomatous inflammation with multiple giant
cells of the Langhans type and no evidence of caseous necrosis. Which of the
following is the most likely diagnosis?
(A) Aspergillosis
(B) Coccidioidomycosis
(C) Histoplasmosis
(D) Sarcoidosis
(E) Tuberculosis
The answer is: D
18. A 70-year-old man presents with the sudden onset of left-sided weakness,
spasticity, and hyperactive and pathologic reflexes. The most serious
consequences of this disorder are the result of damage to which of the following
cell types?
(A) Labile cells
(B) Multipotent adult progenitor cells
(C) Permanent cells
(D) Stable cells
The answer is: C
19. Within minutes of a bee sting, a 23-year-old woman develops generalized
pruritus and hyperemia of the skin, followed shortly by swelling of the face and
eyelids, dyspnea, and laryngeal edema. This reaction is mediated by
(A) antigen-antibody complexes.
(B) cytotoxic T cells.
(C) IgA antibodies.
(D) IgE antibodies.
(E) IgG antibodies.
The answer is: D
20. A 25-year-old woman with membranous glomerulonephritis receives a
kidney transplant. The donor is her HLA-matched sister. She does well initially,
but after several weeks, there is a progressive increase in serum creatinine.
Assuming that this represents acute cellular rejection, an infiltrate with which of
the following types of inflammatory cells is most likely to be a prominent finding
on renal biopsy?

406 ‫صفحة‬ Dr. MOHAMED YAHIA 0900987639


QUESTION BANK OF Dr. MOHAMED YAHIA

(A) Eosinophils
(B) Lymphocytes
(C) Mast cells
(D) Monocytes-macrophages
(E) Neutrophils
The answer is: B
21. A 20-year-old woman presents with malar rash, arthralgias, low-grade fever,
and high titer antibodies to double-stranded DNA and to the Sm (Smith) antigen.
Which of the following forms of hypersensitivity is the primary mechanism of
the abnormalities found in this disorder?
(A) Type I (immediate or anaphylactic) hypersensitivity
(B) Type II (antibody-mediated or cytotoxic) hypersensitivity
(C) Type III (immune complex-mediated disorders) hypersensitivity
(D) Type IV (cell-mediated) hypersensitivity
The answer is: C
22. An HIV-positive intravenous drug user is suspected of having active
tuberculosis, and a tuberculin (Mantoux) intradermal skin test is performed.
After 48 hours, 10 cm of induration is observed. Which of the following are
involved in this form of hypersensitivity reaction?
(A) B cells and antibodies
(B) Basophils and IgE
(C) Immune complexes and complement
(D) Plasma cells and IgM
(E) T cells and macrophages
The answer is: E
23. A 24-year-old woman who had previously been uneventfully transfused
receives a blood transfusion during surgery and shortly thereafter develops
itching, generalized urticaria, laryngeal edema, and dyspnea with wheezing
respiration. She has a past history of recurrent upper respiratory tract infections
and frequent episodes of diarrhea. Laboratory studies are most likely to reveal
decreased concentrations of which of the following immunoglobulins?
(A) IgA
(B) IgD
(C) IgE
(D) IgG
(E) IgM
The answer is: A
24. A 54-year-old woman who has been diagnosed with early-stage breast cancer
undergoes surgery for a lumpectomy to remove a small tumor detected by
mammography. The pathology report confirms the early stage of the cancer and
further comments on the fact that there is significant desmoplasia in the
surrounding tissue. The term desmoplasia refers to

407 ‫صفحة‬ Dr. MOHAMED YAHIA 0900987639


QUESTION BANK OF Dr. MOHAMED YAHIA

(A) an irregular accumulation of blood vessels.


(B) maturation and spatial arrangement of cells.
(C) metastatic involvement of surrounding tissue.
(D) normal tissue misplaced within another organ.
(E) proliferation of non-neoplastic fibrous connective tissue.
The answer is: E
25. A 24-year-old woman with a history of heavy and painful menstrual periods
has been having difficulty conceiving despite months of trying to become
pregnant. Further workup includes a bimanual pelvic examination and an
ultrasound, which demonstrates a mass in the uterus that is presumed to be a
leiomyoma. This mass is a
(A) benign tumor of mesenchymal tissue.
(B) benign tumor of surface epithelium.
(C) malignant tumor of epithelial tissue.
(D) malignant tumor of glandular epithelium.
(E) malignant tumor of mesenchymal tissue.
The answer is: A
26. A 58-year-old man with a 700-pack-per- year smoking history presents to
the emergency department with shortness of breath and hemoptysis. Portable
chest radiography demonstrates a large mass centrally located within the left
lung field. The serum calcium is 13.0 mg/dL (normal 8.5 to 10.2). The metabolic
abnormality described here is likely due to elaboration of which substance?
(A) Adrenocorticotropic hormone–like substance
(B) Antidiuretic hormone
(C) Carcinoembryonic antigen
(D) Erythropoietin
(E) Parathyroid-related hormone
The answer is: E

408 ‫صفحة‬ Dr. MOHAMED YAHIA 0900987639


QUESTION BANK OF Dr. MOHAMED YAHIA

PRETEST PATHOLOGY

1. A 59-year-old man is found to have a 3.5-cm mass in the right upper lobe of
his lung. A biopsy of this mass is diagnosed as a moderately differentiated
squamous cell carcinoma. Workup reveals that no bone metastases are present,
but laboratory examination reveals that the man’s serum calcium levels are 11.5
mg/dL. This patient’s paraneoplastic syndrome is most likely the result of the
ectopic production of which of the following substances?

a. Parathyroid hormone
b. Parathyroid hormone-related peptide
c. Calcitonin
d. Calcitonin-related peptide
e. Erythropoietin

The answer is: B

2. Pyogenic bacterial infections and ischemic brain infarcts most


characteristically produce which one of the listed types of necrosis?

a. Caseous necrosis
b. Coagulative necrosis
c. Fat necrosis
d. Fibrinoid necrosis
e. Liquefactive necrosis

The answer is: E

3. During acute inflammation, histamine-induced increased vascular


permeability causes the formation of exudates (inflammatory edema). Which of
the following cell types is most likely to secrete histamine and cause this
increased vascular permeability?

a. Endothelial cells
b. Fibroblasts
c. Lymphocytes
d. Mast cells
e. Neutrophils

The answer is: D

4. Histologic sections of lung tissue from a 68-year-old woman with congestive


heart failure and progressive breathing problems reveal numerous hemosiderin-

409 ‫صفحة‬ Dr. MOHAMED YAHIA 0900987639


QUESTION BANK OF Dr. MOHAMED YAHIA

laden cells within the alveoli. Which of the following is the cell of origin of these
“heart failure cells”?

a. Endothelial cells
b. Eosinophils
c. Lymphocytes
d. Macrophages
e. Pneumocytes

The answer is: D

5. What type of leukocyte actively participates in acute inflammatory processes


and contains myeloperoxidase within its primary (azurophilic) granules and
alkaline phosphatase in its secondary (specific) granules?

a. Neutrophils
b. Eosinophils
c. Monocytes
d. Lymphocytes
e. Plasma cells

The answer is: A

6. A 37-year-old man presents with a cough, fever, night sweats, and weight loss.
A chest x-ray reveals irregular densities in the upper lobe of his right lung.
Histologic sections from this area reveal groups of epithelioid cells with rare
acid-fast bacilli and a few scattered giant cells. At the center of these groups of
epithelioid cells are granular areas of necrosis. What is the source of these
epithelioid cells?

a. Bronchial cells
b. Fibroblasts
c. Lymphocytes
d. Monocytes
e. Pneumocytes

The answer is: D

7. A 47-year-old man presents with pain in the midportion of his chest. The pain
is associated with eating and swallowing food. Endoscopic examination reveals
an ulcerated area in the lower portion of his esophagus. Histologic sections of
tissue taken from this area reveal an ulceration of the esophageal mucosa that is
filled with blood, fibrin, proliferating blood vessels, and proliferating fibroblasts.
Mitoses are easily found, and most of the cells have prominent nucleoli. Which of
the following statements best describes this ulcerated area?

a. Caseating granulomatous inflammation

410 ‫صفحة‬ Dr. MOHAMED YAHIA 0900987639


QUESTION BANK OF Dr. MOHAMED YAHIA

b. Dysplastic epithelium
c. Granulation tissue
d. Squamous cell carcinoma
e. Noncaseating granulomatous inflammation

The answer is: C

8. Which of the following changes best describes the pathophysiology involved in


the production of pulmonary edema in patients with congestive heart failure?

a. Decreased plasma oncotic pressure


b. Widespread endothelial damage
c. Increased hydrostatic pressure
d. Increased vascular permeability
e. Acute lymphatic obstruction

The answer is: C

9. A 22-year-old second-year medical student develops a “red” face after being


asked a question during lecture. Which of the following statements best describes
this vascular reaction?

a. Active hyperemia
b. Acute congestion
c. Nonpalpable purpura
d. Passive hyperemia
e. Petechial hemorrhage

The answer is: A

10. A 53-year-old man after recovering from a myocardial infarction is advised


by his physician to take one “baby” aspirin every day to reduce the chance of his
developing a second myocardial infarction. The theory behind this advice is that
aspirin decreases the formation of thrombi within the coronary arteries by
inhibiting the platelet formation of which substance?

a. Fibrinogen
b. Prostacyclin
c. Thrombomodulin
d. Thromboplastin
e. Thromboxane

The answer is: E

11. Assuming that the levels of all of the other coagulation factors are within
normal limits, which of the following laboratory findings is most consistent with
an individual who is not taking any medication but has a familial deficiency of
coagulation factor VII?

411 ‫صفحة‬ Dr. MOHAMED YAHIA 0900987639


QUESTION BANK OF Dr. MOHAMED YAHIA

Prothrombin Time (PT Partial Thromboplastin Time (PTT)

a. Prolonged Normal
b. Normal Prolonged
c. Shortened Normal
d. Normal Shortened
e. Shortened Prolonged

The answer is: A

12. Ten minutes after being stung by a wasp, a 30-year-old man develops
multiple patches of red, irregular skin lesions over his entire body. These lesions
(urticaria) are pruritic, and a new crop of lesions develops the next day. Which
one of the following statements correctly describes an important component of
the pathomechanism of this immune response?

a. Activated T lymphocytes stimulate smooth-muscle cells


b. IgA is attached to basophils and mast cells
c. IgA is attached to lymphocytes and eosinophils
d. IgE is attached to basophils and mast cells
e. IgE is attached to lymphocytes and eosinophils

The answer is: D

13. After receiving incompatible blood, a patient develops a transfusion reaction


in the form of back pain, fever, shortness of breath, and hematuria. Which one
of the following statements best classifies this type of immunologic reaction?

a. Systemic anaphylactic reaction


b. Systemic immune complex reaction
c. Delayed type hypersensitivity reaction
d. Complement-mediated cytotoxicity reaction
e. T cell-mediated cytotoxicity reaction

The answer is: D

14. Which of the following is the definition of an allograft?

a. A graft between a human and an animal


b. A graft between two individuals of different species
c. A graft between two individuals of the same species
d. A graft between two individuals of the same inbred strain
e. A graft between identical twins

The answer is: C

412 ‫صفحة‬ Dr. MOHAMED YAHIA 0900987639


QUESTION BANK OF Dr. MOHAMED YAHIA

‫شيت الجامعة‬
1. An example of a tissue or organ composed of Permenant Parenchymal cells is:
a) Liver
b) Bone Marrow
c) Small Intestinal Mucosa
d) Heart
e) Renal Tubules
. The type of necrosis that occurs in Peripancreatic tissue in Acute Pancreatitis
is:
a) Liquefaction
b) Fat
c) Coagulation
d) Gummatous
e) Fibrinoid
3. Change of Columnar Epithelium of the Bronchi into Mature Squamous
Epithelium is called:
a) Metaplasia
b) Dysplasia
c) Hyperplasia
d) Neoplasia
e) Hypertrophy
4. Metastatic Calcification:
a) Is due to hypercalcemia
b) Is due to hypocalcemia
c) Occurs in Necrotic Tissue
d) Occurs at sites of Chronic Inflammation
e) Is due to malignancy
5. Which of the following is a feature of Irreversible Cell Injury:
a) Glycogen stores are depleted

413 ‫صفحة‬ Dr. MOHAMED YAHIA 0900987639


QUESTION BANK OF Dr. MOHAMED YAHIA

b) Cytoplasmic sodium increases


c) Nuclei undergo karyorrhexis
d) Intracellular pH diminishes
e) Blebs form on cell membranes
6. Which of the following processes that occur in the breast allows a mother to
feed the infant:
a) Stromal hypertrophy
b) Epithelial dysplasia
c) Steatocyte atrophy
d) Ductul epithelial metaplasia
e) Lobular hyperplasia
7. An Amputated lower limb from a diabetic patient showing black discoloration
of the skin and soft tissues with areas of yellowish exudates is characterized as:
a) Neoplasia
b) Gangrenous Necrosis
c) Coagulopathy
d) Hemosiderosis
e) Gas gangrene
8. Focal fat necrosis, with flecks of chalky tan-white material seen in the
omentum is most often associated with the following:
a) Gangrenous appendicitis
b) Chronic salpingitis
c) Acute pancreatitis
d) Hepatitis
e) Acute gastritis
9. In a 60 year old male, Gangrene of toes is most likely associated with:
a) Diabetes Mellitus
b) Heart Failure
c) Blunt force trauma
d) AIDS

414 ‫صفحة‬ Dr. MOHAMED YAHIA 0900987639


QUESTION BANK OF Dr. MOHAMED YAHIA

e) Type III hypersensitivity reaction

10. The presence of Columnar Epithelium with Goblet cells in the Lower
Oesophagus is most consistent with:
a) Dysplasia
b) Hyperplasia
c) Carcinoma
d) Ischaemia
e) Metaplasia

11. Which of the following processes explain the appearance of Calcium


deposition in Tuberculous Lymph Nodes:
a) Dystrophic calcification
b) Apoptosis
c) Hypercalcaemia
d) Metastatic calcification
e) Neoplastic change

12. Lipofuscin deposition is most likely to result from:


a) Nuclear pyknosis
b) Myocardial fiber hypertrophy
c) Coagulative necrosis
d) Autophagocytosis
e) Anaerobic glycolysis

13. Which of the following cellular changes is most likely to present irreversible
cellular injury:
a) Epithelial dysplasia
b) Cytoplasmic fatty metamorphosis
c) Nuclear pyknosis

415 ‫صفحة‬ Dr. MOHAMED YAHIA 0900987639


QUESTION BANK OF Dr. MOHAMED YAHIA

d) Atrophy
e) Anaerobic Glycolysis

14. Which of the following is an Anti-Oxidant:


a) Glutathione Peroxidase
b) Catalase
c) Hydrogen peroxide
d) NADPH oxidase
e) Myeloperoxidase

15. Which of the following is the most likely pathologic alteration following
occlusion of the left middle cerebral artery by a sterile thrombus:
a) Cerebral softening from liquefactive necrosis
b) Pale infarction with coagulative necrosis
c) Predominantly the loss of glial cells
d) Recovery of damaged neurons if the vascular supply is re-established
e) Wet gangrene with secondary bacterial infection

16. The action of putrefactive bacteria on necrotic tissue results in:


a) Coagulation
b) Infarction
c) Gangrene
d) Embolism
e) Caseation

17. Which of the following type of necrosis is most commonly associated with
ischaemic injury:
a) Coagulation Necrosis
b) Liquefaction Necrosis
c) Caseous Necrosis

416 ‫صفحة‬ Dr. MOHAMED YAHIA 0900987639


QUESTION BANK OF Dr. MOHAMED YAHIA

d) Fat Necrosis
e) Gangrenous Necrosis

18. Enzymes involved in the inactivation of free radicals include:


a) Superoxide dismutase
b) Vitamin E
c) Both A and B
d) Neither A nor B

e) -antitrypsin

19. Hepatocytes are an example of:


a) Permenant cells
b) Stable cells
c) Labile cells
d) Metaplasia
e) Mesenchymal cells

20. Calcium may play a role in cell injury by:


a) Causing ATP depletion
b) Activating phospholipases
c) Inducing autophagocytosis
d) Regulating pyknosis
e) Reducing intracellular pH
21. A common manifestation of sublethal cell injury (reversible injury) in organs
such as the heart and liver is:
a) Glycogen accumulation
b) Fatty change
c) Calcium deposition
d) Apoptosis
e) Deposition of melanin

417 ‫صفحة‬ Dr. MOHAMED YAHIA 0900987639


QUESTION BANK OF Dr. MOHAMED YAHIA

22. An area of keratinizing squamous epithelium lining a major bronchus is an


example of:
a) Heterotopia
b) Metaplasia
c) Dysplasia
d) Atrophy
e) Neoplasia

23. The enzymes responsible for liquefaction in an abscess are derived mainly
from:
a) Tissue
b) Serum
c) Lymph
d) Neutrophils
e) Lymphocytes
24. Each of the following is an example of hyperplasia except:
a) Enlargement of one kidney after surgical removal of the other kidney
b) Changes in the thyroid gland in response to increased demand for thyroid
hormones
c) Changes in breast tissue during pregnancy
d) Enlargement of lymph nodes during a viral infection
e) Enlargement of skeletal muscles after weight training

25. Select the wrong statement. Apoptosis:


a) Occurs in singles cells or in small cluster of cells
b) Is programmed cell death
c) Is seen in toxic or viral hepatitis
d) Appears cheesy in appearance
e) Does not elicit an inflammatory reaction

418 ‫صفحة‬ Dr. MOHAMED YAHIA 0900987639


QUESTION BANK OF Dr. MOHAMED YAHIA

26. Intracellular system(s) sensitive to cell injury is/are:


a) Aerobic respiration
b) Maintenance of the integrity of cell membrane
c) Synthesis of protein
d) Integrity of genetic apparatus
e) All of the above

27. Which of the following can undergo apoptosis:


a) Cells infected with virus
b) Cells with DNA damage
c) Increased oxidant within the cell
d) None of the above
e) All of the above
28. Apoptosis:
a) Massive necrosis
b) Foreign body phagocytosis
c) Lipoprotein synthesis
d) Programmed cell death
e) Wet gangrene

29. Enzymes involved in the inactivation of free radicals include:


a) Superoxide dismutase
b) Vitamin E
c) Both A an B
d) Neither A nor B

e) -antitrypsin

30. The following is true about hyperplasia except:

419 ‫صفحة‬ Dr. MOHAMED YAHIA 0900987639


QUESTION BANK OF Dr. MOHAMED YAHIA

a) Can be physiological
b) Is a precancerous condition
c) Is reversible
d) Is due to excess hormone stimulation
e) Can be associated with hypertrophy

31. Early clumping of nuclear chromatin is most closely associated with:


a) Reduced intracellular pH
b) Increased intracellular pH
c) Denatured proteins and RNA loss
d) Decreased DNA synthesis
e) Release of lysosomal enzymes

40. Digestion of tissue with soap formation and calcification is characteristic of:
a) Coagulation Necrosis
b) Caseous Necrosis
c) Fibrinoid Necrosis
d) Liquefaction Necrosis
e) Enzymatic Fat Necrosis

41. All of the following findings represent changes seen within cells that
represent reversible cellular injury, except:
a) Fatty change in hepatocytes
b) Neuronal cell swelling
c) Skeletal muscle fiber anaerobic glycolysis
d) Renal tubular cell nuclear pyknosis
e) Kupffer cell iron deposition

42. Which is most likely to happen following a stroke with loss of blood supply to
a lobe of the brain:

420 ‫صفحة‬ Dr. MOHAMED YAHIA 0900987639


QUESTION BANK OF Dr. MOHAMED YAHIA

a) Cerebral softening from liquefactive necrosis


b) Pale infarction with coagulative necrosis
c) Predominantly the loss of glial cells
d) Recover of damaged neurons if the vascular supply is restablished
e) Wet gangrene with secondary bacterial infection
43. The best example of Dystrophic Calcification is seen in a:
a) 55 year old woman with metastases from breast carcinoma &
hypercalcemia
b) Healing granuloma in a 41 year old man with pulmonary tuberculosis
c) Gangrenous lower extremity in a 50 year old woman with diabetes mellitus
d) 62 year old woman with a recent cerebral infarction
e) Abscess of the left fallopian tube in a 19 year old woman with Neisseria
Gonnorrheae Infection

44. Which of the following cellular changes represents the best evidence for
irreversible cellular injury:
a) Epithelial dysplasia
b) Cytoplasmic fatty metamorphosis
c) Nuclear pyknosis
d) Atrophy
e) Anaerobic glycolysis

45. The presence of differentiated columnar epithelium with goblet cells in lower
esophagus is consistent with:
a) Dysplasia
b) Hyperplasia
c) Carcinoma
d) Ischemia
e) Metaplasia

421 ‫صفحة‬ Dr. MOHAMED YAHIA 0900987639


QUESTION BANK OF Dr. MOHAMED YAHIA

46. Which of the following will show hypertrophy:


a) The uterine myometrium in pregnancy
b) The female breast at puberty
c) The liver following partial resection
d) The ovary following menopause
e) The cervix with chronic inflammation

47. Which of the following is most likely to give rise to Metaplasia:


a) Tanning of the skin following sunlight exposure
b) Lactation following pregnancy
c) Vitamin A deficiency
d) Acute Myocardial Infarction
e) Acute Tubular Necrosis

48. Which of the following is deposited in myocardium in advanced age:


a) Hemosidrin
b) Lipochrome
c) Glycogen
d) Cholesterol
e) Calcium Salts

49. Which of the following tissues is most likely to be least affected by Ischemia:
a) Skeletal muscle
b) Small intestinal epithelium
c) Retina
d) Myocardium
e) Hippocampus
50. An amputated foot of a diabetic will most likely show:
a) Neoplasia

422 ‫صفحة‬ Dr. MOHAMED YAHIA 0900987639


QUESTION BANK OF Dr. MOHAMED YAHIA

b) Gangrenous Necrosis
c) Coagulopathy
d) Hemosiderosis
e) Caseation

51. Which of the following processes explains the appearance of calcium


deposition in a tuberculous lymph node:
a) Dystrophic calcification
b) Apoptosis
c) Hypercalcemia
d) Metastatic calcification
e) Neoplastic change

52. Which of the following types of necrosis is grossly opaque & "chalk" white:
a) Coagulation necrosis
b) Liquefaction
c) Caseous necrosis
d) Fat necrosis
e) Gangrenous necrosis

53. Dystrophic Calcification is most closely associated with:


a) Hypercalcemia
b) Necrosis
c) Chronic irritation
d) Diminished blood supply
e) Increased workload

54. Deposition of Calcium in dead or dying tissue is:


a) Dystrophic calcification
b) Metastatic calcification

423 ‫صفحة‬ Dr. MOHAMED YAHIA 0900987639


QUESTION BANK OF Dr. MOHAMED YAHIA

c) Both A and B
d) Neither A nor B

55. Which of the following types of necrosis is characterized by amorphous


granular debris:
a) Coagulation Necrosis
b) Liquifaction Necrosis
c) Caseous Necrosis
d) Fat Necrosis
e) Gangrenous Necrosis
56. Hypertrophy is most closely associated with:
a) Hypercalcemia
b) Necrosis
c) Chronic Irritation
d) Diminished Blood Supply
e) Increased Work Load

57. Which of the following is most susceptible to liquefaction necrosis following


ischemic injury:
a) Pancreas
b) Liver
c) Spleen
d) Brain
e) Intestine

58. Hypoplasia is due to:


a) Cell Loss
b) Atrophy
c) Inadequate development
d) Disuse

424 ‫صفحة‬ Dr. MOHAMED YAHIA 0900987639


QUESTION BANK OF Dr. MOHAMED YAHIA

e) Ischemia

59. The loss of individual cell through fragmentation of individual cell nucleus is
best described as:
a) Coagulative Necrosis
b) Mitochondrial Poisoning
c) Phagocytosis
d) Apoptosis
e) Liquefaction

60. The light brown perinucleur pigment seen in an old man muscle fiber is:
a) Hemosidrin
b) Lipofuscin
c) Glycogen
d) Melanin
e) Calcium
61. Hypertensive Enlargement of the heart is a form of:
a) Fatty infiltration
b) Hypoplasia
c) Glycogen storage
d) Hypertrophy
e) Hyperplasia

62. Gangrene of the big toe in a 60 year old female is most likely associated with:
a) Diabetes Mellitus
b) Temporal Arteritis
c) AIDS
d) Type III Hypersensitivity
e) Carcinoma of the Bronchus

425 ‫صفحة‬ Dr. MOHAMED YAHIA 0900987639


QUESTION BANK OF Dr. MOHAMED YAHIA

63. Focal Fat Necrosis is associated with:


a) Fibrinous Pericarditis
b) Chronic Appendicitis
c) Acute Pancreatitis
d) Periportal Fibrosis
e) Gas Gangrene

64. Two days after myocardial infarction, histology of the heart will show:
a) Fibroblasts and Collagen
b) Granulation Tissue
c) Necrotic Muscle & Neutrophils
d) Granulamatous Inflammation
e) Aneurysmal Dilation

65. Cigarette Smoking does not cause:


a) Squamous Metaplasia
b) Smooth Muscle Hyperplasia
c) Defective Ciliary Action
d) Damage of Airway
e) Inhibition of Alveolar Leukocytes & Macrophages
66. Which of the following is responsible for gangrene of lower extremeties:
a) Obesity
b) Hyperlipidemia
c) Cigarette Smoking
d) Diabetes Mellitus
e) Hypercalcemia

67. Which of the following is not a feature of reversible cell injury:


a) Swollen cell

426 ‫صفحة‬ Dr. MOHAMED YAHIA 0900987639


QUESTION BANK OF Dr. MOHAMED YAHIA

b) Swollen mitochondria
c) Pyknotic Nucleus
d) Dilated Endoplasmic
e) Cell surface blebs

68. Changes of color in Gangrene is due to:


a) Deposition of amyloid
b) Breakdown of hemoglobin
c) Deposition of melanin
d) Deposition of lipofusin
e) Deposition of calcium salts

69. Which of the following is not a feature of reversible cell injury:


a) Increased DNA synthesis
b) Reduction of ATP
c) Failure of Sodium Pump
d) Influx of Calcium
e) Accumulation of Metabolites

70. Which of the following is a feature of Colliquative Necrosis:


a) Complete loss of architecture
b) Cyst formation
c) Occurs in tissues with high water content
d) A and C
e) All of the above
71. Which of the following is (are) true about apoptosis:
a) It is programmed cell death
b) It requires cell energy
c) Induces a severe inflammatory reaction

427 ‫صفحة‬ Dr. MOHAMED YAHIA 0900987639


QUESTION BANK OF Dr. MOHAMED YAHIA

d) Both A and B
e) Both B and C

72. Calcium may play a role in cell injury by:


a) Causing ATP depletion
b) Activating Phospholipases
c) Inducing Autophagocytosis
d) Regulating Pyknosis

73. Irreversible cellular changes include all of the following except:


a) Hydropic Change
b) Pyknosis
c) Karyorrhexis
d) Karyolysis

74. Abnormal pigmentation is seen in the following except:


a) Conn's Syndrome
b) Pernicious Anemia
c) Tattooing
d) Addison's Disease
e) Pregnancy

75. The following are true about Apoptosis except:


a) It is programmed cell death
b) Affects a group of cells
c) Is energy dependent
d) There is chromatin condensation
e) Does not elicit an inflammatory reaction
76. Which of the following is not a feature of Necrosis:

428 ‫صفحة‬ Dr. MOHAMED YAHIA 0900987639


QUESTION BANK OF Dr. MOHAMED YAHIA

a) Karyolysis
b) Karyorrhecsis
c) Cytoplasmic Basophilia
d) Pyknosis
e) Loss of RNA

77. Which of the following is not a feature of Coagulative Necrosis:


a) Deposition of Fibrinolysis
b) Tissues are firm and swollen
c) Ghost outline of cell are seen
d) Architecture is maintained
e) The tissue is deeply eosinophilic

78. Which of the following is most likely associated with Caseation Necrosis:
a) Diabetic Gangrene
b) Gas Gangrene
c) Myocardial Infarction
d) Inpissated Pus
e) Splenic Infarction

79. The following are known causes of Atrophy except:


a) Aging
b) Starvation
c) Increased trophic hormone secretion
d) Immobilization
e) Denervation

80. Dystrophic Calcification is:


a) Reversible

429 ‫صفحة‬ Dr. MOHAMED YAHIA 0900987639


QUESTION BANK OF Dr. MOHAMED YAHIA

b) Due to reduction of pH in tissues


c) Causes renal failure
d) Associated with primary hyperparathyroidism
e) Occurs in dead parasites in the body
81. Which of the following pathologic changes would be considered irreversible?
a) Fatty change in liver cells
b) Hydrophic vacuolization or renal tubular epithelial cells
c) Karyoloysis in myocardial cells
d) Glycogen deposition in hepatocyte nuclei
e) Hyperplasia of the breast during pregnancy

82. Squamous Metaplasia can occur in all of the following sites except:
a) Nose
b) Salivary Gland Ducts
c) Jejunum
d) Renal Pelvis
e) Gall Bladder

83. Colliquative Necrosis occurs in:


a) Spleen
b) Liver
c) Heart
d) Brain
e) Kidneys

84. The following are features of Colliquative Necrosis:


a) Complete loss of architecture
b) Cyst formation
c) Occurs in tissues with high water content

430 ‫صفحة‬ Dr. MOHAMED YAHIA 0900987639


QUESTION BANK OF Dr. MOHAMED YAHIA

d) A & C
e) All of the above

85. Which of the following is significant in Reperfusion Injury:


a) Generation of Free Radicals
b) Decreased pH
c) Cytokines released by damaged hypoxic cells
d) Both A & B
e) Both A & C

86. Reperfusion Injury is due to:


a) Generation of Oxygen Free Radicals
b) Additional recruitment of polymorphs by cytokines from hepatic cells
c) Both A & B
d) None of the above

87. Which of the following is a feature of irreversible cell injury:


a) Swelling of Endoplasmic Reticulum
b) Formation of Blebs
c) Karyolysis of Nuclei
d) Fatty Change
e) Mitochondrial Swelling

88. A common manifestation of sublethal cell injury seen in organs such as the
heart and liver is:
a) Glycogen accumulation
b) Fatty change
c) Calcium deposition
d) Apoptosis

431 ‫صفحة‬ Dr. MOHAMED YAHIA 0900987639


QUESTION BANK OF Dr. MOHAMED YAHIA

e) Deposition of Amyloid

89. In the process of necrosis, a reduction in the size of the nucleus and a
condensation of nuclear material is known as:
a) Karyolysis
b) Pyknophrasia
c) Karyorrhexis
d) Pyknosis
e) Metachromasia

90. A cardinal sign of Inflammation is:


a) Vasoconstriction
b) Redness
c) Leukocyte Margiation
d) Vasodilation
e) Slowing of the Circulation
91. After initiating an acute inflammatory event, the third in a sequence of
changes in vascular flow is:
a) Vasoconstriction
b) Redness
c) Leukocyte Margiation
d) Vasodilation
e) Slowing of the Circulation

92. Leukocytes capable of significant Phagocytosis include:


a) Lymphocyte
b) Plasma Cell
c) Both A and B
d) Neither A nor B

432 ‫صفحة‬ Dr. MOHAMED YAHIA 0900987639


QUESTION BANK OF Dr. MOHAMED YAHIA

93. Which of the following is most associated with Acute Inflammation:


a) Neutrophils
b) Connective Tissue
c) Macrophages
d) Granulation Tissue
e) Granuloma

94. Inflammatory conditions which have a large number of eosinophils in the


exudates include:
a) Asthma
b) Parasitic disease
c) Both A and B
d) Neither A nor B

95. By definition, which cells are involved in organization:


a) Kidney tubular and Liver Parenchymal cells
b) Macrophages and Endothelial Cells
c) Endothelial Cells and Fibroblasts
d) Fibroblasts and Fat Cells
96. Phagocytosis is enhanced by:
a) C5a
b) Bradykinin
c) Lysozyme
d) C3b
e) Serotonin

97. The tumor of inflammation is due primarily to:


a) Arteriolar Dilation
b) Venous Dilation

433 ‫صفحة‬ Dr. MOHAMED YAHIA 0900987639


QUESTION BANK OF Dr. MOHAMED YAHIA

c) Capillary Dilation
d) Increased Intracellular Fluid
e) Increased Extracellular Fluid

98. A mediator of acute inflammation that causes increased vascular


permeability and pain is:
a) Endotoxin
b) Complement
c) Histamine
d) Bradykinin
e) Endogenous Pyrogen

99. Multinucleated Giant Cells of the foreign body type origin from:
a) Nuclear Division of Granulocytes
b) Atypical Regeneration of Epithelium
c) Megakaryocytes
d) Fusion of Macrophages
e) Multiplication of Nuclei Surrounding Fibrocytes

100. Histamine is thought to be the direct cause of:


a) Leukocytosis
b) Emigration
c) Phagocytosis
d) Increased Vascular Permeability
e) All of the Above

101. An Abscess is best defined as:


a) A local defect in the surface of a tissue
b) An abnormal accumulation between two surfaces
c) Any area of tissue necrosis

434 ‫صفحة‬ Dr. MOHAMED YAHIA 0900987639


QUESTION BANK OF Dr. MOHAMED YAHIA

d) A localized collection of pus


e) An epithelium-lined sac filled with viscous fluid

102. Mediators of fever in inflammation include:


a) Macrophage product similar to interleukin I
b) Prostaglandins
c) Both A and B
d) Neither A nor B

103. Phagocytes from an individual deficient in myeloperoxiase (MPO) will most


likely be less effective in:
a) Killing Bacteria
b) Recognizing Bacteria
c) Marginating in vessels
d) Undergoing Chemotaxis
e) Engulfing Bacteria

104. A purulent exudates is generally characterized by the presence of:


a) Mucous
b) Macrophages and connective tissue
c) Neutrophils and necrotic debris
d) Serous Fluid
e) Precipitated Protein

105. Granuloma formation is most frequently associated with:


a) Acute inflammation
b) The Healing Process
c) Wound Contraction
d) Fibroblasts and Neovasularization
e) A persistent irritant

435 ‫صفحة‬ Dr. MOHAMED YAHIA 0900987639


QUESTION BANK OF Dr. MOHAMED YAHIA

106. Which of the following best describes a granuloma:


a) A small nodule of granulation tissue
b) A tumor composed of granulocytes
c) A small hard mass of fibroblasts and collagen
d) Inflammation primarily composed of lymphocytes
e) An aggregate of activated macrophages

107. The most characteristic feature of granulation tissue is the:


a) Resemblance to a Granuloma
b) Growth of Fibroblasts and New Capillaries
c) Character of the Exudate
d) A form of Gangrenous Necrosis
e) Presence of Monocytes and Fibroblasts

108. Which of the following is the most efficient killer of bacteria in neutrophils:
a) Spontaneous dismutation of O2 to H2O
b) Conversion of H2O2 to HOCl by Myeloperoxiase
c) Oxidative metabolism giving rise to OH and O2
d) Hyrolyzing Bacterial Coated with Lysozyme
e) Action of Hydrolytic Enzymes

109. The emigration and accumulation of neutrophils in the acute inflammation


reaction is primarily the result of:
a) Active hyperemia
b) Hydrostatic Pressure
c) Increased Microvascular Permeability
d) Chemotaxis
e) None of the above

436 ‫صفحة‬ Dr. MOHAMED YAHIA 0900987639


QUESTION BANK OF Dr. MOHAMED YAHIA

110. Which of the following is expressed on endothelial cells in acute


inflammation for leukocyte adhesion:
a) Interferon Gamma
b) Hageman Factor
c) Lysozyme
d) E-selectin
e) Prostacyclin

111. Fever and Leukocytes in which most leukocytes are neutrophils typical of:
a) Acute Bacterial Infection
b) Acute Viral Infection
c) Chronic Viral Infection
d) Parasitemia
e) Foreign Body Giant Cell Reaction

112. Epitheloid Cells and Langerhans Giant Cells in granulomas are derived
from:
a) Neutrophils
b) Eosinophils
c) Mast Cells
d) Macrophage
e) Lymphocytes

113. Prostaglandins are formed from Arachidonic Acid through the action of
which enzyme pathway?
a) Cyclooxygenase
b) Lipoxygenase
c) Myeloperoxidase
d) Phospholipase A

437 ‫صفحة‬ Dr. MOHAMED YAHIA 0900987639


QUESTION BANK OF Dr. MOHAMED YAHIA

e) Glutathione Reductase

114. The cavity of an Abscess contains:


a) Caseous Necrosis
b) Hyalin
c) Giant Cells
d) Pus
e) Granulation Tissue

115. Transudate can be found in the following settings except:


a) Congestive Heart Failure
b) Nephrotic Syndrome
c) Superior Venacaval Obstruction
d) Pericarditis
e) None of the Above

116. The diagnostic feature of granuloma is the presence of:


a) Giant cells
b) Epitheloid cells
c) Lymphocytes
d) Fibrosis
e) Caeseation

117. The following have a granuloma except:


a) T.B
b) Syphilis
c) Leprosy
d) Schistosomiasis

438 ‫صفحة‬ Dr. MOHAMED YAHIA 0900987639


QUESTION BANK OF Dr. MOHAMED YAHIA

e) Staphylococcal Infection

118. Formation of the epitheloid cells is mediated by:


a) IL-1
b) IL-2
c) Histamine
d) Gamma Interferon
e) Serotonin

119. The first inflammatory cell to show at the site of acute inflammation is the:
a) Monocyte
b) B-lymphocyte
c) T-lymphocyte
d) Neutrophils
e) Mast cell

120. The following are true about an exudate except:


a) It can be present as edema
b) It has a high protein content

c) It has a specific gravity  1.020


d) It may contain inflammatory cells
e) It can be due to congestive heart failure

121. Which of the following is the most powerful chemotactic agent:


a) Histamine
b) Prostaglandin
c) Hageman Factor

439 ‫صفحة‬ Dr. MOHAMED YAHIA 0900987639


QUESTION BANK OF Dr. MOHAMED YAHIA

d) Bradykinin
e) Complement C5a

122. A biopsy of a region of acute inflammation would most likely reveal all of
the following except:
a) Fibrinous exudate
b) Neutrophils
c) Edema
d) Necrosis
e) Fibrous connective tissue

123. Which of the following chemical mediators is most important in the


development of Granulomatous Inflammation:
a) Interferon gamma
b) Bradykinin
c) Complement C5a
d) Histamine
e) Prostaglandin E2

124. Following an attack of chest infection for three days, a patient becomes very
ill and his chest x-rays shows a 3cm rounded density. He is most likely having:
a) Hypertrophic scar
b) Abscess formation
c) Regeneration
d) Bronchogenic carcinoma
e) Progression to chronic inflammation

440 ‫صفحة‬ Dr. MOHAMED YAHIA 0900987639


QUESTION BANK OF Dr. MOHAMED YAHIA

125. Which of the following cells is the most important in the development of
tuberculous granuloma:
a) Macrophages
b) Fibroblast
c) Neutrophil
d) Mast Cell
e) Platelet

126. Which of the following inflammatory processes would cause interstitial


fibrosis and nodules in the lungs of a person who inhaled silica dust:
a) Neutrophilic infiltration with release of leukotrienes
b) Foreign body giant cell formation
c) Plasma cell production of immunoglobin
d) Histamine release by mast cells
e) Release of growth factors by macrophages

127. Which of the following mediators are predominantly responsible for pain in
acute inflammation:
a) Complement C3b and IgG
b) Interleukin-1 and tumour necrosis factor
c) Histamine and Serotonin
d) Prostaglandin and Bradykinin
e) Leukotriene and E-selectin

128. Which of the following cell types is most likely to be most characteristic of a
foreign body reaction:
a) Mast cell
b) Eosinophil

441 ‫صفحة‬ Dr. MOHAMED YAHIA 0900987639


QUESTION BANK OF Dr. MOHAMED YAHIA

c) Giant cell
d) Neutrophils
e) Plasma cell

129. Which of the following is needed for anti-microbial killing during


Phagocytosis:
a) Glutathione peroxidase
b) C3b
c) Interleukin 1
d) NADPH oxidase
e) Myeloperoxidase

130. Deficiency of which of the following will reduce phagocytosis by


neutrophils:
a) C3a
b) Leukotriene A4
c) C3b
d) Leukotriene B4
e) C5a

131. All of the following are true of granulation tissue except:


a) Contains epitheloid cells
b) Contains myofibroblast
c) Contains proliferating capillaries
d) Important in wound contraction
e) Contains fibronectin

132. Matrix components in early granulation tissue include:


a) Fibronectin
b) Type III collagen

442 ‫صفحة‬ Dr. MOHAMED YAHIA 0900987639


QUESTION BANK OF Dr. MOHAMED YAHIA

c) Both A and B
d) Neither A nor B
e) Granuloma

133. Apoptosis does not induce inflammation because:


a) The injury is too mild to induce inflammation
b) Phagocytic cells secrete cytokines that inhibit inflammation
c) The process occurs only in avascular tissues
d) Apoptosis occurs only in immune deficient tissues
e) The patient is using antibiotics

134. Osponization is the:


a) Formation of free radicals
b) Degradation of bacteria by lysozymes
c) Engulfment of antigen by leukocytes
d) Coating of antigen by antibodies
e) Processing of antigens by antigen presenting cells
135. Which of the following is not a characteristic of a Granuloma:
a) Macrophages
b) Giant Cells
c) Polymorphonuclear Leukocytes
d) Lymphocytes
e) Epitheloid Cells

136. Which of the following has a large number of eosinophils in exudates:


a) Sarcoidosis
b) Bronchiectasis
c) Syphilis
d) Bronchial Asthma

443 ‫صفحة‬ Dr. MOHAMED YAHIA 0900987639


QUESTION BANK OF Dr. MOHAMED YAHIA

e) Chronic Bronchitis

137. Which of the following are involved in organization:


a) Endothelial cells and osteocytes
b) Endothelial cells and chondroblasts
c) Endothelial cells and fibroblasts
d) Fibroblasts and fat cells
e) Fibroblasts and plasma cells

138. The tumour of inflammation is due primarily to:


a) Arteriolar dilation
b) Venous dilation
c) Capillary dilation
d) Increased intracellular fluid
e) Increased extracellular fluid

139. One of the following is a Chemotactic Factor:


a) Histamine
b) Bradykinin
c) Interleukin 1
d) Leukotriene B4
e) Serotonin

140. Phagocytosis is enhanced by:


a) C1
b) Bradykinin
c) Lysozyme
d) Osponin
e) Serotonin

444 ‫صفحة‬ Dr. MOHAMED YAHIA 0900987639


QUESTION BANK OF Dr. MOHAMED YAHIA

141. Which one of the following is a harmful effect of acute inflammation:


a) Dilution of toxins
b) Phagocytosis
c) Formation of fibrin
d) Swelling of tissue
e) Stimulation of immune response

142. The high protein content of the inflammatory exudates is mainly due to:
a) Protein released from dead tissue
b) Production of protein by inflammatory cells
c) Increased capillary permeability
d) Increased capillary hydrostatic pressure
e) Increased blood flow in the site of inflammation

143. Which of the following is expected to be found in sputum of an asthmatic


child:
a) IgE antibodies
b) Activate macrophages
c) Eosinophils
d) Foreign body giant cell
e) Asbestos bodies

144. Phagocytosis is enhanced by:


a) Prostaglandin E2
b) Bradykinin
c) Lysozymes
d) Osponin
e) Serotonin

445 ‫صفحة‬ Dr. MOHAMED YAHIA 0900987639


QUESTION BANK OF Dr. MOHAMED YAHIA

145. Which of the following are involved in organization:


a) Liver parenchymal cells
b) Macrophages and endothelial cells
c) Endothelial cells and fibroblasts
d) Fibroblasts and fat cells
e) Neurons and glial cells

146. Which part of the microcirculation is most consistently involved in the


permeability changes and exudation of acute inflammation:
a) Small arteries
b) Arterioles
c) Capillaries
d) Venules
e) Veins

147. Chronic Inflammation is characterized by the presence of:


a) Polymorphs, monocytes and exudate
b) Plasma cells, lymphocytes, fibroblasts and monocytes
c) Plasma cells, polymorphs, lymphocytes and monocytes
d) Plasma cells, basophils, eosinophils, fibroblasts and monocytes

148. The following are true of macrophages except:


a) They are phagocytic cells
b) They produce interleukins
c) Are antigen-presenting cells
d) They are derived from blood monocytes
e) They are the main cells in chronic inflammation

446 ‫صفحة‬ Dr. MOHAMED YAHIA 0900987639


QUESTION BANK OF Dr. MOHAMED YAHIA

149. Which of the following is true about an Exudate:


a) Occurs in heart failure
b) Is due to low plasma oncotic pressure
c) Has a low specific gravity
d) Has a high protein content
e) Is due to lymphatic obstruction

150. The most important factor in the formation of acute inflammatory exudates
is:
a) Increased Hydrostatic Pressure
b) Increased Blood Flow
c) Chemotaxis
d) Increased Vascular Permeability
e) Lymphatic Obstruction

151. Increased Vascular Permeability is due to:


a) Damage to endothelial cells
b) Widening of gaps between endothelial cells
c) Slowing of blood flow
d) Both A & B
e) Both A & C

152. Infertility and short stature in a woman with a 45x karyotype are typical of:
a) Rlinefelter Syndrome
b) Turner Syndrome
c) Down Syndrome
d) Achondroplasia Dwarfism
e) Cystic Fibrosis

447 ‫صفحة‬ Dr. MOHAMED YAHIA 0900987639


QUESTION BANK OF Dr. MOHAMED YAHIA

153. Down Syndrome is:


a) Trisomy 21
b) Trisomy 18
c) Trisomy 13
d) Autosomal Dominant
e) Mothers are less than 30 years old

154. Mutations in mitochondrial genes are:


a) Inherited from the father
b) Common cause of disease
c) Inherited from the mother
d) Autosomal Recessive
e) Affects only males

155. There are no Y-linked disorders because:


a) There are no mutations occurring in the Y chromosome
b) The affected males die before adulthood
c) Affected males are infertile
d) Defects in the Y chromosome are autosomal dominant
e) None of the above
156. X-linked disorders:
a) Males are always carriers
b) Females are always diseased
c) Are inherited from the father
d) Are inherited from the mother
e) Sickle cell anemia is an example

157. Autosomal recessive disorders:


a) Are inherited from the father only

448 ‫صفحة‬ Dr. MOHAMED YAHIA 0900987639


QUESTION BANK OF Dr. MOHAMED YAHIA

b) Are inherited from the mother only


c) Are inherited commonly due to new mutations
d) Are usually more severe than autosomal dominant disorders
e) None of the above

158. All of the following are autosomal recessive except:


a) Cystic Fibrosis
b) Sickle Cell Anemia
c) Thalassemia
d) Hemochormatosis
e) Diabetes Mellitus

159. Mendelian disorders are due to:


a) Chromosomal abnormalities
b) Single gene defect
c) Environmental Factors
d) Mitochondrial gene mutations
e) X-linked mutations

160. Which of the following is inherited as autosomal recessive disorders:


a) Familial polyposis coli
b) Hemopilia A
c) Cystic Fibrosis
d) Neurofibromatosis
e) Rheumatic Heart Disease

161. Which of the following is inherited as autosomal dominant disorders:


a) Familial polyposis coli
b) Hemopilia A

449 ‫صفحة‬ Dr. MOHAMED YAHIA 0900987639


QUESTION BANK OF Dr. MOHAMED YAHIA

c) Cystic Fibrosis
d) Diabetes
e) Rheumatic Heart Disease

162. Which of the following is inherited as an X-linked recessive trait:


a) Familial polyposis coli
b) Hemopilia A
c) Cystic Fibrosis
d) Neurofibromatosis
e) Rheumatic Heart Disease

163. The primary role of which of the following in the closure of wounds healing
by second intention:
a) Neutrophils
b) Connective Tissue
c) Macrophages
d) Granulation Tissue
e) Granuloma
164. The process of regeneration:
a) Does not restore prior function
b) Invariability leads to scar formation
c) Refers to healing by proliferation of stromal elements
d) Occurs in tissues composed of labile and stable cells
e) Is synonymous with repair

165. The following factors delay healing of damaged tissue:


a) Scurvy
b) Ionizing Radiation
c) Immobilization
d) Infections

450 ‫صفحة‬ Dr. MOHAMED YAHIA 0900987639


QUESTION BANK OF Dr. MOHAMED YAHIA

e) Glucocorticoids

166. Repair of which of the following involves regeneration and connective tissue
repair:
a) Bone fracture
b) Renal tubular necrosis
c) Both A and B
d) Neither A nor B

167. Which of the following does not promote fracture healing:


a) Steroid therapy
b) Adequate blood supply
c) Immobilization
d) Adequate Vitamin C supply

168. Neurogenic, Septic and Cardiogenic shock are all characterized by:
a) Equal prognosis if untreated
b) Need for blood transfusion
c) Peripheral vasodilation at onset
d) Normal blood volume at early stage
e) Progression to irreversible shock

169. The causes of Hypovolemic shock do not include:


a) Severe vomiting
b) Severe diarrhea
c) Extensive burning
d) Myocardial Infarction
e) Severe Trauma

451 ‫صفحة‬ Dr. MOHAMED YAHIA 0900987639


QUESTION BANK OF Dr. MOHAMED YAHIA

170. Following a hypotensive shock which lasted for hours, which of the
following tissues is most likely to withstand the ischemia:
a) Skeletal muscle
b) Small Intestinal Epithelium
c) Retina
d) Myocardium
e) Hippocampus

171. What type of shock is most associated with severe burns:


a) Cardiogenic shock
b) Hypovolemic shock
c) Septic shock
d) Neurogenic shock

172. The causes of Hypovolemic shock do not include:


a) Severe vomiting
b) Severe diarrhea
c) Blood Loss
d) Acute hemorrhagic pancreatitis
e) Extensive Burning

173. The following are true about arterial thrombi except:


a) Are usually due to endothelial injury
b) Can be precipitated by atherosclerosis
c) Grow in the direction of blood flow
d) Lead to ischemia and infarction
e) Commonly cause systemic emboli

174. The following are true regarding venous thrombi except:


a) Commonly are due to stasis

452 ‫صفحة‬ Dr. MOHAMED YAHIA 0900987639


QUESTION BANK OF Dr. MOHAMED YAHIA

b) Always cause infarction


?c) Grow in the direction of blood flow
d) Can cause pulmonary emboli
e) Commonly occur in the deep leg veins

175. Thrombosis can be caused by all of the following except:


a) Endothelial injury
b) Turbulent blood flow
c) Leiden mutation
d) Factor VIII deficiency
e) Atrial Fibrillation

176. Which of the following are sequelae to thrombosis:


a) Embolization
b) Organization and Recanalization
c) Propagation
d) Both A and B
e) All of the above

177. Which one of the following does not predispose to thrombosis:


a) Endothelial damage
b) Vascular Stasis
c) Thrombocytopenia
d) Formation of aggregates
e) Activation of the coagulation mechanism

178. Which one of the following is not a cause of thrombosis:


a) Damage of endothelial lining
b) Aneurysms

453 ‫صفحة‬ Dr. MOHAMED YAHIA 0900987639


QUESTION BANK OF Dr. MOHAMED YAHIA

c) Polycythenia
d) Dissiminated Carcinomatosis
e) High level of antithrombin III
179. The following are associated with DIC except:
a) Widespread deposition of fibrin with microcirculation
b) Consumption of coagulation factors
c) Haemorrhagic diathesis (Bleeding Tendency)
d) Thrombocytosis (Increased Platelet Count)
e) Activation of Plasminogen

180. Disseminated Intravascular Coagulation is associated with the following


except:
a) Microangiopathic hemolytic anemia
b) Thrombocytosis
c) Low fibrinogen level
d) Disseminated malignancy
e) Crush injuries

181. The following predispose to deep vein thrombosis except:


a) Malnutrition
b) Cancer
c) Factor VIII deficiency
d) Abdominal Surgery
e) Immobilization

182. The following are true of Thrombi except:


a) Are formed from blood consitituents
b) Always cause complete vascular occlusion and infarction
c) Are complicated by embolization
d) May undergo dystrophic calcification

454 ‫صفحة‬ Dr. MOHAMED YAHIA 0900987639


QUESTION BANK OF Dr. MOHAMED YAHIA

183. Which of the following is not associated with Thrombosis:


a) Cancer
b) Prolonged bed rest
c) Sickle cell anemia
d) Hemophilia
e) Tissue damage

184. Fat emboli can be precipitated by:


a) Excessive fats intake
b) Rupture of an atheromatous plaque
c) Rupture of the uterus
d) Fracture of the femur
e) Changes in atmospheric pressure

185. Paradoxial emboli are:


a) Commonly pulmonary emboli
b) Are caused by myocardial infarction
c) Due to the presence of an interventricular on inter atrial defect
d) Due to sudden changes in the atmospheric pressure
e) Caused by trauma

186. Pulmonary emboli have each of the following characteristics except:


a) Commonly arise in leg veins
b) Pass through the right heart before reaching the lung

455 ‫صفحة‬ Dr. MOHAMED YAHIA 0900987639


QUESTION BANK OF Dr. MOHAMED YAHIA

c) Consistently cause infarction in the lung


d) Are usually thrombotic emboli

187. A detached intravascular solid, liquid or gas carried to a site distant from its
point of origin is most associated with:
a) Ascites
b) Petechiae
c) Infarction
d) Emboli
e) Hematoma

188. Infarction of the spleen is usually due to:


a) Hypersplenism
b) Congestion
c) Arterial Embolism
d) Deposition of connective tissue and pigment
e) Venous Thrombosis

189. In an area of myocardial infarction, which of the following will be most


effective in promoting healing:
a) Histamine
b) Immunoglobulin G
c) Complement C3b
d) Leukotriene B4
e) VEGF (vascular endothelial growth factor)

190. Red Infarcts develop in:


a) Spleen
b) Liver
c) Kidney

456 ‫صفحة‬ Dr. MOHAMED YAHIA 0900987639


QUESTION BANK OF Dr. MOHAMED YAHIA

d) Adrenal Glands
e) Intestine

191. Which of the following is most significant in the diagnosis of early acute
myocardial infarction:
a) ESR
b) ASO titre
c) Troponin
d) LDH
e) Total CK

192. Which of the following is not a major risk factor for atherosclerosis:
a) Family history
b) Hyperlipidemia
c) Physical inactivity
d) Diabetes
e) Hypertension
193. Organs which are less susceptible than others to infarction because of a dual
blood supply include:
a) Liver and Lungs
b) Liver and Kidney
c) Lung and Spleen
d) Spleen and Kidney
e) Pancreas and Lung

194. Which cells are targeted by HIV:


a) CD4 + cells

457 ‫صفحة‬ Dr. MOHAMED YAHIA 0900987639


QUESTION BANK OF Dr. MOHAMED YAHIA

b) Plasma cells
c) Macrophages
d) Both A & B
e) Both A & C

195. Asthma is an example of:


a) Type I hypersensitivity reaction
b) Type II hypersensitivity reaction
c) Type III hypersensitivity reaction
d) Type IV hypersensitivity reaction
e) Type V hypersensitivity reaction

196. IgE plays a major role in:


a) Type I hypersensitivity reaction
b) Type II hypersensitivity reaction
c) Type III hypersensitivity reaction
d) Type IV hypersensitivity reaction
e) Type V hypersensitivity reaction

197. Acute graft rejection is caused by:


a) Type I hypersensitivity reaction
b) Type II hypersensitivity reaction
c) Type III hypersensitivity reaction
d) Type IV hypersensitivity reaction
e) Type V hypersensitivity reaction
198. The following are true about SLE except:
a) It is commoner in females
b) Anti nuclear antibodies are diagnostic
c) It can affect the joints

458 ‫صفحة‬ Dr. MOHAMED YAHIA 0900987639


QUESTION BANK OF Dr. MOHAMED YAHIA

d) It is associated with a butterfly rash


e) It can cause anemia

199. HIV primarily affects:


a) CD8 lymphocytes
b) Cells expressing CD4 molecules
c) Neutrophils
d) Basophils
e) Eosinophils

200. The following are true about Type I hypersensitivity except:


a) It can be localized or systemic
b) Extrinsic asthma is an example
c) The predominant antibody involved is IgG
d) Degranulation of mast cells is the pathogenesis pathway
e) Can be diagnosed by skin test

201. Of the following histopathologic finding, the one best indicates that a
neoplasm is malignant is:
a) Pleomorphism
b) Atypia
c) Invasion
d) Increased Nuclear/Cytoplasmic Ratio
e) Necrosis

202. Which of the following infectious agents is associated with transformation of


tissue macrophages to epitheloid cells:
a) Mycobacterium leprae
b) Pseudomonas aeruginosa
c) Cytomegalovirus

459 ‫صفحة‬ Dr. MOHAMED YAHIA 0900987639


QUESTION BANK OF Dr. MOHAMED YAHIA

d) Giardia Lamblia
e) Staphylococcus aureus

203. Which of the following complications is most likely to occur in a 76 year old
woman who has a swollen leg followed by fracture of femoral trochanter and
who has been immobilized:
a) Gangrenous Necrosis of the Foot
b) Haematoma of the Thigh
c) Disseminated Intravascular Coagulation (DIV)
d) Pulmonary Thromboembolism
e) Soft Tissue Sarcoma

204. Generation of which of the following substances by the major inflammatory


cell type would be responsible for clearing microorganism in an inflammatory
focus:
a) Platelet Activated Factor
b) Prostaglandins
c) Kallikrein
d) Leukotriene
e) Hydrogen Peroxide
205. Which of the following cell types releases chemical mediators responsible for
allergic rhinitis:
a) Neutrophils
b) Mast Cells
c) CD4+ Cells
d) NK Cells
e) Macrophages

206. Which of the following chemical mediators is most likely to be involved in


anaphylaxis following intravenous penicillin treatment:
a) Interleukin I
b) Bradykinin

460 ‫صفحة‬ Dr. MOHAMED YAHIA 0900987639


QUESTION BANK OF Dr. MOHAMED YAHIA

c) Complement C5a
d) Histamine
e) Thromboxane
207. Which of the following cellular process is most likely to occur in the
myocardium of a patient with myocardial infarction who is being treated with
fibrinolytic agents:
a) Apoptosis
b) Free Radical Injury
c) Fatty Changes
d) Accumulation of Lipofuscin
e) Accumulation of Amyloid

208. Which of the following will predict a better prognosis for breast cancer:
a) The tumor is small in size
b) No metastasis are found in sampled lymph node
c) Numerous mitosis are seen
d) Tumor shows mild hyperchromatism
e) Tumor cells show marked pleomorphism

209. Which of the following is characteristic of neoplasm:


a) Recurrence following excision
b) Rapid increase in size
c) Sensitivity to radiation therapy
d) Uncontrolled (autonomous growth)
e) Necrosis

210. Widespread Oedema is likely in:


a) Lymphatic Obstruction
b) Capillary Damage
c) Inflammations

461 ‫صفحة‬ Dr. MOHAMED YAHIA 0900987639


QUESTION BANK OF Dr. MOHAMED YAHIA

d) Hypoalbuminemia
e) Cancer of the Stomach

211. Acute Pulmonary Oedema of the heart failure results from:


a) Increased Plasma Colloid Osmotic Pressure
b) Decreased Plasma Colloid Osmotic Pressure
c) Increased Vascular Permeability
d) Increased Vascular Hydrostatic Pressure
e) Increased Blood Viscosity

212. DIC following extensive surgery is due to:


a) Entrance of tissue thromboplastin into circulation
b) Hypoxia, acidosis and shock coexisting with extensive surgery
c) Protein C Deficiency
d) Both A and B
e) Both B and C

213. Which of the following is not a feature of stem cells:


a) Capable of self renewal
b) Capable of linkage generation
c) They are the predominant population in a tissue
d) They belong to the terminal differentiation compartment
e) Both A and B

214. Which of the following is not a feature of benign tumors:


a) Grow Slowly
b) Well Demonstrated
c) Infiltrate Adjacent Cells

462 ‫صفحة‬ Dr. MOHAMED YAHIA 0900987639


QUESTION BANK OF Dr. MOHAMED YAHIA

d) Can Secrete Hormones


e) Named by adding suffix 'oma' to type of cell

215. Which of the following malignant tumors has the best prognosis:
a) Hepatocellular Carcinoma
b) Renal Cell Carcinoma
c) Basal Cell Carcinoma
d) Squamous Cell Carcinoma of Cervix
e) Squamous Cell Carcinoma of Skin

216. The contents of a blister is an example of which type of exudate:


a) Serous
b) Fibrinous
c) Purulent
d) Cellular
e) Haemaorrhagic

217. Reduced Plasma Oncotic Pressure is the most important cause of


generalized edema in:
a) Congestive Heart Failure
b) Lung Cancer
c) Constrictive Pericarditis
d) Head Trauma
e) Nephrotic Syndrome

218. Which of the following diseases is characterized by noncaseating


granulomas?
a) Sarcoidosis
b) Tuberculosis
c) Histoplasmosis

463 ‫صفحة‬ Dr. MOHAMED YAHIA 0900987639


QUESTION BANK OF Dr. MOHAMED YAHIA

d) Diptheria
e) Measles

219. Nutmeg Liver is caused by:


a) Viral Hepatitis
b) Acute Hepatic Congestion
c) Chronic Hepatic Congestion
d) Fibrosis of the Liver
e) None of the Above

220. Which of the following is not a predisposing factor for pyelonephritis:


a) Pregnancy
b) Diabetes Mellitus
c) Renal Artery Stenosis
d) Prostatic Hypertrophy
e) Renal Calculi

221. Poststreptococcal glomerulonephritis is most closely associated with:


a) Subepithelial deposits
b) Fibrin Leakage
c) Peripheral Linear IgG deposits by immunoflorescence
d) Berger's Disease
e) Tubular immunoflorescence

222. Crescents are most likely associated with:


a) Subepithelial deposits
b) Fibrin Leakage
c) Peripheral Linear IgG deposits by immunoflorescence

464 ‫صفحة‬ Dr. MOHAMED YAHIA 0900987639


QUESTION BANK OF Dr. MOHAMED YAHIA

d) Berger's Disease
e) Tubular immunoflorescence

223. Dense deposits disease is most closely associated with:


a) Alport's Syndrome
b) Benign Recurrent Hematuria
c) Membranoproliferative glomerulonephritis Type II
d) Membranoproliferative glomerulonephritis Type I
e) Amyloidosis

224. Low Serum Complement Levels are present in:


a) Systemic Lupus Nephritis
b) Acute Post-streptococcal nephritis
c) Both A and B
d) Neither A nor B

225. Papillary Necrosis is associated with all of the following except:


a) Sickle Cell Anemia
b) Diabetes Mellitus
c) Analgesic Abuse
d) Wegener's Granulomatosis

226. All of the following are true of renal cell carcinoma except:
a) The tumor cells secrete erythropoietin
b) It originates from the glomeruli
c) The tumor cells are rich in glycogen
d) The tumor cell is often associated with polycythemia

465 ‫صفحة‬ Dr. MOHAMED YAHIA 0900987639


QUESTION BANK OF Dr. MOHAMED YAHIA

227. One of the following is not true about amoebic liver abscess:
a) There is Neutrophilia
b) The patient is febrile
c) Jaundice is always present
d) The right lobe of the liver is commonly involved
e) Metronidazole is used for treatment

228. Complications of Plasmodium falciparum malaria do not include:


a) Bone Marrow Aplasia
b) Nephrotic Syndrome
c) Hypoglycaemia
d) Haemoglobinuria
e) Severe Gastroenteritis

229. The Chemical Carcinogen, Alfatoxin B1, derived from a fungus which may
contaminate peanuts, most commonly induces:
a) Transitional Cell Carcinoma of the Lung
b) Adenocarcinoma of the Rectum
c) Hepatocellular Carcinoma
d) Squamous Cell Carcinoma of the Skin
e) Renal Cell Carcinoma

230. Which of the following is not true of Amyloid:


a) It is protein in nature
b) Complicates Bronchiectasis
c) Causes Nephrotic Syndrome
d) In Multiple Myeloma, it is of the AA type
e) Demonstrated microscopically using crystal violet

466 ‫صفحة‬ Dr. MOHAMED YAHIA 0900987639


QUESTION BANK OF Dr. MOHAMED YAHIA

231. Which of the following is seen in Typhoid Fever?


a) Haemaphagocytosis is Observed
b) Characteristically there is Tachycardia
c) The appendix is almost always involved
d) There is leucocytosis
e) Necrotizing Granuloma is a feature

232. One of the following is not a Tumor Marker:


a) Thromboxan A2
b) Alpha Fetoprotein
c) Carcinoembryonic Antigen
d) Prostate Specific Antigen
e) Chorionic Gonadotrophin

233. Which of the following is a Tumor Marker?


a) Thromboxane A2
b) Acetylcholine
c) Prostaglandin
d) ACTH
e) Ferritin

234. Which of the following causes Lymphatic Oedema?


a) Sarcoidosis
b) Schistosomiasis
c) Filariasis
d) Syphilis
e) Leishmaniasis

467 ‫صفحة‬ Dr. MOHAMED YAHIA 0900987639


QUESTION BANK OF Dr. MOHAMED YAHIA

235. Which of the following does NOT cause Hypokalaemia?


a) Diueretics
b) Vomiting
c) Diarrhoea
d) Addison's Disease
e) Metabolic Alkalosis

236. The following are associated with Cachexia except:


a) Malignancy
b) Anemia
c) Wasting
d) Hypoplasia
e) Tumour Necrosis Factor

237. Acute effects of Ionizing Radiation do not include:


a) Leukemia
b) Anemia
c) Skin Desquamation
d) Pulmonary Oedema
e) Vomitting

238. Primary stage of Syphilis is best diagnosed by:


a) VDRL Test
b) RPR Test
c) Widal Test
d) Smear seen under dark ground illumination
e) TPHA

239. Granuloma Formation is not a feature of:


a) Tuberculoid Leprosy

468 ‫صفحة‬ Dr. MOHAMED YAHIA 0900987639


QUESTION BANK OF Dr. MOHAMED YAHIA

b) Sarcoidosis
c) Schistosoma manson infection
d) Tuberculosis
e) Cerebral Malaria

240. Pathological effects of Shock do not include:


a) Gastric Ulceration
b) D.I.C.
c) Acute Renal Tubular Necrosis
d) Metabolic Alkalosis
e) Adult Respiratory Distress Syndrome

241. Peg-Shaped (Hutchinson's Teeth) are characteristically seen in babies born


to mothers suffering from:
a) Mucosal Leishmaniasis
b) Syphilis
c) Sarcoidosis
d) Intestinal Tuberculosis
e) AIDS

242. In Amoebiasis:
a) Condylomata Lata is a feature
b) Granuloma Formulation is characteristic
c) The small intestine is always involved
d) The perianal skin may be affected
e) There is an intense infiltration in the lesions by polymorphs

243. Consequences of Necrosis do not include:


a) Dystrophic Calcification
b) Resolution

469 ‫صفحة‬ Dr. MOHAMED YAHIA 0900987639


QUESTION BANK OF Dr. MOHAMED YAHIA

c) Inflammatory Reaction
d) Fibrosis
e) Increased Cellular Enzymes in the Blood

244. In Lepromatous Leprosy:


a) Epitheloid Granuloma Formation is characteristic
b) There is loss of the Rete Ridges of the epidermis
c) Bacilli are scanty
d) Spontaneous cure may occur
e) Lepromin test is negative

245. One of the following is not true about bilharzial portal hypertension:
a) Splenomegaly is of the congestive type
b) Iron deficiency anemia is a complication
c) Periportal fibrosis is the cause
d) There is an increased incidence of hepatocellular carcinoma
e) Hepatic failure is not an early complication

246. Gummas most often appear in:


a) Spleen
b) Pancreas
c) Liver and Testes
d) Kidneys
e) Thyroid
247. Hepar Lobatum is liver showing:
a) Miliary Tuberculosis
b) Multiple Gummas and Fibrosis
c) Periportal Fibrosis
d) Amoebic Abscess

470 ‫صفحة‬ Dr. MOHAMED YAHIA 0900987639


QUESTION BANK OF Dr. MOHAMED YAHIA

e) Sarcoid Granulomas

248. Metaplasia is most closely associated with:


a) Hypercalcemia
b) Necrosis
c) Chronic Irritation
d) Diminished Blood Supply
e) Increased Workload

249. Chest X-Ray of the Lung from a patient suffering of sarcoidosis


characteristically shows:
a) Apical Cavitation
b) Lobar Consolidation
c) Dilated Bronchi
d) Bilateral Enlarged Hilar Lymph Nodes
e) Pleural Thickening

250. Primary sites of Tuberculosis include all of the following except:


a) Skin
b) Lungs
c) Bones
d) Tonsils
e) Intestine

251. The histology of a sarcoid lesion characteristically shows the following


except:
a) Epitheloid cells and Langhans's Giant Cells
b) Asteroid Bodies
c) Shauman Bodies
d) Caseous Necrosis

471 ‫صفحة‬ Dr. MOHAMED YAHIA 0900987639


QUESTION BANK OF Dr. MOHAMED YAHIA

e) A Cuff of Lymphocytes

252. In Bilharzial Portal Hypertension:


a) There is increased incidence of hepatocellular carcinoma
b) Liver cirrhosis occurs in 90% of cases
c) Hepatic Failure is an early complication
d) Haematemesis is a common complication
e) Splenomegaly is due to proliferation of macrophages

253. One of the following is NOT a feature of visceral leishmaniasis:


a) Weight Loss
b) Splenomegaly
c) Bone Marrow Failure
d) Intermittent Fever
e) Bleeding Tendency

254. The following are not features of the primary focus in pulmonary
tuberculosis except:
a) Usually undergoes cavitation
b) Commonly occurs in the apex of the lung
c) May give rise to miliary tuberculosis
d) Is known as Assman's Focus
e) Is a common cause of haemoptysis

255. During the early bacteraemic phase of Typhoid Fever, diagnosis is


established by:
a) Blood Culture
b) Widal Test

472 ‫صفحة‬ Dr. MOHAMED YAHIA 0900987639


QUESTION BANK OF Dr. MOHAMED YAHIA

c) Stool Culture
d) Urine Culture
e) Peritoneal Fluid Culture

256. Pulmonary Oedema does not occur in:


a) Severe Pneumonia
b) Uraemia
c) Lung Fibrosis
d) Inhalation of Toxic Fumes
e) Mitral Stenosis

257. The following occur in P. falciparum infection except:


a) Jaundice
b) Generalized Lymphadenopathy
c) Acute Renal Failure
d) Cerebral Petechial Hemorrhages
e) Splenomegaly

258. Which of the following is a feature of Lepromatous Leprosy:


a) Scanty bacilli
b) Granuloma formation
c) Negative Lepromin Test
d) Thickening of nerves
e) Spontaneous recovery

259. The following are true of Amyloidosis except:


a) It is lipid in nature
b) Causes Splenomegaly

473 ‫صفحة‬ Dr. MOHAMED YAHIA 0900987639


QUESTION BANK OF Dr. MOHAMED YAHIA

c) Is a common cause of Hepatic Failure


d) Deposited Extracellularly
e) Is a known cause of Nephrotic Syndrome

260. In Amoebic Liver Abscess:


a) There is marked neutrophilia
b) The liver in not enlarged
c) Jaundice is a constant feature
d) The left lobe is commonly affected
e) The patient is afebrile

261. Which of the following is not a feature of Typhoid Intestinal Lesions?


a) Organisms commonly settle in the mucosal lymphoid aggregates
b) Ulceration of Peyer's Patches
c) Infiltration by Plasma Cells
d) Infiltration by Neutrophils
e) Infiltration by Lymphocytes
262. Benign Tumours:
a) Commonly Metastasize
b) Are Encapsulated
c) Have hyperchromatic nuclei
d) Infiltrate surrounding tissue
e) Commonly recur after removal

263. The carrier state of Typhoid Fever is due to:


a) Persistent Bacteraemia
b) Persistent Duodenal Lesions
c) Persistent Intestinal Ulcers
d) Typhoid Cholecystitis

474 ‫صفحة‬ Dr. MOHAMED YAHIA 0900987639


QUESTION BANK OF Dr. MOHAMED YAHIA

e) Persistent Rose Spots

264. The following cells are capable of regeneration except:


a) Neurons
b) Hepatocytes
c) Renal Tubular Cells
d) Colonic Mucosal Cells
e) Cells lining the Respiratory Airways

265. In Typhoid Fever:


a) There is Tachycardia
b) The large intestine is commonly affected
c) The white cell count is characteristically very high
d) Rose spots appear in the intestinal mucosa
e) The gallbladder is infected

266. Serious complications of Typhoid Fever include:


a) Intestinal Haemorrhage
b) Amyloidosis
c) Perforation of Intestine
d) Peritonitis
e) All of the Above

267. The sites of Primary Tuberculosis include:


a) Oesophagus
b) Stomach
c) Oropharynx
d) Ileocecal
e) Both C and D

475 ‫صفحة‬ Dr. MOHAMED YAHIA 0900987639


QUESTION BANK OF Dr. MOHAMED YAHIA

268. Oedema of Congestive Heart Failure is due to:


a) Generalized increase in venous pressure
b) Triggered rennin-angiotensin-aldosterone axis
c) Increased salt intake
d) Both A and B
e) Both B and C

269. Which of the following is not used for diagnosis of Typhoid Fever:
a) Blood Culture
b) Widal
c) Stool Culture
d) VDRL
270. The activation of chemical procarcinogen requires activation by:
a) Neuropeptidases
b) Dismutases
c) Collagenase
d) P450 dependent Oxygenase
e) Alkaline Phosphotase

271. Immune-Complex Glomerulonephritis is seen with:


a) Primary TB
b) Tuberculoid Leprosy
c) Lepromatous Leprosy
d) Typhoid Fever
e) Amoebic Dysentery

476 ‫صفحة‬ Dr. MOHAMED YAHIA 0900987639


QUESTION BANK OF Dr. MOHAMED YAHIA

272. Exposure to UV light will give rise to which of the following tumours:
a) Hepatocellular Carcinoma
b) Basal Cell Carcinoma
c) Squamous Cell Carcinoma
d) Both A and C
e) Both B and C

273. Who of the following is more likely to develop Squamous Cell Carcinoma of
the Skin:
a) Sudanese
b) Nigerians
c) Kenyans
d) Scandinavian living in Zimbabwe
e) Bantu South African

274. In Acute Hemopoetic Syndrome following Ionizing Radiation there is:


a) Anemia
b) Lymphopenia
c) Thrombocytopenia
d) Both B and C
e) All of the Above

275. Which of the following is used for screening of food handlers for Typhoid:
a) Blood Culture
b) Stool Culture
c) Widal
d) Sputum Culture

477 ‫صفحة‬ Dr. MOHAMED YAHIA 0900987639


QUESTION BANK OF Dr. MOHAMED YAHIA

e) Stools Hatching Test

276. Which of the following is most likely to develop skin cancer:


a) Clerk Typist
b) Laboratory Technologist
c) Heavy Truck Driver
d) Radiology Technologist
e) Supermarket Supervisor
277. Which of the following is associated with Leukemia's and Lymphomas:
a) Cyclophosphamide
b) Chloramephenicol
c) Chloroquine
d) Cigarette Smoking
e) Heavy Alchohol Consumption

278. Granuloma Formation in Schistosomiasis is stimulated by:


a) Adult Female Antigens
b) Adult Male Antigens
c) Soluble Egg Antigen
d) Cercarial Antigens
e) All of the Above

279. Hepatomegaly and very high leukocytes are features of:


a) Visceral Leishmaniasis
b) Schistosomal Hepatosplenomegaly
c) Typhoid Fever
d) Extra Intestinal Amoebiasis
e) Food Poisoning

478 ‫صفحة‬ Dr. MOHAMED YAHIA 0900987639


QUESTION BANK OF Dr. MOHAMED YAHIA

280. Which of the following is a feature of Chemical Carcinogens:


a) They are Hetrophilic
b) Amphophilic
c) Neutrophilic
d) Nucleophilic
e) Inactive Cytochrome Oxidase
281. Sarcoidosis is characterized by the following EXCEPT:
a) Asteroid Bodies
b) Schuman Bodies
c) Caseating Granuloma
d) Langhans Giant Cells
e) Foreign Body Giant Cell

282. Which of the following can give rise to Nitrosamins when ingested:
a) Fresh Vegetable
b) Citric Fruits
c) Preserved Food
d) Biscuits
e) Beef

283. Individuals with Sickle Cell Trait are resistant to Malaria because:
a) They have low hemoglobin
b) Anopheles mosquito does not bite them
c) Their red blood cells sickle when parasitized and are removed by spleen
d) Their red blood cells contain antiparasite antibodies
e) None of the Above

284. Sandy Patches are feature of:


a) Primary Tuberculosis

479 ‫صفحة‬ Dr. MOHAMED YAHIA 0900987639


QUESTION BANK OF Dr. MOHAMED YAHIA

b) Urinary Schistosomiasis
c) Tuberculoid Leprosy
d) Sarcoidosis
e) Tertiary Syphilis

285. Pipe-Stern Fibrosis is:


a) Delayed Wound Healing
b) Dystrophic Calcification
c) Liver Changes in Schistosomiasis
d) Hilar Lymph Nodes in Primary TB
e) Healed Primary Chancre

286. Asteroid Bodies are characteristically seen in:


a) Secondary Tuberculosis
b) Tertiary Syphilis
c) Amyloidosis
d) Sarcoidosis
e) Tuberculoid Leprosy

287. The hallmark of Syphilis is:


a) Endarteritis Obliterans
b) Abominal Aneurysm
c) Heavy Plasma Cell Infiltrate
d) Both A and B
e) Both A and C

288. Syphilitic Aortic Aneurysm:

480 ‫صفحة‬ Dr. MOHAMED YAHIA 0900987639


QUESTION BANK OF Dr. MOHAMED YAHIA

a) Occurs in Thoracic Aorta


b) Is due to Ischemic Destruction of Media and Elastic Fibre
c) There is Endarteritis of Vasa Vasora
d) All of the Above
e) None of the Above

289. Snail Track Ulcers are a feature of:


a) Cutaneous Leishmaniasis
b) Typhoid Fever
c) Extra Intestinal Amoebiasis
d) Secondary Syphilis
e) Schistosomal Dermatitis

290. Bilateral Hilar Lymphadenopathy is a feature of:


a) Secondary Tuberculosis
b) Tertiary Tuberculosis
c) Amyloidosis
d) Sarcoidosis
e) Tuberculoid Leprosy

291. Which of the following is not true about Syphilitic Primary Chancre:
a) Occurs commonly on external genitalia
b) Contains treponema pallidum spiroecele
c) Has a dense plasma cell infiltrate
d) Does not heal spontaneously
e) Occurs 3 weeks after contact with an infected individual

292. Which of the following is not a feature of Schistosomal Portal Hypertension:


a) Ascites

481 ‫صفحة‬ Dr. MOHAMED YAHIA 0900987639


QUESTION BANK OF Dr. MOHAMED YAHIA

b) Oesophageal Varices
c) Markedly Disturbed Liver Function
d) Coput Medusa
e) All of the Above

293. Eosinophilia of Schistosomiasis is stimulated by:


a) C3b
b) C5a
c) TH2 inducing IL-4 and IL-5 production
d) Tremor Necrosis Factor
e) None of the Above

294. Which of the following is most likely to occur as a late complication of


urinary schistosomiasis:
a) Oesophageal Varices
b) Ascites
c) Carcinoma of Urinary Bladder
d) Pulmonary Hypertension
e) Cor Pulmonale

295. Immune-Complex Glomerulonephritis is most likely associated with:


a) Tuberculoid Leprosy
b) Primary Tuberculosis
c) Typhoid Fever
d) Schistosomiasis
e) Extra Intestinal Amoebiasis

296. Hydroureter and Hydronephrosis are complications of:


a) Sarcoidosis
b) Widespread Cancer

482 ‫صفحة‬ Dr. MOHAMED YAHIA 0900987639


QUESTION BANK OF Dr. MOHAMED YAHIA

c) Typhoid Fever
d) Urinary Schistosomiasis
e) Portal Hypertension

297. Which of the following is most affected by Ionizing Radiation:


a) Kidney
b) Liver
c) Bone
d) Bone Marrow
e) Striated Muscle
298. Which of the following tissues is most affected by Sarcoidosis:
a) The Liver
b) The Kidneys
c) The Lungs
d) Skin
e) Bone

299. Flask-shaped Ulcers are characteristically seen in:


a) Intestinal Tuberculosis
b) Bacillary Dysentery
c) Amoebiasis
d) Intestinal Schistosomiasis
e) Typhoid Fever
300. Which of the following is associated with inflammation and ulceration of
Peyer Patches:
a) Intestinal Tuberculosis
b) Bacilliary Dysentery
c) Amoebiasis
d) Intestinal Schistosomiasis
e) Typhoid Fever

483 ‫صفحة‬ Dr. MOHAMED YAHIA 0900987639


QUESTION BANK OF Dr. MOHAMED YAHIA

301. Ionizing Radiation has a maximum effect at:


a) High Oxygen Tension
b) Low Oxygen Tension
c) High CO2 Tension
d) Low CO2 Tension
e) High Nitrogen Tension

302. Pathological Lesions of Typhoid Fever are best seen in:


a) Ileo-Caecal Valve
b) Appendix
c) Peyers Patches
d) Anal Mucosa
e) 2nd Part of Duodenum

303. Which of the following is an Immune-Complex Disease:


a) Rheumatoid Arthritis
b) Bronchial Asthma
c) Chronic Osteomyelitis
d) Acute Glomerulonephritis
e) Secondary Tuberculosis

304. Fibrosis following Ionizing Radiation is due to:


a) Direct Damage and Necrosis to Tissue
b) Stimulation of Fibroblast Proliferation by Cytokines
c) Ischaemia due to injury of Endothelial Cells
d) Replacement of Atrophic Parenchymal Cells
t e) All of the Above

484 ‫صفحة‬ Dr. MOHAMED YAHIA 0900987639


QUESTION BANK OF Dr. MOHAMED YAHIA

305. The killing of Parasites in Leishmaniasis is mediated by:

a) -Interferon

b) TNF-
c) IL-4
d) Both A and B
e) Both A and C

306. Which of the following involved in pathogenesis of septic shock:


a) Bacterial Wall Lipopolysaccharides
b) IL-1
c) IL-8
d) All of the Above
e) None of the Above

307. In Sudan, Post Kalazar Dermal Leishmaniasis:


a) Soon follows treatment of visceral leishmaniasis
b) The lesion heals spontaneously within few months
c) There is deep ulceration of the lesions
d) Both A and B
e) Both A and C

308. Which of the following is used for diagnosis of visceral leishmaniasis:


a) Blood Culture
b) Nasal Smear
c) Skin Snip

485 ‫صفحة‬ Dr. MOHAMED YAHIA 0900987639


QUESTION BANK OF Dr. MOHAMED YAHIA

d) Widal Test
e) Bone Marrow Smear

309. Which of the following is associated with deposition of Amyloid AL:


a) Osteoarthritis
b) Intestinal Schistosomiasis
c) Multiple Myeloma
d) Systemic Lupus Erythematous
e) Crohn's Disease

310. Which of the following statements is not true about Unconjugated Bilirubin:
a) It is Protein Bound
b) It is Water Insoluble
c) It is Excreted in Urine
d) Can cross the Blood-Brain Barrier
e) It is increased in Hemolytic Anemia

311. Post Hepatic Jaundice is characterized by:


a) Pale Stools
b) Absence of Bilirubinuria
c) Unconjugated Hyperbilirubinemia
d) Hematuria
e) Elevated Serum Transaminases
312. Primary Tuberculosis is Characterized by:
a) Cavitation and Fibrosis
b) Majority of patients are Asymptomatic
c) Hilar Lymph Nodes are not affected
d) It is mainly disease of adults
e) Mainly affects the apex of the lung

486 ‫صفحة‬ Dr. MOHAMED YAHIA 0900987639


QUESTION BANK OF Dr. MOHAMED YAHIA

313. When a Cercaria penetrate the skin of man, it is transformed into:


a) Metacaria
b) Schistomulum
c) Sporocyst
d) Miracidium
e) Redia

314. Schistosoma Mansoni and Japonicum can cause:


a) Hematuria
b) Hepatocellular Carcinoma
c) Rectal Polyps
d) Malabsorption
e) Colorectal Carcinoma

315. Condyloma Latum is seen in:


a) Secondary Syphilis
b) Extra Intestinal Amoebiasis
c) Intestinal Schistosomiasis
d) Sarcoidosis
e) Secondary TB

316. Gummas are seen in:


a) Tertiary Syphilis
b) Intestinal Amoebiasis
c) Leishmaniasis
d) Sarcoidosis
e) Secondary TB

487 ‫صفحة‬ Dr. MOHAMED YAHIA 0900987639


QUESTION BANK OF Dr. MOHAMED YAHIA

317. Which type of Amyloid is associated with Long Term Haemoialysis:


a) B2- Microglobulin
b) AL
c) AA
d) Amyloid Endocrine (AE)
e) None of the Above

318. Select the incorrect statement. In the etiology of human carcinogenesis:


a) Food components may play a role
b) Dietary Fat is known to initiate colon cancer
c) Certain occupational exposures are known risk factors
d) Consumption of moldy food is a risk factor

319. All of the following are thought to be related to the capacity for tumor cells
to invade except:
a) Elaboration of Type IV Collagenase
b) Expression of Laminin Receptors
c) Mitotic Rate
d) Production of Proteases

320. Which of the following are characteristics of Malignant Neoplasms:


a) Encapsulation
t b) Expansile Growth
c) Both A and B
d) Neither A nor B

321. Which of the following is the best candidate as the origin of a Metastatic
Neoplasm in the Liver:
a) Carcinoma in situ of the uterine cervix
b) Adenocarcinoma of the cecum

488 ‫صفحة‬ Dr. MOHAMED YAHIA 0900987639


QUESTION BANK OF Dr. MOHAMED YAHIA

c) Renal Cell Carcinoma, right kidney


d) Germ Cell Neoplasm of the testis
e) Transitional Cell Carcinoma of the bladder

322. The most important enzyme system for the Metabolic Activation of
Chemical Carcinogens is:
a) Glutathione Reductase
b) Cytochrome-P450-dependednt Monooxygenases
c) Epoxide Hydrolase
d) UDP-Glucuronic Acid Transferase

323. The ability of a Chemical Carcinogen to induce cancer in an animal is


influence by:
a) DNA Repair Capacity of the Animal
b) Ability of the Animal to Detoxify the Carcinogen
c) Ability of the Animal to Activate the Carcinogen
d) Presence of the Tumor Promoter
e) All of the Above

324. Which factor(s) contribute(s) to selection of a favored site of Metastatic


Deposits for a Tumor:
a) The Ploidy of Tumor Cells
b) Compatibility of Cell Surface Characteristics of Tumor and the Receptor
Tissue Site
c) Both A and B
d) Neither A nor B

325. Which of the following is not a Tumor Marker:

489 ‫صفحة‬ Dr. MOHAMED YAHIA 0900987639


QUESTION BANK OF Dr. MOHAMED YAHIA

a) AFP (Alfa-Feto Protein)


b) CEA (Carcino Embryonic Antigen)
c) PSA (Prostale Specific Antigen)
d) Aflatoxin
e) ACTH

326. The late Mitsuda Reaction is strongly positive in:


a) Lepromatous Leprosy
b) Tuberculoid Leprosy
c) Indeterminate Leprosy
d) Borderline Leprosy
e) Borderline Lepromatous Leprosy
327. Snail Track Ulcers are a feature of:
a) Intestinal Amoebiasis
b) Muco-cutaneous Leishmaniasis
c) Dermal Sarcoidosis
d) Secondary Syphilis
e) Post-Kale Agar Dermal Leishmaniasis

328. Which of the following is not a feature of Secondary Syphilis:


a) Widespread Skin Rash
b) Generalized Lymphadenopathy
c) Condyloma Lata
d) Hepar Lobatum
e) Abundant Spirochaete

329. Which of the following is a Non-Specific Test for Syphilis:


a) VDRL (test)
b) Widal Test

490 ‫صفحة‬ Dr. MOHAMED YAHIA 0900987639


QUESTION BANK OF Dr. MOHAMED YAHIA

c) Kvein Test
d) Mantoux Test
e) Lepromin Test

330. Asteroid and Shauman Bodies are seen in:


a) Primary Tuberculosis
b) Secondary Tuberculosis
c) Sarcoidosi\
d) Tertiary Syphilis
e) Visceral Leishmaniasis

331. Syphilis Transmission is:


a) Venereal
b) Through Placenta
c) Accidental Inoculation
d) Both A and B
e) All of the Above

332. Which of the following is not a feature of Lepromatous Leprosy:


a) Involvement and perforation of Nasal Cartilage
b) Production of Antibodies to various antigens
c) Multinucleated Giant Cells containing Asteroid Bodies
d) Lepromatous Cells contain many Bacilli
e) Sub epidermal zone is clear of Infiltrate

333. Which of the following is not a feature of Primary Chanre of Syphilis:

491 ‫صفحة‬ Dr. MOHAMED YAHIA 0900987639


QUESTION BANK OF Dr. MOHAMED YAHIA

a) It is a hard very painful nodule


b) Develops at a site of infection
c) Persists for weeks and heals spontaneously
d) Leaves minimal scarring
e) Contains many spirochaetes
334. Which of the following is not a Malignant Tumor:
a) Lung Hamartoma
b) Liver Hepatoma
c) Brain Glioma
d) Skin Melanoma
e) Lymphoma

335. Peg-Shaped (Hutchinson's Teeth) are seen:


a) Mucutaneous Leishmaniasis
b) Secondary Syphilis
c) Oral Candida (fungal) infection
d) Congenital Syphilis
e) Gumma of Gingiva

336. Formation of Epitheloid Cells from Macrophages is induced by:


a) IL-10
b) IL-4
c) C3b

d) -Interferon
e) IgM

492 ‫صفحة‬ Dr. MOHAMED YAHIA 0900987639


QUESTION BANK OF Dr. MOHAMED YAHIA

337. Which of the following clinical patterns of Leprosy is associated with a well
developed cell mediated response:
a) Lepromatous Leprosy
b) Tuberculoid Leprosy
c) Borderline Leprosy
d) Indeterminate Early Stage
e) Borderline Lepromatous

338. Mycobacterium leprae grows in the laboratory in which of the following


media:
a) Blood agar
b) Maconkey agar
c) Lowenston-Johnson Medium
d) All of the Above
e) None of the Above

339. Which of the following Tumors does not Metastasize:


a) Skin Squamous Cell Carcinoma
b) Male Breast Carcin/oma
c) Gastric Carcinoma
d) Skin Basal Cell Carcinoma
e) Melano Carcinoma

340. The following are associated with Cardiovascular Syphilis, except:


a) Tree-Bark appearance of Aorta
b) Inflammation of Vasa Vasora
c) Aneurysm of Arch of Aorta
d) Aortic Stenosis

493 ‫صفحة‬ Dr. MOHAMED YAHIA 0900987639


QUESTION BANK OF Dr. MOHAMED YAHIA

e) Occlusion of Coronary Ostia

341. Which of the following is a Specific Test for Syphilis:


a) VDRL (test)
b) Widal Test
c) Kvein Test
d) Mantoux Test
e) TPI Test

342. Bilateral Hilar Lymphadenopathy Lung involvement by Non-Caseating


Granuloma is characteristically seen in:
a) Primary Tuberculosis
b) Post-Primary (secondary) Tuberculosis
c) Tuberculoid Leprosy
d) Sarcoidosis
e) Bacteriuria Phase of Enteric Fever

343. The hallmarks of Syphilis is (are):


a) Endarteritis Obliteraus
b) Heavy Plasma Cell Infiltrate
c) Epitheloid Granuloma
d) Both A and B
e) All of the Above

344. A Psoas Cold Abscess is seen in:


a) Staphylococcal Infection of an Ischemic Toe
b) Pyogenic Infection of the Pleural Cavity

494 ‫صفحة‬ Dr. MOHAMED YAHIA 0900987639


QUESTION BANK OF Dr. MOHAMED YAHIA

c) Extra Intestinal Amoebiasis


d) Infiltration of Caseous Material from Vertebra by Polymorph
e) Leprosy Ulcers

345. Sites of Primary Intestinal Tuberculosis are:


a) The Gum
b) Oropharynx
c) Ileac Caezal Valve Regie
d) Both B and C
T e) All of the Above

346. Which of the following is (are) features of Post Primary (Secondary


Tuberculosis):
a) Can be due to Re-Infection
b) Can be due to Re-Activation
c) Commonly affects the Upper Lobe
d) Both B and C
e) All of the Above

347. Which of the following is (are) portal entry of Lepra Bacilli:


a) Skin
b) Sexual Transmission
c) Upper Respiratory Tract
d) Both A and B
T e) Both A and C

348. Immuno Deficient Patients are prone to the following except:


a) Bacterial Infections
b) Viral Infections
c) Autoimmune Diseases

495 ‫صفحة‬ Dr. MOHAMED YAHIA 0900987639


QUESTION BANK OF Dr. MOHAMED YAHIA

d) Tumors
e) None of the Above

349. The type of Amyloid associated with Chronic Inflammation is:


a) AA
b) AL
c) B2 Amyloid
d) Transthyrefin
e) B2 Microglobulin

350. Amyloid Material is deposited:


a) Intracellularly
b) Extracellularly
c) In the Kidney only
d) Both A and B
e) Both A and C

351. Under Polarized Light, Amyloid Stained with Congo Red appears:
a) Pink
b) Blue
c) Vacuoles
d) Apple Green Birifringence
e) Granulated

352. Visceral Leishmaniasis is best diagnosed by the following:


a) Demonstration of LD bodies in Bone Marrow
b) Demonstration of LD bodies in Lymph Nodes
c) Demonstration of LD bodies in Spleen
d) Demonstration of LD bodies in Skin

496 ‫صفحة‬ Dr. MOHAMED YAHIA 0900987639


QUESTION BANK OF Dr. MOHAMED YAHIA

e) Leishmania Test

353. The course of Leishmania Infection is determined by:


a) The host T-cell Response
b) The species of the Leishmania
c) The host B-cell Response
d) Both A and B
e) Both A and C

354. The following are true about V. leishmaniasis except:


a) It is not common in Sudan
b) It can cause Thrombocytopenia
c) It is transmitted by the Sand Fly
d) It is a Zoonotic Infection
e) It can cause Immunodeficiency

355. Unconjugated Hyperbilirubinemia can be found in all of the following


except:
a) Sickle Cell Anemia
b) Viral Hepatitis
c) Stone in the Common Bile Duct
d) Malaria
e) Gilbert's Syndrome

356. In Rotor Syndrome:


a) There is decreased excretion of bilirubin by the hepatocytes
b) Defective Conjugation of Bilirubin
c) The liver is pigmented
d) There is increased urobilinogen in urine
e) None of the Above

497 ‫صفحة‬ Dr. MOHAMED YAHIA 0900987639


QUESTION BANK OF Dr. MOHAMED YAHIA

357. The following are causes of Secondary Immunodeficiency except:


a) Infection
b) Malnutrition
c) Aging
d) DiGeorge's Syndrome
e) Radiotherapy

358. Cellular Immunodeficiceny predisposes to all of the following except:


a) Viral Infections
T b) Pyogenic Infections
c) Fungal Infections
d) TB
T e) Malignancy

359. The worst prognosis is found in Amyloidosis associated with:


a) Alzahimer's
b) Multiple Myloma
c) Chronic Inflammation
d) Localized Type
e) Renal Failure

360. Plasma Bilirubin is:


a) 5% Unconjugated
b) 95% Conjugated
c) 95% Unconjugated
d) Crosses the Blood Brain Barrier in Adults
e) Is normally filtered by kidneys

498 ‫صفحة‬ Dr. MOHAMED YAHIA 0900987639


QUESTION BANK OF Dr. MOHAMED YAHIA

361. Kala-zar means:


a) Black Fever
b) Yellow Fever
c) Black Ulcer
d) Cutaneous Leishmaniasis
e) Black Skin
362. Sickle Cell Anemia mode of inheritance is:
a) Autosomal Dominant
b) Autosomal Recessive
c) Sex-Linked Recessive
d) Random Mutation
e) None of the Above

363. Sickle Cell Anemia first presents as:


a) Adult Life
b) Birth
c) Few Months after Birth
d) Pre Notaily
e) All of the Above

364. Morphologic changes in Hemolytic Anemia includes:


a) Reticulocytosis
b) Increased Namboblasts in the Bone Marrow
c) Increased Lymphoblasts in the Bone Marrow
T d) A and B
e) B and C

365. Causes of Megaloblastic Anemia:


a) B12 Deficiency

499 ‫صفحة‬ Dr. MOHAMED YAHIA 0900987639


QUESTION BANK OF Dr. MOHAMED YAHIA

b) B6 Deficiency
c) Folate Deficiency
d) A and B
e) B and C

366. Causes of Iron Deficiency Anemia include all of the following except:
a) Peptic Ulcers
b) Hemorrhoids
c) Low Dietary Iron
d) Intra-Vascular Hemolysis
e) Menorrhagia

367. The causes of Aplastic crisis in Sickle Cell Anemia is usually:


a) Infection by Coxakie Virus
b) Infection by Mumps Virus
c) Infection by Molania
d) Infection of Paro Virus
e) Exhaustion of Erythropoiesis

368. The Causative Agent of Osteomyelitis in Sickle Cell Anemia is:


a) Shigelle
b) Salmonella
c) Stapli aurevs
d) Streptococci
e) E-coli
369. In Cold Agglutinin Hemolytic Anemia the Antibodies are:
a) Ig G
T b) Ig M
c) Ig A

500 ‫صفحة‬ Dr. MOHAMED YAHIA 0900987639


QUESTION BANK OF Dr. MOHAMED YAHIA

d) Ig E
e) Ig D

370. Hypochromic Microcytic RBCs are found in:


a) Iron Deficiency Anemia
b) B12 Deficiency
c) Folate Deficiency
d) Anemia of Chronic Disease
e) Sickle Cell Anemia

371. Lactating Mothers will most likely develop:


T a) Iron Deficiency Anemia
b) B12 Deficiency
c) Folate Deficiency
d) Hemolytic Anemia
e) None of the Above

372. In Onchocerciasis there is:


a) Inflammatory Infiltrate around adult O. volvulus
b) Inflammatory Infiltrate filarial or O. volvulus
c) Collection of Filaria subcuatenously
d) Collection of Adult O. volculus in the muscle
e) None of the Above

373. Mazzotti Reaction is:


a) Damage of the skin by O. volvulus worms
b) Damage of the skin by wolbachia
c) Due to treatment with Antifilarial Drugs
d) Due to treatment with Doxycyclin

501 ‫صفحة‬ Dr. MOHAMED YAHIA 0900987639


QUESTION BANK OF Dr. MOHAMED YAHIA

e) None of the Above

374. Chagas Disease can present as the following except:


a) Dilated Cardiomyopathy
b) Cardiac Arryhthmias
c) Dilated Colon
d_ Leptomeningitis
e) Dilated Esophagus

375. The Second Common Cause of Blindness in Africa is:


a) Diabetes Mellitus
b) Trachoma
c) Onchocericiasis
d) Glucoma
e) Hypertension
376. Somatostatin is:
a) Growth Hormone Releasing Hormone
T _b)Growth Hormone Inhibiting Hormone
c) Insulin Like Growth Factor-1
d) Growth Hormone
e) Corticotrophic Releasing Hormone

377. The following stimulate Growth Hormone Release except:


a) Sleep
b) Exercise
c) Hyperglycemia
d) Ingestion of certain Amino Acids

502 ‫صفحة‬ Dr. MOHAMED YAHIA 0900987639


QUESTION BANK OF Dr. MOHAMED YAHIA

e) None of the Above

378. Prolactin Release is stimulated by the following except:


a) Sleep
b) TSH
c) TRH
d) Stress
e) Pregnancy
379. FSH in Males:
a) Stimulates Testosterone Secretion
b) Inhibits LH Secretion
T c) Stimulates Spermotogenesis
d) Inhibits Testosterone Secretion
e) Is activated by Inhibin

380. LH is released by:


T a) Anterior Pituitary
b) Posterior Pituitary
c) Hypothalamus
d) Corpus Luteum
e) Graffion Follicles

381. Early Features of Pituitary Failure include:


a) TSH Deficiency
b) Growth Hormone Deficiency
c) Thyroxine Deficiency
d) Cortisol Deficiency
T e) ACTH Deficiency

503 ‫صفحة‬ Dr. MOHAMED YAHIA 0900987639


QUESTION BANK OF Dr. MOHAMED YAHIA

382. Excess Growth Hormone in Adults causes:


a) Giantisim
b) Acromegally
c) Dwarfism
d) Hypoglycemia
e) Has No Effects
383. Excessive Growth Hormone leads to the following except:
a) Excessive Sweating
b) Greasy Skin
c) Acne
T d) Decreased Head Circumference
e) Diabetes Mellitus

384. Physiological Hyperprolactinemia can be caused by the following except:


a) Stress
b) Sleep
c) Suckling
T d) Prolactinoma
e) Pregnancy

385. Diabetes Insipidus is:


a) Excessive ADH
T b) Deficient ADH
c) Excessive Insulin
d) Deficient Insulin
e) None of the Above

386. The following cause Polyuria and Polydypsia except:


a) Head Injury

504 ‫صفحة‬ Dr. MOHAMED YAHIA 0900987639


QUESTION BANK OF Dr. MOHAMED YAHIA

b) Head Surgery
T c) Hypocalcemia
d) Hypokalemia
e) Chronic Renal Failure

387. The main source of Androgens in males is:


a) Adrenal Cortex
b) Adrenal Medulla
T c) Testis
d) Anterior Pituitary
e) Supra Renal Gland

388. The test for investigation of Adrenal Hypofunction is:


T a) Low Dose Dexo Methasone Test
b) Synacthen Test
c) Insulin Test
d) Glucose Tolerance Test
e) None of the Above

389. In Addison's Disease:


a) Hypertension
b) Gain of Weight
c) Bruises
T d) Hyperpigmentation
e) Hyperglycemia

390. Causes of Secondary Adrenal Hypofunction:


a) Autociuvene Adrenalitis
b) T.B of Adrenals

505 ‫صفحة‬ Dr. MOHAMED YAHIA 0900987639


QUESTION BANK OF Dr. MOHAMED YAHIA

c) Adrenal Ectomy
T d) Pituitary Tumors
e) Meningo-Coccal Septicemia

391. Concerning Addison's Disease:


a) There may be Hypotension
b) Plasma Renin is low
c) There is Hyponatremia
d) A and B
T e) A and C

392. Which of the following is a feature of Cardiovascular Syphillis?


T a) Dissecting Aortic Aneurysm
b) Mitral Stenosis
c) Hemopericardium
d) Mycotic Aneurysm of the Thoracic Aorta
e) Angina Pectoris

393. Active Hyperaemia of acute inflammation occurs in response of:


a) Histamine
b) Interleukin I
c) Prostacyclin
T d) Leukotriene B4
e) Selectins

394. Which of the following is a Hormone Secreting Tumor?

506 ‫صفحة‬ Dr. MOHAMED YAHIA 0900987639


QUESTION BANK OF Dr. MOHAMED YAHIA

a) Breast Carcinoma
b) Carcinoma of the Pancreas
c) Fibroadenoma of the Breast
T d) Bronchogenic Carcinoma
e) Carcinoma of the Cervix Uteri

395. Which of the following features are of Visceral Leishmaniasis, except:


a) Bone Marrow Failure
b) Weight Loss
c) Epistaxis
d) Splenomegaly
e) Pancytopenia

396. Which of the following is a complication of Cardiogenic Shock:


a) Chronic Pyelonephritis
T b) Acute Tubular Necrosis
c) Nodular Glomerulosclerosis
d) Renal Vein Thrombosis
e) Rapidly Progressive Glomerulonephritis

397. Which of the following tumors is likely to rise because of loss of Tumor
Suppressor Gene:
a) Infiltrating Ductal Carcinoma of Breast
b) Small Cell Anaplastic Carcinoma of the Lung
c) Retinoblastoma of the Eye
d) Cerebral Astrocytomy
e) Chronic Myeloid Leukemia

398. Which of the following is most likely to be found in a 30 year old female who
has SLE:

507 ‫صفحة‬ Dr. MOHAMED YAHIA 0900987639


QUESTION BANK OF Dr. MOHAMED YAHIA

a) Severe Atherosclerosis
b) Glomerular Immune Deposits
c) Amyloid Deposition
T d) Vascular Chronic Inflammation
e) Anti-Glomerular Basement Membrane Antibody

399. Which of the following is a cause of Lymphopenia:


a) Iron Deficiency
T b) HIV Infection
c) Splenectomy
d) Rubella Infection
e) Tissue Necrosis

400. Ghon Focus is associated with:


a) Emphysema
b) Chronic Bronchitis
c) Bronchiectasis
T d) Tuberculosis
e) Lobar Pneumonia

508 ‫صفحة‬ Dr. MOHAMED YAHIA 0900987639


QUESTION BANK OF Dr. MOHAMED YAHIA

‫شيت المجلس‬
1. A cell consistently seen in an abscess contents:
a. Histocytes
b. Lymphocytes
c. Neutrophil
d. Multinucleated giant cell
e. Plasma cell

2. Which one of the following is a phagocytic cell:


a. B lymphocytes
b. T lymphocyte
c. Kupffer's cell
d. Mesangial cell
e. Mast cell

3. One of the following statements is not true about apoptosis:


a. programmed cell death
b. involves individual cells
c. energy dependent
d. provokes an inflammatory response
e. suprest in neoplastic growth

4. Which one of the following is not a useful effect of acute inflammation:


a. accumulation of phagocytic cell
b. formation of fibrin
c. dilution of toxins
d. stimulation of the immune response
e. swelling of tissue

5. Least expected to be seen in a tissue exposed to radiation is:


a. double stranded chromosal breaks
b. formation of free radicals
c. abnormal mitotic figures
d. metaplastic change
e. endothelial swelling

6. Squamous metaplasia is least likely to be seen in:


a. nasal mucosa in Vit. A deficiency
b. bronchial mucosa in bronchietesis
c. synovium in rheumatoid arthritis
d. gall bladder mucosis in cholelithiasis
e. urinary bladder mucosa in schistosomiasis

509 ‫صفحة‬ Dr. MOHAMED YAHIA 0900987639


QUESTION BANK OF Dr. MOHAMED YAHIA

7. One of the following seriously interferes with healthy and repair of damaged
tissue:
a. age
b. sex
c. hypertension
d. immobilization
e. diabetes mellitus

8. Cloudy swelling reflects:


a. lysosomal disruption
b. increased cytplasmic water
c. description of endoplasmic reticulum
d. mitochondrial shrinkage
e. intracellular fat accumulation

9. Prostaglandin are:
a. products of arachidonic acid metabolism
b. inhibited by aspirin
c. inhibited by azothioprins
d. vasodilatation
e. important in the formation of mucus barrier in the stomach

10. The following may be associated with muscular atrophy Except:


a. denervation
b. ischemia
c. use of anabolic steroids
d. malnutrition
e. fixation in plaster of Paris in treating fractures

11. The following statements are true about malignant neoplasm Except:
a. infiltrate adjacent tissue
b. can metastasis
c. cause loss of weight
d. majority occur in children
e. some secrete ectopic hormone

12. which of the following is not a features of malignant neoplasms:


a. presence of cellular atypia
b. loss of polarity
c. presence of abnormal mitotic figures
d. low nuclear /cytoplasmic ratio
e. cell membrane irregularity

13. The following are about lysosomes Except:


a. are membrane – bound intracytoplasmic organelles
b. play a role in hereditary storage disease
c. are involved in detoxication of steroids
d. play a role in body defense mechanism
e. contain hydrolytic enzymes

510 ‫صفحة‬ Dr. MOHAMED YAHIA 0900987639


QUESTION BANK OF Dr. MOHAMED YAHIA

14. One of the following is not valuable in reporting an incisional biopsy from a
malignant tumor:
a. histological type of tumor
b. histological grading of tumor
c. extension of tumor to the lines of surgical excision
d. presence of necrosis
e. presence of vascular invasion

15. Which of the following are a stable cells:


a. Neurons
b. Hepatocytes
c. Histiocytes
d. Squamous cell
e. Neutrophils

16. Surgical staging contributes least to the management and prognosis of cancer of:
a. the breast
b. the bowel
c. the brain
d. the uterine cervix
e. the urinary bladder

17. Malignant tumors are least likely to cause:


a. cachexia
b. myocardial infarction
c. discriminated intravascular coagulation
d. hypercalcemia
e. peripheral neuropathy

18. Epithelioid cells in a tubercle granuloma are derived from:


a. mast cells
b. neutrophils
c. histocytes
d. langerhan's cells
e. lymphochytes

19. One of the following is a benign tumor:


a. nephroblastoma
b. lymphoma
c. leukemia
d. carcinoid tumor
e. melanoma

511 ‫صفحة‬ Dr. MOHAMED YAHIA 0900987639


QUESTION BANK OF Dr. MOHAMED YAHIA

20. Myocardial necrosis caused by sudden occlusion of a major branch of coronary


artery is best described as:
a. liquefaction necrosis
b. coagulation necrosis
c. fat necrosis
d. caseons necrosis
e. fibrinoid necrosis

21. Not a common cause of malena:


a. chronic duodenal ulcer
b. carcinoma of colon
c. chromic gastric ulcer
d. Crohn's disease
e. esophagealvarices

22. The best established factor promoting the migration of epidermal cells in wound
healing is:
a. high concentration of vit. C in the exudates
b. loss of contact inhibition
c. raised local temperature
d. secretion of local mitosis regulators (chalones)
e. secretion of wound hormones (trephones)

23. Healing of a fractured bone starts by:


a. cellus formation
b. migration of esteositis
c. transformation of asteoblasts into osteochytes
d. hematoma formation
e. transformation of fibroblasts into osteoblasts

24. Fat embolism commonly may lead to death by causing:


a. acute renal failure
b. hepatic decompensation
c. gastro-intestinal hemorrhage
d. adrenal infarction
e. pulmonary edema

25. Viruses have been implicated in the pathogenesis of the following human tumors
except:
a. nasopharyngeal carcinoma
b. Hodgkins disease
c. colonic carcinoma
d. hepatocellular carcinoma
e. cervical carcinoma

512 ‫صفحة‬ Dr. MOHAMED YAHIA 0900987639


QUESTION BANK OF Dr. MOHAMED YAHIA

26. The following are true about cell death Except:


a. by apoptosis is energy dependent
b. as a result of viral infection implies integration of viral DNA into host
DNA
c. in ischemia is associated with increased intracellular free calcium
d. induced by toxic T lymphocytes occurs through the action of soluble
mediators
e. is associated with loss of selective permeability

27. One of the following is not associated with disseminated intravascular


coagulation:
a. septicemia
b. von Willebrand's disease
c. carcinoma of the pancreas
d. amniotic fluid embolism
e. burns

28. The following compounds mediate increased vascular permeability in acute


inflammation Except:
a. C5a
b. Interleukin 6
c. Histamine
d. Leukotrienes C4 Da& E4
e. Prostaglandins

29. One of the following is not true with raised intracranial pressure:
a. leads to systemic hypertension
b. is associated with bradycardia
c. leads to papilledema
d. may cause brain stem hemorrhage
e. may lead to pulmonary edema

30. A "granuloma" is best defined as:


a. a bacterial infection of long standing
b. a chronic inflammatory lesion, usually involving some local swelling
c. a collection of granulocytes
d. a form of tuberculosis
e. a rare variety of malignant tumor seen chiefly n the nose

31. Final event in the healing of a fracture is:


a. formation of blood clot
b. formation of callus
c. provisional union of bone ends
d. migration of ostoblasts from bone ends into penostem
e. remodeling to restore normal structure

32. Minor loss of differentiation is the correct definition for:

513 ‫صفحة‬ Dr. MOHAMED YAHIA 0900987639


QUESTION BANK OF Dr. MOHAMED YAHIA

a. anaplasia
b. aplasia
c. dysplasia
d. heteroplasia
e. metaplasia

33. Major risk factors associated with development of coronary and generalized
atherosclerosis included all the following Except:
a. cigarette smoking
b. elevated high-density lipoproteins (HDL) levels
c. diabetes mellitus
d. hypertension
e. hypercholesterolaemia

34. Disorders that predispose to thrombosis include all the following Except:
a. pancreatic carcinoma
b. pregnancy
c. vitamin K deficiency
d. sickle cell anemia
e. severe burns

35. One of the following agents is not known to be carcinogenic:


a. Cyclophosphamide
b. Asbestos
c. Arsenic
d. saccharin
e. vinyl chloride

36. In hypoxic cell injury swelling of the cells occurs because:


a. intracytoplasmic lipids accumulate
b. intracytoplasmic protein accumulate
c. intracytoplasmic glycogen increase
d. intracytoplasmiclipofusin accumulate
e. water enters the cells

37. In immunologic reactions lymphocytes make substances (cytokines) that are


chemotactic for all of the following cells Except:
a. Neutrophuils
b. platelets
c. eosinophils
d. macrophages

38. All of the following neoplasms are malignant Except:


a. glomustumor
b. Ewing'stumor
c. Wilmstumor
d. Seminoma
e. Histiocytosis

514 ‫صفحة‬ Dr. MOHAMED YAHIA 0900987639


QUESTION BANK OF Dr. MOHAMED YAHIA

39. congenital malformations are associated with all of the following conditions
Except:
a. fetal chromosomal abnormalities
b. maternal thalidomide use
c. rubella infection in the first 8 weeks of pregnancy
d. cytomegalovirus infection in the second trimester of pregnancy
e. heavy cigarette smoking throughout pregnancy

40. Malignancies that commonly occur in children younger than 5 years of age
include all of the following Except:
a. Leukaemia
b. Wilmstumor
c. Retinoblastoma
d. Rhabdomyosarcoma
e. Osteosarcoma

41. All of the following conditions increase in incidence with obesity Except:
hypertension
a. diabetes mellitus
b. hyperlipidemia
c. rheumatoid arthritis
d. gall stones

42. All of the following statements are true about iron as a nutrient Except:
a. daily requirements are greater for young women than for young men
b. normally most dietary iron is not absorbed
c. in Western World deficiency in adult males is often due to gastro-intestinal
blood loss
d. deficiency causes a drop in the transferin iron-binding capacity
e. deficiency causes abnormal mitochondrial and cytochrome function

43. Immediate consequences of a first ischemic cardiac event include all of the
following Except:
a. angina pectoris
b. sudden death without infarction
c. trans-mural infarction
d. cardiac rupture
e. no symptoms

44. All of the following commonly contribute to postoperative atelectasis after


uncomplicated abdominal surgery Except:
a. adult respiratory distress syndrome
b. diaphragmatic elevation
c. voluntary suppression of coughing
d. excessive bronchial secretions
e. limitation of respiratory

515 ‫صفحة‬ Dr. MOHAMED YAHIA 0900987639


QUESTION BANK OF Dr. MOHAMED YAHIA

45. All of the following statements about disseminate intravascular coagulation are
true Except:
a. the disorder is characterized by wide-spread thrombosis
b. the disorder is characterized by wide-spread hemorrhage
c. it most often present as a primary (idiopathic) condition
d. the brain is the organ most often involved
e. the disorder is associated with muci-secretary adenocarcinoma

46. Cancer patients receiving medical treatment often develop eosophigitis from all of
the following Except:
a. antibiotic toxicity
b. viral infection
c. chemotherapeutic agent toxicity
d. radiation
e. fungal infection

47. Uremia is associated with all of the following abnormalities Except:


a. peripheral neuropathy
b. gastritis
c. polycythemia
d. pericarditis
e. diffuse alveolar damage

48. the role of extracellular matrix (ECM) includes:


a. determination of cell orientation
b. control of cell growth
c. maintenance of cell differentiation
d. all of the above
e. none of the above

49. Which of the following is the commonest cause of edema:


a. increased hydrostatic pressure
b. reduced plasma osmotic pressure
c. lymphatic obstruction
d. acute inflammation
e. sodium retention

50. Which of the following is the most confirmative feature of malignancy in size
a. progressive increase in size
b. infiltration of adjacent tissue
c. presence of metastasis
d. cellular pleomorphism
e. mitotic activity

51. In acute inflammation vasodilatation is the main cause of one of the following
cardinal signs:

516 ‫صفحة‬ Dr. MOHAMED YAHIA 0900987639


QUESTION BANK OF Dr. MOHAMED YAHIA

a. Pain
b. Swelling
c. Redness
d. Hotness
e. Loss of function

52. Which one of the following is a tumor of epithelial origin:


a. Hemangioma
b. Meningioma
c. Hepatoma
d. Melanoma
e. Astrocytoma

53. First to occur in cell injury is:


a. breaks in plasma membrane
b. depletion of ATP
c. loss of specialized processes
d. decreases metabolic activity
e. swelling and disruption of endoplasmic reticulum

54. The most serious effect t of ionizing radiation is:


a. cell necrosis
b. alopecia
c. genetic mutations
d. fibrosis
e. infertility

55. Hypoxia is an important and common cause of cell injury and death. Which of the
following is the commonest cause of hypoxia:
a. Ischemia
b. pneumonia
c. anemia
d. CO poisoning
e. high attitude

56. The following are true about the functions of the cytoskeleton Except:
a. intracellular transport for organelles and molecules
b. maintenance of basic cell structure
c. production of intracellular energy
d. cell mobility
e. conveying all cell and cell-extracellular matrix signals to the nucleus

57. Which of the following is related to dystrophic calcification:


a. renal failure
b. Paget's disease
c. Hyperparathyroidism
d. athroma
e. vitamin D-related disorders
58. One of the following statements is not true for prostaglandins:
a. they are products of arachidonic acid metabolism

517 ‫صفحة‬ Dr. MOHAMED YAHIA 0900987639


QUESTION BANK OF Dr. MOHAMED YAHIA

b. inhibited by aspirin
c. inhibited by azothioprens
d. are vasodilators
e. increase vascular permeability

59. The following conditions are associated with acquired defects in leukocyte
function, which one is most frequently seen:
a. thermal injury
b. diabetes mellitus
c. hemodialysis
d. leukemia
e. sepsis

60. The following are characteristic of chronic inflammation Except:


a. infiltration with mononuclear cells
b. tissue destruction
c. production of chemical mediators of inflammation
d. angiogenesis
e. fibrosis

61. Chronic inflammation arises in the following setting Except:


a. viral infections
b. persistent microbial infections
c. prolonged exposure to potentially toxic
d. exposure to ionizing radiation
e. auto-immune disease

62. The following are steps in the process of angiogenesis Except:


a. proteolytic degradation of the parent vessel basement membrane (BM)
b. migration of endothelial cells from original capillary towards an angiogenic
stimulus
c. formation of a thrombus at the site of migrating endothelial cells
d. proliferation of endothelial cells behind the leading edge of migrating cells
e. recruitment and proliferation of pericytes and smooth muscle cells

63. The following are high risk for thrombosis Except:


a. prolonged bed risk or immobilization
b. myocardial infarction
c. tissue damage (surgery, fracture, burns)
d. nephrotic syndrome
e. disseminate intravascular coagulation

64. Which of the listed categories of shock rank first as a cause of death in intensive
care units:
a. cardiogenic shock
b. hypovolemic shock

518 ‫صفحة‬ Dr. MOHAMED YAHIA 0900987639


QUESTION BANK OF Dr. MOHAMED YAHIA

c. septic shock
d. neurogenic shock
e. anaphylactic shock

65. Correct statements about oncogenic viruses include:


a. all of them contain envelop
b. they have a DNA step in their replication cycle
c. contain SSRNA
d. they are infectious to human
e. they spread by budding

66. Hepatitis A
a. is the most common type of viral hepatitis
b. is spread by the fecal–oral route
c. has a carrier state
d. virus causes direct damage to the liver
e. caused by DSRNA virus

67. The following associations are correct:


a. hepatitis A- long incubation
b. hepatitis A- carrier state
c. hepatitis B- chronic liver disease
d. hepatitis E- liver cancer
e. hepatitis D- acute icteric disease

68. The following statements about the hepatitis B virus are correct:
a. the whole virus is called the Dane particle
b. the virus replicates in the blood stream
c. HBsAg is produced by replication in nucleus
d. HBcAg is formed in the liver cell cytoplasm
e. the damage caused by it is the result of an immunological reaction to the
virus

69. Correct statements about serological changes in hepatitis B include:


a. HBc Ag is not usually seen in the blood
b. persistent HBeAg correlates with increased severity and infertility of the
disease
c. high tires of total anti-HBc suggests continuous viral replication
d. appearance of anti-HBe relates to chromic disease
e. high titersofIgM – anti – HBs suggests immunity

70. Viruses multiply in:


a. blood agar
b. Mcconkey agar
c. cooked meat medium

519 ‫صفحة‬ Dr. MOHAMED YAHIA 0900987639


QUESTION BANK OF Dr. MOHAMED YAHIA

d. tissue culture
e. serum slopes

71. The genetic material of viruses is composed of:


a. both DNA and RNA
b. either DNA or RNA
c. ribosomes
d. transfer RNA
e. MRNA

72. Extra cellular virus is called:


a. Plasmid
b. Prion
c. virion
d. capsid
e. peplomer

73. Spikes on the surface of viruses may be:


a. sex pili
b. fimbriae
c. hemagglutinins
d. flasella
e. ribosmomes

74. Viruses multiply by:


a. binary fission
b. mitosis
c. meiosis
d. a complex life cycle
e. producing conidia

75. HIV (Human immuno-deficiency virus) is:


a. adenovirus
b. enterovirus
c. retrovirus
d. rotavirus
e. myxovirus

76. Hepatitis B virus is transmitted by:


a. blood – sucking insects
b. droplets
c. blood transfusion
d. kissing
e. ice-cream

77. The following drug is used in the treatment of HIV infection:


a. zidovudin (AZT)
b. foscarnet
c. acyclovir

520 ‫صفحة‬ Dr. MOHAMED YAHIA 0900987639


QUESTION BANK OF Dr. MOHAMED YAHIA

d. ribanvirin
e. marboran

78. Metabolic features of IDDM


a. diabetic nephropathy
b. feature of glycation
c. low LDL
d. reduced FFA level
e. low urine p11

79. The following are causes of hyperlipidemia Except:


a. primary hypothyroidism
b. D.M.
c. nephrotic syndrome
d. thyrotoxicosis
e. ladies on the contraceptive pills

80. Indication for measuring plasma lipid include all Except:


a. Xanthelasma
b. Hypertension
c. D.M
d. family history of arterial disease
e. thyrotoxicosis

81. Control of H2O depends on:


a. water intake
b. reduced ADH secretion
c. reduced urine volume
d. release of rennin
e. urine PH

82. In hemophilia A which of the following tests is diagnostic :


a. prolonged BT
b. prolonged PT
c. prolonged APTT
d. low F VIII level
e. poor clot retraction

83. A hypochromic, microcytic blood picture is seen in:


a. severe megaloblasticanemia
b. sickle cell anemia
c.  - thalassaemia
d. sideroblasticanemia
e. anemia due to chromic disease

84. Thrombocytopenia is a known feature of:


a. acute malaria infection
b. brucellosis
c. chronic myeloid leukemia

521 ‫صفحة‬ Dr. MOHAMED YAHIA 0900987639


QUESTION BANK OF Dr. MOHAMED YAHIA

d. sickle cell anemia


e. prolonged steroid use

85. All are essential for hemopoiesis Except:


a. G.CSF
b. GM – CSF
c. Erythropoietin
d. IL-12
e. Vit. B12

86. In acute leukemia:


a. ESR is low
b. WBC count is invariably high
c. there is hyperuricemia
d. there is platelet dysfunction
e. there is iron deficiency

87. Fresh frozen plasma (FFP) is the best choice for:


a. platelet dysfunction
b. ITP
c. Hemophilia A
d. Liver dysfunction
e. Hypersplenism

88. The diagnostic test for homozygous SS disease (sickle cell anemia) is:
a. sickling test
b. peripheral film examination for sickle cells
c. reticulocyte count
d. Hb electrophoresis
e. urine examination for urobiliogen

89. A 20 years old man had pain around his Rt.knee, x-ray showed an osteolytic
lesion extending to subchondral bone. To diagnose GCT (Giant Cell Tumor) the
microscopic picture would show:
a. small round cells with indistinct cell boundary
b. spindle cells and giant cells
c. giant cells and plenty of cartilage cells
d. fibroblast and lymphocytes
e. reactive bone

90. Tuberculosis of the spine, early presentation in:


a. back pain
b. paraplegia
c. kyphos defects
d. marked weight loss and anemia
e. cold abscesses
91. The tuberculosis granuloma shows the following features EXCEPT:
a. caseating necrosis
b. bone destruction
c. abundance of reactive bone

522 ‫صفحة‬ Dr. MOHAMED YAHIA 0900987639


QUESTION BANK OF Dr. MOHAMED YAHIA

d. lymphocytes and plasma cells


e. giant cells with multiple nuclei

92. The following are common causes of paraplegia in T.B. spine EXCEPT:
a. compression caused by necrotic bone
b. abscess formation compressing the cord
c. disc prolapse
d. vascular occlusion due to inflammatory process
e. cord may be stretching over the apex of kyphos deformity

93. The following mechanisms are used by some parasites to evade attack by the
immune system EXCEPT:
a. antigenic disguise
b. development of capsules
c. resistance to ability of macrophage in killing
d. immunosuppression
e. antigenic variation

94. In anemic jaundiced patient the most probable diagnosis is:


a. severe iron deficiency
b. aplastic anemia
c. hepatitis C
d. pernicious anemia
e. sideroblasticanemia.

95. The most serious effect of anemia is:


a. fatty change in liver
b. chronic renal failure
c. splenomegaly
d. tissue hypoxia
e. malabsorption

96. Bone marrow examination is mandatory in:


a. pancytopenia
b. chronic myeloid leukemia
c. sickle cell anemia
d. severe megaloblasticanemia
e. chronic lymphocytic anemia

97. The following are known complications of sickle cell anemia EXCEPT:
a. aplastic crisis
b. hemolytic crisis
c. sequestration crisis
d. osteomyelitis
e. acute salpingitis

98. Thrombosis is not associated with:


a. vascular stasis
b. thrombocytopenia
c. activation of coagulation mechanism

523 ‫صفحة‬ Dr. MOHAMED YAHIA 0900987639


QUESTION BANK OF Dr. MOHAMED YAHIA

d. endothelial damage
e. formation of platelet aggregates

99. In severe injuries a patient with pelvic fracture may lose blood in the range of:
a. 1-4 units
b. 2-4 units
c. 2-6 units
d. 2-3 units
e. 1-3 units

100. An infected wound is best allowed to heal by:


a. Debridement and primary suture.
b. Debridement and secondary suture.
c. Drainage, debridement and allowed to heal by secondary intension.
d. Antibiotic therapy and daily dressings.
e. Debridement and delayed primary suture

101. The amount of IV fluids to be given to post operativesurgical patients is best


judged by :
a. The pulse rate.
b. Frequent BP measurements
c. Urine output.
d. Fluid balance charting.
e. The degree of dryness of the tongue.

102. The MOST likely mechanism of action of ciprofloxacin is inhibition of:


a. tetrahydrofolic acid synthesis
b. DNA synthesis
c. mRNA synthesis
d. aminoacyl transfer RNA
e. initiation complex (peptide bond and polysomeformation)

103. The most effective method for reducing the incidence of nosocomial infection
is:
a. laminar flow rooms
b. wearing surgical masks
c. immunization
d. giving antibiotics
e. hand washing

104. The most common cause of acute limb ischemia is :


a. Atherosclerosis.
b. Arterial thrombosis.
c. Arterial injury.
d. Arterial embolism.

524 ‫صفحة‬ Dr. MOHAMED YAHIA 0900987639


QUESTION BANK OF Dr. MOHAMED YAHIA

e. Diabetic angiopathy

105. In complete obstructive of the common bile duct the following is true except.
a. Increased direct serum bilirubin.
b. Increased indirect serum bilirubin.
c. Increased serum alkaline phosphatase .
d. Presence of uroblinogen in the urine.
e. Presence of bilirubin in the urine.

106. Nosocomial infection of wounds is commonly caused by:


a. Klebsiella
b. E. coli
c. Staph. aureus
d. Proteus vulgaris
e. Streptococcus faecalis

107. The following is least likely to affect healing:


a. protein calorie malnutrition
b. excessive administration of glucocorticoids
c. controlled diabetes mellitus
d. vitamin C deficiency
e. ultraviolet light

108. In acute inflammation which one of the following events occurs first?
a. increased vascular permeability
b. leukocyte cellular parementation
c. chemotaxispavementation
d. vasodilatation
e. leukocyte cellular aggregation

109. Which of the following conditions is not associate with defects in leukocyte
function:
a. diabetes mellitus
b. sepsis
c. hemodialysis
d. leukemia
e. leishmaniasis

110. The most characteristic feature of granulation tissue is:


a. the presence of neutrophils and monocytes
b. formation of new capillaries
c. presence of epithelial cells and new capillaries
d. presence of foreign body giant cells
e. presence of epitheloid cells
111. Chronic inflammation is associated with the following EXCEPT:
a. increased plasma globulin level
b. exudation more than proliferation
c. mononuclear cellular infiltrate
d. persistence of causative agent
e. suppuration

525 ‫صفحة‬ Dr. MOHAMED YAHIA 0900987639


QUESTION BANK OF Dr. MOHAMED YAHIA

112. Which of the following is not a fundamental physiological change in


malignancy:
a. evasion of apoptosis
b. limitless replication potential
c. successful DNA repair
d. sustained angiogenesis
e. ability to invade and metastasize

113. Paraneoplastic syndromes do not include:


a. hypercalcaemia
b. myasthenia gravis
c. anemia
d. thrombocytopenia
e. Cushing syndrome

114. Escherichia coli is known to cause the following EXCEPT:


a. hospital acquired urinary tract infection
b. neonatal septicemia
c. acute watery diarrhea
d. non-gonococcal urethritis
e. acute cholycystitis

115. In gas gangrene the following factors are important EXCEPT:


a. dead muscles
b. ischemia
c. clostridia welchii
d. leaving the wound open
e. closing the wound after good washing

116. The sample needed for acid-base balance evaluation is:


a. whole blood venous sample
b. plasma
c. serum
d. arterial blood
e. capillary blood

117. Natural antibodies are produced by:


a. B1 lymphocytes
b. B2 lymphocytes
c. Pre-lymphocytes
d. Plasma cells
e. Thymocytes

118. One of the following is not considered a pattern of cellular adaptation:


a. hypoplasia
b. metaplasia
c. anaplasia
d. hypertrophy
e. atrophy

526 ‫صفحة‬ Dr. MOHAMED YAHIA 0900987639


QUESTION BANK OF Dr. MOHAMED YAHIA

119. Which of the following statements about staphylococcus areus is correct?


a. it cannot withstand drying
b. it can grow aerobically but not anaerobically
c. infection are not usually by hand from colonized sites in the nose, axilla or
perineum
d. carriage is lower in persons with dermatologic disease than normal individuals
e. phage typing (of stasis) is useful in defecting strains causing epidemic in
hospital

120. Serum analysis for drug concentration is necessary for monitoring treatment
with:
a. chloramphenicol
b. ampiclox
c. gentamycin
d. fucidic acid
e. clavulonic acid

121. All of the following organisms are associated with hospital wound infection
EXCEPT:
a. Staphylococcus aureus
b. Coliforms
c. Β-hemolytic streptococci
d. d-hemolytic streptococci
e. clostridium perfringens

122. Amyloid is least likely to be deposited in:


a. liver
b. spleen
c. brain
d. kidney
e. thyroid

123. Widespread edema is most likely caused by:


a. lymphatic obstruction
b. capillary damage
c. inflammation
d. hypoalbuminemia
e. urticaria

124. The best test for investigating long-term diabetic control in type 2 diabetes is:
a. fasting plasma glucose
b. GTT
c. Plasma insulin
d. Hb A1C
e. pancreatic β-cell antibodies

125. The metabolic response after major surgery includes which of the following?
a. hypoglycemia

527 ‫صفحة‬ Dr. MOHAMED YAHIA 0900987639


QUESTION BANK OF Dr. MOHAMED YAHIA

b. water deficit
c. hyperkalemia
d. tendency to ketosis
e. saline responsive metabolic alkalosis

126. Which of the following microorganisms the ATE surgeon can have vaccine
for:
a. hepatitis A virus
b. hepatitis B virus
c. hepatitis C virus
d. HIV virus
e. Rubella virus

127. Which of the following cells produce IgE:


a. eosinophils
b. mast cells
c. osteoclasts
d. basophils
e. plasma cells

128. Liquefaction necrosis occurs after infarction of:


a. heart
b. kidney
c. brain
d. liver
e. spleen

129. Healing by first intention is characterized by:


a. presence of large tissue defect
b. wound contraction
c. abundant necrotic debris
d. marked angiogenesis
e. abundant granulation tissue formation

130. A major characteristic of mycobacterium tuberculosis is:


a. penicillinase production
b. requirement for oxygen
c. rapid growth
d. acid fastness
e. lipopolysaccharide antigen

131. Prophylactic antibiotics are used in the following situations EXCEPT:


a. normal delivery
b. when operating on gastrointestinal tract
c. in repairing fistulae

528 ‫صفحة‬ Dr. MOHAMED YAHIA 0900987639


QUESTION BANK OF Dr. MOHAMED YAHIA

d. when operating in the central nervous system


e. when operating on joints

132. In hypovolemic shock the most appropriate intravenous solution to give is:
a. 5% dextrose in water
b. dextrose saline
c. dextran
d. ringer's lactate solution
e. Darrow's solution

133. Which of the following is not used as a tumor marker:


a. carcino embryonic antigen (CEA)
b. prostate specific antigen (PSA)
c. C-reactive protein
d. thyroglobulin
e. monoclonal antigen

134. Septic shock is least likely to by caused by:


a. Pseudomonas aeroginosa
b. Escherichia coli
c. Proteus esp.
d. Corynebacteriumdiphtheriae
e. Klebsiellapneumoniae

135. A prolonged PTT with normal PT is seen in deficiency of:


a. F VII
b. FII
c. F VIII
d. F XIII
e. F XI

136. Massive blood transfusion is defined as:


a. 5 pints in 24 hours
b. half blood volume in 24 hours
c. one blood volume in 24 hours
d. one blood volume in 48 hours
e. twice blood volume in 24 hours

137. The least complication of massive blood transfusion is:


a. hyperkalemia
b. hypercalcemia
c. bleeding tendency
d. pulmonary edema
e. leukopenia

138. Labile cells are found in:


a. liver
b. brain
c. bone marrow
d. heart

529 ‫صفحة‬ Dr. MOHAMED YAHIA 0900987639


QUESTION BANK OF Dr. MOHAMED YAHIA

e. renal tubules

139. Signet – ring cells are characteristic of:


a. hypernephrone
b. myeloma
c. mucus-producing carcinoma
d. fibrosarcoma
e. leiomyoma

140. Edema is not expected to occur with:


a. constriction of post-capillary venules
b. hypoalbuminemia
c. arterular vasoconstriction
d. activation of bradykinin
e. lymphatic obstruction

141. Clinical manifestations of cancer may not include:


a. myopathies
b. venous thrombosis
c. xerodermapigmentosa
d. chachexia
e. lymphatic metastasis

142. One of the following is not a response in acute inflammation:


a. active hyperemia
b. early migration of histocytes
c. activation of compliment
d. formation of transudate
e. local edema

143. Fatty change does not occur in:


a. liver
b. kidney
c. breast
d. brain
e. heart

144. Malignant cells characteristically show the following EXCEPT:


a. increased nuclear/cytoplasmic ratio
b. chromosomal abnormalities
c. regular cell membrane
d. mitotic activity
e. abnormalities of the P53 gene

145. In cell injury one of the following is not true:


a. morphologic changes are due to biochemical derangement
b. myocardial cells are most susceptible to injury
c. nuclear changes are due microsomal enzyme activity
d. severity of cell damage is partly related to duration of insult

530 ‫صفحة‬ Dr. MOHAMED YAHIA 0900987639


QUESTION BANK OF Dr. MOHAMED YAHIA

e. cloudy swelling is a reversible change

146. First to occur in atherosclerosis is:


a. fragmentation of internal elastic lamina
b. deposition of lipid in the smooth muscle cells
c. collagen formation
d. calcification
e. necrosis

147. The most influential factor in prognosis in malignant tumors is:


a. histological grade of tumor
b. clinical stage of the tumor
c. hormone dependence
d. mitotic index
e. site of origin

148. The most frequent cause of intra-cerebral hemorrhage is:


a. ruptured aneurysm
b. trauma
c. blood dyscreaseas
d. angiomas
e. hypertension vascular disease

149. The epitheloid cells of a tubercle are derived from:


a. Polymorph-nuclear leukocytes
b. mononuclear phagocyte
c. mast cells
d. Langerham's cell
e. lymphocytes

150. Deoxyribonucleic acid (DNA) is found in:


a. the nucleus
b. the chromosomes
c. both chromosomes and nucleus
d. cytoplasm
e. nuclear membrane

151. In acute inflammation the main means (way) of increased escape of plasma
from capillaries and venules is:
a. fenestration of basement membrane
b. gaps between endothelia cells produced by active contraction of cells
c. necrosis of short stretch of the wall
d. reduction of charges on muco-polysaccharides receptors in the basement
membrane
e. transfer throughout endothelial cells by enlarged pinocytotic vesicles

152. The gross texture and resilience of a given tumor are largely influenced by:
a. degree of malignancy
b. type of epithelium present
c. presence or absence connective tissue stroma

531 ‫صفحة‬ Dr. MOHAMED YAHIA 0900987639


QUESTION BANK OF Dr. MOHAMED YAHIA

d. interface between normal tissue and tumor


e. relative blood supply

153. The involution of the thymus that occurs with aging is an example of the
process of programmed cell death, best described as:
a. pyknosis
b. karyorexia
c. apoptosis
d. karyolysis
e. pinocytosis

154. The following cells are not capable of regeneration:


a. hepatocyts
b. colonic mucosal epithelial cells
c. renal tubular cells
d. myocardial muscle cells
e. vascular endothelial cells

155. Venous dissemination is most characteristic of:


a. carcinoma of the stomach
b. fibro-adenoma of breast
c. gynecomastia
d. osteogenic sarcoma
e. giant cell tumor of bone

156. Cancer cells derive their energy primarily from:


a. aerobic glycolysis
b. utilizing nitrogen
c. utilizing Co2
d. utilizing Co
e. anaerobic glycolysis

157. Rosette formation is characteristic of:


a. neuroblastoma
b. thecoma
c. lymphosarcoma
d. ganglioneuroma
e. astrocytoma

158. Which one of the following is not an acceptable characteristic of a granuloma?


a. composed of altered macrophages
b. composed of fused macrophages (giant cells)
c. composed of epithelial cells
d. composed of mixture of chronic inflammatory cells
e. composed of polymorph nuclear leukocytes, cellular debris, fibrin

159. the role of extracellular matrix (ECM) includes:

532 ‫صفحة‬ Dr. MOHAMED YAHIA 0900987639


QUESTION BANK OF Dr. MOHAMED YAHIA

a. determination of cell orientation


b. control of cell growth
c. maintenance of cell differentiation
d. transmission of intracellular signals
e. transmission of intercellular signals

160. Osteosclerotic bone metastasis is seen mostly with cancer of:


a. gut
b. prostate
c. lung
d. thyroid
e. kidneys

161. Tumor metastasis in bone most likely seen in carcinoma of:


a. cervix
b. stomach
c. prostate
d. colon
e. ovary

162. The presence of which of the following is helpful in the diagnosis of


ankylosing spondylitis:
a. female patient
b. serum ANA
c. RF in serum
d. HLA-B27
e. HLA B28

163. Surgical equipment and linen are usually sterilized by:


a. hot air oven at 160ºC
b. glutaraldehide
c. 20% formaldehyde
d. autoclave at 120ºC
e. ethylene oxide
164. Which of the following complications is most likely to occur in a 76 years old
women who has a swollen leg following fracture of femoral trochanter?
a. gangrenous necrosis of the foot
b. hematoma of the thigh
c. disseminated intravascular coagulation (DIC)
d. pulmonary thromboembolism
e. soft tissue neoplasm

165. Hemarthrosis is the main feature in:


a. thrombasthemia
b. I.T.P
c. fibrinogen deficiency
d. factor VIII deficiency
e. Von Willebrand disease

533 ‫صفحة‬ Dr. MOHAMED YAHIA 0900987639


QUESTION BANK OF Dr. MOHAMED YAHIA

166. Which of the following is a common site of secondary deposits from


malignant tumors:
a. spleen
b. cartilage
c. liver
d. eye
e. striated muscle

167. The presence of calcification in the soft tissue of the upper arm following
severe blunt trauma is best described as:
a. metaplasia
b. hyperplasia
c. hypertrophy
d. dystrophy
e. dysplasia

168. An example of intraepithelial spread of malignant tumor is shown in:


a. malignant melanoma
b. keratoacanthoma
c. basal cell carcinoma
d. Paget's disease of the nipple
e. Squamous cell carcinoma

169. An infected wound is best treated by:


a. debridement and primary suture
b. debridement and secondary suture
c. drainage, debridement and tertiary suture
d. drainage, debridement and allowed to heal by secondary intention
e. antibiotic therapy and daily drainage

170. A solitary nodule with signs of thyrotoxicosis is most likely to be:


a. a papillary carcinoma
b. follicular cell carcinoma
c. an adenoma
d. a colloid cyst
e. a dominant nodule of a multinodulargoiter

171. An abscess is defined as:


a. a circumscribed collection of neutrophil with necrotic cellular debris
b. a localized defect that result from sloughing of necrotic tissue
c. a localized proliferation of fibroblasts and small blood vessels
d. an aggregate of macrophages
e. subcutaneous cellulites

172. Testicular torsion occurs when there is:


a. occlusion of the testicular artery

534 ‫صفحة‬ Dr. MOHAMED YAHIA 0900987639


QUESTION BANK OF Dr. MOHAMED YAHIA

b. venous occlusion
c. epididmoorchits
d. occlusion of the cremastric artery
e. thrombosis of the pampino form plexus

173. Surgical instruments are best sterilized by:


a. hot air oven
b. a solution of formalin
c. a solution of dettol
d. auto-claving
e. water boiling

174. Cystosarcomaphalludes of the breast:


a. is a truly benign tumor of the breast
b. has a fibrous capsule
c. recurs after excision
d. is a vascular
e. infiltrates adjacent breast tissue

175. In intestinal obstruction with vomiting the most grave sequale is:
a. dehydration
b. hemo-concentration
c. hypokalemia
d. hyponatremia
e. hypocalcemia

176. Lymphatic spread of carcinoma of the breast to the contra-lateral breast is


through
a. submammary lymphatic plexuses
b. intramammary lymphatic plexus
c. intra-axillary lymphatic plexus
d. intrmammary lymphatic plexus
e. internal mammary lymphatic plexus
177. pleomorphic adenoma of the parotid gland:
a. has an anatomical capsule
b. best treated by enculation
c. infiltrates the facial nerve
d. has a surgical capsule
e. occurs commonly on the deep parotid lobe

178. the most serious sequelae of plummer–vinson syndrome:


a. iron deficiency
b. glossitis
c. occurrence of esophageal mucosal webs
d. angular stomatitis
e. kolynichia

179. The most grave sequelae of acute pancreatitis is:


a. increased serum analysis
b. presence of pancreatic ascites

535 ‫صفحة‬ Dr. MOHAMED YAHIA 0900987639


QUESTION BANK OF Dr. MOHAMED YAHIA

c. pleural effusion
d. pancreatic necrosis
e. occurrence of a pseudo cyst

180. One of the following is a benign tumor:


a. Nephroblastoma
b. Lymphoma
c. leutioma
d. carcinoid tumor
e. melanoma

181. The last sign of acute inflammation to be described is:


a. Pain
b. Swelling
c. loss of function
d. redness
e. hotness

182. Atherosclerosis is characterized by all the following EXCEPT:


a. onset in midlife
b. formation of essential lesion in the intima
c. disintegration of the internal elastic lamina in advanced lesions
d. relatively numerous lesions in large arteries and fewer in smaller arteries
e. plaque formations that cause little reduction in luminal size of large
arteries

183. A 59old male is found to have a 3.5-cm mass in the right upper lobe of his
lung a biopsy of this mass is diagnosed as moderately differentiated squamous cell
carcinoma. Work up reveal no bone metastasis, but laboratory examination
revealed calcium level of 11.5 mg/dL, this patient paraneoplastic syndrome is
most likely the result of ectopic production of:
a. parathyroid hormone
b. parathyroid hormone related peptide
c. calcitonin
d. calcitonin related peptide
e. erythropiotin

184. Which one of the listed statements best describes chemotaxis:


a. abnormal fusion of phagosomes to primary lysosomes
b. attachment of chemicals to extracellular material to increase phagocytosis
c. dilation of blood vessels by chemotherapeutic drugs
d. movement towards a certain sites or source
e. transmigration of cells from blood vessels into tissue

536 ‫صفحة‬ Dr. MOHAMED YAHIA 0900987639


QUESTION BANK OF Dr. MOHAMED YAHIA

185. After receiving incompatible blood, a patient develops transmission reaction in


the form of back pain, fever, shortness of breath and hematuria. This type of
immunologic reaction is classified as:
a. systemic anaphylactic reaction
b. systemic immune complex reaction
c. delayed type hypersensitivity reaction
d. complement-mediated cytotoxicity reaction
e. T cell- mediated cytotoxicity reaction

186. Which one of the following is the most common congenital heart defect that
cause an initial left-to-right shunt:
a. tetraology of fallot
b. coarctation of the aorta
c. ventricular septal defect
d. atrial septal defect
e. patent ductusarteriosus

187. An elderly patient who was waiting for a thoracotomy became acutely short of
breath, hypotensive with elevated jugular venous pressure and muffled heart
sound. This triad of signs is most suggestive of:
a. chronic pericarditis
b. chronic pericardial effusion
c. cardiac tamponade
d. dissecting aortic aneurysm
e. right heart failure

188. Which one of the listed substances is secreted by malignant plasma cells in
individuals with multiple myeloma and is the osteoclast-activating factor (OAF)
that produces the characteristic lytic bone lesion?
a. interleukin 1
b. interleukin 6
c. tumor necrosis factor β
d. transforming growth factor β
e. platelet-derived growth factor

189. A 7-year-old boy accidentally inhales small peanut-which lodges in one of his
bronchi. A CXR showed mediastinal shifting towards the site of obstruction. The
best description for the lung changes that result from this obstruction is:
a. absorptive atelectasis
b. compression atelectasis
c. contraction atelectasis
d. patchy atelectasis
e. hyaline membrane disease

190. Histologic sections of lung tissue from an individual with adult respiratory
distress syndrome (ARDS) are most likely to reveal:
a. angioinvasive infiltrates of pleomorphic lymphoid cells

537 ‫صفحة‬ Dr. MOHAMED YAHIA 0900987639


QUESTION BANK OF Dr. MOHAMED YAHIA

b. deposits of needle-like crystals from the membranes of eosinophils


c. infiltrate in groups of inflammatory cells having intercellular bridges
d. irregular membranes of edema, fibrin, and dead cells lining alveoli
e. plexiform lesions within pulmonary arterioles

191. What type of cyst is characteristically located in the lateral or anterolateral


portion of the neck, is derived from the pharyngeal apparatus,
a. branchial cleft cyst
b. dentigerous cyst
c. odontogenickeratocyst
d. radicular cyst
e. thyroglossal duct cyst

192. A 49-year-old female presents with increasing problems of swallowing food,


x-ray studies with contrast reveal that she has a markedly dilated esophagus above
the level of the lower esophageal sphincter (LES). No lesions are seen within the
lumen of the esophagus. This patient's symptoms are most likely caused by:
a. decreased LES resting pressure
b. absence of myenteric plexus in the body of esophagus
c. absence of myenteric plexus at the LES
d. absence of submucosal plexus in the body of esophagus
e. absence of submucosal plexus at the LES

193. Familial polyposis coli is characterized by:


a. autosomal recessive pattern of inheritance
b. multiple hamartomatous polyps throughout the colon
c. 100% risk of carcinoma
d. an association with fibromotosis and multiple osteomas
e. an association with tumor of central nervus system

194. The Astler-Coller modification of the Duke's classification is used to classify


cancers of :
a. colon
b. liver
c. lung
d. pancreas
e. stomach

195. An 18-year-old woman presents with abdominal pain localized to the right
lower quadrant, nausea and vomiting, mild fever an elevation of the peripheral
leukocyte count to 17.000. examination of the surgically resected appendix is
most likely to reveal:
a. an appendix with a normal appearance
b. neutrophils within the muscular walled
c. lymphoid hyperplasia and multinucleated giant cells within the muscular
walled

538 ‫صفحة‬ Dr. MOHAMED YAHIA 0900987639


QUESTION BANK OF Dr. MOHAMED YAHIA

d. a dilated lumen filled mucus


e. a yellow tumor nodule at the tip of the appendix

196. A 44-year-old male presents with sudden of severe right upper quadrant
(RUQ), abdominal pain, temper hepatomegaly and hematemesis. These symptoms
are suggestive of Budd-Chiari syndrome, a disorder that is caused by:
a. obstruction of common bile duct
b. obstruction of intrahepatic sinusoids
c. thrombosis of hepatic artery
d. thrombosis of hepatic vein
e. thrombosis of portal vein

197. The combination of episodic elevation of serum transamenase level along with
fatty change in hepatocytes is most suggestive of infection with:
a. hepatitis A virus
b. hepatitis B virus
c. hepatitis C virus
d. hepatitis D virus
e. hepatitis E virus

198. Which one of the following tumors is most likely to be associated with
primary sclerosing cholangitis:
a. adenocarcinoma of gall bladder
b. adenocarcinoma of pancreas
c. cholangiocarcinoma
d. hepatoblastoma
e. hepatocellular carcinoma

199. A 55-year-old female with painful chronic diarrhea, multiple recurrent


duodenal ulcer, and increased basal gastric acid output is most likely to have:
a. a gastrin – secreting tumor of the pancreas
b. serotonin tumor of the ileum
c. a somatostatin secreted tumor of duodenum
d. an epinephrine tumor of adrenal medulla
e. an erythropoietin secreting tumor of liver

200. Which is the most aggressive variant tumor of the breast:


a. colloid carcinoma
b. comedo carcinoma
c. infiltrating ductal carcinoma
d. infiltrating lobular carcinoma
e. lobular carcinoma in situ

201. The part of a long bone initially involved in hematogenous osteomyelitis is


the:

539 ‫صفحة‬ Dr. MOHAMED YAHIA 0900987639


QUESTION BANK OF Dr. MOHAMED YAHIA

a. metaphyseal region
b. diaphsysis
c. epiphysis
d. area around entrance of the nutrient artery
e. medullary cavity

202. Which one of the listed abnormalities is most likely to produce a spinal cord
lesion that destroys both bone and the disk space (cartilage):
a. metastatic carcinoma
b. multiple myeloma
c. non-Hodgkin's lymphoma
d. syringomyelia
e. tuberculosis

203. Aminoglycides are:


a. bactericidal
b. active staphylococci
c. active against anaerobic bacteria
d. quickly absorbed from gut
e. may have toxic effects

204. Which of the following antibiotics is most toxic to humans


a. trimethoprim
b. rifampicin
c. polymyxin
d. sulphamethoxazole
e. penicillin

205. A young man who developed systemic fungal infection was advised to avoid
antifungal agents with renal toxicity because he has history of renal disease.
Which of the following drugs should be avoided?
a. 5-fluocytosine
b. Amphotericin B
c. Ketocorazole
d. Mycostatin
e. Vonconazole

206. Methicilian resistant staphylococcus aureus (MRSA) is best treated with:


a. benzyle penicillin
b. fluocloxacillin
c. tetracycline
d. vancomycin
e. trimethoprim

207. More than 50% of nonspecific urethritis is caused by:


a. Neisseria gonorrhoea
b. Ureaplasmaurealyticum

540 ‫صفحة‬ Dr. MOHAMED YAHIA 0900987639


QUESTION BANK OF Dr. MOHAMED YAHIA

c. Chlamydia trachomatis
d. Trichomorasvaginalis
e. Group B streptococci

208. The origin of organisms causing urinary tract infection is mostly:


a. exogenous
b. zoonotic
c. endogenous
d. environmental
e. anthropophylic

209. The following organisms may cause septic arthritis EXCEPT:


a. streptpyogens
b. staph aureus
c. streptococcus pneumoniae
d. streptococcus viridans
e. mycoplasma

210. Which one of the following organisms causes gas gangrene?


a. Streptococcus pyogenes
b. Pseudomonas aerugenosa
c. Clostridium perfringens
d. Bacillus anthracis
e. Staphylococcus aureus

211. Which of the following organisms can produces an exotoxin that causes
myocarditis and pseudomembrane at the throat?
a. Corynebacteriumdiphtheriae
b. Streptococcus pyogenes
c. Bordetelia pertussis
d. Salmonella typhi
e. b and c only

212. Which of the following organisms is associated with the clinical condition
called pseudomembranous enterocolitis following long abuse of antibiotics?
a. Staphylococcus aureus
b. Bacillus cereus
c. Clostridium difficile
d. Pseudomonas aerugenosa
e. Group B hemolytic streptococcui

213. Which of the following organisms is the most recovered in hospital acquired
infections?
a. E. coli
b. S. aureus

541 ‫صفحة‬ Dr. MOHAMED YAHIA 0900987639


QUESTION BANK OF Dr. MOHAMED YAHIA

c. Streptococcus pneumoniae
d. Mycoplasma pneumonia
e. Pseudomonas aerugenosis

214. Which of the following organisms is a menace to a hospital burn unit


a. Streptococcus pyogenes
b. Escherichia coli
c. Enterococcus faecalis
d. Pseudomonas aeruginosa
e. Haemophilusinfluenzae

215. HBs antigen is found in the following EXCEPT :


a. shared syringes
b. saliva
c. blood
d. water
e. semen

216. Epstein – Barr virus has been associated with:


a. acquired immune deficiency
b. hepatoma
c. nasopharyngeal carcinoma
d. papilloma
e. osteosarcoma

217. Presence of helicobacter pylori is identified by production of:


a. catalose
b. oxidase
c. urease
d. hydrogen sulphide
e. growth at 37˚C

218. Which of the following is an agent of actinomycetoma?


a. actinomycesisraelii
b. madurellamycetomatis
c. nocardiabrasiliensis
d. aspergillusnidulans
e. curvularialunata

219. The most likely hazard for both health personnel and patients in a dialysis
unities:
a. malaria
b. syphilis
c. hepatitis B
d. typhoid fever
e. herpes virus

542 ‫صفحة‬ Dr. MOHAMED YAHIA 0900987639


QUESTION BANK OF Dr. MOHAMED YAHIA

220. Soil contaminated wound will most likely change oxidation reduction
potential leading to growth of:
a. Escherichia coli
b. Clostridium spp
c. Staph aureus
d. Pseudomonas aeruginosa
e. Streptococcus faecalis

221. The most common virulence factor found in the gram-negative enteric
organisms is:
a. endotoxin
b. exotoxin
c. H-antigen
d. K-antigen
e. heavy capsule

222. In which of the following sites is the presence of anaerobes considered


abnormal?
a. mouth
b. blood
c. vagina
d. gut
e. skin

223. Several organisms considered commensals are causing disease nowadays. This
is mainly due to:
a. misuse of antibiotics
b. increased poverty
c. increased incidence of diabetes
d. increased immunocompromised patients
e. development of new strains

224. The organism of which of the following diseases is unable to produce spores?
a. Gasgangrene
b. anthrax
c. diphtheria
d. pseudomembraneous
e. tetanus

225. The smallest human intestinal tape worm is:


a. Taeniasaginata
b. Taeniasolium
c. D. latum
d. E. granulosus
e. H. nana

543 ‫صفحة‬ Dr. MOHAMED YAHIA 0900987639


QUESTION BANK OF Dr. MOHAMED YAHIA

226. The following parasite can cause intestinal obstruction:


a. E. vermicularis
b. T. trichiura
c. A. lumbricoides
d. T. saginata
e. T. solium

227. The commonest site of the hydatid cyst is the:


a. spleen
b. bone marrow
c. lungs
d. brain
e. subcutaneous tissues

228. A drug that causes a false increase of serum creatinine in Sudan is:
a. fansidar
b. flagyl
c. cephalexin
d. mycostatin
e. suramin

229. Both serum ALP & GGT are elevated in:


a. cholestasis
b. Ca bronchus
c. placenta previa
d. Paget's disease of bone
e. hydatidiform mole

230. A mixed metabolic and respiratory acidosis occur in:


a. renal tubular acidosis
b. methanol poisoning
c. Ca bronchus secreting ACTH
d. cardiogenic shock
e. acetazolamide

231. Reactive hypoglycemia occurs in association with:


a. panhypopituitarism
b. retroperitoneal sarcomas
c. severe malaria
d. total gastrectomy
e. thyroid storm

232. Causes of secondary hypercholesterolemia include:


a. choleithiasis

544 ‫صفحة‬ Dr. MOHAMED YAHIA 0900987639


QUESTION BANK OF Dr. MOHAMED YAHIA

b. strawberry gallbladder
c. primary biliary cirrhosis
d. severe fatty change of liver
e. glycogen storage disease

233. Low K and high bicarbonate is seen in:


a. thiazide therapy
b. purgative users
c. over-treatment on respirator
d. hysterical over-breathing
e. carcinoma of the bronchus

234. Innate immunity in comparison to adaptive immunity is characterized by:


a. memory
b. lymphocytes are the main cells
c. phagocytes are the main cells
d. production of antigen recognition molecules
e. slow development (over 13 days)

235. The antibody molecule that is transferable through the placenta is:
a. IgG
b. IgA
c. IgM
d. IgD
e. IgE

236. The following antigen–antibody interaction is infact a physiological


interaction that occurs in vivo
a. ELISA
b. precipitation
c. western blot
d. RAST
e. gel immunodiffusion

237. The effector phase of the adaptive immune response is the phase when:
a. foreign antigen is first recognized
b. a clone of lymphocytes is expanded
c. expanded lymphocytes are differentiated
d. foreign antigen is eliminated by antibody
e. all above

238. HIV infects immune cells through which of the following cell-surface
molecules:
a. CD3
b. CD8
c. CD40
d. CD5

239. TH1 cytokines are predominantly associated with:


a. visceral leishmaniasis

545 ‫صفحة‬ Dr. MOHAMED YAHIA 0900987639


QUESTION BANK OF Dr. MOHAMED YAHIA

b. tuberculoid leprosy
c. lepromatous leprosy
d. SLE
e. acute poststreptococcal glomerulonephritis

240. The defect in chronic granulomatous disease is:


a. defective intracellular killing of bacteria in neutrophils
b. low neutrophil course
c. poor neutrophil chemostasis
d. defective T cell cytotoxicity
e. defective opsonisation

241. Platelet transfusion is indicated in non-bleeding patients with:


a. chronic I.T.P.
b. hypersplenism
c. myelodysplastic syndrome
d. fibrinogen deficiency
e. a platelet count of 10.000
242. Chronic DIC is associated with:
a. abruptio placentae
b. snake bites
c. disseminated malignancy
d. uterine fibrosis
e. post-surgery

243. Chronic lymphocytic leukemia:


a. has a peak incidence in younger age groups
b. is primarily a T-cell disorder
c. is associated with the Philadelphia chromosome
d. commonly transforms into acute leukemia
e. may be asymptomatic

244. IgM + IgD are simultaneously expressed as antigen receptors on the follwing
cell:
a. macrophage
b. T lymphocyte
c. B lymphocyte
d. plasma cell
e. epithelial cell

245. Which of the following antibodies is the main constituent of pooled


intravenous immunoglobulins:
a. IgG
b. IgA
c. IgM
d. IgD
e. IgE

246. Transient hypogammaglobulinaemia of infancy usually:


a. presents at the first month in infants

546 ‫صفحة‬ Dr. MOHAMED YAHIA 0900987639


QUESTION BANK OF Dr. MOHAMED YAHIA

b. is more severe in premature babies


c. results from placental absorption of IgG
d. predispose to autoimmune disease in adulthood
e. is deficiency of IgG, IgA and IgM together

247. Which of the following is NOT a live vaccine:


a. vaccinia
b. BCG
c. sabin vaccine
d. hepatitis B
e. measles

248. The advantages of a live vaccine include the following EXCEPT:


a. single small dose often enough
b. given by natural route
c. stable vaccine
d. stimulates systemic and local immunity
e. stimulates natural infection
249. The pathogenesis requirements for immune complex-induced
glomerulonephritis include:
a. red blood cell and complement interaction
b. lymphocytes
c. neutrophils
d. kidney – derived antigen
e. large aggregated immune complex

250. Several types of immunologic rejection reactions can occur following organ
transplantation. The rejection reaction that is caused by the presence of preformed
antibodies in the recipient is referred to as:
a. acute
b. hyper acute
c. chronic immediate
d. accelerated

251. prevention of and recovery from cancer may involve all of the following
EXCEPT:
a. natural killer (NK) cells
b. macrophages
c. cytotoxic T cells
d. suppressor T cells
e. tumor – specific antibody

252. Which of the following procedures gives the most sensitive measure of
antibody?
a. precipitation
b. agglutination
c. radial immunodiffusion
d. radioimmunoassay
e. immunoelectrophoresis

547 ‫صفحة‬ Dr. MOHAMED YAHIA 0900987639


QUESTION BANK OF Dr. MOHAMED YAHIA

253. What is included in the indicator system of a complement fixation test?


a. specific antibody and complement
b. specific antigen and complement
c. sensitized red blood cells
d. the patients heat inactivated serum
e. Guinea pig serum

254. Regarding the p53 protein, one of the following statements is incorrect:
a. it is transcription factor
b. it is found only in cells that have undergone malignant transformation
c. it is a regulator of apoptosis
d. it suppresses cell division
e. a mutation of p53 in the primary defect in the Li Fraumeni syndrome

255. Which disease does not involve genomic imprinting?


a. Prader-Willi syndrome
b. Angelman syndrome
c. Beckwith-Wiedemann syndrome
d. Turner's syndrome
e. myotonic dystrophy

256. Bacterial causes of urinary tract infections are:


a. Eschericia coli
b. Staphylococcus aureus
c. Staphylococcus saprophyticus
d. Bacteriodesfragilis
e. Klebseilla

257. Eumycetoma due to Madurellamycetomatis is amendable to treatment with:


a. clotrimazole
b. ketoconazole
c. amphotericin B
d. itraconazole
e. griseofulvin

258. The following are caused by S. aureus EXCEPT:


a. scaled skin syndrome
b. glomerulonephritis
c. food poisoning
d. toxic shock syndrome
e. pemphigus vulgaris

259. Lancefield classification of streptococci depends on:


a. carbohydrate in the cell wall
b. haemolysin
c. leucocidin
d. all of the above

548 ‫صفحة‬ Dr. MOHAMED YAHIA 0900987639


QUESTION BANK OF Dr. MOHAMED YAHIA

e. b & c only

260. The following organisms are sensitive to penicillin EXCEPT:


a. streptococcus pyogenes
b. bacillus cereus
c. enerococcusfaecalis
d. staphylococcus epidermidis

261. The following organisms cause food poisoning EXCEPT:


a. bacillus cereus
b. enterococcus faecalis
c. staphylococcus aureus
d. clostridium perfringens

262. H. pylori can survive in the gastric mucosa because of the following EXCEPT:
a. urease activity generates ammonia which neutralizes gastric acid
b. possessing flagella sustains motility of the organism
c. microaerophilic conditions of the mucus gel
d. curved and spiral shape of the organism
e. nature of the stomach chyme

263. The most neonatal meningitis is caused by Streptococcus, it is usually:


a. Streptococcus pyogenes (group A)
b. S.agalactiae (group B)
c. S.sanguis (viridans streptococci)
d. Haemophilusinfluenzae
e. S.mitis (viridans streptococci)

264. Which of the following is best for preparing skin before surgery:
a. 80% alcohol
b. 5% chlorine (hypochlorite)
c. Tincture iodine
d. Iodophor
e. Hydrogen peroxide

265. Which of the following toxins affects the intestine only?


a. Bacillus cereus
b. Staphylococcus aureus
c. Clostridium tetani
d. Cl. Botulinm
e. Cl. Perfringens

266. The commonest method of transmission of hospital infection is:


a. through inhalation of contaminated air
b. transfusion of contaminated blood
c. through contact with contaminated hands
d. use of non-sterile surgical tools
e. ingestion of undercooked food

549 ‫صفحة‬ Dr. MOHAMED YAHIA 0900987639


QUESTION BANK OF Dr. MOHAMED YAHIA

267. The larvae of the following develop in the alveolar tissues:


a. Enterobiusvermicularis
b. Ancylostomaduodenale
c. Trichuristrichiura
d. Ascarislumbricoides
e. Strongyloidesstercoralis

268. There is pancytopenia in:


a. trypanosomiasis
b. toxaplasmosis
c. visceral leishmaniasis
d. intestinal schistosomiasis
e. hydatid cyst disease

269. Cell-mediated Immunity involves the following mechanisms:


a. lgG
b. T-lymphocytes
c. Eosinophils
d. Complement
e. IgE

270. Pulmonary edema may occur in patients suffering from:


a. major trauma
b. Osteosarcoma
c. right-sided heart failure
d. hypoproteinaemia
e. nematode infections

271. The following tumors may produce hormones except:


a. choriocarcinoma .
b. bronchial carcinoma
c. fibroma of the ovary
d. islet cell tumors of the pancreas
e. chromophobe pituitary adenoma

272. Gangrene is necrosis together with:


a. Desiccation
b. colliquative necrosis
c. involvement of a limb
d. infection of the tissues with Gram positive organisms
e. putrefaction

273. Hyperkalemia commonly occurs:


a. following severe buns
b. in Conn's syndrome
c. following nephroticsyndrome
d. in the Zollinger-Ellison syndrome
e. in the carcinoid syndrome

274. The following are true about Blood which is to be used for transfusion except:

550 ‫صفحة‬ Dr. MOHAMED YAHIA 0900987639


QUESTION BANK OF Dr. MOHAMED YAHIA

a. should be stored at 4C0


b. transmit malaria parasites
c. needs to be tested for complement content
d. may be used after storage for platelet replacement
e. should be stored in an acid anticoagulant

275. The following are mainly intracellular parasites except:


a. plasmodium faciparum
b. Leishmaniadonovani
c. Trypanosomagambiense
d. Plasmodium vivex
e. Toxoplasma Gondi

276. The healing of an incised wound is associated with the following EXCEPT:
a. a lag phase
b. a demolition phase
c. a proliferative phase
d. a contractile phase
e. B maturation phase

277. The plasma acid phosphatase concentration increases in :


a. Paget's disease (osteitisdeformans)
b. idiopathic hypercalciuria
c. prostatic cancer
d. medullary carcinoma of the thyroid
e. rickets

278. The following are true about anaphylaxis except :


a. develops 24 hours after the initial stimulus
b. causes a weal and flare response
c. is produced by IgEantibody
d. causes eosinophilia
e. causes degranulation of basophils and mast cells

279. Graft versus host disease:


a. may follow bone marrow transplants
b. may follow blood transfusion
c. can be suppressed by cyclosporins
d. can be suppressed by tetracyclines
e. can be suppressed by cyclophosphamide

280. The following coagulation factors are generated in the liver except:
a. Factor II
b. Factor VII
c. Factor VI
d. Factor IX
e. Factor X

281. Primary thyrotoxicosis is always accompanied by the following except:


a. increased iodine uptake by the gland

551 ‫صفحة‬ Dr. MOHAMED YAHIA 0900987639


QUESTION BANK OF Dr. MOHAMED YAHIA

b. a raised protein bound iodine


c. tachycardia
d. hypercalcemia
e. pernicious anemia

282. Hemolytic jaundice is associated with the following except:


a. an increase in the concentration of bilirubin diglucuronide in the bile
b. the absence of bilirubin in the urine
c. anincrease in the serumalkaline phosphatase
d. an increase in unconjugated bilirubin in the serum
e. an increase in urobilinogen in the urine

283. Gall stones are not associated with the following diseases:
a. contraceptive pills
b. cirrhosis of the liver
c. hereditary spherocytosis
d. obesity
e. raised serum triglycerides

284. The following are immunosuppressive drugs except :


a. Azathioprine
b. Vincristine
c. Adriamycine
d. Cyclophosphamide
e. Chlorpropamide

285. Antibiotics which inhibit the synthesis of mucopeptide in the wall of a


bacterium include:
a. cycloserine
b. cephalosporins
c. penicillin
d. semisynthetic-uerivatives of penicillin
e. erythromycin

286. Woven bone is found:


a. in bone forming in a model of cartilage
b. in diaphysis of long bones
c. in bones forming in sheets of differentiating mesenchyme
d. replacing lamellar bone in healing fractures
e. surrounding the ends of ununited fractures

287. The metabolic effects following a severe injury include the following except:
a. respiratory alkalosis
b. accelerated gluconeogenesis
c. mobilization of fat stores
d. decreased aldosterone secretion
e. protein catabolism

288. The commonest hemolytic disorder in the world is:


a. congenital spherocytosis

552 ‫صفحة‬ Dr. MOHAMED YAHIA 0900987639


QUESTION BANK OF Dr. MOHAMED YAHIA

b. disseminated lupus erythematosus


c. malaria
d. G6PD deficiency
e. sickle cell disease

289. When using an autoclave to sterilize surgical drapes and instruments the
following are essential except:
a. the load should be tightly packed
b. the containers in which the loads are packed should be pervious to steam
c. air should be completely removed from the chamber prior to the admission of
steam
d. a vacuum must be made at the end of the cycle
e. an adequate indication of autoclave efficiency should be included in the load

290. The following infections are common in immunodeficient patients except:


a. Pneumocystis carini
b. diphtheria
c. TB
d. Cytomegalovirus
e. Candida albicans

291. The compensatory mechanisms available to preserve the organism as a whole


in the 'shock state' Include:
a. increase CO2 in the blood
b. a fall in the PO2 of the blood
c. decreased pulmonary compliance
d. an increased sympathoadrenal discharge
e. hemoconcentration

292. Consecutive clot :


a. occurs in arteries distal to a thrombotic obstruction
b. occurs in the collateral branches of an artery followingobstruction to the main
vessel
c. occurs in veins after the cessation of blood flow
d. dose not extends proximally to the entrance of the next venous tributary
e. is formed of coralline thrombus

293. Acute heart failure occurs in the fallowing except:


a. in rheumatic fever
b. in diphtheria
c. in myxoedema
d. following myocardial infarction
e. in acute nephritis

294. The following are the chemical mediators involved in acute inflammation:

553 ‫صفحة‬ Dr. MOHAMED YAHIA 0900987639


QUESTION BANK OF Dr. MOHAMED YAHIA

a. complement
b. histamine
c. prostaglandins
d. bradykinin
e. lymphokine
295. Antibodies may be detected in vitro by the following except:
a. Precipitation
b. complement fixation
c. ELISA
d. lymphocyte transitiontest
e. RIA(radio-immunoassay)

296. The following belong to the mononuclear phagocyte system except:


a. Macrophages
b. Mast cells
c. Epithelioid cells
d. Monocytes
e. Kupffer'scells

297. The following is the chief characteristics of delayed hypersensitivity reactions:


a. the development of a polymorph-nuclearleukocyte infiltrate
b. the reaction has reached its maximum Intensity at 4 hours
c. an individual can be passively sensitized with serum
d. it is associated with T-Iymphocyte function
e. complement activation is an essential factor

298. For osteoporosis the following is true compared to osteomalacia:


a. the radiographic density of the skeleton is reduced in the former and not the
latter
b. the remaining bone in the former presents a normal histological
appearance
c. major changes occur in the epiphyses in the former
d. pseudo-fractures are commoner in the former than the tader
e. excess osteoid tissue is present in the former

299. Uretero-colic anastomosis is followed by the following except:


a. ascending pyelonephritis
b. absorption of ammonium salts
c. absorption of urea from the colon
d. metabolic alkalosis
e. hyperchloremia

300. Necrosis dose not occurs as a concomitant feature of chronic inflammation in:
a. leprosy
b. tuberculosis
c. syphilis
d. actinomycosis
e. coccidiomycosis

554 ‫صفحة‬ Dr. MOHAMED YAHIA 0900987639


QUESTION BANK OF Dr. MOHAMED YAHIA

301. Post operative infection delays wound healing because:


a. the wound becomes packed with leucocytes
b. many of the organisms involved produce spreading factors which may destroy
the interceIlular ground substance
c. collagen is destroyed
d. capillary loops fail to develop
e. fibroblasts are diminished in number

302. Hemorrhagic lesions may occur as a result of:


a. vitamin B deficiency
b. vitamin C deficiency
c. retinol deficiency
d. the nephrotic syndrome
e. penicillin therapy

303. The following infections may be successfully prevented by the administration


of a vaccine except:
a. herpes simplex
b. rabies.
c. hepatitis B
d. poliomyelitis
e. yellow fever

304. Septicemia is associated with the fallowing except:


a. bacteremia
b. toxemia
c. the multiplication of bacteria in the blood stream
d. invasion of the blood stream by organisms multiplying elsewhere, e.g. the
peritoneum
e. multiple hemorrhagic foci in the tissues

305. General factors predisposing to wound infection do not include the following
except:
a. uncontrolled diabetes
b. hypogammaglobulinaemia
c. low platelet count
d. agranulocytopenia
e. eosinophilia

306. Acute inflammation is not caused by:


a. Streptococcus pneumonia
b. Mycobacterium tuberculosis
c. Neisseria meningitides.
d. Mycobacterium leprae
e. Borreliavincentj

555 ‫صفحة‬ Dr. MOHAMED YAHIA 0900987639


QUESTION BANK OF Dr. MOHAMED YAHIA

307. In liver failure the following biochemical abnormalitiesmay be found except:


a. a decrease inthe plasma albumin concentration
b. an increase in the plasma globulin
c. an increase in the blood ammonium concentration
d. a rise in the blood urea
e. impaired glucose tolerance

308. Disseminated intravascular coagulation is a complication of the following


except:
a. many obstetrical complications
b. malignant disease
c. polycythemia vera
d. massive blood transfusion
e. endotoxemic shock

309. The findings of the following substances in excessive quantities in the blood
may be due to the presence of a specific type of tumor except:
a. Noradrenaline
b. 5-hydroxytryptemine
c. carcinoembryonic antigen
d. prostaglandins
e. calcium

310. Chromosome abnormalities may occur in the following except:


a. in Klinefelter's syndrome
b. Turner's syndrom
c. as a result of ionizing radiation
d. in Down's syndrome
e. in Christmas disease

311. Exfoliative cytology is useful for the diagnosis of in the following except:
a. CA uterus
b. broncnial cancer
c. multiple myeloma
d. cervical cancer
e. vesical cancer

312. The following tumors may secrete hormones except:


a. carcinoid tumors
b. choriocarcinoma
c. benign teratoma of the ovary
d. monodermalteratoma of the ovary
e. Seminoma

556 ‫صفحة‬ Dr. MOHAMED YAHIA 0900987639


QUESTION BANK OF Dr. MOHAMED YAHIA

313. Pus contains the following except:


a. Lipids
b. Fibrin
c. Breakdown products
d. Plasma cells
e. Polymorpho-nuclear leucocytes

314. Ototoxicity is not a well recognized complication following the administration


of:
a. Streptomycin
b. Gentamicin
c. Neomycin
d. Bacitracin
e. Kanamycine

315. The following infection is not associated with the development of cancer:
a. clostridial infections
b. HSV infection
c. EBV infection
d. HIV infection
e. schistosomiasis

316. An enlarged lymph node which is excised is found by histological


examination to be packed with tubercles. consisting of epithelioid cells and giant
cells. The tuberculin and Heaf test are negative. Which of the following diseases
should then be considered of the probable cause of the Iymphadenopathy:
a. Hodgkin's disease
b. Tuberculosis
c. Sarcoidosis
d. Syphilis
e. Toxoplasmosis

317. Pure water depletion in the surgical patient does not follow:
a. reduced intake of water
b. dysphagia
c. severe diarrhea
d. persistent fever
e. the development of diabetes insipidus.

318. The renin-angiotensin-aldosterone system regulates the following except:


a. potassium balance
b. sodium balance
c. fluid volume
d. blood pressure
e. nitrogen balance

319. Ischemic necrosis is a recognized complication of fractures of the following


bones except:
a. talus

557 ‫صفحة‬ Dr. MOHAMED YAHIA 0900987639


QUESTION BANK OF Dr. MOHAMED YAHIA

b. calcaneum
c. scaphoid
d. capitulum
e. femoral head

320. The following type of immunity provides immediate but short-lived


protection:
a. active
b. passive
c. adaptive
d. cellular
e. phagocytosis

321. The part of an immunoglobulin molecule that passes biological functions but
no antibody function is:
a. CHI domain
b. CL domain
c. fab end
d. FC end
e. Both CHI and CL

322. The function of interferon is:


a. enzymatic
b. pyrogenic
c. induction of resistance to avoid infection
d. enhancing phagocytosis
e. antigen presentation

323. Which of the following cells is primarily responsible for antigen processing
and presentation?
a. basophil
b. neutrophil
c. NK cell
d. eosinophil
e. macrophage

324. The measurement of the following complement component is clinical


screening test for C1 inhibitor deficiency:
a. C1
b. C3
c. Factor I
d. C4
e. C9

325. The MHC (HLA) class of molecules that is found on all nucleated cells of the
body and is necessary cytotoxic T cell function is class:
a. I
b. II

558 ‫صفحة‬ Dr. MOHAMED YAHIA 0900987639


QUESTION BANK OF Dr. MOHAMED YAHIA

c. III
d. I & II
e. II & III

326. Anaphylactic shock is caused by mediators released mainly by:


a. macrophages
b. eosinophils
c. lymphocytes
d. mast cells
e. neutrophils

327. Type I hypersensitivity is characterized by:


a. complement activation
b. T helper 1 activation
c. immediate release of histamine
d. immune complex formation and precipitation
e. all above

328. Which of the following cytokines are secreted by tumor cells:


a. INF gama
b. TNF alpha
c. IL- 2
d. IL- 4
e. IL-10

329. The following tumor marker can be used to diagnose cancer:


a. CEA
b. PSA
c. CA19-9
d. CA125
e. All above

330. The following method is a semi-quantitative method for assessment of serum


antibody:
a. agglutination
b. radioimmunoassay
c. single radial immunodiffusion (SRID)
d. nephlometry
e. lurbidometry

331. An adjuvant:
a. is an immunogen
b. is an activator of macrophages
c. may be a soluble complex of aluminum and calcium salts
d. enhances quick release of immunogen from site of injection
e. is formed of pure water
332. The following is not a property of an immunogen:
a. large size molecule
b. foreingerness
c. complex molecule e.g. protein

559 ‫صفحة‬ Dr. MOHAMED YAHIA 0900987639


QUESTION BANK OF Dr. MOHAMED YAHIA

d. route of administration decisive


e. requires HLA

333. Which of the following is commonly used to maintain a low level of


microbiological contamination in the atmosphere of surgical theatres.
a. Filtration
b. Radiation
c. Formaldehyde
d. Dry heat
e. Gluteraldehyde

334. The basis for Mycobacterium tuberculosis virulence is largely due to:
a. Production of exotoxin
b. Presence of lipids in cell wall
c. Presence of wax in cell wall
d. Multiplication within macrophages
e. Presence of mycolic acid

335. Chemoprophylaxis against staphylococci in case of hip or cardiac replacement


is obtained by
a. Pipercillin
b. Amikacin
c. Ciprofloxacin
d. Vancomycin
e. Gentamicin

336. Which of following is routinely used to establish presence of


psuedomembranous colitis
a. Gram stain of stools
b. CW Hare of clostridium difficile
c. Detection of toxin in stool
d. Enzyme immunoassay
e. Trial treatment with vancomycin

337. Which of the following antibiotics is recommended for treatment of a brain


abscess because of its ability to penetrate the CNS
a. Wide spectrum penicillin
b. Tetracycline
c. Clindamycin
d. Cefuroxime
e. Chloramphenicol

338. Incubation period before development of Frank disease depends on


a. Integrity of skin
b. Secretion by mucosal epithelium
c. Secreted acid and enzymes.

560 ‫صفحة‬ Dr. MOHAMED YAHIA 0900987639


QUESTION BANK OF Dr. MOHAMED YAHIA

d. Adaptation of organism to environment


e. Vector of disease

339. Ionizing radiation affects killing of micro-organisms by:


a. Damaging DNA
b. Increasing osmolarity
c. Interrupting protein synthesis
d. Damaging the cell wall
e. Generating heat

340. In tetanus:
a. active immunity is produced by giving human antitoxin immunoglobulin
b. the disease caused by gram negative anaerobic bacilli
c. the longer the incubation period the better is the prognosis
d. benzyl penicillin is the drug of choice
e. the disease is caused by the endotoxin of colestridiumtetani

341. Helicobacter pylori:


a. is a gram-negative organism
b. is associated with active chronic (type B) gastritis
c. is more often found in the body than the antrum of the stomach
d. activates complement by the alternative pathway
e. colonizes normal small intestinal mucosa

342. Enteritis can be caused by:


a. colestridiumbotulinum
b. salmonella typhimurium
c. branhamellacatarralis
d. yerseniaenterocolitica
e. proteusmirabales

343. Regarding AIDS


a. the causative agent is HIV
b. the causative agent belongs to the retrovirus group
c. T lymphcytes of the helper/inducer subset are infected
d. cytotoxic/suppressor T cells are very markedly increased
e. the incidence of non-Hodgkins lymphomas is increased in AIDs

344. Actinomycosis
a. is a bacterial disease
b. causative agent is commensal in the mouth
c. pus contains sulphur granules composed of filaments
d. is a nosocomial infection
e. can be controlled by an effective vaccine

345. The following are beta-lactamase resistant:


a. methicillin
b. amoxicillin
c. cephalexin

561 ‫صفحة‬ Dr. MOHAMED YAHIA 0900987639


QUESTION BANK OF Dr. MOHAMED YAHIA

d. flucloxacillin
e. penicillin

346. Pseudomonas aeruginosa:


a. is a common cause of burn infection
b. can multiply in certain disinfectants
c. can cause bacteraemia
d. is an important cause of cystic fibrosa infection
e. causes otitis externa

347. The following interfere with protein synthesis:


a. tetracycline
b. vancomycin
c. penicillin
d. ciprofloxacin
e. chloramphenicol

348. The Epstein-Barr virus (EBV) has a proven positive association with the
following conditions:
a. carcinoma of the cervix
b. infectious mononucleosis
c. human T-cell lymphoma
d. Burkitt's lymphoma
e. undifferentiated nasopharyngeal carcinoma

349. Poliovirus
a. selectively attacks the motor neuron cells in the anterior horn of the spinal
cord
b. is most readily isolated from the feces of the infected subjects
c. occurs in the three different types
d. produces neurological signs in the majority of those infected
e. gains entry to the central nervous system across the blood-brain barrier

350. The following microorganisms are sensitive to penicillin:


a. entercoccusfaecalis
b. streptococcus pyogenes
c. staphylococcus saprophyticus
d. colestridiumtetani
e. kelbseilla

351. The following microorganisms are common causes of nosocomial infections


a. anaerobic streptococci
b. staphylococcus epidermideis
c. pseudomonas aeroginosa
d. streptococcus pneumonia

562 ‫صفحة‬ Dr. MOHAMED YAHIA 0900987639


QUESTION BANK OF Dr. MOHAMED YAHIA

e. aspergillusfumigatus

352. Nephritis following streptococcal pharyngitis is due to:


a. spread of infection to the kidney
b. spread of streptococcal toxins
c. deposition of immunological complexes
d. a specific strain of streptococcus pyogenes
e. incomplete treatment of streptococcal tonsillitis

353. The following diseases can be of zoonotic origin EXCEPT:


a. anthrax
b. pulmonary nocardiosis
c. Dengue fever
d. tuberculosis
e. rabies

354. Which of the following organisms causes a urinary tract infection resistant to
the majority of antibiotics:
a. Proteus mirabilis
b. Klebsiella spp.
c. Staphylococcus saprophyticus
d. Pseudomonas aeruginosa
e. Enterococcus faecalis

355. All of the following apply to Mycoplasma pneumoniae except:


a. main mode of transmission through aerosols
b. associated with a low infecting dose
c. resulting pneumonia treated with 3rd. generation cephalosporins
d. associated with production of cold agglutinins
e. major cause of atypical pneumonia

356. Primary lung aspergillosis in Sudan is mostly associated with:


a. Aspergillusnidulans.
b. A.umigatus.
c. A.niger.
d. A.flavus.
e. A.glaucus.

357. Urinary tract infection caused by Pseudomonas aeruginosa is best treated with.
a. Cloxacillin
b. Vancomycin
c. Kanamycin
d. Piperacillin
e. Gentamicin

563 ‫صفحة‬ Dr. MOHAMED YAHIA 0900987639


QUESTION BANK OF Dr. MOHAMED YAHIA

358. Nocardiosisof the lung is mainly caused by:


a. Nocrdiacaviae.
b. N.asteroides.
c. N.brasiliensis.
d. N.nova.
e. N.farcinica.

359. Antibiotic associated diarrhea is commonly seen with the use of :


a. Gentamicin
b. Clindamycin
c. Cephalexin
d. Metronidazole
e. Vancomycin

360. Which of the following viruses is associated with nasopharyngeal carcinoma:


a. herpes virus.
b. Epstein–Barr virus
c. Adnovirus
d. Cytomegalovirus
e. Varicella virus

361. Which of the following atypical mycobacteria causes cervical


lymphadenopathy:
a. Mycobacterium ulcerans
b. M.kansasii
c. M.avium-intracellulare
d. M.chelonei.
e. M.marinum.

362. A 65 year old man suffered a massive myocardial infarction that was
complicated by shock and prolonged hypotension. On arrival in the emergency
department he was found to have focal neurological signs in addition to features
consistent with low-output cardiac failure. Despite the best efforts of the medical
team he died the next day. At autopsy the most likely change you would expect to
see in a brain would be:
a. Acute hemorrhagic change
b. Coagulative necrosis
c. Granulomatous change
d. Lacunar infarct
e. Liquefactive necrosis

363. A skin biopsy from an anorexic 16 year old girl showed cellular atrophy.
During atrophy:
a. The cell disappears
b. Cellular organelles swell
c. Cell size decreases
d. Cell size increases
e. Protein synthesis increases

564 ‫صفحة‬ Dr. MOHAMED YAHIA 0900987639


QUESTION BANK OF Dr. MOHAMED YAHIA

364. A 35 year old man is a habitual smokerIf a biopsy is taken from the respiratory
tract in this man. the epithelium of respiratory tract is most likely to show:
a. Mucous hyperplasia
b. Smooth muscle hyperplasia
c. Squamous cell anaplasia
d. Squamous cell hypertrophy
e. Stratified squamous metaplasia

365. Apoptotic cells usually exhibit distinctive morphological features. Which one
of the following morphological features is usually seen in pure apoptosis ?
a. Cellular swelling
b. Chromatin condensation
c. Early disruption of the plasma membrane
d. Nuclear stabilization
e. Phagocytosis of apoptotic bones by neutrophils

366. A healthy 26 year old man fractured his right tibia In a road traffic accident.
His right leg was stabilized in a plaster cast. The cast was removed from his leg
after 8 weeks of immobilization. Which of the following changes is most likely to
have taken place in his gastrocnemius muscle after this time?
a. Decrease in the number of muscle fibers
b. Decrease in the number of nerve fibers
c. Increase in the number of fast fibers
d. Increase in the mitochondrial content
e. Increase in the number of satellite cells

367. A 73-years old woman with long-standing hypertension and aortic stenosis
died suddenly one morning . An autopsy was performed on her body. At autopsy
her heart weighed 540 g. The size of her heart is most likely to be the result of
which of the following processes involving the myocardial fibers?
a. Fatty degeneration
b. Fatty infiltration
c. Hyperplasia
d. Hypertrophy
e. Edema

368. Metaplasia is reversible change in which one adult cell type (epithelial or
mesenchymal) is replaced by another adult cell type. In which of the following
situations is the process of epithelial metaplasia most likely to have occurred?
a. Acute myocardial infarction
b. Lactation following pregnancy
c. Tanning of the skin following sunlight exposure
d. Urinary obstruction due to an enlarged prostate
e. Vitamin A deficiency

369. A 55 year old woman with chronic atrial fibrillation suddenly developed an
acute abdomen and was rushed to the emergency department at emergency
laparoscopy most of the small bowel loops were dusky to purple red in color. Her

565 ‫صفحة‬ Dr. MOHAMED YAHIA 0900987639


QUESTION BANK OF Dr. MOHAMED YAHIA

mesenteric veins were patent the most probable underlying pathological process
is:
a. Coagulative necrosis
b. Dry gangrene
c. Gas gangrene
d. Liquefactive gangrene
e. Wet gangrene

370. A histopathology report describes hyaline degeneration Which of the


following microscopic descriptions is most characteristic of hyaline degeneration?
a. Accumulation of lipids in cells
b. Homogeneous, ground-glass. pink-staining appearance of cells
c. Presence of calcium salts with destruction of cellular detail
d. Pyknotic densely stained nucleus
e. Totally amorphous appearance with no cell membrane discernable

371. Which of the following cells is an example of a permanent cell, on the basis of
classification according to regenerative ability?
a. Acinar cell of the pancreas
b. Colonic mucosal cell
c. Erythrocyte
d. Hepatocyte
e. Osteocyte

372. Which of the following provides an example of concomitant hyperplasia and


hypertrophy?
a. Breast enlargement at puberty
b. Cystic hyperplasia of the endometrium.
c. Enlargement of skeletal muscle in athletes
d. Left ventricular cardiac hypertrophy
e. Uterine growth during pregnancy

373. You are looking at a histopathology slide that shows changes associated with
cell injury which of the following cell changes associated with injury is most
likely to be accompanied by disruption of the cell membrane?
a. Apoptosis
b. Cloudy swelling
c. Coagulative necrosis
d. Hydropic change
e. Pyknosis

374. The histopathologist reported the presence of epithelloid cells in a biopsied


cervical lymph node from a patient with tuberculosis. Epithelioid cells are
transformed from:
a. Epithelial cells
b. Eosinophils
c. Lymphocytes

566 ‫صفحة‬ Dr. MOHAMED YAHIA 0900987639


QUESTION BANK OF Dr. MOHAMED YAHIA

d. Macrophages
e. Neutrophils

375. A lump was excised and sent for histo-pathological examination. The histo-
pathologist reported seeing what appeared to bean abnormal amount and
arrangement of normal tissue than is appropriate or normal for the area in which
the tissue arises. This is best described as:
a. Carcinosarcoma
b. Embryonal tumor
c. Hamartoma
d. Mixed tumor
e. Teratoma

376. A histopathology report states that the periphery of hematoma is infiltrated by


new capillaries, fibroblasts and collagen. What is this process called?
a. Embolisation
b. Lysis of the clot
c. Organisation of the hematoma
d. Recanalisation
e. Thrombosis

377. Which of the following terms best describes the passage of leukocytes through
the blood vessel wall?
a. Diapedesis.
b. Euperiporesis
c. Migration
d. Phagocytosis
e. Pavement

378. A 16 year old boy with a 1 day history of sore throat was seen by his GP. On
physical examination the most prominent finding was a pharyngeal purulent
exudates. Which of the following types of inflammation does this boy have?
a. Acute inflammation
b. Abscess formation
c. Chronic inflammation
d. Granulomatous inflammation
e. Resolution of inflammation

379. A 15 year old girl presented to the emergency department complaining of


chronic cough with fever and weight loss for the past month. A chest X-ray
revealed multiple nodules. ranging from 1 cm to 4 cm In size. some of which
demonstrated cavitation (in the upper lobes). A sputum sample revealed the
presence of acid-fast bacilli. Which of the following cells is the most important
cell in the development her lung lesions?
a. Fibroblast
b. Neutrophil
c. Macrophage

567 ‫صفحة‬ Dr. MOHAMED YAHIA 0900987639


QUESTION BANK OF Dr. MOHAMED YAHIA

d. Mast cell
e. Platelet

380. In systemic inflammation. Which of the following statements about TNF is


true?
a. It inhibits apoptotic cell death
b. It inhibits the acute phase reaction
c. It is a soluble intracytoplasmic peptide
d. It is an appetite suppressant
e. It is mainly produced by basophils

381. The histopathology report for a granulomatous lesion suggests chronic


inflammation. Which cell types are most commonly seen in tissue undergoing
chronic inflammation?
a. Eosinophiles
b. Lymphocytes
c. Mast cells
d. Neutrophils
e. Platelets

382. A histopathology report describes the presence of granulation tissue in a


lesion. Which of the following features is characteristic of granulation tissue?
a. Giant cells and fibroblasts
b. Giant cells and lymphocytes
c. Giant cells, plasma cells and lymphocytes
d. Neutrophils and necrotic tissue
e. Proliferation of new capillaries, with fibroblasts and new collagen
formation

383. Which of the following terms best describes the unidirectional migration of
leucocytes towards a target?
a. Chemotaxis
b. Diapedesis
c. Endocytosis
d. Margination
e. Meiosis

384. What is the origin of the cells of the mononuclear phagocyte system?
a. Bone marrow
b. Liver
c. Lymph nodes
d. Thymus
e. Spleen

385. In human to human renal transplant the graft is called?


a. Allograft
b. Autograft
c. Isograft
d. Synngraft
e. Xenograft

568 ‫صفحة‬ Dr. MOHAMED YAHIA 0900987639


QUESTION BANK OF Dr. MOHAMED YAHIA

386. A 26-year-old woman with blood group type A Rh-negative is pregnant with
her second child. Her first child is Rh- positive and the father is also Rh- positive.
The second child most likely to be at risk of developing:
a. An autoimmne disease
b. ABO incompatibility
c. Drug-induced hemolytic anemia
d. Neutropenia
e. Hemolytic disease of the newborn

387. A 25-year-old man with persistent low-grade fever and cough had a blood
smear which showed a marked increase in the number of cells with a large bi-
lobed nucleus which of the following cell types was seen in the smear?
a. Basophils
b. Eosinophils
c. Lymphocytes
d. Monocytes
e. Neutrophils

388. Which of the following white blood cells will be the predominant cell type
seen under the microscope if a blood smear Is made from a normal healthy
individual?
a. Basophils
b. Eosinaphils
c. Lymphocytes
d. Monocytes
e. Neutrophils

389. Which of the following cytokines has anti-inflammatory properties?


a. Granulocyte macrophage colony-stimulating factor
b. Interleukin-2
c. lnterleukin-3
d. Interieukin-10
e. Tumor necrosis factor (TNF)

390. Arachidonic acid is one of the essential fatty acids required by most mammals.
It isessential for the synthesis of which of the following mediators of
Inflammation ?
a. Bradykinin
b. Interferon gamma
c. Interleukin-l
d. Prostaglandins
e. Tumor necrosis factor

391. Which one of the following is a primary function of the Kupffer cells in the
liver?
a. Protein synthesis
b. Recycling of old red blood cells
c. Secretion of mucus
d. Storage of fat-soluble vitamins

569 ‫صفحة‬ Dr. MOHAMED YAHIA 0900987639


QUESTION BANK OF Dr. MOHAMED YAHIA

e. Synthesis of intrinsic factor

392. A healthy 45-year-old woman has a routine health checkShe has no chest pain
cough or fever A chest X-ray is taken however which shows a peripheral coin
lesion 2.5cm in diameter in the right mid-lung field which of the following
biological characteristics would best distinguish this lesion as a neoplasm rather
than a granuloma?
a. Necrosis
b. Rapid increase in size
c. Recurrence following excision
d. Sensitivity to radiation or chemotherapy
e. Uncontrolled (autonomous) growth

393. A 76 year old man with back painwas diagnosed with metastatic prostatic
carcinoma involving the lumbar spine. High serum levels of which of the
following tumor markers will aid the diagnosis of prostatic carcinoma?
a. Acid phosphatase
b. Alkaline phosphatase
c. Alpa-fetoprotien
d. CEA
e. PSA

394. Hypercalcemia is probably the most common paraneoplastic syndrome. The


most likely mechanism by which cancer causes this abnormality is:
a. bone metastasis
b. Increased level of vitamin D
c. Increased level of parathyroid hormone (PTH).
d. Increase d level of PTH-related protein
e. Renal failure

395. Which one of the following is a benign tumor?


a. Adenocarcinoma
b. Fibrosarcoma
c. Hematoma
d. Osteogenic sarcoma
e. Warthin's tumor

396. Which of the following features is taken into account when staging a cancer?
a. Basophilia
b. Local invasion
c. nuclear/ cytoplasmic ratio
d. Number of mitotic figures
e. Pleomorphism

397. A 60 year old woman with suspected carcinoma cervix is seen by the
gynaecologist. The most reliable and easy method to confirm the diagnosis is:
a. Blood culture

570 ‫صفحة‬ Dr. MOHAMED YAHIA 0900987639


QUESTION BANK OF Dr. MOHAMED YAHIA

b. Colposcopy
c. Laparotomy
d. Pap smear
e. Ultrasound

398. Which of the following Is the most common primary malignant tumor of the
thyroid ?
a. Anaplastic carcinoma
b. Follicular carcinoma
c. Large-cell carcinoma
d. Medullary carcinoma
e. Papillary carcinoma

399. Carcinoma of the oral cavity:


a. Is more common in women
b. Is predisposed to by erythroplasia
c. Is predisposed t o by hairy leukoplakia
d. Is predominantly adenocarcinoma in type
e. Occurs most frequently on the hard palate

400. In which of the following sites is a tumor morelikely to occur in adults than in
children?
a. Bone
b. Central nervous system
c. Kidney
d. Lung
e. Soft t issue

401. Which of the following sites in newborns most commonly gives rise to tumors
derived from all three germ-cell layers?
a. Central nervous system
b. Mediastinum
c. Ovaries
d. Sacrococcygeal area
e. Testis

402. Which of the following is the most common predisposing factor for
adenocarcinoma of the esophagus?
a. Exposure to nitrosamines
b. Gastro-esophageal reflux disease
c. Human papilloma virus infection
d. Smoking
e. Tylosispalmaris

403. Which of the following structures is found in Gram negative bacteria but not
in Gram-positive bacteria?
a. Capsule
b. Cell wall
c. Cytoplasmic membrane

571 ‫صفحة‬ Dr. MOHAMED YAHIA 0900987639


QUESTION BANK OF Dr. MOHAMED YAHIA

d. Endospore
e. Outer membrane

404. A 21 year old woman who had been feeling unwell since returning from a
holiday trip to India, presented to the emergency department on day 16 after the
onset of fever. She had a positive Widel test. The most likely organism
responsible for the fever is:
a. Bacteroidesfragilis
b. Escherichia coli
c. Klebsiellaspecies
d. Mycobacterium tuberculosis
e. Salmonella typhi

405. The virulence of bacteria is related to:


a. Toxin and enzyme production
b. The resistance of the patient
c. The number of bacteria
d. The age of the patient
e. The portal of entry

406. Brucellosis is transmitted by:


a. Cats
b. Ticks
c. Flies
d. Mosquito bite
e. Unpasteurized milk

407. An abscess containing sulphur granules is feature of:


a. Actinomycosis
b. Amoebiasis
c. Brucellosis
d. Histoptasmosis
e. Staphylococcus aureusinfection

408. A 6 year old child is brought to the emergency department by his parents
complaining of earache. On examination, there is greenish pus discharging from
his right ear. Which of the following organisms ismost likely to be responsible for
this child's ear infection?
a. Klebsiellaspecies
b. Pseudomonas aeruginosa
c. Staphylococcus aureus
d. Streptococcus pyogenes
e. Streprococcuspneumoniae

409. Infective endocarditis is mostly caused by the following micro-organism:


a. Echerichia coli
b. Haemophilusinfluenza
c. Streptococcus pneumonia
d. Streptococcus viridans

572 ‫صفحة‬ Dr. MOHAMED YAHIA 0900987639


QUESTION BANK OF Dr. MOHAMED YAHIA

e. Staphylococcus aureus

410. A 22 year old medical student presented In the emergency department with
sore throat and lymphadenopathy. Her peripheral smear shows atypical
lymphocytes. What is the most probable infectious cause for this patient' s clinical
presentation?
a. Cytomegalovirus
b. Epstein- Barr virus
c. Herpes Simplex virus
d. HIV
e. Mycobacterium tuberculosis

411. Demineralized bone matrix, which is a clinically useful bone graft substitute
or enhancer, exerts its effectiveness by what mechanism:
a. Cytokine activity
b. Creation of hyperosmotic environment
c. Calcium stimulation of stem cells
d. Structural mechanical support
e. Exogenous recombinant bone morphogenetic protein activity

412. Features of DIC do not include:


a. Increase level of fibrin degradation products
b. Thrombocytopenia
c. Decrease fibrinogen
d. Decrease coagulation factor
e. Increased clotting time

413. The most documented association between cancer & virus is in the case of:
a. Human immunodeficiency virus
b. Human papilloma virus
c. Pox virus
d. Herpes simplex virus
e. Vericella virus

414. The most powerful chemotactic agent of the following is:


a. Histamine
b. Prostaglandin
c. Hageman factor
d. Bradykinin
e. C5a

415. One of the following not associated with malignant transformation of cells:
a. Asbestos filters
b. Chronic malaria
c. Nitrosamines
d. U.V light
e. Human papilloma virus

416. An example of combined hypertrophy & hyperplasia is:


a. Skeletal muscles enlargement in athletes

573 ‫صفحة‬ Dr. MOHAMED YAHIA 0900987639


QUESTION BANK OF Dr. MOHAMED YAHIA

b. Gravid uterus
c. Cardiac muscle hypertension
d. Proliferative endometrium following menstruation
e. Endemic goiter

417. The least likely site of primary tuberculosis infection:


a. Bone
b. Uterine tubes
c. Brain
d. Lungs
e. Intestine

418. Hypertrophy is defined as:


a. Increase in cell number
b. Decrease in cell number
c. Increase in cell size
d. Decrease in cell number
e. Cell with high nuclear cytoplasmic ratio

419. A visualization by the electron microscope, the structure most likely not to be
a cytoplasmic organelle is (are):
a. Lysosome
b. Golgi complex
c. Desmosomes
d. Endoplasmic reticulum
e. Microbodies

420. Containment of antibacterial resistance can be achieved by the following


methods:
a. Using new & broad spectrum, injectable antibacterial drugs
b. Effective infection control policy
c. Restricting the prescription of some highly effective drugs only by consultant
d. Reducing patient self medication
e. Topical use of antibacterial instead of oral use

421. Traveler's diarrhea is most probably caused by:


a. Staphylococcus aureus
b. Eschericia coli
c. B. cereus
d. Cl. Botulinum
e. V. cholerae

422. The most powerful hyperlipidemia associated with IHD is:


a. Hyper chylomicremia
b. Increased VLDL level
c. Increased LDL level
d. Increased HDL level
e. Increased lipo-protein "a"

574 ‫صفحة‬ Dr. MOHAMED YAHIA 0900987639


QUESTION BANK OF Dr. MOHAMED YAHIA

423. The primary mechanism by which aspirin its effectiveness is direct inhibition
of:
a. Cyclooxygenase
b. Lipoxygenase
c. Phospholipase
d. Leukotrines
e. Thromboxane A2

424. Caseous necrosis is most commonly associated with:


a. Brain infarction
b. Streptomyces somaliensis infection
c. Tuberculosis
d. Coccidosis
e. Sarcoidosis

425. The following statement about hepatitis B carrier is correct:


a. They should be treated with interferon
b. Adult are more likely to remain carrier that are children
c. Carrier with HBe Ag are highly infective
d. Carrier with reverse transcriptase in the serum are highly infective
e. 10% of the developed from maternal transmission

426. Which of the following organisms is isolated in acute necrotizing fasciaitis:


a. Group A streptococcus
b. Group D streptococcus
c. Pseudomonas aeroginosa
d. Staphylococcus aureus
e. Clostiridiumdifficile

427. Diagnosis of multiple myeloma can be confirmed by:


a. ESR
b. Skeletal radiological survey
c. Serum protein estimation
d. Urine examination
e. Bone marrow biopsy

428. Which one of the following clinical signs is most important of severe
dehydration:
a. Tachycardia
b. Decreased voiding
c. Thirst
d. Reduced skin turgor
e. Dry Axillary fold

429. Action of parathormone is mediated by:


a. Calcitonin
b. Glucagon

575 ‫صفحة‬ Dr. MOHAMED YAHIA 0900987639


QUESTION BANK OF Dr. MOHAMED YAHIA

c. Prostaglandin
d. Cyclic AMP
e. ACTH

430. Which one of the following is not a clinical feature of primary


hyperparathyroidism:
a. Bone pain
b. Pathological fractures
c. Tetany
d. Peptic ulcer
e. Muscle weakness

431. Nitric acid is an inflammatory mediator synthesized by:


a. Recutiophils
b. Qornophils
c. Lymphocytes
d. Monocytes
e. Basophils

432. Which are of the following of vascular endothelial changes that allow
increased fluid permeability:
a. Cytoplasm transcytosis of the fluid
b. Gaps in basement membrane
c. Increase number of phagolysomes
d. Endothelial gaps due to endothelial cell contraction
e. Fluid escapes without morphological changes

433. Which factor that does not impair wound healing:


a. Tissue hypoxia
b. Good vascular supply
c. Excels of corticosteroid hormone
d. Deficiency of zinc
e. Deficiency of vit. C

434. Increased vascular permeability in acute inflammation:


a. Is caused by endothelial cell contraction occurring mainly in venules
b. Leads to formation of watery transudate
c. Is an irreversible process
d. Is the main cause of cellular migration into tissue
e. Cause increased viscosity of the blood

435. Hydrogen peroxide myeleperoxidase halide system mainly kills:


a. Viruses
b. Fungi
c. Parasites

576 ‫صفحة‬ Dr. MOHAMED YAHIA 0900987639


QUESTION BANK OF Dr. MOHAMED YAHIA

d. Bacteria
e. Rickettsia

436. A 60 years old hypertensive male presented with a pulsating abdominal mass.
A CTscan revealed an abdominal aneurysm. The most likely cause is:
a. High blood pressure on arterial wall
b. Old syphilis
c. Atherosclerosis
d. Endarteritis oblitertans of vasa vasorum
e. Chlamydia infection

437. After 2 weeks in hospital following a facture of her Lt. femur a 76 years old
woman now has Lt. leg swollen below the knee. There is pain on movement of the
leg. Which of the following complications is most likely to occur after these
events:
a. Gangrenous necrosis of the foot
b. Hematoma
c. DIC
d. Pulmonary thrombo-embolism
e. Soft tissue sarcoma

438. Which of the following infectious agents induce the transformation tissue
macrophages to epitheloid cells:
a. Mycobacterium leprae
b. Pseudomonas aerouginosa
c. Cytomegalovirus
d. Giardia lamblia
e. Treponemapallidum

439. An exudate is mostly the result of:


a. Increase venous pressure
b. Sodium retention
c. Decreased plasma oncotic pressure
d. Increase capillary permeability & increased hydrostatic pressure
e. Lymphatic obstruction

440. Blood pH is affected by:


a. PO2
b. Blood glucose
c. Protein metabolism
d. Bicarbonate
e. Plasma Na+

441. A 43 years old male complained of mild burning epigastric pain following
meals for 3 years. Endoscopy & biopsy from the lower esophagus was done which

577 ‫صفحة‬ Dr. MOHAMED YAHIA 0900987639


QUESTION BANK OF Dr. MOHAMED YAHIA

showed presence of columnar epithelium with goblet cells. Which one of the
following mucosal alteration is represendted by these findings:
a. Dysplasia
b. Hyperplasia
c. Carcinoma
d. Ischemia
e. Metaplasia

442. Generation of which of the following substances within the cells is most likely
to reduce the number of free radicals:
a. Glutathione peroxidase
b. Caspase
c. Hydrogen peroxide
d. NADPH oxidase
e. Meyloperoxidase
443. Which of the following cellular changes represent irreversible cellular injury:
a. Dysplasia
b. Fatty changes
c. Nuclear pyknosis
d. Atrophy
e. Anaerobic glycolysis

444. Most important cells in the pathogenesis of atheromatous plaques is the:


a. Neutrophils
b. T lymphocytes
c. Macrophages
d. Smooth muscle cells
e. Fibroblast

445. A 40 years old woman has a sudden onset of severe abdominal pain. Physical
examination & investigations showed acute pancreatitis with increased serum
lipase. Which of following cellular changes is most likely to accompany these
findings:
a. Coagulative necrosis
b. Dry gangrene
c. Fat necrosis
d. Apoptosis
e. Liquifactive necrosis

446. Tumor cachexia is related to:


a. Depression
b. Intestinal malbsorption
c. Tumor necrosis factor
d. Defective deglutition
e. Gastric metastatic deposition

447. Tumor with definitive genetic basis include:


a. Bronchogenic carcinoma
b. Vaginal adenocarcinoma
c. Basal cell carcinoma

578 ‫صفحة‬ Dr. MOHAMED YAHIA 0900987639


QUESTION BANK OF Dr. MOHAMED YAHIA

d. Colonic adenocarcinoam
e. Ovarian teratoma

448. Hypertrophy due to hormonal stimulation is associated with:


a. Acromegaly
b. Uterine fibroids
c. Athletic foot
d. Turner’s syndrome
e. Hypersplenism

449. Dysplasia:
a. Is irreversible
b. Occurs in ulcerative colitis
c. May progress into metaplasia
d. Is associated with invasion of basement membrane
e. Is usually seen in the uterine endo-cervix

450. The following factor promotes wound healing:


a. Glucocorticoids
b. High carbohydrate intake
c. Scurvy
d. Wound suture
e. Fall in temperature

451. Granulation tissue:


a. Contains Langhan’s giant cells
b. Contains numerous granulomas
c. Mast cells helps in degradation
d. Contains thin walled capillaries
e. Is found following injury to the brain

452. The following tumor rarely metastasized but locally very destructive:
a. Sequamous cell carcinoma of the bladder
b. Basal cell carcinoma
c. Carcinoid tumor in the stomach
d. Glioblastoma
e. Hemangioma

453. Apoptosis:
a. Require energy
b. Occurs in helicobacter pylori infection
c. Is a cause of inflammation
d. Is reversible
e. Is monogenic

454. Disturbances in shock include:


a. Metabolic alkalosis

579 ‫صفحة‬ Dr. MOHAMED YAHIA 0900987639


QUESTION BANK OF Dr. MOHAMED YAHIA

b. Hyperkalemia
c. Hypernatremia
d. Hypoglycemia
e. Decrease level of fatty acids in the plasma

455. Edema occurs in:


a. Malnutrition
b. Nephritic syndrome
c. Lymphatic obstruction
d. Liver failure
e. Cardiac failure

456. Deficiency resulting in increased susceptibility to viral infections is due to


primarily to a deficiency in:
a. Macrophages
b. B-cells
c. T-cells
d. Natural killer cells
e. Polymorph nuclear cells

457. A CSF profile that consist of elevated PMNL, depressed glucose & elevated
protein suggests:
a. Brain abscess
b. Encephalitis
c. Fungal meningitis
d. Viral meningitis
e. Untreated bacterial meningitis

458. The pathological specimen where Aspergillus heads can be seen directly in:
a. Sputum
b. Ear swab
c. Biopsy
d. CSF
e. Nail clipping

459. A single painless indurated ulcer on the genitalia is indicative of:


a. Herpes
b. Lymphogranulomavenerium
c. Chancroid
d. Syphilis
e. Wart

460. Which one of the following organisms can resist intracellular killing by
phagocytes:
a. Mycobacterium tuberculosis
b. Streptococcus pneumoniae
c. Haemophilusinflauenzae
d. Staphylococcus saprophyticus
e. Campylobacter jejunei

580 ‫صفحة‬ Dr. MOHAMED YAHIA 0900987639


QUESTION BANK OF Dr. MOHAMED YAHIA

461. When cultured on chocolate blood agar under appropriate conditions colonies
of helicobacter pylori appear after:
a. 1-2 days
b. 2-3 days
c. 4-6 days
d. 7 days
e. 8 days

462. Serum assay of drug is necessary in patient treated with:


a. Amikacin
b. Clavulanic acid
c. Carbencillin
d. Fusidic acid
e. Cefutaxime

463. Which of the following antibiotics will affect killing of microorganism by


action on the cell wall:
a. Trimethoprim
b. Tetracycline
c. Vancomycin
d. Ciprofloxacin
e. Chloramphenicol

464. Maximum duration to keep peripheral lines to prevent blood stream infection
is:
a. 24 hours
b. 48 hours
c. 72 hours
d. 7 days
e. 10 days

465. The common microorganism isolated causing hospital acquired infection:


a. Viruses
b. Fungi
c. Bacteria
d. Parasites
e. Reckettsia

581 ‫صفحة‬ Dr. MOHAMED YAHIA 0900987639


QUESTION BANK OF Dr. MOHAMED YAHIA

CHAPTER 5

174 Question

582 ‫صفحة‬ Dr. MOHAMED YAHIA 0900987639


QUESTION BANK OF Dr. MOHAMED YAHIA

PRETEST
PHARMACOLOGY

1. Which route of administration is most likely to subject a drug to a first pass


effect?

a. Intravenous
b. Inhalational
c. Oral
d. Sublingual (SL)
e. Intramuscular

The answer is: C

2. The pharmacokinetic value that most reliably reflects the amount of drug
reaching the target tissue after oral administration is the

a. Peak blood concentration


b. Time to peak blood concentration
c. Product of the V d and the first-order rate constant
d. Vd
e. Area under the blood concentration-time curve (AUC)

The answer is: E

3. Of the following characteristics, which is unlikely to be associated with the


process of facilitated diffusion of drugs?

a. The transport mechanism becomes saturated at high drug concentrations


b. The process is selective for certain ionic or structural configurations of the
drug
c. If two compounds are transported by the same mechanism, one will
competitively inhibit the transport of the other
d. The drug crosses the membrane against a concentration gradient and the
process requires cellular energy
e. The transport process can be inhibited noncompetitively by substances that
interfere with cellular metabolism

The answer is: D

4. In comparing the following possible routes, which is associated with the


excretion of quantitatively small amounts of drugs or their metabolic
derivatives?

583 ‫صفحة‬ Dr. MOHAMED YAHIA 0900987639


QUESTION BANK OF Dr. MOHAMED YAHIA

a. Biliary tract
b. Kidneys
c. Lungs
d. Feces
e. Milk

The answer is: E

5. Of the following, which is a phase II biotransformation reaction?

a. Sulfoxide formation
b. Nitro reduction
c. Ester hydrolysis
d. Sulfate conjugation
e. Deamination

The answer is: D

6. Which of the following is unlikely to be associated with oral drug


administration of an enteric-coated dosage form?

a. Irritation to the gastric mucosa with nausea and vomiting


b. Destruction of the drug by gastric acid or digestive enzymes
c. Unpleasant taste of the drug
d. Formation of nonabsorbable drug-food complexes
e. Variability in absorption caused by fluctuations in gastric emptying time

The answer is: E

7. A 19-year-old male being treated for leukemia develops fever. You give agents
that will cover bacterial, viral, and fungal infections. Two days later, he develops
acute renal failure. Which drug was most likely responsible?

a. Vancomycin
b. Ceftazidime
c. Amphotericin B
d. Acyclovir

The answer is: C

8. A 26-year-old female with acquired immunodeficiency syndrome (AIDS)


develops cryptococcal meningitis. She refuses all intravenous medication. Which
antifungal agent can be given orally to treat the meningeal infection?

a. Ketoconazole
b. Amphotericin B
c. Fluconazole
d. Nystatin

584 ‫صفحة‬ Dr. MOHAMED YAHIA 0900987639


QUESTION BANK OF Dr. MOHAMED YAHIA

The answer is: B

9. Why is vitamin B 6 usually prescribed with isoniazid (INH)?

a. It acts as a cofactor for INH


b. It prevents some adverse effects of INH therapy
c. Like INH, it has tuberculostatic activity
d. It prevents metabolism of INH

The answer is: B

10. A 19-year-old woman is diagnosed with tuberculosis (TB). Before prescribing


a drug regimen, you take a careful medication history because one of the drugs
commonly used to treat TB induces microsomal cytochrome P450 enzymes in the
liver. Which drug is this?

a. Isoniazid
b. Rifampin
c. Pyrazinamide
d. Ethambutol
e. Vitamin B 6

The answer is: B

11. Amantadine, a synthetic antiviral agent used prophylactically against


influenza A 2 , is thought to act by

a. Preventing production of viral capsid protein


b. Preventing virion release
c. Preventing penetration of the virus into the host cell
d. Preventing uncoating of viral DNA
e. Causing lysis of infected host cells by release of intracellular lysosomal
enzymes

The answer is: D

12. Streptomycin and other aminoglycosides inhibit bacterial protein synthesis


by binding

a. Peptidoglycan units in the cell wall


b. Messenger RNA (mRNA)
c. DNA
d. 30S ribosomal particles
e. RNA polymerase

The answer is: D

585 ‫صفحة‬ Dr. MOHAMED YAHIA 0900987639


QUESTION BANK OF Dr. MOHAMED YAHIA

13. A patient with AIDS is treated with a combination of agents, which includes
zidovudine. What is the mechanism of action of zidovudine?

a. Inhibition of RNA synthesis


b. Inhibition of viral particle assembly
c. Inhibition of viral proteases
d. Inhibition of nucleoside reverse transcriptase
e. Inhibition of nonnucleoside reverse transcriptase

The answer is: D

14. A jaundiced one-day-old premature infant with an elevated free bilirubin is


seen in the premature-baby nursery. The mother received an antibiotic
combination preparation containing sulfamethizole for a urinary tract infection
(UTI) one week before delivery. You suspect that the infant’s findings are caused
by the sulfonamide because of the following mechanism:

a. Enhanced synthesis of bilirubin


b. Competition between the sulfonamide and bilirubin for binding sites on
albumin
c. Inhibition of bilirubin degradation
d. Inhibition of urinary excretion of bilirubin

The answer is: B

15. A 30-year-old male with a two-year history of chronic renal failure requiring
dialysis consents to transplantation. A donor kidney becomes available. He is
given cyclosporine to prevent transplant rejection just before surgery. What is
the most likely adverse effect of this drug?

a. Bone marrow depression


b. Nephrotoxicity
c. Oral and GI ulceration
d. Pancreatitis
e. Seizures

The answer is: B

16. The use of chloramphenicol may result in

a. Bone marrow stimulation


b. Phototoxicity
c. Aplastic anemia
d. Staining of teeth
e. Alopecia

The answer is: C

586 ‫صفحة‬ Dr. MOHAMED YAHIA 0900987639


QUESTION BANK OF Dr. MOHAMED YAHIA

17. Ampicillin and amoxicillin are in the same group of penicillins. Which of the
following statements best characterizes amoxicillin?

a. It has better oral absorption than does ampicillin


b. It can be used in penicillinase-producing organisms
c. It is classified as a broad-spectrum penicillin
d. It does not cause hypersensitivity reactions
e. It is effective against Pseudomonas

The answer is: A

18. A 60-year-old male with AIDS develops a systemic fungal infection that is
treated with fluconazole. What is the mechanism of action of fluconazole?

a. It inhibits ergosterol synthesis


b. It inhibits DNA synthesis
c. It inhibits peptidoglycan synthesis
d. It inhibits protein synthesis

The answer is: A

19. A 50-year-old male diabetic develops an external otitis from which


Pseudomonas organisms are cultured. Topical therapy with polymyxin is
effective. What is the mechanism of action of polymyxin?

a. Inhibition of cell-wall synthesis


b. Formation of reactive cytotoxic products that interfere with DNA synthesis
c. Disruption of membrane permeability
d. Inactivation of protein sulfhydryl groups
e. Inhibition of protein synthesis by binding to transfer RNA (tRNA)

The answer is: A

20. Which of the following best describes ampicillin’s effect on therapeutically


administered estrogens?

a. It decreases estrogen metabolism


b. It decreases the enterohepatic circulation of estrogen
c. It decreases the plasma protein binding of estrogen
d. It decreases the renal excretion of estrogen
e. It decreases the sensitivity of estrogen at its site of action

The answer is: B

21. A 36-year-old female with a chronic UTI treated with ciprofloxacin is not
responsive to the antibiotic. Which of the following agents that she might have
been taking for other reasons would decrease the effectiveness of ciprofloxacin?

587 ‫صفحة‬ Dr. MOHAMED YAHIA 0900987639


QUESTION BANK OF Dr. MOHAMED YAHIA

a. An antacid
b. An antihistamine
c. A nonsteroidal anti-inflammatory
d. An anxiolytic
e. A multivitamin not containing iron

The answer is: A

22. A 20-year-old male with herpes simplex of the lips is treated with famciclovir.
What is the mechanism of action of famciclovir?

a. Cross-linking of DNA
b. Strand breakage of DNA
c. Inhibition of viral DNA synthesis
d. Inhibition of nucleotide interconversions
e. Inhibition of a viral kinase

The answer is: C

23. A patient being treated with a combination of drugs for pulmonary


tuberculosis develops a decrease in visual acuity and red-green color blindness
resulting from retrobulbar neuritis. Which of the following agents is responsible
for these findings?

a. INH
b. Streptomycin
c. Rifampin
d. Pyrizinamide
e. Ethambutol

The answer is: E

24. The most effective drug for immunosuppression of rejection of the


allografted kidney is

a. Azathioprine
b. Cyclosporine
c. 5-fluorouracil (5-FU)
d. Cyclophosphamide
e. Vincristine

The answer is: B

25. The phase of the cell cycle that is resistant to most chemotherapeutic agents
and requires increased dosage to obtain a response is the

a. M phase
b. G 2 phase

588 ‫صفحة‬ Dr. MOHAMED YAHIA 0900987639


QUESTION BANK OF Dr. MOHAMED YAHIA

c. S phase
d. G 0 phase
e. G 1 phase

The answer is: D

26. A nucleophilic attack on deoxyribonucleic acid (DNA) that causes the


disruption of base pairing occurs as a result of the administration of

a. Cyclophosphamide
b. 5-FU
c. Methotrexate
d. Prednisone
e. Thioguanine

The answer is: A

27. The antineoplastic chemotherapeutic agent that is classified as an alkylating


agent is

a. Thioguanine
b. Busulfan
c. Bleomycin
d. Vincristine
e. Tamoxifen

The answer is: B

28. Which of the following is a chemotherapeutic drug that possesses a


mechanism of action involving alkylation?

a. Cyclophosphamide
b. Methotrexate
c. Tamoxifen
d. 5-FU
e. Doxorubicin

The answer is: A

29. A nine-year-old boy is diagnosed with acute lymphoblastic leukemia. He is


maintained on methotrexate. A recent platelet count is below normal, and a stool
guaiac is 4+. Which of the following agents should be administered to counteract
methotrexate toxicity?

a. N-acetyl- L -cysteine
b. Vitamin K
c. Penicillamine
d. Leucovorin

589 ‫صفحة‬ Dr. MOHAMED YAHIA 0900987639


QUESTION BANK OF Dr. MOHAMED YAHIA

e. Deferoxamine

The answer is: D

30. Cardiotoxicity limits the clinical usefulness of which one of the following
antitumor antibiotics?

a. Dactinomycin
b. Doxorubicin
c. Bleomycin
d. Plicamycin
e. Mitomycin

The answer is: B

31. Binding to the enzyme dihydrofolate reductase is the mechanism of action for

a. Procarbazine
b. Paclitaxel
c. Methotrexate
d. Ifosfamide
e. Cladribine

The answer is: C

32. Which of the following is considered to be the effective mechanism of action


of the vinca alkaloids?

a. Inhibition of the function of microtubules


b. Damage and prevention of repair of DNA
c. Inhibition of DNA synthesis
d. Inhibition of protein synthesis
e. Inhibition of purine synthesis

The answer is: A

33. The tumor that is least susceptible to cell-cycle-specific (CCS) anti- cancer
agents is

a. Acute lymphoblastic leukemia


b. Acute granulocytic leukemia
c. Burkitt’s lymphoma
d. Adenocarcinoma of the colon
e. Choriocarcinoma

The answer is: D

590 ‫صفحة‬ Dr. MOHAMED YAHIA 0900987639


QUESTION BANK OF Dr. MOHAMED YAHIA

34. A 32-year-old cancer patient, who has smoked two packs of cigarettes a day
for 10 years, presents a decreased pulmonary function test. Physical examination
and chest x-rays suggest preexisting pulmonary disease. Of the following drugs,
which is best not prescribed?

a. Vinblastine
b. Doxorubicin
c. Mithramycin
d. Bleomycin
e. Cisplatin

The answer is: D

35. Of the following, which is not a CCS agent

a. Mercaptopurine (6-MP)
b. 5-FU
c. Bleomycin
d. Busulfan
e. Vincristine

The answer is: D

36. A 25-year-old female post–renal transplant shows signs of acute renal


allograph rejection. Of the following agents, which should be administered?

a. Interferon α
b. Aldesleukin
c. Muromonab-CD3
d. Sargramostim
e. Filgrastim

The answer is: C

37. A 50-year-old female with rheumatoid arthritis has developed erosions in her
wrist bones. Which of the following agents should be administered?

a. Allopurinol
b. Asparaginase
c. Methotrexate
d. Streptozocin
e. 6-MP
f. Azathioprine
g. Pentostatin
h. Leucovorin
i. Bacille Calmette-Guérin (BCG) vaccine

The answer is: C

591 ‫صفحة‬ Dr. MOHAMED YAHIA 0900987639


QUESTION BANK OF Dr. MOHAMED YAHIA

38. A 40-year-old female post–renal transplant has developed evidence of


osteoporosis, most likely due to cyclosporine. Which of the following agents
might replace cyclosporine?

a. Allopurinol
b. Asparaginase
c. Methotrexate
d. Streptozocin
e. 6-MP
f. Azathioprine
g. Pentostatin
h. Leucovorin
i. BCG vaccine

The answer is: F

39. A 34-year-old male with Hodgkin’s disease is treated with the adriamycin,
bleomycin, vinblastine, and decarbazine (ABVD) regimen. What is the
mechanism of action of vinblastine?

a. Scission of DNA strands


b. Inhibition of dihydrofolate reductase
c. Inhibition of enzymes involved in purine metabolism
d. Prevention of assembly of tubulin dimers into microtubules
e. Inhibition of topoisomerase

The answer is: D

40. A 60-year-old male with hematuria is found to have a small localized tumor
of the bladder that is diagnosed as a carcinoma. Which of the following agents
should be given intravesicularly?

a. Allopurinol
b. Asparaginase
c. Methotrexate
d. Streptozocin
e. 6-MP
f. Azathioprine
g. Pentostatin
h. Leucovorin
i. BCG vaccine

The answer is: I

41. A 45-year-old female has a bone marrow transplant for treatment of ovarian
cancer. Cyclosporine is given as an immunosuppressant. What is the mechanism
of action of cyclosporine?

592 ‫صفحة‬ Dr. MOHAMED YAHIA 0900987639


QUESTION BANK OF Dr. MOHAMED YAHIA

a. Direct destruction of proliferating lymphoid cells


b. Inhibition of T cell response to cytokines
c. Inhibition of folic acid metabolism
d. Inhibition of factors that stimulate T cell growth
e. Inhibition of enzymes that are related to purine metabolism

The answer is: D

42. A young adult patient with acute granulocytic leukemia, treated with a
combination of cytarabine and thioguanine, is no longer responsive to the
therapy. The nonresponsiveness of the patient is thought to be due to
thioguanine. What is the mechanism of resistance to thioguanine?

a. Decreased uptake
b. Increased efflux
c. Increased alkaline phosphatase activity
d. Increased production of trapping agents
e. Increased DNA repair

The answer is: C

43. A 45-year-old male has an insulinoma. Which of the following agents is the
treatment of choice?

a. Cyclophosphamide
b. Carboplatin
c. Vincristine
d. Streptozocin
e. Bleomycin

The answer is: D

44. A 50-year-old female is treated with paclitaxel. Of the following, how is


paclitaxel classified?

a. An alkylating agent
b. An antimetabolite
c. A plant alkaloid
d. An antibiotic
e. A hormonal agent

The answer is: C

45. A 41-year-old female is treated for endometrial cancer with tamoxifen. Of the
following, how is tamoxifen classified?

a. An alkylating agent
b. An antimetabolite

593 ‫صفحة‬ Dr. MOHAMED YAHIA 0900987639


QUESTION BANK OF Dr. MOHAMED YAHIA

c. A plant alkaloid
d. An antibiotic
e. A hormonal agent

The answer is: E

46. A 35-year-old female is being treated for cervical cancer with cisplatin. Of
the following, how is cisplatin classified?

a. An alkylating agent
b. An antimetabolite
c. A plant alkaloid
d. An antibiotic
e. A hormonal agent

The answer is: A

47. A 16-year-old male treated for acute lymphocytic leukemia develops severe
lumbar and abdominal pain. His serum amylase is markedly elevated. Which of
the following agents most likely caused these findings?

a. 6-MP
b. Asparaginase
c. Doxorubicin
d. Methotrexate
e. Vincristine

The answer is: B

48. A 60-year-old female treated for breast cancer develops leucopenia and
severe stomatitis and oral ulcerations. Which of the following agents most likely
caused these findings?

a. 5-FU
b. Paclitaxel
c. Cyclophosphamide
d. Tamoxifen
e. Carboplatin

The answer is: A

49. A 45-year-old male on combination therapy for remission- maintenance acute


lymphocytic leukemia develops suprapubic pain, dysuria, and hematuria.
Evidence of hemorrhage and inflammation is apparent on cystoscopy of the
urinary bladder. Which of the following agents most likely caused these
findings?

a. 6-MP

594 ‫صفحة‬ Dr. MOHAMED YAHIA 0900987639


QUESTION BANK OF Dr. MOHAMED YAHIA

b. Methotrexate
c. Cyclophosphamide
d. Doxorubicin
e. Carmustine

The answer is: C

50. A 45-year-old female treated for ovarian cancer develops difficulty hearing.
Which of the following agents most likely caused these findings?

a. Paclitaxel
b. Doxorubicin
c. Bleomycin
d. 5-FU
e. Cisplatin

The answer is: E

51. Of the many types of adrenergic receptors found throughout the body, which
is most likely responsible for the cardiac stimulation that is observed following
an intravenous injection of epinephrine?

a. α 1 -adrenergic receptors
b. α 2 -adrenergic receptors
c. β 1 -adrenergic receptors
d. β 2 -adrenergic receptors
e. β 3 -adrenergic receptors

The answer is: C

52. The nonselective β-adrenergic blocking agent that is also a competitive


antagonist at α 1 -adrenoceptors is

a. Timolol
b. Nadolol
c. Pindolol
d. Acebutolol
e. Labetalol

The answer is: E

53. The reversible cholinesterase inhibitor indicated in the treatment of


Alzheimer’s disease is

a. Tacrine
b. Edrophonium
c. Neostigmine
d. Pyridostigmine

595 ‫صفحة‬ Dr. MOHAMED YAHIA 0900987639


QUESTION BANK OF Dr. MOHAMED YAHIA

e. Ambenonium

The answer is: A

54. Hypotension, bradycardia, respiratory depression, and muscle weakness, all


unresponsive to atropine and neostigmine, would most likely be due to

a. Diazoxide
b. Isofluorphate
c. Tubocurarine
d. Nicotine
e. Pilocarpine

The answer is: D

55. Ritodrine hydrochloride is used in the treatment of

a. Parkinson’s disease
b. Bronchial asthma
c. Depression
d. Hypertension
e. Premature labor

The answer is: E

56. The skeletal muscle relaxant that acts directly on the contractile mechanism
of the muscle fibers is

a. Gallamine
b. Baclofen
c. Pancuronium
d. Cyclobenzaprine
e. Dantrolene

The answer is: E

57. A predictably dangerous side effect of nadolol that constitutes a


contraindication to its clinical use in susceptible patients is the induction of

a. Hypertension
b. Cardiac arrhythmia
c. Asthmatic attacks
d. Respiratory depression
e. Hypersensitivity

The answer is: C

596 ‫صفحة‬ Dr. MOHAMED YAHIA 0900987639


QUESTION BANK OF Dr. MOHAMED YAHIA

58. All of the following drugs are used topically in the treatment of chronic wide-
angle glaucoma. Which of these agents reduces intraocular pressure by
decreasing the formation of the aqueous humor?

a. Timolol
b. Echothiophate
c. Pilocarpine
d. Isofluorphate
e. Physostigmine

The answer is: A

59. The cholinomimetic drug that is useful for treating postoperative abdominal
distention and gastric atony is

a. Acetylcholine (ACh)
b. Methacholine
c. Carbachol
d. Bethanechol
e. Pilocarpine

The answer is: D

60. Neostigmine will effectively antagonize skeletal muscle relaxation produced


by

a. Pancuronium
b. Succinylcholine
c. Diazepam
d. Baclofen
e. Nicotine

The answer is: A

61. Pralidoxime chloride is a drug that

a. Reduces the vesicular stores of catecholamines in adrenergic and


dopaminergic neurons
b. Blocks the active transport of choline into cholinergic neurons
c. Reactivates cholinesterases that have been inhibited by organophosphate
cholinesterase inhibitors
d. Stimulates the activity of phospholipase C with increased formation of inositol
triphosphate
e. Inhibits the reuptake of biogenic amines into nerve terminals

The answer is: C

597 ‫صفحة‬ Dr. MOHAMED YAHIA 0900987639


QUESTION BANK OF Dr. MOHAMED YAHIA

62. Which of the following antimuscarinic drugs is used by inhalation in the


treatment of bronchial asthma?

a. Dicyclomine hydrochloride
b. Cyclopentolate hydrochloride
c. Ipratropium bromide
d. Methscopolamine bromide
e. Trihexyphenidyl hydrochloride

The answer is: C

63. The cholinesterase inhibitor that is used in the diagnosis of myasthenia


gravis is

a. Edrophonium chloride
b. Ambenonium chloride
c. Malathion
d. Physostigmine salicylate
e. Pyridostigmine bromide

The answer is: A

64. Epinephrine may be mixed with certain anesthetics, such as procaine, in


order to

a. Stimulate local wound repair


b. Promote hemostasis
c. Enhance their interaction with neural membranes and their ability to depress
nerve conduction
d. Retard their systemic absorption
e. Facilitate their distribution along nerves and fascial planes

The answer is: D

65. The skeletal muscles that are most sensitive to the action of tubocurarine
are the

a. Muscles of the trunk


b. Muscles of the arms and legs
c. Respiratory muscles
d. Muscles of the head, neck, and face
e. Abdominal muscles

The answer is: D

66. The drug of choice for the treatment of anaphylactic shock is

a. Epinephrine

598 ‫صفحة‬ Dr. MOHAMED YAHIA 0900987639


QUESTION BANK OF Dr. MOHAMED YAHIA

b. NE
c. Isoproterenol
d. Diphenhydramine
e. Atropine

The answer is: A

67. Both phentolamine and prazosin

a. Are competitive antagonists at α 1 -adrenergic receptors


b. Have potent direct vasodilator actions on vascular smooth muscle
c. Enhance gastric acid secretion through a histamine-like effect
d. Cause hypotension and bradycardia
e. Are used chronically for the treatment of primary hypotension

The answer is: A

68. A 60-year-old male with congestive heart failure (CHF) is treated with
dobutamine. Select the mechanism of action of dobutamine.

a. α-adrenergic agonist
b. α-adrenergic antagonist
c. β-adrenergic agonist
d. β-adrenergic antagonist
e. Mixed α and β agonist
f. Mixed α and β antagonist

The answer is: E

69. A 58-year-old male with angina is treated with atenolol. Select the mechanism
of action of atenolol.

a. α-adrenergic agonist
b. α-adrenergic antagonist
c. β-adrenergic agonist
d. β-adrenergic antagonist
e. Mixed α and β agonist
f. Mixed α and β antagonist

The answer is: D

70. A 75-year-old female with CHF is treated with carvedilol. Select the
mechanism of action of carvedilol.

a. α-adrenergic agonist
b. α-adrenergic antagonist
c. β-adrenergic agonist
d. β-adrenergic antagonist

599 ‫صفحة‬ Dr. MOHAMED YAHIA 0900987639


QUESTION BANK OF Dr. MOHAMED YAHIA

e. Mixed α and β agonist


f. Mixed α and β antagonist

The answer is: F

71. A 35-year-old male with a pheochromocytoma is treated with labetalol. Select


the mechanism of action of labetalol.

a. α-adrenergic agonist
b. α-adrenergic antagonist
c. β-adrenergic agonist
d. β-adrenergic antagonist
e. Mixed α and β agonist
f. Mixed α and β antagonist

The answer is: F

72. Which of the following agents will increase pulse pressure?

a. Metoprolol
b. Dopamine
c. Isoproterenol
d. Epinephrine
e. Albuterol

The answer is: D

73. A male patient is brought to the emergency department (ED) following


ingestion of an unknown substance. He is found to have an elevated temperature,
hot and flushed skin, dilated pupils, and tachycardia. Of the following, which
would most likely cause these findings?

a. Propranolol
b. Methylphenidate
c. Prazosin
d. Guanethidine
e. Atropine

The answer is: E

74. Of the following structures, which does not respond to β-adrenergic receptor
stimulation?

a. Ciliary muscle of the iris


b. Radial muscle of the iris
c. Bronchial muscle
d. Atrioventricular (AV) node
e. Sinoatrial (SA) node

600 ‫صفحة‬ Dr. MOHAMED YAHIA 0900987639


QUESTION BANK OF Dr. MOHAMED YAHIA

The answer is: B

75. A 16-year-old male treated for bronchial asthma develops skeletal muscle
tremors. Which of the following agents may be responsible for this finding?

a. Ipratropium
b. Zileuton
c. Beclomethasone
d. Cromolyn
e. Salmeterol

The answer is: E

76. Of the following, which will not be blocked by atropine and scopolamine?

a. Bradycardia
b. Salivary secretion
c. Bronchoconstriction
d. Skeletal muscle contraction
e. Miosis

The answer is: D

77. Which of the following agents should a patient take for a stuffy, runny nose?

a. Oxymetazoline
b. Albuterol
c. Clonidine
d. Terbutaline
e. Metoprolol

The answer is: A

78. A 65-year-old male has a blood pressure of 170/105 mmHg. Which of the
following would be effective in lowering this patient’s blood pressure?

a. Methylphenidate
b. Terbutaline
c. Dobutamine
d. Pancuronium
e. Prazosin
f. Scopalamine

The answer is: E

79. A 10-year-old male displays hyperactivity and is unable to focus on his


schoolwork because of an inability to focus on the activity. Which of the
following might prove effective in this patient?

601 ‫صفحة‬ Dr. MOHAMED YAHIA 0900987639


QUESTION BANK OF Dr. MOHAMED YAHIA

a. Methylphenidate
b. Terbutaline
c. Dobutamine
d. Pancuronium
e. Prazosin
f. Scopalamine

The answer is: A

80. Nicotine in low doses may cause

a. Decreased tone and motor activity of the small intestine


b. Stimulation of the respiratory rate and depth
c. Miosis
d. Bradycardia

The answer is: B

81. Which of the following agents might mask the hypoglycemia in treated
diabetics?

a. An α-adrenergic agonist
b. An α-adrenergic antagonist
c. A β-adrenergic agonist
d. A β-adrenergic antagonist
e. A cholinergic agonist
f. A cholinergic antagonist

The answer is: D

82. Of the following effects, which is not elicited by activation of the


parasympathetic nervous system?

a. Decreased heart rate


b. Increased tone of longitudinal smooth muscles of the intestine
c. Contraction of skeletal muscles
d. Contraction of the detrusor of the urinary bladder
e. Secretion of fluid from the lacrimal glands

The answer is: C

83. Which of the following occurs in the treatment of glaucoma with a β-


adrenergic antagonist?

a. Decreased aqueous humor secretion


b. Pupillary dilator muscle fiber contraction
c. Dilation of the uveoscleral veins
d. Direct opening of the trabecular meshwork

602 ‫صفحة‬ Dr. MOHAMED YAHIA 0900987639


QUESTION BANK OF Dr. MOHAMED YAHIA

e. Circular pupillary constrictor muscle contraction

The answer is: A

84. A 75-year-old female in congestive heart failure (CHF) is unable to climb a


flight of stairs without experiencing shortness of breath. Digoxin is administered
to improve cardiac muscle contractility. Within two weeks, she has a marked
improvement in her symptoms. What cellular action of digoxin accounts for
this?

a. Inhibition of cyclic adenosine 5′-monophosphate (cAMP) synthesis


b. Inhibition of mitochondrial calcium (Ca 2+ ) release
c. Inhibition of the sodium (Na + ) pump
d. Inhibition of β-adrenergic stimulation
e. Inhibition of adenosine triphosphate (ATP) degradation

The answer is: C

85. In a patient who has had attacks of paroxysmal atrial tachycardia, an ideal
prophylactic drug is

a. Adenosine
b. Procainamide
c. Lidocaine
d. Nifedipine
e. Verapamil

The answer is: E

86. The therapeutic action of β-adrenergic receptor blockers such as propranolol


in angina pectoris is believed to be primarily the result of

a. Reduced production of catecholamines


b. Dilation of the coronary vasculature
c. Decreased requirement for myocardial oxygen
d. Increased peripheral resistance
e. Increased sensitivity to catecholamines

The answer is: C

87. A 59-year-old female with mild CHF is treated with furosemide. What is its
primary mechanism of action?

a. Inhibition of sodium-potassium (Na + ,K + ) adenosine triphosphatase


(ATPase)
b. Inhibition of Na + ,K + , chloride (Cl − ) co-transporter
c. Inhibition of Na + ,Cl − co-transporter
d. Inhibition of Cl − transporter

603 ‫صفحة‬ Dr. MOHAMED YAHIA 0900987639


QUESTION BANK OF Dr. MOHAMED YAHIA

e. Inhibition of Ca 2+ divalent cation (Ca 2+ ) transporter

The answer is: B

88. A positive Coombs’ test and hemolytic anemia may follow the administration
of which antihypertensive drug?

a. Methyldopa
b. Clonidine
c. Guanabenz
d. Prazosin
e. Captopril

The answer is: A

89. Which of the following is an antiarrhythmic agent that has relatively few
electrophysiologic effects on normal myocardial tissue but suppresses the
arrhythmogenic tendencies of ischemic myocardial tissues?

a. Propranolol
b. Procainamide
c. Quinidine
d. Lidocaine
e. Disopyramide

The answer is: D

90. A 59-year-old male with a history of rheumatic heart disease is found to have
atrial fibrillation (AF), for which he is treated with digoxin. Treatment with
digoxin converts his AF to a normal sinus rhythm and most likely results in a
decrease in which of the following?

a. The length of the refractory period


b. The velocity of shortening of the cardiac muscle
c. The conduction velocity in the atrioventricular (AV) node
d. The atrial maximum diastolic resting potential

The answer is: C

91. A 65-year-old female receives digoxin and furosemide for CHF . After several
months, she develops nausea and vomiting. Serum K + is 2.5 mEq/L.
Electrocardiogram (EKG) reveals an AV conduction defect. What cellular effect
is causing these new findings?

a. Increased intracellular K +
b. Increased intracellular cyclic guanosine 5′-monophosphate (cGMP)
c. Increased intracellular Ca 2+
d. Increased intracellular norepinephrine

604 ‫صفحة‬ Dr. MOHAMED YAHIA 0900987639


QUESTION BANK OF Dr. MOHAMED YAHIA

e. Increased intracellular nitric oxide (NO)

The answer is: C

92. Which of the following drugs recommended for the lowering of blood
cholesterol inhibits the synthesis of cholesterol by blocking 3-hydroxy-3-
methylglutaryl–coenzyme A (HMG-CoA) reductase?

a. Lovastatin
b. Probucol
c. Clofibrate
d. Gemfibrozil
e. Nicotinic acid (NA)

The answer is: A

93. The EKG of a patient who is receiving digitalis in the therapeutic dose range
would be likely to show

a. Prolongation of the QT interval


b. Prolongation of the PR interval
c. Symmetric peaking of the T wave
d. Widening of the QRS complex
e. Elevation of the ST segment

The answer is: B

94. A 45-year-old male takes simvastatin for hypercholesterolemia; how- ever,


his cholesterol level remains above target at maximal doses. Cholestyramine is
added to the therapeutic regimen. What drug-drug interaction can occur?

a. The combination will not lower cholesterol more than either agent alone
b. The combination causes elevated very-low-density lipoprotein (VLDL)
c. Cholestyramine inhibits gastrointestinal (GI) absorption of simvastatin
d. Simvastatin is a direct antagonist of cholestyramine

The answer is: C

95. In a hypertensive patient who is taking insulin to treat diabetes, which of the
following drugs is to be used with extra caution and advice to

the patient?

a. Hydralazine
b. Prazosin
c. Guanethidine
d. Propranolol
e. Methyldopa

605 ‫صفحة‬ Dr. MOHAMED YAHIA 0900987639


QUESTION BANK OF Dr. MOHAMED YAHIA

The answer is: D

96. Which of the following drugs is considered to be most effective in relieving


and preventing ischemic episodes in patients with variant angina?

a. Propranolol
b. Nitroglycerin
c. Sodium nitroprusside
d. Nifedipine
e. Isosorbide dinitrate

The answer is: D

97. A 47-year-old male is seen in the medicine clinic with recently diagnosed
mixed hyperlipidemia. An antihyperlipidemic is administered that favorably
affects levels of VLDL, low-density lipoprotein (LDL), and high- density
lipoprotein (HDL) and inhibits cholesterol synthesis. This drug is:

a. Lovastatin
b. Colestipol
c. Niacin
d. Probucol
e. Neomycin

The answer is: C

98. If quinidine and digoxin are administered concurrently, which of the


following effects does quinidine have on digoxin?

a. The absorption of digoxin from the GI tract is decreased


b. The metabolism of digoxin is prevented
c. The concentration of digoxin in the plasma is increased
d. The effect of digoxin on the AV node is antagonized
e. The ability of digoxin to inhibit the Na + K + -stimulated ATPase is reduced

The answer is: C

992. Drugs that block the catecholamine uptake process (e.g., cocaine, tri- cyclic
antidepressants, and phenothiazines) are apt to block the antihypertensive action
of which of the following drugs?

a. Propranolol
b. Guanethidine
c. Prazosin
d. Hydralazine
e. Diazoxide

The answer is: B

606 ‫صفحة‬ Dr. MOHAMED YAHIA 0900987639


QUESTION BANK OF Dr. MOHAMED YAHIA

100. A 44-year-old obese male has a significantly high level of plasma


triglycerides. Following treatment with one of the following agents, his plasma
triglyceride levels decrease to almost normal. Which agent did he receive?

a. Neomycin
b. Lovastatin
c. Cholestyramine
d. Gemfibrozil

The answer is: D

101 A 64-year-old male with arteriosclerotic heart disease (AHD) and CHF who
has been treated with digoxin complains of nausea, vomiting, and diarrhea. His
EKG reveals a bigeminal rhythm. The symptoms and EKG findings occurred
shortly after another therapeutic agent was added to his regimen. A drug-drug
interaction is suspected. Which agent was involved?

a. Lovastatin
b. Hydrochlorothiazide
c. Phenobarbital
d. Nitroglycerin
e. Captopril

The answer is: B

102. Nicotinic acid in large doses used to treat hyperlipoproteinemia causes a


cutaneous flush. The vasodilatory effect is due to

a. Release of histamine
b. Production of local prostaglandins
c. Release of platelet-derived growth factor (PDGF)
d. Production of NO
e. Ca channel blockade

The answer is: B

103. One type of hyperlipoproteinemia is characterized by elevated plasma levels


of chylomicra, normal plasma levels of β-lipoproteins, and the inability of any
known drug to reduce lipoprotein levels. This is which of the following types of
hyperlipoproteinemia?

a. Type I
b. Type IIa, IIb
c. Type III
d. Type IV
e. Type V

The answer is: A

607 ‫صفحة‬ Dr. MOHAMED YAHIA 0900987639


QUESTION BANK OF Dr. MOHAMED YAHIA

104. A 69-year-old male with angina develops severe constipation following


treatment with

a. Propranolol
b. Captopril
c. Verapamil
d. Dobutamine
e. Nitroglycerin

The answer is: C

105. Angiotensin converting enzyme (ACE) inhibitors are associated with a high
incidence of which of the following adverse reactions?

a. Hepatitis
b. Hypokalemia
c. Agranulocytosis
d. Proteinuria
e. Hirsutism

The answer is: D

106. A 45-year-old male post–myocardial infarction (post-MI) for one week is


being treated with intravenous (IV) heparin. Stool guaiac on admission was
negative, but is now 4+, and he has had an episode of hematemesis. The heparin
is discontinued, and a drug is given to counteract the bleeding. What drug was
given?

a. Aminocaproic acid
b. Dipyridamole
c. Factor IX
d. Protamine
e. Vitamin K

The answer is: D

107 Patients with genetically low levels of N-acetyltransferase are more prone to
develop a lupus erythematosus–like syndrome with which of the following
drugs?

a. Propranolol
b. Procainamide
c. Digoxin
d. Captopril
e. Lidocaine

The answer is: B

608 ‫صفحة‬ Dr. MOHAMED YAHIA 0900987639


QUESTION BANK OF Dr. MOHAMED YAHIA

108. Which of the following anemias would be treated with cyanocobalamin


(vitamin B 12 )?

a. Anemia in infants who are undergoing rapid growth


b. Anemia associated with cheilosis, dysphagia, gastritis, and hypochlorhydria
c. Anemia associated with small, bizarre cells poorly filled with hemoglobin
d. Anemia associated with infestation by Diphyllobothrium latum
e. Bleeding from a gastric ulcer

The answer is: D

109. The preferred agent to combat extreme digoxin overdose is

a. K+
b. Ca 2+
c. Phenytoin
d. Fab fragments of digoxin antibodies
e. Magnesium (Mg 2+ )

The answer is: D

110. Significant relaxation of smooth muscle of both venules and arterioles is


produced by which of the following drugs?

a. Hydralazine
b. Minoxidil
c. Diazoxide
d. Sodium nitroprusside
e. Nifedipine

The answer is: D

111. Precautions advisable when using lovastatin include

a. Serum transaminase measurements


b. Renal function studies
c. Acoustic measurements
d. Monthly complete blood counts
e. Avoidance of bile acid sequestrants

The answer is: A

112. The first-line drug for treating an acute attack of reentrant


supraventricular tachycardia (SVT) is

a. Adenosine
b. Digoxin
c. Propranolol

609 ‫صفحة‬ Dr. MOHAMED YAHIA 0900987639


QUESTION BANK OF Dr. MOHAMED YAHIA

d. Phenylephrine
e. Edrophonium

The answer is: A

113. A 60-year-old male, following hospitalization for an acute myocardial


infarction, is treated with warfarin. What is the mechanism of action of
warfarin?

a. Increase in the plasma level of factor IX


b. Inhibition of thrombin and early coagulation steps
c. Inhibition of synthesis of prothrombin and coagulation factors VII, IX, and X
d. Inhibition of platelet aggregation in vitro
e. Activation of plasminogen
f. Binding of Ca 2+ ion cofactor in some coagulation steps

The answer is: C

114. A 39-year-old pregnant female requires heparin for thromboembolic


phenomena. What is the mechanism of action of heparin?

a. Increase in the plasma level of factor IX


b. Inhibition of thrombin and early coagulation steps
c. Inhibition of synthesis of prothrombin and coagulation factors VII, IX, X
d. Inhibition of platelet aggregation in vitro
e. Activation of plasminogen
f. Binding of Ca 2+ ion cofactor in some coagulation steps

The answer is: B

116. A 42-year-old male with an acute MI is treated with alteplase. What is the
mechanism of action of alteplase?

a. Inhibition of platelet thromboxane production


b. Antagonism of ADP receptor
c. Glycoprotein IIb/IIIa antagonist
d. Inhibition of the synthesis of vitamin K–dependent coagulation factors
e. Activation of plasminogen from plasmin

The answer is: E

116. A 65-year-old male with a previous history of a stroke is treated with


ticlopidine as prophylaxis for preventing further stroke. What is the mechanism
of action of ticlopidine?

a. Inhibition of platelet thromboxane production


b. Antagonism of ADP receptor
c. Antagonism of glycoprotein IIb/IIIa

610 ‫صفحة‬ Dr. MOHAMED YAHIA 0900987639


QUESTION BANK OF Dr. MOHAMED YAHIA

d. Inhibition of the synthesis of vitamin K–dependent coagulation factors


e. Activation of plasminogen to plasmin

The answer is: B

117. A 40-year-old female is to have angioplasty following an acute MI. As part


of her treatment, she is given intravenously administered eptifibatide. What is
the mechanism of action of eptifibatide?

a. Inhibition of platelet thromboxane production


b. Antagonism of ADP receptor
c. Antagonism of glycoprotein IIb/IIIa
d. Inhibition of the synthesis of vitamin K–dependent coagulation factors
e. Activation of plasminogen from plasmin

The answer is: C

118. Administration of which of the following antianginal agents results in


antianginal effects for only 10 hours, despite detectable therapeutic plasma levels
for 24 hours?

a. Atenolol
b. Transdermal nitroglycerin
c. Amlodipine
d. Amyl nitrite

The answer is: B

119. A 70-year-old female is treated with sublingual nitroglycerin for her


occasional bouts of angina. Which of the following is involved in the action of
nitroglycerin?

a. α-adrenergic activity
b. Phosphodiesterase activity
c. Phosphorylation of light chains of myosin
d. Norepinephrine release
e. cGMP

The answer is: E

120. A 56-year-old female has recently developed essential hypertension, for


which she is receiving chlorothiazide to lower her blood pressure. Which of these
ions would not increase in concentration in her urine?

a. K+
b. Cl −
c. Ca 2+
d. Na +

611 ‫صفحة‬ Dr. MOHAMED YAHIA 0900987639


QUESTION BANK OF Dr. MOHAMED YAHIA

e. Mg 2+

The answer is: C

121. A 60-year-old female with deep-vein thrombosis (DVT) is given a bolus of


heparin, and a heparin drip is also started. Thirty minutes later, she is bleeding
profusely from the intravenous site. The heparin is stopped, but the bleeding
continues. You decide to give protamine to reverse the adverse effect of heparin.
How does protamine act?

a. It causes hydrolysis of heparin


b. It changes the conformation of antithrombin III to prevent binding to heparin
c. It activates the coagulation cascade, overriding the action of heparin
d. It combines with heparin as an ion pair, thus inactivating it

The answer is: D

122. A 47-year-old female comes to the emergency department (ED) with severe
crushing chest pain of one hour’s duration. Electrocardiogram and blood
chemistries are consistent with a diagnosis of acute MI. Streptokinase is chosen
as part of the therapeutic regimen. What is its mechanism of action?

a. It activates the conversion of fibrin to fibrin-split products


b. It activates the conversion of plasminogen to plasmin
c. It inhibits the conversion of prothrombin to thrombin
d. It inhibits the conversion of fibrinogen to fibrin

The answer is: B

123. Of the following agents, which is best avoided in a patient with a history of
chronic congestive heart failure (CHF)?

a. Hydrochlorothiazide
b. Amiloride
c. Mannitol
d. Ethacrynic acid
e. Spironolactone

The answer is: C

124. Furosemide inhibits the sodium-potassium-dichloride (Na + , K + , 2Cl 2 − )


co-transporters that are located in the

a. Collecting duct
b. Ascending limb of the loop of Henle
c. Descending limb of the loop of Henle
d. Proximal tubule
e. Distal convoluted tubule

612 ‫صفحة‬ Dr. MOHAMED YAHIA 0900987639


QUESTION BANK OF Dr. MOHAMED YAHIA

The answer is: B

125. Of the following agents, which is best avoided when a patient is being
treated with an aminoglycoside?

a. Metolazone
b. Triamterene
c. Furosemide
d. Spironolactone
e. Acetazolamide

The answer is: C

126. Hyperkalemia is a contraindication to the use of which of the following


drugs?

a. Acetazolamide
b. Chlorothiazide
c. Ethacrynic acid
d. Chlorthalidone
e. Spironolactone

The answer is: E

127 A reduction in insulin release from the pancreas may be caused by which of
the following diuretics?

a. Triamterene
b. Chlorothiazide
c. Spironolactone
d. Acetazolamide
e. Amiloride

The answer is: B

128. Acute uric acid nephropathy, which is characterized by the acute


overproduction of uric acid and by extreme hyperuricemia, can best be
prevented with which of the following?

a. Antidiuretic hormone (ADH) [vasopressin (VP)]


b. Cyclophosphamide
c. Allopurinol
d. Amiloride
e. Sodium chloride (NaCl)

The answer is: C

613 ‫صفحة‬ Dr. MOHAMED YAHIA 0900987639


QUESTION BANK OF Dr. MOHAMED YAHIA

129. The release of ADH is suppressed by which of the following drugs to


promote a diuresis?

a. Guanethidine
b. Acetazolamide
c. Chlorothiazide
d. Ethanol
e. Indomethacin

The answer is: D

130. Conservation of K ions in the body occurs with which of the following
diuretics?

a. Furosemide
b. Hydrochlorothiazide
c. Triamterene
d. Metolazone
e. Bumetanide

The answer is: C

131. Spironolactone can be characterized by which one of the following


statements?

a. It is biotransformed to an inactive product


b. It binds to a cytoplasmic receptor
c. It is a more potent diuretic than is hydrochlorothiazide
d. It interferes with aldosterone synthesis
e. It inhibits Na reabsorption in the proximal renal tubule of the nephron

The answer is: B

132. Which of the following diuretics could be added to the therapeutic regimen
of a patient who is receiving a direct vasodilator for the treatment of
hypertension?

a. Acetazolamide
b. Triamterene
c. Spironolactone
d. Mannitol
e. Hydrochlorothiazide

The answer is: E

614 ‫صفحة‬ Dr. MOHAMED YAHIA 0900987639


QUESTION BANK OF Dr. MOHAMED YAHIA

133. A patient develops acute gout following treatment with which of the
following?

a. Acetazolamide
b. Allopurinol
c. Triamterene
d. Spironolactone
e. Furosemide

The answer is: E

124. A patient with nephrogenic diabetes insipidus is best treated with which of
the following?

a. Hydrochlorothiazide
b. Triamterene
c. Furosemide
d. Spironolactone
e. Acetazolamide

The answer is: A

135. Which of the following is unlikely to cause a drug-drug interaction with the
thiazides?

a. Adrenal corticosteroids
b. Anticoagulants (oral)
c. Aminoglycosides
d. Uricosuric agents
e. Insulin

The answer is: C

136. Of the following diuretic agents, which would be least likely to indirectly
cause an increased binding of digoxin to cardiac tissue sodium– potassium–
adenosine triphosphatase (Na + ,K + ,ATPase)?

a. Hydrochlorothiazide
b. Furosemide
c. Amiloride
d. Acetazolamide
e. Metolazone

The answer is: C

615 ‫صفحة‬ Dr. MOHAMED YAHIA 0900987639


QUESTION BANK OF Dr. MOHAMED YAHIA

137. A patient with compromised renal hemodynamics is given a trial of


mannitol. Of the following, which is the least likely to be associated with the
effect of mannitol?

a. Retention of water in the tubular fluid


b. Ability to be metabolically altered to an active form
c. Capacity to be freely filtered
d. Effectiveness as nonelectrolytic, osmotically active particles
e. Ability to resist complete reabsorption by the renal tubule

The answer is: B

138. A patient develops hyperglycemia, hyperuricemia, and hypomagnesemia on


which of the following diuretic agents?

a. Hydrochlorothiazide
b. Triamterene
c. Spironolactone
d. Acetazolamide
e. Amiloride

The answer is: A

139. Of the following, which adverse reaction is not associated with furosemide?

a. Hyperglycemia
b. Tinnitus
c. Fluid and electrolyte imbalance
d. Hypotension
e. Metabolic acidosis

The answer is: E

140. A 35-year-old male has renal stones and increased calcium (Ca) in the urine
that is associated with normal serum Ca and parathyroid hormone levels. Which
of the following agents could be used to treat this patient?

a. Furosemide
b. Acetazolamide
c. Triamterene
d. Hydrochlorothiazide
e. Vasopressin

The answer is: D

141. In which of the following are thiazide diuretics ineffective?

a. Edema caused by CHF

616 ‫صفحة‬ Dr. MOHAMED YAHIA 0900987639


QUESTION BANK OF Dr. MOHAMED YAHIA

b. Edema induced by glucocorticoids


c. Hypertension with or without edema
d. Ascites
e. Glaucoma

The answer is: E

617 ‫صفحة‬ Dr. MOHAMED YAHIA 0900987639


QUESTION BANK OF Dr. MOHAMED YAHIA

BRS PHARMACOLOGY

1 . Somatostatin interacts with

(A) G i -protein–coupled receptor


(B) G q -protein–coupled receptor
(C) Ligand-activated ion channel
(D) Receptor-activated tyrosine kinase
(E) Intracellular nuclear receptor

The answer is: A

2 . Cortisol is capable of targeting intranuclear receptors secondary to its ability


to

(A) Recruit intracellular kinases


(B) Undergo autophosphorylation
(C) Diffuse through lipid membranes
(D) Interact with G-proteins
(E) Interact with adenylyl cyclase

The answer is: C

3 . Which of the following parameters is used to indicate the ability of a drug to


produce the desired therapeutic effect relative to a toxic effect?

(A) Potency
(B) Intrinsic activity
(C) Therapeutic index
(D) Efficacy
(E) Bioavailability

The answer is: C

4 . A 64-year-old woman with a history of multiple abdominal surgeries due to


Crohn’s disease presents to the emergency room with constipation and feculent
emesis. A diagnosis of small bowel obstruction is made, and she is taken to the
operating room for lysis of adhesions and resection of stenosed region of small
bowel. Postoperatively, the patient is noted to have elevated blood pressure, and
oral metoprolol is administered; however, no improvement of hypertension is
observed. This is likely due to

(A) The first-pass effect


(B) Decreased passage of drug through intestine
(C) Decreased GI blood flow

618 ‫صفحة‬ Dr. MOHAMED YAHIA 0900987639


QUESTION BANK OF Dr. MOHAMED YAHIA

(D) Destruction of drug by stomach acid


(E) Increased protein binding of the drug

The answer is: B

5 . An important feature of congestive heart failure (CHF) regarding drug action


is

(A) Impaired blood flow to the intestine


(B) Increased protein binding of various drugs
(C) Increased volume of distribution
(D) Increased drug elimination
(E) Altered drug kinetics

The answer is: C

6 . Which of the following is the term used to describe the elimination rate via
metabolism catalyzed by alcohol dehydrogenase when the enzyme is saturated?

(A) Zero-order kinetics


(B) First-order elimination
(C) Clearance
(D) Biotransformation
(E) Redistribution

The answer is: A

7 . A 69-year-old woman is being treated in the intensive care unit for presumed
staphylococcal sepsis. To avoid problems with possible resistance, she is
empirically given IV vancomycin while waiting for the culture results to come
back. Vancomycin is a renally excreted drug. The patient’s routine laboratory
work-up reveals a creatinine value of 3.2, indicating acute renal failure. What
specific considerations will have to be made with regard to adjustments of the
prescribed medication?

(A) She will have to be switched to an oral (per nasogastric tube)


vancomycin preparation
(B) The patient will need to be water restricted to decrease the volume of
distribution
(C) No changes to the current regimen will be made because the condition
of the patient is life-threatening and the drug needs to be administered
regardless
(D) The dose of vancomycin will need to be reduced because of increased
accumulation
(E) Dosage adjustments will have to be made because the patient is
currently ventilated

619 ‫صفحة‬ Dr. MOHAMED YAHIA 0900987639


QUESTION BANK OF Dr. MOHAMED YAHIA

The answer is: D

8 . Glucuronidation reactions

(A) Are considered phase I reactions


(B) Require an active center as the site of conjugation
(C) Include the enzymatic activity of alcohol dehydrogenase
(D) Located in mitochondria are inducible by drugs
(E) Require nicotinamide adenine dinucleotide phosphate (NADPH) for the
enzymatic reaction

The answer is: B

9 . A 38-year-old woman presents to her psychiatrist with a request to try a


different antidepressant medication, since she doesn’t feel her current
medication is helping. She even felt so depressed that she started drinking
heavily in the past couple of months. The doctor wants to try imipramine;
however, since this drug is known to undergo an extensive first-pass effect, he
orders a hepatic function panel before prescribing it, given the patient’s recent
history of alcohol use. What is the rationale for the doctor’s decision?

(A) In the presence of hepatic dysfunction, drugs with a high first-pass


metabolism reach high systemic concentrations
(B) The results of the hepatic function panel may reveal a particular
susceptibility to the drug
(C) Bioavailability of imipramine is increased by the fraction of drug removed
by the first pass
(D) The drug is more rapidly metabolized by the liver when hepatic
aminotransferase levels are elevated
(E) Solubility of the drug is affected in the face of hepatic damage

The answer is: A

10 . A 43-year-old man who was recently fired from a well-paying job decides to
commit suicide and ingests a jarful of his antiseizure medication, phenobarbital.
His wife finds him at home sleeping, but notices that he has diminished
breathing, low body temperature, and skin reddening. She brings him to the ER,
where he is appropriately diagnosed with barbiturate overdose. The patient is
given bicarbonate to alkalinize his urine. How does alkalinization of urine with
bicarbonate help to overcome the toxic effects of Phenobarbital in this situation?

(A) It increases glomerular filtration


(B) It decreases proximal tubular secretion
(C) It decreases distal tubular reabsorption
(D) It enhances drug metabolism
(E) It decreases untoward side effects

620 ‫صفحة‬ Dr. MOHAMED YAHIA 0900987639


QUESTION BANK OF Dr. MOHAMED YAHIA

The answer is: C

11 . Erythromycin is prescribed ‘‘qid,’’ or four times daily, because of its short


half-life. The rationale for such a frequent dosing schedule is

(A) To achieve the steady-state plasma concentration of the drug


(B) To avoid the toxicity of the drug because of its low therapeutic index
(C) To aid more complete distribution of the drug
(D) To inhibit the first-pass metabolism of the drug
(E) To ensure that the drug concentration remains constant over time

The answer is: A

12 . A 13-year-old boy suffers two tonic-clonic seizures within 1 week. He is


diagnosed with epilepsy, and phenytoin therapy is started. To achieve proper
drug concentrations in plasma, the patient is first given a loading dose, followed
by maintenance doses. The blood level of phenytoin is frequently monitored to
adjust the maintenance dose as needed. What is the rationale behind such a
regimen?

(A) If drug is administered at a maintenance dose rate, steady-state


concentration will be reached after two half-lives
(B) A loading dose is administered to achieve the desired plasma
concentration rapidly
(C) The maintenance dose rate usually does not equal the elimination rate,
which is why the loading dose is required
(D) Loading dose of the drug does not depend on the volume of distribution,
whereas the maintenance dose does
(E) The maintenance dose rate does not depend on clearance of the drug,
whereas the loading dose does

The answer is: B

13 . Following IV administration, the initial rates of drug distribution to


different tissues depend primarily on which of the following parameters?

(A) Blood flow to the tissues


(B) Fat content of the tissues
(C) Degree of ionization of the drug in the tissues
(D) Active transport of the drug out of different cell types
(E) Specific organ clearances

The answer is: A

14 . A drug is administered in the form of an inactive pro-drug. The pro-drug


increases the expression of a cytochrome P-450 that converts the pro-drug to its

621 ‫صفحة‬ Dr. MOHAMED YAHIA 0900987639


QUESTION BANK OF Dr. MOHAMED YAHIA

active form. With chronic, long-term administration of the pro-drug, which of


the following will be observed?

(A) The potency will decrease


(B) The potency will increase
(C) The efficacy will decrease
(D) The efficacy will increase

The answer is: B

15 . If the oral dosing rate of a drug is held constant, what will be the effect of
increasing the bioavailability of the preparation?

(A) Increase the half-life for first-order elimination


(B) Decrease the first-order elimination rate constant
(C) Increase the steady-state plasma concentration
(D) Decrease the total body clearance
(E) Increase the volume of distribution

The answer is: C

16 . You administer to a patient an oral maintenance dose of drug calculated to


achieve a steady-state plasma concentration of 5 mcg/L. After dosing the patient
for a time sufficient to reach steady state, the average plasma concentration of
drug is 10 mcg/L. A decrease in which of the following parameters explains this
higher than anticipated plasma drug concentration?

(A) Bioavailability
(B) Volume of distribution
(C) Clearance
(D) Half-life

The answer is: C

17 . Which of the following factors will determine the number of drug–receptor


complexes formed?

(A) Efficacy of the drug


(B) Receptor affinity for the drug
(C) Therapeutic index of the drug
(D) Half-life of the drug
(E) Rate of renal secretion

The answer is: B

18 . Which of the following is an action of a noncompetitive antagonist?

(A) Alters the mechanism of action of an agonist

622 ‫صفحة‬ Dr. MOHAMED YAHIA 0900987639


QUESTION BANK OF Dr. MOHAMED YAHIA

(B) Alters the potency of an agonist


(C) Shifts the dose–response curve of an agonist to the right
(D) Decreases the maximum response to an agonist
(E) Binds to the same site on the receptor as the agonist

The answer is: D

19. A 35-year-old diabetic woman presents to the emergency room with signs and
symptoms of urinary tract infection, including fever, dysuria, and bacteriuria.
Given that she is diabetic, she is admitted for treatment with intravenous
ciprofloxacin. What is the mechanism of this drug?

(A) Inhibition of the 30s ribosome


(B) Inhibition of the 50s ribosome
(C) Inhibition of bacterial cell wall synthesis
(D) Inhibition of RNA synthesis
(E) Inhibition of DNA gyrase

The answer is: E

20. A 25-year-old man presents with recurrent bouts of hypoglycemia with


mental status changes that are rapidly reversed by eating. He is not diabetic, and
his serum levels of insulin are markedly elevated. His C-peptide levels are also
elevated. You begin treating the patient for a presumed insulinoma with which of
the following agents?

(A) Cyclophosphamide
(B) Melphalan
(C) Carmustine
(D) Thiotepa
(E) Streptozocin

The answer is: E

21. A 73-year-old woman with breast cancer and history of congestive heart
failure is placed on a chemotherapy regimen that includes the use of
methotrexate (MTX) after her mastectomy. This agent’s activity is related to its
ability to do what?

(A) Carbamylate intracellular macromolecules


(B) Indirectly inhibit DNA synthesis
(C) Block chromosomal migration and cell differentiation
(D) Complex with DNA to form crosslinks
(E) Inhibit estrogen-dependent tumor growth

The answer is: B

623 ‫صفحة‬ Dr. MOHAMED YAHIA 0900987639


QUESTION BANK OF Dr. MOHAMED YAHIA

22. A 53-year-old man presents with changes in bowel frequency and pencil-thin
stools with occasional bright red blood in the stool. A further work-up, including
computed tomography (CT) scanning of the chest, abdomen, and pelvis,
demonstrates lesions consistent with metastasis in the liver. His therapy will
likely include which of the following chemotherapeutic agents?

(A) Carmustine
(B) Fluorouracil
(C) Leuprolide
(D) Temozolamide
(E) Tamoxifen

The answer is: B

23. A 53-year-old woman with breast cancer undergoes a breast-conserving


lumpectomy and lymph node biopsy. The pathology report returns with mention
of cancer cells in two of eight lymph nodes removed. Following radiation
therapy, chemotherapy is started that includes the use of paclitaxel. Which side
effect is the patient likely to complain of?

(A) Blood in the urine


(B) Easy bruising
(C) Hot flashes
(D) Shortness of breath
(E) Numbness and tingling

The answer is: E

24. A 74-year-old man with a 100-pack/year history of smoking is evaluated for


hemoptysis. A computed tomography (CT) scan of the chest shows numerous
pulmonary nodules. A nodule on the pleural surface is selected for CT-guided
biopsy by the interventional radiologist. The biopsy report is small-cell
carcinoma of the lung, and chemotherapy containing etoposide is started. This
drug works by

(A) Inhibiting topoisomerase II


(B) Inhibiting dihydrofolate reductase
(C) Alkylating double-stranded DNA
(D) Stabilizing microtubules, with resultant mitotic arrest
(E) Causing DNA chain scission and fragmentation

The answer is: A

25. A 56-year-old woman with metastatic breast cancer is started on


chemotherapy. Her initial treatment will include both cyclophosphamide and
doxorubicin. Careful attention is required because of doxorubicin’s well-
documented toxicity, which is

624 ‫صفحة‬ Dr. MOHAMED YAHIA 0900987639


QUESTION BANK OF Dr. MOHAMED YAHIA

(A) Hemorrhagic cystitis


(B) Acne
(C) Peripheral neuropathy
(D) Hot flashes
(E) Cardiomyopathy

The answer is: E

26. A world-class cyclist was diagnosed with metastatic testicular cancer with
lesions in both his lung and brain. He forgoes the standard treatment for his
condition because he learns one of the drugs typically used for his condition
could ultimately compromise his pulmonary function. Which of the following is
included in the standard regimen and is associated with his feared complication?

(A) Cisplatin
(B) Busulfan
(C) Aminoglutethimide
(D) Bleomycin
(E) Cyclophosphamide

The answer is: D

27. A 35-year-old otherwise healthy man presents with fullness in the inguinal
region with swelling of the ipsilateral leg. A computed tomography (CT) scan
demonstrates several confluent enlarged lymph nodes. Biopsy specimens
demonstrate malignant CD20 + B cells. A diagnosis of diffuse B-cell lymphoma is
made. Which of the following biologics will likely be given to the patient?

(A) Traztuzumab
(B) Rituxan
(C) Dactinomycin
(D) l-Asparaginase
(E) Interferon-a

The answer is: B

28. A 56-year-old man complains of fatigue and malaise. On physical


examination he has significant splenomegaly. His white blood cell count is
dramatically elevated, and the physician suspects leukemia. Chromosomal
studies indicate a (9:22) translocation, the Philadelphia chromosome, confirming
the diagnosis of chronic myelocytic leukemia (CML). Which of the following
might be used in his treatment?

(A) Anastrozole
(B) Rituximab
(C) Imatinib
(D) Gefitinib

625 ‫صفحة‬ Dr. MOHAMED YAHIA 0900987639


QUESTION BANK OF Dr. MOHAMED YAHIA

(E) Amifostine

The answer is: C

29. A 37-year-old man presents with changes in Bowel habits for the last several
months. He comPlains of small stool caliber alongwith occasional blood in his
stools. Colonoscopy reveals the diagnosis of colon adenocarcinoma. Further
work-up demonstrates that there are metastatic lesions in His liver. The
oncologist recommends the use of becacizamab. This agent

(A) Inhibits cell cycle progression


(B) Induces differentiation of cells
(C) Blocks signaling by EGFR
(D) Inhibits angiogenesis
(E) Inhibits HER2/neu signaling

The answer is: D

30. A 54-year-old woman complains of headaches, nausea, and vomiting. A


computed tomography (CT) scan of the head reveals a large mass in the frontal
lobe. She underwent surgery to remove the mass, which was shown to be a
glioblastoma multiforme(GBM). In addition to receiving radiation, which agent
should be given?

(A) Thalidomide
(B) Cisplatin
(C) Thioguanine
(D) Temozolomide
(E) Mercaptopurine

The answer is: D

31. A 63-year-old African-American man with a history of prostate cancer had


his prostate removed 10 years ago. His prostate-specific antigen levels have
begun to rise again, and he complains of back pain, suggesting metastatic
disease. A computed tomography (CT) scan demonstrates enlarged paraaortic
lymph nodes and osteoblastic lesions of his lumbar spine. Therapy with which
agent should be started?

(A) Anastrozole
(B) Leuprolide
(C) Tamoxifen
(D) Mitotane
(E) Prednisone

The answer is: B

626 ‫صفحة‬ Dr. MOHAMED YAHIA 0900987639


QUESTION BANK OF Dr. MOHAMED YAHIA

32. A 56-year-old woman with a significant smoking history was diagnosed with
small-cell lung cancer 2 years ago and was successfully treated. Now on follow-
up computed tomography (CT) scan there are several new pulmonary nodules,
and the oncologist elects to begin second-line chemotherapy with a DNA
topoisomerase I inhibitor. Which of the following is such an agent?

(A) Ciprofloxacin
(B) Etoposide
(C) Vinorelbine
(D) Teniposide
(E) Irinotecan

The answer is: E

33. A 42-year-old premenopausal woman recently underwent partial mastectomy


and radiation therapy for a small tumor in her breast. There were no lymph
nodes involved, and the tumor was estrogen-receptor positive. The oncologist
explains that there is little advantage to adding systemic chemotherapy in such
an early-stage cancer but does recommend that the patient take tamoxifen.
Which of the following is a concerning side effect of tamoxifen?

(A) Thromboembolism
(B) Bowel perforation
(C) Aplastic anemia
(D) Myelosuppression
(E) Hypotension

The answer is: A

627 ‫صفحة‬ Dr. MOHAMED YAHIA 0900987639


QUESTION BANK OF Dr. MOHAMED YAHIA

CHAPTER 6

497 Question

628 ‫صفحة‬ Dr. MOHAMED YAHIA 0900987639


QUESTION BANK OF Dr. MOHAMED YAHIA

PRETEST ANATOMY

1. Which of the following is a characteristic of the female (compared with the


male) pelvis?

A heart-shaped (as opposed to an oval-shaped) pelvic inlet


A relatively deep (as opposed to shallow) false pelvis with ilia that are flared
A pelvic outlet of smaller diameter
A subpubic angle of about 85°

The answer is: D

2. The gubernaculum is a continuous mesenchymal condensation extending from


the caudal pole of each gonad through the inguinal canal to the labioscrotal
swelling, inferiorly. In the female the gubernaculums becomes which of the
following?

Canal of Nuck
Ligament of the ovary or proper ligament of the ovary
Round ligament of the uterus
Round ligament of the uterus and the ligament of the ovary or proper ligament of
the ovary
Suspensory ligament of the ovary

The answer is: D

3. Parts of some human skeletal remains are brought to you as coroner of a rural
community. The pelvis is complete, yet the individual bones of the pelvis, the
ilium, ischium, and pubis have just started to fuse together. The subpubic angle
you estimate at 60°and the pelvic brim has a distinctive heart shaped
appearance. On the basis of this information, you guess the remains are of which
of the following?

3-year-old male
4-year-old female
14-year-old male
30-year-old female
80-year-old male

The answer is: C

4. Following vaginal childbirth, a woman experienced urinary incontinence,


particularly when coughing. This was most likely caused by tearing of which of
the following?

629 ‫صفحة‬ Dr. MOHAMED YAHIA 0900987639


QUESTION BANK OF Dr. MOHAMED YAHIA

Puborectalis muscle
Obturator internus muscle
Pubococcygeus muscle
Superficial transverse perineal muscle
Piriformis muscle

The answer is: A

5. When one touches the upper medial thigh or scrotum of most young males, the
testicles are pulled upwards towards the external inguinal ring. This is called the
cremasteric reflex. The efferent limb of the cremasteric reflex is provided by
which of the following?

Femoral branch of the genitofemoral nerve


Genital branch of the genitofemoral nerve
Ilioinguinal nerve
Pudendal nerve
Temperature differential between core body temperature and scrotal temperature

The answer is: A

6. if You have to do an abdominal and pelvic computer tomography (CT) to look


for ureteric calculi. What specific location(s) will you look for in the CTs for
obstructing calculi?

At the junction of the renal pelvis with the ureters


As the ureters cross the cranial edge of the greater pelvis
As the ureters cross the external iliac artery at the pelvic brim
As the ureters pass through the wall of the bladder
a, b, and c
a, c, and d

The answer is: F

7. You deliver a full-term baby boy who is healthy and receives an Apgar score
of 9 out of 10. You do note that his scrotum is rather large compared to his penis
and when he cries and strains, the scrotum gets even bigger. You palpate for
testes and epididymides and think both are present and don’t feel any abnormal
structures. You tell the parents the newborn has which of the following?

Cryptorchidism
Direct inguinal hernia
Varicocele
Hydrocele
Klinefelter’s syndrome

The answer is: D

630 ‫صفحة‬ Dr. MOHAMED YAHIA 0900987639


QUESTION BANK OF Dr. MOHAMED YAHIA

8. A 6-year-old boy badly bruised his perineum on the horizontal bar of his
bicycle as he was learning to ride a bike. Blood extended into his scrotum, and
onto the anterior abdominal wall from 3 in. below his umbilicus to just anterior
to his anus, but did not pass into his thigh. Which anatomical layers most likely
explain the distribution of extravasated blood?

Superficial membranous fascia and Camper’s fascia


Superficial membranous fascia and transversalis fascia
Dartos fascia and the perineal membrane
Superficial membranous fascia and the perineal membrane
Deep perineal fascia and inferior fascia of the pelvic diaphragm

The answer is: D

9. Fructose, a source of energy for spermatozoa, is found primarily in secretions


from which of the following organs?

Bulbourethral glands
Epididymis
Prostate
Seminal vesicles
Testis

The answer is: D

10. Benign prostatic hypertrophy results in obstruction of the urinary tract.


Patients present with weak urine flow, increased difficulty initiating urination,
and increase frequency of urination since the bladder often is not fully emptied.
Benign prostatic hypertrophy is associated with enlargement of which of the
following?

Entire prostate gland


Lateral/posterior lobes
Mucosal and submucosal regions
Anterior region

The answer is: C

11. A rectal cancer that occurs within the anal canal penetrates the mucosa and
basement membrane. Which nodes would you most likely harvest at the same
time you removed the cancerous growth to send to pathology to determine if
there has been metastasis?

Superficial inguinal nodes


Inguinal nodes and internal iliac nodes
Superficial inguinal, internal iliac and preaortic inferior mesenteric nodes
Internal iliac and external iliac nodes

631 ‫صفحة‬ Dr. MOHAMED YAHIA 0900987639


QUESTION BANK OF Dr. MOHAMED YAHIA

External iliac, superficial inguinal, and preaortic nodes

The answer is: C

12. In the male, the homologue of the vaginal artery is which of the following?

Obturator artery
Internal pudendal artery
Middle rectal artery
Umbilical artery
Inferior vesical artery

The answer is: E

13. A 24-year-old woman seeking assistance for apparent infertility has been
unable to conceive despite repeated attempts in 5 years of marriage. She reveals
that her husband fathered a child in a prior marriage. Although her menstrual
periods are fairly regular, they are accompanied by extreme lower back pain.
The lower back pain during menstruation experienced by this woman probably
is referred from the pelvic region. The pathways that convey this pain sensation
to the central nervous system involve which of the following?

Hypogastric nerve to L1–L2


Lumbosacral trunk to L4–L5
Pelvic splanchnic nerves to S2–S4
Pudendal nerve to S2–S4

The answer is: C

14. The body of the uterus tends to wander within the pelvic cavity. However the
cervix of the uterus tends to remain fairly firmly in place most of the time.
Which of the following would be found immediately inferior to the left cardinal
(lateral cervical) ligament?

Ovarian neurovascular bundle


Uterine tube
Round ligament of the uterus
Ureter
Ovarian artery and vein

The answer is: D

15. The most important measurement of the pelvic outlet, indicating the
SMALLEST dimension, is the transverse midplane diameter. It is measured
between which of the following?

Ischial spines
Ischial tuberosities

632 ‫صفحة‬ Dr. MOHAMED YAHIA 0900987639


QUESTION BANK OF Dr. MOHAMED YAHIA

Lower margin of the pubic symphysis to the sacroiliac joint


Sacral promontory to the inferior margin of the pubic symphysis

The answer is: A

16. At delivery, caudal analgesia is induced by administration of anesthetic into


the epidural space in the sacral region. The needle is introduced via which of the
following?

Anterior sacral foramina


Dural sac
Intervertebral foramina
Posterior sacral foramina
Sacral hiatus

The answer is: E

17. Which structure is most susceptible to unintentional damage during a


hysterectomy?

Uterine artery
Ureter
Urinary bladder
Urethra
Kidney

The answer is: B

18. Which of the following arteries may occasionally arise as a branch of the
external iliac artery or inferior epigastric artery instead of as a branch of the
internal iliac artery?

Internal pudendal artery


Obturator artery
Superior gluteal artery
Umbilical artery
Uterine artery

The answer is: B

19. A couple comes to your office because they have been unable to conceive a
child after 1 year of trying. You examine the man and notice a darkish mass and
fullness of the left scrotum/spermatic cord compared to the smaller right
scrotum/spermatic cord. You suggest he follow up with an urologist because you
suspect which of the following?

Undiagnosed cryptorchidism of the right testicle


Acquired varicocele

633 ‫صفحة‬ Dr. MOHAMED YAHIA 0900987639


QUESTION BANK OF Dr. MOHAMED YAHIA

Acquired left femoral hernia


Acquired right direct femoral hernia
Congenital absence of the pampiniform plexus on the right side

The answer is: B

20. Which of the following is one of the roles of the sympathetic chain in the
pelvis?

Bladder contraction
Cutaneous function (sudomotor, vasomotor, pilomotor)
Erection in males
Erection in both male and female

The answer is: B

21. Both the autonomic and vascular systems need to function properly for
successful male sexual function. Which of the following statements concerning
erection, emission, and ejaculation in the male is correct?

Contraction of the internal urethral sphincter is under control of the


parasympathetic nervous system
The parasympathetic nerves stimulate closure of helical arteries
Sympathetic neurons stimulate the helicine arteries to dilate and increase blood
flow to the corpora cavernosum
Parasympathetic innervation stimulates emission of seminal fluid
Contraction of the bulbospongiosus muscles impedes the drainage of blood from
the corpus spongiosum

The answer is: E

22. Which of the following locations is optimal for fertilization of an ovulated egg
by sperm to occur?

Ampulla of the oviduct


Uterus
Infundibulum of the oviduct
Isthmus of the oviduct
Cervical canal of the uterus

The answer is: A

23. Which of the following contains the ovarian neurovascular bundle?

Broad ligament
Mesosalpinx
Mesovarium
Suspensory ligament of the ovary

634 ‫صفحة‬ Dr. MOHAMED YAHIA 0900987639


QUESTION BANK OF Dr. MOHAMED YAHIA

Transverse cervical ligament

The answer is: D

24. Pap smears are the collection of cells from the uterine cervix to look for
cytological evidence of transformation to cancerous forms, most typically due to
a viral infection. While most women do not report any discomfort associated
with the collection of cervical cells, a few women do. To which of the following
somatic areas does the uterine cervix refer pain to?

Epigastric region
Medial thigh and buttock
Inguinal and pubic regions, anterior labia majora, medial thigh
Lateral leg and perineum
Subcostal and umbilical regions

The answer is: D

25. Episiotomies are performed to control tearing that can occur during a
vaginal delivery. When performing a mediolateral episiotomy, an OB- GYN will
likely cut through several structures of the perineum. What perineal structures
must be sutured back together following a typical mediolateral episiotomy?

Vaginal wall, pubococcygeus, and piriformis muscles


Vaginal wall, pubococcygeus, and iliococcygeus muscles
Vaginal wall, bulbospongiosus, and superficial transverse perineal muscles
Vaginal wall, prepuce, and rectus abdominis muscle
Vaginal wall, sacrospinous and sacrotuberous ligaments

The answer is: C

26. Which anatomical region or structure does not drain into the superficial
inguinal lymph nodes and thus should be EXCLUDED from your differential
diagnosis list?

Penis
Scrotum
Testicles
Anus
Epididymides

The answer is: C

635 ‫صفحة‬ Dr. MOHAMED YAHIA 0900987639


QUESTION BANK OF Dr. MOHAMED YAHIA

27. What would you tell your patient regarding how to perform a pudendal
nerve block and an advantage to performing the injection transvaginally versus
injecting through perineal skin?

The pudendal nerve serves the skin around the anterior/lateral entrance of the
vagina; the nerve wraps around the ischial spine, which is used as landmark;
transvaginal administration is less painful since the upper portion of the vagina
has fewer pain receptors
The pudendal nerve serves the skin around the posterior/lateral entrance of the
vagina; the nerve wraps around the ischial spine, which is used as a landmark;
transvaginal administration is less painful since the upper portion of the vagina
has fewer pain receptors
The pudendal nerve serves the skin around the anterior/lateral entrance of the
vagina; the nerve wraps around the ischial tuberosity, which is used as
landmark; transvaginal administration is less painful since the upper portion of
the vagina has fewer pain receptors
The pudendal nerve serves the skin around the posterior/lateral entrance of the
vagina; the nerve wraps around the ischial tuberosity, which is used as a
landmark; transvaginal administration is less painful since the upper portion of
the vagina has fewer pain receptors

The answer is: B

636 ‫صفحة‬ Dr. MOHAMED YAHIA 0900987639


QUESTION BANK OF Dr. MOHAMED YAHIA

BRS ANATOMY

1. A 68-year-old woman with uterine carcinoma undergoes surgical resection.


This cancer can spread directly to the labia majora in lymphatics that follow
which of the following structures?

Pubic arcuate ligament


Suspensory ligament of the ovary
Cardinal (transverse cervical) ligament
Suspensory ligament of the clitoris
Round ligament of the uterus

The answer is: E

2. A 17-year-old boy suffers a traumatic groin injury during a soccer match. The
urologist notices tenderness and swelling of the boy’s left testicle that may be
produced by thrombosis in which of the following veins?

Left internal pudendal vein


Left renal vein
Inferior vena cava
Left inferior epigastric vein
Left external pudendal vein

The answer is: B

3. On a busy Saturday night in Chicago, a 16-year-old boy presents to the


emergency department with a stab wound from a knife that entered the pelvis
above the piriformis muscle. Which of the following structures is most likely to
be damaged?

Sciatic nerve
Internal pudendal artery
Superior gluteal nerve
Inferior gluteal artery
Posterior femoral cutaneous nerve

The answer is: C

4. A 22-year-old woman receives a deep cut in the inguinal canal 1 in. lateral to
the pubic tubercle. Which of the following ligaments is lacerated within the
inguinal canal?

Suspensory ligament of the ovary


Ovarian ligament

637 ‫صفحة‬ Dr. MOHAMED YAHIA 0900987639


QUESTION BANK OF Dr. MOHAMED YAHIA

Mesosalpinx
Round ligament of the uterus
Rectouterine ligament

The answer is: D

5. A 29-year-old carpenter sustains severe injuries of the pelvic splanchnic nerve


by a deep puncture wound, which has become contaminated. The injured
parasympathetic preganglionic fibers in the splanchnic nerve are most likely to
synapse in which of the following ganglia?

Ganglia in or near the viscera or pelvic plexus


Sympathetic chain ganglia
Collateral ganglia
Dorsal root ganglia
Ganglion impar

The answer is: A

6. A 59-year-old woman comes to a local hospital for uterine cancer surgery. As


the uterine artery passes from the internal iliac artery to the uterus, it crosses
superior to which of the following structures that is sometimes mistakenly ligated
during such surgery?

Ovarian artery
Ovarian ligament
Uterine tube
Ureter
Round ligament of the uterus

The answer is: D

7. A 29-year-old woman is admitted to a hospital because the birth of her child is


several days overdue. Tearing of the pelvic diaphragm during childbirth leads to
paralysis of which of the following muscles?

Piriformis
Sphincter urethrae
Obturator internus
Levator ani
Sphincter ani externus

The answer is: D

8. A 37-year-old small business manager receives a gunshot wound in the pelvic


cavity, resulting in a lesion of the sacral splanchnic nerves. Which of the
following nerve fibers would primarily be damaged?

638 ‫صفحة‬ Dr. MOHAMED YAHIA 0900987639


QUESTION BANK OF Dr. MOHAMED YAHIA

Postganglionic parasympathetic fibers


Postganglionic sympathetic fibers
Preganglionic sympathetic fibers
Preganglionic parasympathetic fibers
Postganglionic sympathetic and parasympathetic fibers

The answer is: C

9. A young couple is having difficulty conceiving a child. Their physician at a


reproduction and fertility clinic explains to them that

The ovary lies within the broad ligament


The glans clitoris is formed from the corpus spongiosum
Erection of the penis is a sympathetic response
Ejaculation follows parasympathetic stimulation
Fertilization occurs in the infundibulum or ampulla of the uterine tube

The answer is: E

10. A 46-year-old woman has a history of infection in her perineal region. A


comprehensive examination reveals a tear of the superior boundary of the
superficial perineal space. Which of the following structures would most likely
be injured?

Pelvic diaphragm
Colles fascia
Superficial perineal fascia
Deep perineal fascia
Perineal membrane

The answer is: E

11. A 58-year-old man is diagnosed as having a slowly growing tumor in the


deep perineal space. Which of the following structures would most likely be
injured?

Bulbourethral glands
Crus of penis
Bulb of vestibule
Spongy urethra
Great vestibular gland

The answer is: A

12. An elderly man with a benign enlargement of his prostate experiences


difficulty in urination, urinary frequency, and urgency. Which of the following
lobes of the prostate gland is commonly involved in benign hypertrophy that
obstructs the prostatic urethra?

639 ‫صفحة‬ Dr. MOHAMED YAHIA 0900987639


QUESTION BANK OF Dr. MOHAMED YAHIA

Anterior lobe
Middle lobe
Right lateral lobe
Left lateral lobe
Posterior lobe

The answer is: B

13. A 59-year-old man is diagnosed with prostate cancer following a digital


rectal examination. For the resection of prostate cancer, it is important to know
that the prostatic ducts open into or on which of the following structures:

Membranous part of the urethra


Seminal colliculus
Spongy urethra
Prostatic sinus
Prostatic utricle

The answer is: D

14. A 29-year-old woman with a ruptured ec-topic pregnancy is admitted to a


hospital for culdocentesis. A long needle on the syringe is most efficiently
inserted through which of the following structures?

Anterior fornix of the vagina


Posterior fornix of the vagina
Anterior wall of the rectum
Posterior wall of the uterine body
Posterior wall of the bladder

The answer is: B

15. A 37-year-old man is suffering from carcinoma of the skin of the penis.
Cancer cells are likely to metastasize directly to which of the following lymph
nodes?

External iliac nodes


Internal iliac nodes
Superficial inguinal nodes
Aortic (lumbar) nodes
Deep inguinal nodes

The answer is: C

16. A 42-year-old woman who has had six children develops a weakness of the
urogenital diaphragm. Paralysis of which of the following muscles would cause
such a symptom?

640 ‫صفحة‬ Dr. MOHAMED YAHIA 0900987639


QUESTION BANK OF Dr. MOHAMED YAHIA

Sphincter urethrae
Coccygeus
Superficial transversus perinei
Levator ani
Obturator internus

The answer is: A

17. A 43-year-old man has a benign tumor located near a gap between the
arcuate pubic ligament and the transverse perineal ligament. Which of the
following structures is most likely compressed by this tumor?

Perineal nerve
Deep dorsal vein of the penis
Superficial dorsal vein
Posterior scrotal nerve
Deep artery of the penis

The answer is: B

18. An obstetrician performs a median episiotomy on a woman before


parturition to prevent uncontrolled tearing. If the perineal body is damaged, the
function of which of the following muscles might be impaired?

Ischiocavernosus and sphincter urethrae


Deep transverse perineal and obturator internus
Bulbospongiosus and superficial transverse perineal
External anal sphincter and sphincter urethrae
Bulbospongiosus and ischiocavernosus

The answer is: C

19. A 22-year-old man has a gonorrheal infection that has infiltrated the space
between the inferior fascia of the urogenital diaphragm and the superficial
perineal fascia. Which of the following structures might be inflamed?

Bulb of the penis


Bulbourethral gland
Membranous part of the male urethra
Deep transverse perineal muscle
Sphincter urethrae

The answer is: A

20. A 39-year-old man is unable to expel the last drops of urine from the urethra
at the end of micturition because of paralysis of the external urethral sphincter
and bulbospongiosus muscles. This condition may occur as a result of injury to
which of the following nervous structures?

641 ‫صفحة‬ Dr. MOHAMED YAHIA 0900987639


QUESTION BANK OF Dr. MOHAMED YAHIA

Pelvic plexus
Prostatic plexus
Pudendal nerve
Pelvic splanchnic nerve
Sacral splanchnic nerve

The answer is: C

21. A 21-year-old marine biologist asks about her first bimanual examination,
and it is explained to her that the normal position of the uterus is

Anteflexed and anteverted


Retroflexed and anteverted
Anteflexed and retroverted
Retroverted and retroflexed
Anteverted and retroverted

The answer is: A

22. After his bath but before getting dressed, a 4-year-old boy was playing with
his puppy. The boy’s penis was bitten by the puppy, and the deep dorsal vein was
injured. The damaged vein

Lies superficial to Buck fascia


Drains into the prostatic venous plexus
Lies lateral to the dorsal artery of the penis
Is found in the corpus spongiosum
Is dilated during erection

The answer is: B

23. A 62-year-old man is incapable of penile erection after rectal surgery with
prostatectomy. The patient most likely has a lesion of which of the following
nerves?

Dorsal nerve of the penis


Perineal nerve
Hypogastric nerve
Sacral splanchnic nerve
Pelvic splanchnic nerve

The answer is: E

24. A 23-year-old massage therapist who specializes in women’s health attends a


lecture at an annual conference on techniques of massage. She asks, “What
structure is drained by the lumbar (aortic) lymph nodes?” Which of the
following structures is the correct answer to this question?

642 ‫صفحة‬ Dr. MOHAMED YAHIA 0900987639


QUESTION BANK OF Dr. MOHAMED YAHIA

Perineum
Lower part of the vagina
External genitalia
Ovary
Lower part of the anterior abdominal wall

The answer is: D

25. A sexually active adolescent presents with an infection within the ischiorectal
fossa. Which of the following structures is most likely injured?

Vestibular bulb
Seminal vesicle
Greater vestibular gland
Inferior rectal nerve
Internal pudendal artery

The answer is: D

26. A first-year resident in the urology department reviews pelvic anatomy


before seeing patients. Which of the following statements is correct?

The dorsal artery of the penis supplies the glans penis.


The seminal vesicles store spermatozoa.
The duct of the bulbourethral gland opens into the membranous urethra.
The duct of the greater vestibular gland opens into the vagina.
The anterior lobe of the prostate gland is prone to carcinomatous
transformation.

The answer is: A

27. A 43-year-old woman presents with a prolapsed uterus. Repair of a


prolapsed uterus requires knowledge of the supporting structures of the uterus.
Which of the following structures plays the most important role in the support of
the uterus?

Levator ani
Sphincter urethrae
Uterosacral ligament
Ovarian ligament
Arcuate pubic ligament

The answer is: A

28. A 16-year-old boy presents to the emer- gency department with rupture of
the penile urethra. Extravasated urine from this injury can spread into which of
the following structures?

643 ‫صفحة‬ Dr. MOHAMED YAHIA 0900987639


QUESTION BANK OF Dr. MOHAMED YAHIA

Scrotum
Ischiorectal fossa
Pelvic cavity
Testis
Thigh

The answer is: A

29. A 23-year-old woman visits her obstetrician for an annual checkup. During
vaginal examination, which of the following structures may be palpated?

Apex of the urinary bladder


Fundus of the uterus
Terminal part of the round ligament of theuterus
Body of the clitoris
Uterine cervix

The answer is: E

30. A 53-year-old bank teller is admitted to a local hospital for surgical removal
of a benign pelvic tumor confined within the broad ligament. There is a risk of
injuring which of the following structures that lies in this ligament?

Ovary
Proximal part of the pelvic ureter
Terminal part of the round ligament of the uterus
Uterine tube
Suspensory ligament of the ovary

The answer is: D

31. A 72-year-old man comes to his physician for an annual checkup. Which of
the following structures is most readily palpated during rectal examination?

Prostate gland
Epididymis
Ejaculatory duct
Ureter
Testis

The answer is: A

32. A 48-year-old college football coach under- goes a radical prostatectomy for
a malignant tumor in his prostate. Following surgery, he is incapable of
achieving an erection. Which of the following nerves is most likely damaged
during the surgery?

Sacral splanchnic nerve

644 ‫صفحة‬ Dr. MOHAMED YAHIA 0900987639


QUESTION BANK OF Dr. MOHAMED YAHIA

Pelvic splanchnic nerve


Pudendal nerve
Dorsal nerve of the penis
Posterior scrotal nerve

The answer is: B

33. While performing a pelvic exenteration, the surgical oncologist notices a


fractured or ruptured boundary of the pelvic inlet. Which of the following
structures is most likely damaged?

Promontory of the sacrum


Anterior–inferior iliac spine
Inguinal ligament
Iliac crest
Arcuate pubic ligament

The answer is: A

34. A 32-year-old patient with multiple fractures of the pelvis has no cutaneous
sensation in the urogenital triangle. The function of which of the following nerves
is most likely to be spared?

Ilioinguinal nerve
Iliohypogastric nerve
Posterior cutaneous nerve of the thigh
Pudendal nerve
Genitofemoral nerve

The answer is: B

35. A 22-year-old victim of an automobile accident has received destructive


damage to structures that form the boundary of the perineum. Which of the
following structures is spared?

Pubic arcuate ligament


Tip of the coccyx
Ischial tuberosities
Sacrospinous ligament
Sacrotuberous ligament

The answer is: D

36. A 32-year-old man undergoes vasectomy as a means of permanent birth


control. A physician performing the vasectomy by making an incision on each
side of the scrotum should remember which of the following statements most
applicable to the scrotum?

645 ‫صفحة‬ Dr. MOHAMED YAHIA 0900987639


QUESTION BANK OF Dr. MOHAMED YAHIA

It is innervated by the ilioinguinal and genitofemoral nerves.


It receives blood primarily from the testicular artery.
Its venous blood drains primarily into the renal vein on the left.
Its lymphatic drainage is primarily into upper lumbar nodes.
Its dartos tunic is continuous with the perineal membrane.

The answer is: A

37. A 37-year-old woman complains of a bearing-down sensation in her womb


and an increased frequency of and burning sensation on urination. On
examination by her gynecologist, she is diagnosed with a uterine prolapse. Which
of the following structures provides the primary support for the cervix of the
uterus?

External anal sphincter


Broad ligament of the uterus
Cardinal (transverse cervical) ligament
Round ligament of the uterus
Suspensory ligament of the ovary

The answer is: C

38. A woman is delivering a breech baby. The obstetrician decides that it is best
to perform a mediolateral episiotomy. Which of the following structures should
the obstetrician avoid incising?

Vaginal wall
Superficial transverse perineal muscle
Bulbospongiosus
Levator ani
Perineal membrane

The answer is: D

39. During pelvic surgery, a surgeon notices severe bleeding from the artery that
remains within the true pelvis. Which of the following arteries is most likely to be
injured?

Iliolumbar artery
Obturator artery
Uterine artery
Internal pudendal artery
Inferior gluteal artery

The answer is: C

40. A neurosurgeon performs a surgical resection of a rare meningeal tumor in


the sacral region. He tries to avoid an injury of the nerve that arises from the

646 ‫صفحة‬ Dr. MOHAMED YAHIA 0900987639


QUESTION BANK OF Dr. MOHAMED YAHIA

lumbosacral plexus and remains within the abdominal or pelvic cavity. To which
of the following nerves should he pay particular attention?

Ilioinguinal nerve
Genitofemoral nerve
Lumbosacral trunk
Femoral nerve
Lateral femoral cutaneous nerve

The answer is: C

41. After repair of a ruptured diverticulum, a 31-year-old patient begins to spike


with fever and complains of abdominal pain. An infection in the deep perineal
space would most likely damage which of the following structures?

Ischiocavernosus muscles
Superficial transverse perineal muscles
Levator ani
Sphincter urethrae
Bulbospongiosus

The answer is: D

42. A radiologist interprets a lymphangiogram for a 29-year-old patient with


metastatic carcinoma. Upper lumbar nodes most likely receive lymph from
which of the following structures?

Lower part of the anal canal


Labium majus
Clitoris
Testis
Scrotum

The answer is: D

43. A 49-year-old woman has a large mass on the pelvic brim. Which of the
following structures is most likely compressed by this mass when crossing the
pelvic brim?

Deep dorsal vein of the penis


Uterine tube
Ovarian ligament
Uterine artery
Lumbosacral trunk

The answer is: E

647 ‫صفحة‬ Dr. MOHAMED YAHIA 0900987639


QUESTION BANK OF Dr. MOHAMED YAHIA

44. A 26-year-old man comes to a hospital with fever, nausea, pain, and itching
in the perineal region. On examination by a urologist, he is diagnosed as having
infected bulbourethral (Cowper) glands. Which of the following structures is/are
affected by this infection?

Superficial perineal space


Sphincter urethrae
Production of sperm
Testis
Seminal vesicles

The answer is: B

45. A 21-year-old man is involved in a high- speed motor vehicle accident. As a


result, he has extensive damage to his sphincter urethra. Which of the following
best describes the injured sphincter urethra?

Smooth muscle
Innervated by the perineal nerve
Lying between the perineal membrane and Colles fascia
Enclosed in the pelvic fascia
Part of the pelvic diaphragm

The answer is: B

46. A 6-month-old male infant is admitted to the children’s hospital because he


has no testis in his scrotum. During physical examination, the pediatrician
palpated the testis in the inguinal canal. What is the diagnosis of this condition?

Male pseudohermaphroditism
Hypospadias
Epispadias
Cryptorchid testis
Chordee

The answer is: D

47. An obstetrician is about to perform a pudendal block so a woman can


experience less pain when she delivers her child. He recalls what he learned in
medical school about this nerve. Which of the following statements is correct?

It passes superficial to the sacrotuberous ligament


It innervates the testis and epididymis in a male
It provides motor fibers to the coccygeus
It can be blocked by injecting an anesthetic near the inferior margin of the
ischial spine
It arises from the lumbar plexus

648 ‫صفحة‬ Dr. MOHAMED YAHIA 0900987639


QUESTION BANK OF Dr. MOHAMED YAHIA

The answer is: D

48. A trauma surgeon in the emergency department at a local center examines a


14-year-old boy with extensive pelvic injuries after a hit and run accident. The
surgeon inspects the ischiorectal fossa because it

Accumulates urine leaking from rupture of the bulb of the penis


Contains the inferior rectal vessels
Has a pudendal canal along its medial wall
Is bounded anteriorly by the sacrotuberous ligament
Contains a perineal branch of the fifth lumbar nerve

The answer is: B

49. An elderly man with prostatitis is seen at an internal medicine clinic. The
seminal colliculus of his prostate gland is infected, and its fine openings are
closed. Which of the following structures is/are most likely to be disturbed?

Ducts of the prostate gland


Prostatic utricle
Ducts of the bulbourethral glands
Ejaculatory ducts
Duct of the seminal vesicles

The answer is: D

50. A general surgeon is giving a lecture to a team of surgery residents. She


describes characteristics of structures above the pectinate line of the anal canal,
which include

Stratified squamous epithelium


Venous drainage into the caval system
Lymphatic drainage into the superficial inguinal nodes
Visceral sensory innervation
External hemorrhoids

The answer is: D

51. A 5-month-old boy is admitted to the children’s hospital because of urine


being expelled from the dorsal aspect of the penis. Which of the following
embryologic structures failed to fuse in this patient?

Labioscrotal swellings or folds


Urogenital sinus
Spongy urethra
Phallus
Urethral folds

649 ‫صفحة‬ Dr. MOHAMED YAHIA 0900987639


QUESTION BANK OF Dr. MOHAMED YAHIA

The answer is: C

52. A 78-year-old man has carcinoma of the rectum. The cancer is likely to
metastasize via the veins into which of the following structures?

Spleen
Kidney
Liver
Duodenum
Suprarenal gland

The answer is: C

53. During a partial hysterectomy leaving the ovaries intact, the surgeon
detaches the ovary from the uterus by transecting the ovarian ligament. This
ligament developed from which embryonic structure?

Mesonephric duct
Urogenital folds
Gubernaculum
Processus vaginalis
Paramesonephric ducts

The answer is: C

650 ‫صفحة‬ Dr. MOHAMED YAHIA 0900987639


QUESTION BANK OF Dr. MOHAMED YAHIA

‫شيت الجامعة‬
PELVIS
1. The following statements concerning the uterus are correct except:

a) The fundus is part of the uterus above the openings of uterine tubes

b) The long axis of the uterus is usually bent anteriorly on the

long axis of the vagina (anteversion)

c) The nerve supply of the uterus is from the inferior hypogastric plexuses

d) The anterior surface of the cervix is completely covered with peritoneum

e) The uterine veins drain into the internal iliac veins

2. Concerning the vas deferens, all of the following statements are true except:

a) It emerges from the deep inguinal ring and passes around the lateral
margin of the inferior epigastric artery

b) It crosses the ureter in the region of the ischial spine

c) The terminal part is dilated to form the ampulla

d) It lies on the posterior surface of the prostate but is separated from it by

peritoneum

e) It joins the duct of the seminal vesicle to form the ejaculatory duct

3. Concerning the pelvic part of the ureter, the following statements are true
except:

a) It enters the pelvis in front of the bifurication of the common iliac arteries

b) The ureter enters the bladder by passing directly through its wall, there

being no valvular mechanism at its entrance

c) It has a close relationship to the ischial spine before it turns medially toward
the bladder

651 ‫صفحة‬ Dr. MOHAMED YAHIA 0900987639


QUESTION BANK OF Dr. MOHAMED YAHIA

d) The blood supply of the distal part of the ureter is from the superior vesical
artery

e) It enters the bladder at the upper lateral angle of the trigone

4. Concerning the seminal vesicle, the following statements are true except:

a) The seminal vesicles are related posteriorly to the rectum and can be

palpated through the rectal wall

b) The seminal vesicles are two lobulated organs that store spermatozoa

c) The upper ends of the seminal vesicles are covered by peritoneum

d) The function of the seminal vesicles is to produce a secretion that is added

to the seminal fluid

e) The seminal vesicles are related anteriorly to the bladder, and no

peritoneum separates these structures

5. Concerning the ovary, the following statements are true except:

a) The lymph drainage is into the para-aortic (lumbar) lymph nodes at the

level of the first lumbar vertebra

b) The round ligament of the ovary extends from the ovary to the upper end of
the lateral wall of the body of the uterus

c) The ovarian fossa is bounded above by the external iliac vessels and behind
by the internal iliac vessels

d) The left ovarian artery is a branch of the left internal iliac artery

e) The obturator nerve lies lateral to the ovary

6. Concerning the nerve supply to the urinary bladder, the following statements
are true except:

652 ‫صفحة‬ Dr. MOHAMED YAHIA 0900987639


QUESTION BANK OF Dr. MOHAMED YAHIA

a) The sympathetic postganglionic fibers originate in the first and second

lumbar ganglia

b) The parasympathetic postganglionic fibers originate in the inferior

hypogastric plexuses

c) The afferent sensory fibers arising in the bladder wall reach the spinal cord
via the pelvic splanchnic nerves and also travel with the sympathetic nerves

d) The parasympathetic preganglionic fibers arise from the second, third and

fourth sacral segments of the spinal cord

e) The parasympathetic postganglionic fibers are responsible for closing the

vesical sphincter during ejaculation

7. Concerning the vagina, the following statements are true except:

a) The area of the vaginal lumen around the cervix is divided into four fornices

b) The upper part of the vagina is supported by the levator ani muscles and

transverse cervical ligaments

c) The perineal body lies posterior to and supports the lower part of the vagina

d) The upper part of the vagina is not covered with peritoneum

e) The vaginal wall receives a branch of the uterine artery

8. Concerning the visceral layer of pelvic fascia in the female, the following
statements are true except:

a) In the region of the cervix of the uterus, it is called the parametrium

b) It is condensed to form the pubocervical, transverse cervical, and

sacrocervical ligaments of the uterus

c) It covers the obturator internus muscle

d) It does not become continuous above with the fascia transversalis

e) On the lateral wall of the pelvis, it fuses with the parietal layer of the pelvic
fascia

653 ‫صفحة‬ Dr. MOHAMED YAHIA 0900987639


QUESTION BANK OF Dr. MOHAMED YAHIA

9. The following statements concerning the lymphatic drainage of pelvic


structures are correct except:

a) Lymph from the cervix of the uterus drains into the internal and external

iliac lymph nodes

b) Lymph from the prostate drains into the external iliac lymph

nodes

c) Lymph from the posterior fornix of the vagina drains into the internal and

external iliac lymph nodes

d) Lymph from the trigone of the bladder drains into the internal and external

iliac lymph nodes

e) Lymph from the fundus of the uterus drains into the para-aortic lymph

nodes at the level of the first lumbar vertebrae

10. The following statements concerning the main venous drainage of pelvic
structures are true except:

a) The venous blood from the left ovary drains into the inferior vena
cava

654 ‫صفحة‬ Dr. MOHAMED YAHIA 0900987639


QUESTION BANK OF Dr. MOHAMED YAHIA

b) The venous blood from the prostate drains into the interal iliac veins

c) The venous blood from the urinary bladder drains into the internal iliac
veins

d) The venous blood from the mucous membrane of the rectum drains into the
superior rectal vein

e) The venous drainage of the seminal vesicles drains into the internal iliac

veins

11. The following statements concerning the female urethra are true except:
a) It lies immediately anterior to the vagina

b) Its external orifice lies about 2 in. (5 cm) from the clitoris

c) It is about 1.5in (3.75 cm) long

d) It pierces the urogenital diaphragm

e) It is straight, and only minor resistance is felt as a catheter is passed through


the urethtral sphincter

12. The following structures can be palpated by a vaginal examination except:

a) Sigmoid colon

b) Ureters

c) Perineal body

d) Ischial spines

e) Iliopectineal line

13. The following statements concerning the ischiorectal fossa are true except:

a) The pudendal nerve lies in its lateral wall

b) The floor is formed by the superficial fascia and skin

c) The lateral wall is formed by the obturator internus muscle and its fascia

d) The medial wall is formed in part by the levator ani muscle

655 ‫صفحة‬ Dr. MOHAMED YAHIA 0900987639


QUESTION BANK OF Dr. MOHAMED YAHIA

e) The roof is formed by the urogenital diaphragm

14. The following statements concerning the penis are true except:

a) Its root is formed in the midline by the bulb of the penis which continues

anteriorly as the corpus spongiosum

b) Its roots laterally are formed by the crura, which continue anteriorly as the

corpora cavernosa

c) The penile urethra lies within the corpus spongiosum

d) The glans penis is a distal expansion of the fused corpora cavernosa

e) The penis is suspended from the lower part of the anterior abdominal wall

by two condensations of deep fascia

15. The following statements concerning the perineal structures are correct
except:

a) The anorectal ring is formed by the subcutaneous, superficial and deep

fibers of the external sphincter

b) The urogenital diaphragm is attached laterally to the inferior ramus of the

pubis and the ischial ramus

c) The bulbourethral glands are situated in the deep perineal pouch

d) The anococcygeal body is rarely damaged in child birth

e) The lymph drainage of the skin around the anus is into the medial group of

superficial inguinal nodes

16. The urogenital diaphragm is formed by the following structures except:

a) Deep transverse perineal muscle

b) Perineal membrane

c) Sphincter urethrae

d) Colles' fascia (membranous layer of superficial fascia)

656 ‫صفحة‬ Dr. MOHAMED YAHIA 0900987639


QUESTION BANK OF Dr. MOHAMED YAHIA

e) Parietal pelvic fascia covering the upper surface of the sphincter urethrae

muscle

17. In the male, the following structures can be palpated on rectal examination
except:

a) Bulb of the penis

b) Urogenital diaphragm

c) Anorectal ring

d) The anterior surface of the rectum

e) Ureter

18. The following statements concerning the anal canal are true except:

a) It is about 1.5 in (3.8 cm) long

b) It pierces the urogenital diaphragm

c) It is related laterally to the external anal sphincter

d) It is the site of an important portal-systemic anastomosis

e) The mucous membrane of the lower half receives its arterial supply from

the inferior rectal artery

19. The following statements concerning the subcutaneous part of the external
anal sphincter are correct except:

a) It encircles the anal canal

b) It is not attached to the anococcygeal

c) It is composed f striated muscle fibers

d) It is not responsible for causing the anal canal and the rectum to join at an

acute angle

e) It is innervated by the middle rectal nerve

657 ‫صفحة‬ Dr. MOHAMED YAHIA 0900987639


QUESTION BANK OF Dr. MOHAMED YAHIA

20. The following statements concerning the defecation are true except:

a) The act is often preceded by the entrance of the feces into the rectum, which
gives rise to the desire to defecate

b) The muscles of the anterior abdominal wall contract

c) The external anal sphincters and the puborectalis relax

d) The internal sphincter contracts and causes the evacuation of the feces

e) The mucous membrane of the lower part of the anal canal is extruded

through the anus ahead of the fecal mass

21. The process of ejaculation depends on the following processes except:

a) The sphincter of the bladder contracts

b) The sympathetic preganglionic nerve fibers arising from the first and

second lumbar segments of the spinal cord must be intact

c) The smooth muscle of the epididymis, ductus (vas) deferens, seminal

vesicles and prostate contracts

d) The bulbourethral glands and the urethral glands are active

e) The bulbospngiosus muscles relax

22. The following structures receive innervation from the branches of the
pudendal nerve except:

a) Labia minora

b) Urethral sphincter

c) The posterior fornix of the vagina

d) Ischiocavernosus muscles

e) Skin of the penis or clitoris

658 ‫صفحة‬ Dr. MOHAMED YAHIA 0900987639


QUESTION BANK OF Dr. MOHAMED YAHIA

23. The following structures are closely related to the rectouterine pouch (pouch
of Douglas) except:

a) Anteriorly is situated in the posterior surface of the upper part of the vagina

b) Posteriorly is situated the upper part of the rectum

c) The trigone of the bladder is directly related to its anterior wall

d) Anteriorly is situated in the posterior surface of the body of the uterus

e) Laterally is situated in the sacrocervical ligaments passing forward to the

cervix

24. Support for the uterus either directly or indirectly, is provided by the
following structures except:

a) The perineal body

b) The mesosalpinx

c) The transverse cervical (cardinal) ligaments

d) The levator ani muscles

e) The pubocervical ligaments

25. The following statements regarding the ovary are correct except:

a) It is attached to the posterior layer of the broad ligament

b) It ovulates an ovum into the peritoneal cavity

c) It is attached to the lateral pelvic wall by the round ligament of the ovary

d) It normally is not related to the posterior fornix of the vagina

e) The right ovarian vein drains into the inferior vena cava

26. The following statements regarding the urinary bladder are true except:

a) It lies in the visceral layer of the pelvic fascia beneath the peritoneum

b) When the bladder is empty, the internal surface is wrinkled except at the

trigone, which is smooth

659 ‫صفحة‬ Dr. MOHAMED YAHIA 0900987639


QUESTION BANK OF Dr. MOHAMED YAHIA

c) Parasympathetic nerve fibers innervate the detrusor muscle

d) The trigone is the area between the openings of the urethra and the two

ureters

e) In children, the bladder is located entirely within the pelvis

27. The broad ligament contains all of the following except:

a) The round ligament of the ovary

b) The uterine artery

c) The round ligament of the uterus

d) The uterine tubes

e) Ureters

28. Malignant tumors of the trigone of the bladder spread (metastasize) to the
following lymph nodes:

a) Lumbar

b) Sacral

c) External iliac only

d) External and internal iliac

e) Superficial inguinal

29. In males, traumatic injury to the perineum may rupture the bulb of the penis
or penile urethra. The resulting leakage of blood or urine may be found in all of
the following areas except:

660 ‫صفحة‬ Dr. MOHAMED YAHIA 0900987639


QUESTION BANK OF Dr. MOHAMED YAHIA

a) The anterior abdominal wall

b) The ischiorectal fossa

c) The scrotum

d) The penis

e) The superficial perineal puch

30. The deep perineal space (pouch) contains the:

a) Internal pudendal artery

b) Pudendal artery

c) Sphincter urethtrae

d) Both A and B

e) Both A and C

31. The following is true about the pudendal canal except:

a) Can be palpated throughout its entire length during rectal examination

b) Is a fascial tunnel

c) Contains the terminal branches of the pudendal nerve

d) Lies in the lateral wall of the ischiorectal fossa

32. All of the following statements about the ductus deferens are true except:

a) It opens via the ejaculatory duct into the prostatic urethtra

b) It begins at the head of the epididymis

c) It is one of the contents of the spermatic cord

d) It enters the abdominal cavity at the deep inguinal ring

e) It is retroperitoneal

33. Which one of the following is true regarding the muscles of the pelvis:

661 ‫صفحة‬ Dr. MOHAMED YAHIA 0900987639


QUESTION BANK OF Dr. MOHAMED YAHIA

a) The obturator internus muscle leaves the pelvis through the greater sciatic

foramen

b) The levator ani muscle is composed of the pubococcygeus, puborectalis and


iliococcygeus

c) The pelvic diaphragm is a sling of muscle that separates the true pelvis from
the false pelvis

d) The piriformis is one of the components forming the lateral pelvic walls

34. Which one of the following is true:

a) The urinary bladder in the male sits at a lower level than that of the female

because of the presence of the prostate

b) The scrotum is actually an external genital organ, but is in direct continuity

with male pelvic organs via the vas deferens

c) The efferent ductules unite at the superior aspect of the testis to form the

ductus deferens

d) The gonadal artery in the male and female is a branch from the anterior

division of the internal iliac artery

35. The urethra is located in the:

a) Ischiorectal fossa

b) Deep perineal pouch

c) Superficial perineal pouch

d) Both B and C

e) All of the above

36. The bulb of the penis is located in the:

a) Deep perineal pouch

b) Superficial perineal pouch

662 ‫صفحة‬ Dr. MOHAMED YAHIA 0900987639


QUESTION BANK OF Dr. MOHAMED YAHIA

c) Ischiorectal fossa

d) None of the above

37. Primary support for the uterus is derived from the:

a) Lateral (transverse) cervical ligaments

b) Pubocervical ligaments

c) Sacrocervical ligaments

d) Round ligament of the uterus

e) Broad ligament

38. Which is not considered a usual branch of the anterior division of the
internal iliac artery:

a) Internal pudendal artery

b) Iliolumbar artery

c) Obturator artery

d) Umbilical artery

e) Middle rectal artery

39. Which of the following statements about the rectum is false?

a) The sciatic nerve is derived from the segments L4 and 5; S1, 2 and 3

b) In the pudendal nerve is derived from the segments L3, 4 and 5

c) The pelvic splanchnic nerve is derived from the segments S2, 3 and 4

d) The obturator nerve is derived from the segments L2, 3 and 4

e) The lumbosacral trunk is derived from the segments L4 and 5

40. Which of the following ligaments is attached to the uterus and composed of
peritoneum:

a) Transverse cervical (cardinal) ligaments

663 ‫صفحة‬ Dr. MOHAMED YAHIA 0900987639


QUESTION BANK OF Dr. MOHAMED YAHIA

b) Broad ligament

c) Round ligament

d) Suspensory ligament of ovary

e) Medial umbilical fold

41. Which statement is false about the pudendal nerve?

a) It carries fibers from the spinal cord segments S2-S4

b) It innervates the levator ani muscle

c) It innervates the internal sphincters of bladder and rectum

d) It runs in the ischiorectal fossa

e) It exits the pelvis through the greater sciatic foramen

42. The male urethtra:

a) In rupture urethra urine may be found in the scrotum

b) Its entire epithelial lining is stratified squamous type

c) Is 5 inches long

d) The membranous part receives the ejactulatory ducts

e) Has 3 constrictions

43. The transverse (lateral) cervical ligament is pierced by the:

a) Superior hypogastric plexus

b) Uterine artery

c) Ureter

d) Both B and C

e) All of the above

664 ‫صفحة‬ Dr. MOHAMED YAHIA 0900987639


QUESTION BANK OF Dr. MOHAMED YAHIA

44. The deep perineal space is deep to the perineal membrane. It contains:

a) Membranous urethra

b) Vagina

c) Deep transverse perinea muscles

d) All of the above

e) Both A and C

45. Regarding the spermatic cord, select the false statement:

a) The vas deferens starts in the tail of epididymis

b) The internal spermatic fascia arises from the internal oblique muscle

c) The testicular artery arises from the aorta

d) The cremasteric artery arises from the inferior epigastric artery

e) Lymphatics from the ductus deferens drain to internal iliac nodes

46. Which of the following is not a branch of the internal iliac artery?

a) Inferior vesical

b) Umbilical

c) Superior gluteal

d) Iliolumbar

e) Inferior epigastric

47. Which of the following is true?

a) The urinary bladder in male sits at a lower level than that of the female

because of the presence of the prostate

b) The scrotum is actually an external genital organ, but is in direct continuity

with male pelvic organs via the ductus deferens

c) The efferent ductules inite at the superior aspect of the testis to form the

665 ‫صفحة‬ Dr. MOHAMED YAHIA 0900987639


QUESTION BANK OF Dr. MOHAMED YAHIA

ductus deferens

d) The gonadol artery in the male and female is a branch from the anterior

division of the internal iliac artery

e) The lymphatics of the ovary drain to the internal iliac nodes

48. The following statements concerning the ischio-anal fossa are true except:

a) The pudendal nerve lies in its lateral wall

b) The floor is formed by the superficial fascia and skin

c) The lateral wall is formed by the obturator internus muscle and its fascia

d) The medial wall in part is formed by the levator ani muscle

e) The roof is formed by the urogenital diaphragm

49. Which is statement is not true concerning the anal canal?

a) It is about 1.5 inches (3.75cm) long

b) It is lined by ciliated columnar epithelium

c) It is related laterally to the external anal sphincter

d) It is the site of an important portal-systemic anastomosis

e) The mucous membrane f the lower half receives its arterial supply from the

inferior rectal artery

50. All of the following structures exit the pelvis via the greater sciatic foramen
except:

a) Piriformis muscle

b) Superior gluteal vessels

c) Internal pudendal vessels

d) Obturator internus muscle

666 ‫صفحة‬ Dr. MOHAMED YAHIA 0900987639


QUESTION BANK OF Dr. MOHAMED YAHIA

e) Inferior gluteal vessels

51. The pelvic diaphragm:

a) Forms the inferior boundary of the pelvic cavity

b) Can be divided into levator ani and coccygeus muscles

c) Exhibits midline openings for the rectum and urethra in the male

d) Is attached to the pubis, lateral pelvic wall (arcus tendineus) and ischial

spine

e) All of the above

52. The ovaries are anchored to the lateral pelvic wall by the:

a) Ovarian ligament

b) Broad ligament

c) Suspensory ligament

d) Mesovarium

e) Round ligament

53. Which of the following statements is true?

a) The internal pudendal artery is embedded in a fascial canal within the

urogenital triangle

b) The pudendal nerve is a branch of the lumbar plexus

c) The posterior scrotal (labial) nerves are not branches of the inferior rectal

nerve

d) The pudendal nerve enters the perineum (from the gluteal region) by

passing through the greater sciatic foramen

e) The pudendal nerve supplies the external anal sphincter

667 ‫صفحة‬ Dr. MOHAMED YAHIA 0900987639


QUESTION BANK OF Dr. MOHAMED YAHIA

54. Which of the following has an intra- and extrapelvic component:

a) Broad ligament

b) Ovarian ligament

c) Round ligament of the uterus

d) Transverse cervical (cardinal) ligament

e) More than one of the above

55. Which structures do not pass through the deep perineal pouch of the
urogenital triangle:

a) Urethra

b) Root of the clitoris

c) Deep transverse perineal muscle

d) Internal pudendal artery

e) Vagina

56. The urinary bladder:

a) Is lined by stratified squamous epithelium

b) Has lymphatics which drain to nodes alongside the internal iliac artery

c) Has a trigone whose endothelium is endoderm-derived

d) Has a medial umbilical ligament arising from its apex

e) Lies anterior to the anal canal in the male

57. Concerning the pudendal nerve, select the incorrect statement:

a) Is the main motor and sensory supply of the pelvic floor and perineum

b) Has a root value of S1-3

c) Passes between piriformis and coccygeus to reach the ischioanal fossa

d) Has the inferior rectal nerve as one of its branches which supplies the

668 ‫صفحة‬ Dr. MOHAMED YAHIA 0900987639


QUESTION BANK OF Dr. MOHAMED YAHIA

levator ani

e) Has the perineal nerve as one of its terminal branches

58. Which is not true about the rectum?

a) Receives its blood supply from the superior, middle and inferior rectal

arteries, the superior being the main artery supplying the mucous membrane

b) Receives some of its blood supply from the middle rectal artery- a branch of

the internal iliac artery which may sometimes be absent

c) Receives some of its blood supply from the internal pudendal artery via the

inferior rectal artery

d) Has a venous drainage which includes the middle rectal vein, which drains

to the portal circulation

e) Has a lymph drainage which reaches both the inferior mesenteric and

internal iliac lymph nodes

59. The Prostate Gland:

a) Has peritoneum on its posterior surface

b) Releases its secretions into the ejaculatory duct

c) Is supported by both the urogenital and pelvic diaphragms

d) If healthy, cannot be palpated by rectal digital examination

e) All of the above

ABDOMEN

1. The following statements concerning the liver are true except:

a) The quadrate lobe drains into the right hepatic duct

669 ‫صفحة‬ Dr. MOHAMED YAHIA 0900987639


QUESTION BANK OF Dr. MOHAMED YAHIA

b) The lesser omentum suspends the stomach from the visceral


surface of the liver

c) The left triangular ligament of the liver lies anterior to the abdominal part of

the esophagus

d) The attachment of the hepatic veins to the inferior vena cava is one of the

most important supports of the liver

e) The ligamentum venosum is attached to the left branch of the portal vein in

the porta hepatis

2. The following statements concerning the pancreas are true except:

a) The pancreas receives part of its arterial supply from the splenic

artery

b) The main pancreatic duct opens into the third part of the duodenum

c) The uncinate process of the pancreas projects from the head of the pancreas

d) The bile duct (common bile duct) lies posterior to the head of the pancreas

e) The transverse mesocolon is attached to the anterior border of the pancreas

3. The following statements concerning the ileum are true except:


a) The circular smooth muscle of the lower end of the ileum serves as a

sphincter at the junction of the ileum and cecum

b) The branches of the superior mesenteric artery serving the ileum form more
arcades than those serving the jejunum

c) Peyer's patches are present in the mucous membrane of the lower ileum

along the antimesenteric border

d) The plicae circulares are more prominent at the distal end of the ileum than

in the jejunum

e) The parasympathetic innervation of the ileum is from the vagus nerve

670 ‫صفحة‬ Dr. MOHAMED YAHIA 0900987639


QUESTION BANK OF Dr. MOHAMED YAHIA

4. The hilum of the right kidney contains the following important structures,
except the:

a) Renal Pelvis

b) Tributaries of the renal vein

c) Sympathetic nerve fibres

d) Part of the right suprarenal gland

e) Branches of the renal artery

5. The following statements concerning the left suprarenal gland are incorrect
except:

a) It extends behind the inferior vena cava

b) It is separated from the left kidney by the pararenal fat

c) Its vein drains into the left renal vein

d) It is usually located on the upper pole and lateral border of the left kidney

e) The medulla is innervated by postganglionic sympathetic nerve fibres

```````````````````````````````````````````````````````````````````````````````````````````````````````
````````````````````````

```````````````````````````````````````````````````````````````````````````````````````````````````````
```````````````````````

6. The following statements concerning the abdominal aorta are true except:

a) It bifurcates into the two common iliac arteries in front of the fourth lumbar
vertebra

b) It lies on the right side of the inferior vena cava

671 ‫صفحة‬ Dr. MOHAMED YAHIA 0900987639


QUESTION BANK OF Dr. MOHAMED YAHIA

c) From its anterior surface arise the celiac, superior mesenteric and inferior

mesenteric arteries

d) It enters the abdomen in front of the twelfth thoracic vertebra

e) The thoracic duct leaves the abdomen through the aortic opening of the

diaphragm on the right side of the aorta

7. The following statements concerning the abdominal part of the sympathetic


trunk are NOT TRUE except:

a) It enters the abdomen behind the lateral arcuate ligament

b) The trunk possesses six segmentally arranged ganglia

c) All the ganglia receive white rami communicantes

d) Gray rami communicantes are given off to the lumbar spinal nerves

e) The splanchnic nerves from the thorax join the trunks below the diaphragm

8. The following statements concerning the lumbar plexus are true except:

a) The plexus lies within the psoas muscle

b) The plexus is formed from the posterior rami of the upper four lumbar

nerves

c) The femoral nerve emerges from the lateral border of the psoas muscle

d) The obturator nerve emerges from the medial border of the psoas muscle

e) The iliohypogastric nerve emerges from the lateral border of the psoas

muscle

9. The following veins form important portal-systemic anastamoses, except the:

a) Esophageal branches of the left gastric vein and tributaries of the azygos

vein

b) Superior rectal vein and the inferior vena cava

672 ‫صفحة‬ Dr. MOHAMED YAHIA 0900987639


QUESTION BANK OF Dr. MOHAMED YAHIA

c) Paraumbilical vein and the superficial veins of the anterior abdominal wall

d) Veins of the ascending and descending parts of the colon with the lumbar

veins

e) Veins from the bare areas of the liver with the phrenic veins

10. The following statements concerning the ureters are true except:

a) Both have three anatomic sites that are constricted

b) Both receive their blood supply from the testicular or ovarian arteries

c) Both are separated from the transverse processes of the lumbar vertebrae by

the psoas muscle

d) Both pass anterior to the testicular or ovarian vessels

e) Both lie anterior to the sacroiliac joints

11. Concerning the inferior mesenteric artery, all of the following statements are
true except:

a) Its colic branch supplies the descending colon

b) It gives off the inferior pancreaticoduodenal artery

c) It supplies the sigmoid colon

d) Its branches contribute to the marginal artery

e) It arises from the aorta immediately below the third part of the duodenum

12. Which of the following structures is NOT present within the lesser omentum?

a) Portal vein

b) Bile duct

c) Inferior Vena Cava

673 ‫صفحة‬ Dr. MOHAMED YAHIA 0900987639


QUESTION BANK OF Dr. MOHAMED YAHIA

d) Hepatic artery

e) Lymph Nodes

13. The following statements concerning the liver are true except:

a) Its lymph drainage is to the celiac nodes

b) The quadrate and the caudate lobes are functionally part of


the left lobe

c) Its parasympathetic innervation is from the vagus nerve

d) It receives highly oxygenated blood from the portal vein

e) The triangular ligaments connect the liver to the diaphragm

14. The following differences exist between the ileum and the ascending colon
except:

a) The ascending colon has appendices epiploicae, whereas the ileum

does not

b) The arterial supply to the wall of the ileum is arranged so that it produces

areas of weakness through which mucosal herniations may occur

c) The ascending colon may have a well-developed marginal artery, whereas

the ileum does not

d) The ileum has a longitudinal muscle that forms a continuous layer around

the wall, whereas the ascending colon has tenia coli

e) The ascending colon is retroperitoneal, whereas the ileum is intraperitoneal

15. Which of the following statements is (are) correct regarding the duodenum?
a) The duodenum receives its entire blood supply from the superior mesenteric
artery

b) The first part of the duodenum is completely retroperitoneal

c) The bile duct enters the second (vertical) portion of the duodenum

674 ‫صفحة‬ Dr. MOHAMED YAHIA 0900987639


QUESTION BANK OF Dr. MOHAMED YAHIA

d) The lymph from the fourth part of the duodenum drains into the celiac

nodes

e) The tip of the appendix cannot reach down into the pelvic cavity

16. The following statements concerning the appendix are correct except:

a) The appendix is situated in the right iliac region

b) At the base of the appendix, the teniae coli of the cecum fuse to form a

complete, longitudinal muscle layer in the wall of the appendix

c) Afferent pain nerve fibers accompany the sympathetic nerves and enter the

spinal cord at the level of the tenth thoracic segment

d) The appendix receives its blood supply from a branch of the posterior cecal
artery

e) The tip of the appendix cannot reach down into the pelvic cavity

17. The following statements concerning the pyloric sphincter are correct except:

a) It receives its motor innervation from the sympathetic autonomic nerves

b) It lies on the level of the third lumbar vertebrae

c) It is inhibited by the impulses passing down the vagus nerve

d) It is formed by a thickening of the circular layer of smooth muscle in the

stomach wall

e) The cavity of the pylorus is called the pyloric canal

18. The following structures form the boundaries of the entrance into the lesser
sac (epiploic foramen) except:

a) The inferior vena cava

b) The bile duct

c) The portal vein

d) The quadrate lobe of liver

675 ‫صفحة‬ Dr. MOHAMED YAHIA 0900987639


QUESTION BANK OF Dr. MOHAMED YAHIA

e) The first part of the duodenum

19. After complete occlusion of the origin of the inferior mesenteric artery with a
blood clot, the blood supply of the left portion of the colon is maintained by the
following arteries except:

a) The marginal artery (of Drummond)

b) The middle colic artery

c) The left lumbar arteries

d) Anastomoses between the superior, the middle and the inferior

hemorrhoidal arteries

e) The sigmoid arteries

20. The following statements concerning the gall bladder are correct except :

a) The arterial supply is from the cystic artery, which is a branch of the right

hepatic artery

b) The fundus of the gallbladder is located just beneath the tip of the right

ninth costal cartilage

c) The peritoneum completely surrounds the fundus, the body and the neck

d) The nerves of the gallbladder are derived from the celiac plexus

e) Pain sensation from gallbladder disease may be referred along the phrenic

and supraclavicular nerves to the skin over the shoulder

21. In patients with an obstruction of the superior vena cava, blood may return
to the right atrium through the following anastomotic channels except:

a) The lateral thoracic, the lumbar, and the superficial epigastric veins

b) The superior and the inferior epigastric veins

c) The lateral thoracic, the paraumbilical and the portal veins

d) The posterior intercostal and the lumbar veins

676 ‫صفحة‬ Dr. MOHAMED YAHIA 0900987639


QUESTION BANK OF Dr. MOHAMED YAHIA

e) The lateral thoracic and the superior epigastric veins

22. Which of the following structures may be eroded by a perforating gastric


ulcer on the posterior wall of the stomach?

a) The inferior mesenteric artery

b) The splenic artery

c) The right kidney

d) The second part of the duodenum

e) The quadrate lobe of the liver

23. Which of the following statements regarding the left ureter is correct?

a) It has the inferior mesenteric vein lying on its lateral side

b) Its lumen is constricted at the point where it crosses the brim of the pelvis

c) Its entire arterial supply is derived from the left renal artery

d) It lies in direct contact with the tips of the transverse processes of the

lumbar vertebrae

e) It lies within the peritoneal cavity

24. The following structures are connected to the liver. Which structures provide
the greatest support for the liver?

a) The falciform ligament

b) The coronary ligament

c) The ligamentum teres

d) The hepatic veins joining the inferior vena cava

e) The ligamentum venosum

677 ‫صفحة‬ Dr. MOHAMED YAHIA 0900987639


QUESTION BANK OF Dr. MOHAMED YAHIA

25. A gallstone has eroded through the posterior wall of the gallbladder and
entered the intestinal tract. Which part of the intestinal tract is likely to initially
contain the stone?

a) The sigmoid colon

b) The descending colon

c) The transverse colon

d) The ascending colon

e) The jejunum

26. Which of the following statements regarding the superficial inguinal ring is
incorrect?

a) It is a perforation in the aponeurosis of the external oblique muscle

b) Its greatest width lies above and medial to the pubic tubercle

c) It is strengthened posteriorly by the conjoint tendon

d) The internal spermatic fascia is attached to its margins

e) In males, it allows passage of the spermatic cord and the ilioinguinal nerve

27. A surgeon decides to divide the anterior vagal trunk (vagotomy) as it lies on
the anterior surface the abdominal part of the esophagus as therapy for a
chronic gastric ulcer not responding to medical treatment. Which of the
following is likely to result from this procedure?

a) A loss of secretomotor nerve supply to the mucosal glands of the stomach

b) Hoarse voice because of paralysis of the intrinsic muscles of the larynx on

the left side

c) Increased heart rate because of decreased parasympathetic input to the

cardiac plexus

d) Incontinence because of an absence of parasympathetic input to the bladder

e) Compromised input into the greater splanchnic nerves

678 ‫صفحة‬ Dr. MOHAMED YAHIA 0900987639


QUESTION BANK OF Dr. MOHAMED YAHIA

28. The skin of the umbilicus receives its sensory innervation from:

a) T7

b) T12

c) L1

d) T10

e) L2

29. In both sexes, the inguinal canal is formed by:

a) The descent of the gonad

b) The contraction of the gubernaculum

c) The processus vaginalis

d) The ilioinguinal nerve

e) The contraction of the cremaster muscle

30. The lacunar ligament:

a) Formed from the conjoint tendon

b) Part of the posterior wall of the rectus sheath

c) Not continuous with the inguinal ligament

d) An important medial relation to the femoral ring of the femoral sheath

e) Attached to the inferior ramus of the pubis

31. The following statements concerning the superficial fascia of the anterior
abdominal wall are true except:

a) It has a superficial fatty layer and a deep membranous layer

679 ‫صفحة‬ Dr. MOHAMED YAHIA 0900987639


QUESTION BANK OF Dr. MOHAMED YAHIA

b) Scarpa's fascia fuses with the fascia lata just below the inguinal ligament

c) Camper's fascia is continuous with the Colles' fascia in the perineum

d) It is continuous with the dartos muscle in the wall of the scrotum

e) It does not contribute to the femoral sheath

32. The rectus sheath contains the following structure except:

a) The pyramidalis muscle (when present)

b) The ligamentum teres

c) The inferior epigastric artery

d) The T7-12 anterior rami

e) The rectus abdominis muscle

33. The following statements concerning the epididymis are correct except:

a) It lies within the scrotum and posterior to the testis

b) It has an expanded upper end called the head

c) The tail gives rise to the vas deferens

d) It is supplied by the testicular artery

e) Its lymph drains into the superficial inguinal nodes

34. The jejunum and ileum can be differentiated on the basis of the following
anatomic features except:

a) Numerous (4-5) arterial arcades are associated with the jejunum

b) The plicae circulares are much more prominent in the jejunum than in the

ileum

c) Fat depositions are generally present throughout the mesentery associated

with the ileum

d) The jejunum is located in the upper left region of the abdominal cavity

680 ‫صفحة‬ Dr. MOHAMED YAHIA 0900987639


QUESTION BANK OF Dr. MOHAMED YAHIA

e) Peyer's patches are characteristic of the lower ileum and may be visible on

the surface

35. The spermatic cord contains the following structures except:

a) The scrotal arteries and veins

b) The vas deferens

c) The pampiniform plexus

d) The testicular artery

e) Autonomic nerves

36. The following structures pass through the esophageal hiatus in the
diaphragm except:

a) The left vagus nerve

b) Branches of the left gastric artery

c) The left phrenic nerve

d) The right vagus nerve

e) A tributary of the portal vein

37. The following statements regarding the peritoneum are incorrect except:

a) The parietal peritoneum is only sensitive to stretch

b) The parietal peritoneum lining the anterior abdominal wall is innervated by


the lower six thoracic and the first lumbar spinal nerves

c) The visceral peritoneum is innervated by the posterior primary rami of the

twelfth thoracic and the lumbar spinal nerves

d) The visceral peritoneum is sensitive to pain, temperature and touch

e) The peritoneum lining the lesser sac has no nerve supply

681 ‫صفحة‬ Dr. MOHAMED YAHIA 0900987639


QUESTION BANK OF Dr. MOHAMED YAHIA

38. If the portal vein becomes blocked, the following venous anastomoses are
important in uniting the portal with the systemic venous systems except:

a) Esophageal branches of the left gastric and the azygos veins

b) Veins of the ligamentum teres and the paraumbilical veins

c) The superior and the inferior rectal veins

d) The right colic veins and the lumbar veins

e) The middle and the inferior rectal veins

39. In a patient with cancer of the stomach who requires a total gastrectomy, the
following arteries must be ligated except:

a) The common hepatic artery

b) The short gastric arteries

c) The left and right gastroepiploic arteries

d) The right gastric artery

e) The left gastric artery

40. When performing a midline abdominal paracentesis (tapping the abdominal


cavity) below the umbilicus, the cannula will pass through the following
anatomic structures except:

a) The skin and the fascia

b) The linea alba

c) The transversalis fascia and extraperitoneal fat

d) The rectus abdominis muscle

e) The parietal peritoneum

41. The following statements are correct regarding and inguinal hernia except:

a) The inferior epigastric artery lies medial to the neck of an indirect inguinal
hernia

b) An inguinal hernia is more common than a femoral hernia in females

682 ‫صفحة‬ Dr. MOHAMED YAHIA 0900987639


QUESTION BANK OF Dr. MOHAMED YAHIA

c) A direct inguinal hernia is more common in elderly men than in boys

d) The opening of the sac of a direct inguinal hernia is wide

e) The contents of an indirect inguinal hernia may be strangulated against the

lacunar ligament

42. The following statements regarding the portal vein are true except:

a) It courses through a portion of the lesser omentum

b) It enters the liver at the porta hepatis

c) It receives venous blood from both the large and small intestines

d) It originates at the junction of the superior mesenteric and the splenic veins

e) It passes in front of the neck of the pancreas

43. If the common hepatic artery is unavoidably ligated during surgery, the
arterial supply to the liver is maintained by the following anastomic connections
except:

a) The superior pancreaticoduodenal artery anastomosing with the inferior

pancreaticduodenal artery

b) The right gastric artery anastomosing with the left gastric artery

c) The gastroduodenal artery anastomosing with the splenic artery

d) The esophageal arteries anastomosing with the inferior phrenic arteries

e) The right gastroepiploic artery anastomosing with left gastroepiploic artery

44. The following general statements are incorrect except:

a) The superior and inferior epigastric vessels anastomose on the anterior

surface of the rectus abdominis muscle

b) The inguinal ligament is attached laterally to the anterior inferior iliac spine

c) The abdominal aorta bifurcates below, in front of the fourth lumbar

683 ‫صفحة‬ Dr. MOHAMED YAHIA 0900987639


QUESTION BANK OF Dr. MOHAMED YAHIA

vertebrae

d) The foregut ends in the third (inferior) segment of the duodenum

e) The umbilicus is located approximately at the vertebral level L5

45. The following veins drain directly into the inferior vena cava except:

a) The hepatic veins

b) The renal veins

c) The lumbar veins

d) The inferior mesenteric vein

e) The right testicular (ovarian) vein

46. To tap a hydrocele (collection of fluid in the tunica vaginalis) a cannula must
pierce the following structures except:

a) The skin

b) The fatty subcutaneous tissue (fascia of Camper)

c) Dartos muscle

d) The cremasteric fascia

e) The external and the internal spermatic fasciae

47. Which of the following arteries supplies the suprarenal (adrenal) gland:

a) The aorta

b) The lumbar arteries

c) The superior phrenic artery

d) The testicular (ovarian) artery

e) The subcostal artery

48. The following structures are present in the porta hepatic except:

684 ‫صفحة‬ Dr. MOHAMED YAHIA 0900987639


QUESTION BANK OF Dr. MOHAMED YAHIA

a) Lymph nodes

b) The right and left branches of the portal vein

c) The right and left hepatic ducts

d) The right and left hepatic veins

e) The right and left branches of the hepatic artery

49. The following statements regarding the celiac plexus are true except :

a) The celiac plexus is not a purely sympathetic plexus

b) The celiac plexus does not surround the celiac artery

c) The celiac ganglia are made of nerve cell bodies and nerve fibers

d) Parasympathetic preganglionic fibers pass through the plexus, generally

synapsing with postganglionic neurons within the walls of the organs they

innervate

e) Sympathetic preganglionic fibers to the celiac plexus originate from the

thoracic spinal segments and travel in thoracic splanchnic nerves

50. The right kidney has the following important relationships except:

a) It is related to the neck of the pancreas

b) It is anterior to the right costodiaphragmatic recess

c) It is related to the second part of the duodenum

d) It is related to the right colic flexure

685 ‫صفحة‬ Dr. MOHAMED YAHIA 0900987639


QUESTION BANK OF Dr. MOHAMED YAHIA

e) It is anterior to the right twelfth rib

51. The following structures form the walls of the inguinal canal except the:
a) Conjoint tendon

b) Aponeurosis of external oblique muscles

c) Internal oblique muscle

d) Lacunar ligament

e) Fascia transversalis

52. The following structures and events serve to strengthen the walls of the
inguinal canal except:

a) The inguinal ligament is made tense by extending the hip joint

b) The contracting internal oblique muscle reinforces the anterior wall of the

the canal in front of the weak deep inguinal ring

c) The strong conjoint tendon reinforces the posterior wall of the canal behind

the weak superficial inguinal ring

d) Contraction of the arching fibres of the internal oblique and transverse

abdominis muscles lowers the roof of the canal so that the canal is

practically obliterated

e) After birth, as the result of growth, the deep inguinal ring moves laterally

away from the superficial ring so that the canal becomes oblique and the

two rings no longer lie opposite one another

53. In the female, the inguinal canal contains the following structures except the:
a) Ilioinguinal nerve

b) Remnant of the processus vaginalis

c) Round ligament of the uterus

d) Inferior epigastric artery

686 ‫صفحة‬ Dr. MOHAMED YAHIA 0900987639


QUESTION BANK OF Dr. MOHAMED YAHIA

e) Lymph vessels from the fundus of the uterus

54. All of the following structures concerning the spermatic cord are true except:

a) It extends from the deep inguinal ring to the scrotum

b) It contains the testicular artery

c) It is covered by five layers of spermatic fascia

d) It contains the pampiniform plexus

e) It contains lymph vessels that drain the testis

55. All the following structures are present in the inguinal canal in the male
except:

a) Internal spermatic fascia

b) Genital branch of the genitofemoral nerve

c) Testicular vessels

d) Deep circumflex iliac artery

e) Ilioinguinal nerve

56. All of the following statements concerning the conjoint tendon are true
except:

a) It is attached to the pubic crest and the pectineal line

b) It is formed by the fusion of the aponeuroses of the transverses abdominis

and internal oblique muscles

c) It is attached medially to the linea alba

d) It is continuous with the inguinal ligament

e) It may bulge forward in a direct inguinal hernia

687 ‫صفحة‬ Dr. MOHAMED YAHIA 0900987639


QUESTION BANK OF Dr. MOHAMED YAHIA

57. All of the following statements concerning the an indirect inguinal hernia are
true except:

a) It is the most common form of abdominal hernia

b) The neck of the hernial sac lies medial to the inferior epigastric artery

c) The sac is the remains of the processus vaginalis

d) The hernial sac can extend into the scrotum

e) At the superficial inguinal ring, the hernial sac lies above and medial to the

pubic tubercle

58. To pass a needle into the cavity of the tunica vaginalis in the scrotum, the
following structures have to be pierced except:

a) Skin

b) Dartos muscle and Colles' fascia

c) Tunica albuginea

d) Internal spermatic fascia

e) Cremasteric fascia

59. The following statements are true about muscles forming the posterior
abdominal wall except:

a) The psoas major muscle has a fascial sheath that extends down into the

thigh as far as the lesser trochanter of the femur

b) The quadratus lumborum is covered anteriorly by fascia that forms the

lateral arcuate ligament

c) The iliacus muscles is innervated by the femoral nerve

d) The transverses abdominis muscles does form part of the posterior

abdominal wall

e) The diaphragm does not contribute to the musculature on the posterior

abdominal wall

688 ‫صفحة‬ Dr. MOHAMED YAHIA 0900987639


QUESTION BANK OF Dr. MOHAMED YAHIA

60. Regarding hernias in the inguinal region

a) The sac of an indirect hernia lies posterior to the spermatic cord

b) The sac of a direct hernia lies lateral to the internal ring

c) The sac of a direct hernia lies medial to the inferior epigastric vessels

d) The sac of a direct inguinal hernia usually comes down to the scrotum

e) The sac of the femoral hernia has a wide neck

61. The borders of Hesselbech's trangle include the epigastric vessels, the edge of
the rectus sheath and:

a) Poupart''s ligament (the reflected inguinal ligament)

b) The internal oblique aponeurosis

c) The external oblique aponeurosis

d) Transversalis fascia

e) The conjoint tendon

62. The only statement regarding the small bowel which is true is:

a) The entire small bowel is intra-peritoneal

b) The jejunum is longer in length, large in diameter, and thinner-walled than


the ileum

c) The muscularis, the muscle layer, provides the strength for the

placement of sutures or staples for surgical anastomoses

d) Peyer's patches are most prominent in the distal ileum

e) The marginal artery of Drummond provides the blood supply to the

duodenum

63. The following layers are encountered in a paramedian surgical incision (an
incision lateral to midline) from above downwards:

689 ‫صفحة‬ Dr. MOHAMED YAHIA 0900987639


QUESTION BANK OF Dr. MOHAMED YAHIA

a) The skin, subcutaneous fat, deep fascia, rectus muscle and peritoneum

b) The skin, superficial fascia, subcutaneous fat, anterior rectus sheath, rectus

muscle and peritoneum

c) The skin, subcutaneous fat, anterior rectus sheath, rectus muscle, posterior

rectus sheath and peritoneum

d) The skin, subcutaneous fat, extraperitoneal fat, anterior rectus sheath and

muscle

e) The skin, subcuataneous fat, anterior rectus sheath, peritoneum and

posterior rectus sheath

64. The appendix:

a) Arises from the posteromedial aspect of the caecum below the ileoceccal

valve

b) Has an extremely variable position

c) Is supplied by a branch from the right colic artery

d) Is devoid of lymph nodes

e) Is removed surgically through akocher incision

65. The following are correct regarding the gall-bladder except:

a) The Hartmann's pouch lies close to the fundus

b) The cystic artery will be found in Calot's triangle in the majority of cases

c) The cystic artery has no accompanying vein

d) Bleeding during surgery can be controlled by compressing the hepatic

artery

e) Cholecystectomy can be done through a right paramedian incision

690 ‫صفحة‬ Dr. MOHAMED YAHIA 0900987639


QUESTION BANK OF Dr. MOHAMED YAHIA

66. Below the arcuate line, which of the following structures lies posterior to the
rectus muscle?

a) Linea alba

b) Aponeurosis of transverses abdominis

c) Aponeurosis of internal oblique

d) Transversalis fascia

e) Aponeurosis of external oblique

67. Which of the following structures of the spermatic cord is derived from the
internal oblique muscle?

a) Internal spermatic fascia

b) External spermatic fascia

c) Cremaster muscle

d) Dartos muscle

e) Vas deferens

68. Which of the following organs does not receive its blood supply from the
celiac trunk?

a) Liver

b) Stomach

c) Spleen

d) Jejunum

e) Gall bladder

69. The common bile duct and the main pancreatic duct empty into the:

a) First part of the duodenum

b) Second part of the duodenum

c) Third part of the duodenum

691 ‫صفحة‬ Dr. MOHAMED YAHIA 0900987639


QUESTION BANK OF Dr. MOHAMED YAHIA

d) Gall bladder

e) Vermiform appendix

70. Which of the following organs is retroperitoneal?

a) Spleen

b) Stomach

c) Jejunum

d) Pancreas

e) Transverse colon

71. Which of the following is not a symptom of portal vein hypertension?

a) Esophageal bleeding

b) Hemorrhoids

c) Caput medusae

d) Indirect inguinal hernia

e) Engorged veins of the posterior abdominal wall

72. What structure passes behind the medial arcuate ligament of the abdominal
diaphragm:

a) The aorta

b) The esophagus

c) The psoas major muscle

d) The quadratus lumborum muscle

e) The inferior vena cava

73. Which of the following is not a branch of the superior mesenteric artery?

a) Right colic artery

692 ‫صفحة‬ Dr. MOHAMED YAHIA 0900987639


QUESTION BANK OF Dr. MOHAMED YAHIA

b) Left colic artery

c) Middle colic artery

d) Iliocolic artery

e) Inferior pancreaticoduodenul artery

74. Which of the following is not characteristic of the large intestine?

a) Taenia coli

b) Haustra

c) Appendices epiploicae

d) Vermiform appendix

e) Epiploic foramen

75. Which of the following statements about the liver is false?

a) Its blood supply normally comes from the celiac trunk

b) It is the largest gland in the body

c) It has four lobes right, left, quadrate and caudate

d) It is connected to the stomach by the lesser omentum

e) Its venous drainage is via the portal vein

76. Which of the following is false about the normal pattern of arterial blood
supply to the stomach:

a) Lesser curvature/ Right gastric artery

b) Pylorus/ Left gastric artery

c) Greater curvature/ left gastroepiploic artery

d) Greater curvature/ right gastroepiploic artery

693 ‫صفحة‬ Dr. MOHAMED YAHIA 0900987639


QUESTION BANK OF Dr. MOHAMED YAHIA

e) Fundus/ Short gastric arteries

77. The area around the umbilicus receives sensory innervation from the:

a) Superficial epigastric arteries

b) Anterior branch of the ventral ramus of the spinal nerve T10

c) Subcostal nerve

d) Splanchnic nerve

e) Vagus

78. Inferior the arcuate line, which of the following structures lies posterior to
the rectus muscle?

a) Aponeurosis of external oblique

b) Aponeurosis of internal oblique

c) Aponeurosis of internal oblique and tranversus abdominis

d) Internal oblique fascia

e) Transversalis fascia, peritoneum

79. Which layer of the spermatic cord is derived from the internal oblique
muscle of the abdominal wall?

a) External spermatic fascia

b) Cremaster muscle

c) Internal spermatic fascia

d) Tunica vaginalis

e) Tunica albuginea

80. Which of the following statements about indirect inguinal hernia is false?

a) They enter the abdominal wall lateral to the inferior epigastric artery

b) They are more common in the males than in females

694 ‫صفحة‬ Dr. MOHAMED YAHIA 0900987639


QUESTION BANK OF Dr. MOHAMED YAHIA

c) They follow the path of the inguinal canal

d) They often result from a patent processus vaginalis

e) They involve and outpouching of gut below the inguinal ligament

81. Which of the following is not a site of portal systemic anastomosis?

a) Lower part of esophagus

b) Umbilicus

c) Rectum

d) Greater curvature of the stomach

e) Posterior abdominal wall

82. The conjoint tendon (falx ingunalis) is formed by the:

a) Transverse abdominis muscle or its aponeurosis

b) Internal abdominal oblique muscle or its aponeurosis

c) Both A and B

d) Neither A nor B

83. Which of the following supply the lesser curvature of the stomach?

a) Left gastric artery

b) Right gastric artery

c) Short gastric arteries

d) All of the above

e) Both A and B

695 ‫صفحة‬ Dr. MOHAMED YAHIA 0900987639


QUESTION BANK OF Dr. MOHAMED YAHIA

84. Which of the following is not involved in arterial blood supply to the anterior
abdominal wall?

a) Superior epigastric artery

b) Intercostal arteries

c) Deep circumflex iliac artery

d) Musculophrenic artery

e) Inferior epigastric artery

85. The external oblique muscle:

a) Inserts on the internal surfaces of the seventh through twelfth costal

cartilages

b) Makes up the entire rectus sheath inferior to the umbilicus

c) Originates on the pubic symphysis and pubic crest

d) Extends the trunk

e) Is innervated by the inferior six thoracic spinal nerves (intercostals nerves

and subcostal nerves)

86. Which of the following is the obliterated umbilical vein?

a) Ligamentum venosum

b) Ligamentum teres

c) Median umbilical ligament

d) Lateral umbilical ligament

e) Falciform ligament

87. The hepatic hilum consists of:

a) Hepatic duct

b) Branches of the portal vein

696 ‫صفحة‬ Dr. MOHAMED YAHIA 0900987639


QUESTION BANK OF Dr. MOHAMED YAHIA

c) The hepatic artery

d) The hepatic vein, cystic duct and main pancreatic duct

e) The portal vein, hepatic artery and common hepatic duct

88. The abdominal aorta:

a) Is surrounded by the central portion of the diaphragm

b) Is surrounded by plexus of autonomic nerve fibers

c) Both A and B

d) Neither A nor B

89. Position of the deep inguinal ring relative to the inferior epigastric vessels is:

a) Medial

b) Lateral

c) Posterior

d) Anterior

e) Superior

90. Regarding the pancreas:

a) Exocrine secretions are released into the interstitial spaces

b) The pancreas is invested by a prominent and distinct connective tissue

capsule

c) The pancreas is retroperitoneal except for the tail which is intraperitoneal

d) Centroaciner cells appear acidophilic when stained by routine hematoylin

and eosin methods due to the high number of secretory vesicles

697 ‫صفحة‬ Dr. MOHAMED YAHIA 0900987639


QUESTION BANK OF Dr. MOHAMED YAHIA

e) All of the above

91. Primary arterial supply of which region travels via the superior mesenteric
artery?

a) Infradiaphragmatic foregut

b) Midgut

c) Hindgut

d) Both A and C

e) All of the above

92. The renal arteries:

a) Originate directly from the abdominal aorta

b) Provide arterial blood to the adrenal glands

c) Provide arterial blood to the kidneys

d) Usually form four anterior segmental arteries within the hilum of the kidney

e) All of the above

93. The median, medial and lateral umbilical folds of the anterior abdominal
wall are formed respectively by the:

a) Urachus, obliterated umbilical arteries and inferior epigastric arteries

b) Urachus, obliterated umbilical arteries and veins

c) Obliterated umbilical arteries and veins

d) Urachus and inferior epigastric artery

e) Urachus, obliterated umbilical arteries and superficial epigastric arteries

94. The following statements are true about the cisterna cyli except:

a) Receives lymph from nodes draining the posterior abdominal wall

b) Is in direct continuity with the inferior vena cava

698 ‫صفحة‬ Dr. MOHAMED YAHIA 0900987639


QUESTION BANK OF Dr. MOHAMED YAHIA

c) Is a terminal dilation of the thoracic duct

d) Is generally located inferior to the aortic hiatus

95. The following statements about the kidney are true except:

a) Is in contact with the spleen on the left side

b) Is in contact with duodenum on the right side

c) Is located retroperitoneally

d) Receives its blood supply from a branch of the celiac trunk

96. Select the one best answer:

a) The superior epigasrtic artery is a branch of the external iliac artery

b) The anterior portion of the rectus sheath is formed by the aponeurosis of the
external abdominal oblique and transverses abdominis muscle

c) The inguinal canal terminates at the superficial inguinal ring ( a defect in

the external abdominal oblique muscle)

d) The conjoint tendon consists of lower fibers of the external abdominal

muscle and the inguinal ligament

e) The internal spermatic fascia is derived from the internal oblique muscle

97. Which of the following is not true in relation to the stomach?

a) It is located at the supramesocolic compartment

b) The right gastroepiploic artery running along the greater curvature, is a

branch of the gastrodudodenal artery

c) The cardiac notch is an incisure found at the most dependent point of the

lesser curvature

d) The greater curvature gives attachment to the greater omentum

e) Cardiac orifice is at T11 vertebral level

699 ‫صفحة‬ Dr. MOHAMED YAHIA 0900987639


QUESTION BANK OF Dr. MOHAMED YAHIA

98. Select the correct statement:

a) Falciform ligament

b) The liver occupies the right hypochondriac and epigastric region of the

abdomen

c) The quadrate lobe of the liver is adjacent to the inferior vena cava

d) The caudate lobe is the bed for the gallbladder

e) The cystic bile duct leads into the proximal duodenum

99. The posterior limit of the omental (epiploic) foramen is:

a) The free border of the lesser omentum

b) The hepatic artery

c) The caudate lobe

d) The duodenum

e) The inferior vena cava

100. Indicate the correct statement:

a) The pancreas is located in the epigastric and right hypochondriac regions

b) The pancreas is divided into head, neck and tail

c) The uncinate process projects medially behind the superior mesenteric

vessels

d) The posterior surface of the head of the pancreas is related to the left crus

e) The posterior aspect of the neck of the pancreas is covered with peritoneum
and is adjacent to the pylorus

101. The common bile duct and the pancreatic ducts empty into the:

a) First part of the duodenum

b) Second part of the duodenum

700 ‫صفحة‬ Dr. MOHAMED YAHIA 0900987639


QUESTION BANK OF Dr. MOHAMED YAHIA

c) Third part of the duodenum

d) Second part of the duodenum (8cm) from the pylorus

e) Middle of the 2nd part of the duodenum

102. Which of the following is retroperitoneal?

a) Spleen

b) Stomach

c) Jejunum

d) Head of pancreas

e) Transverse colon

103. What structure passes behind the medial arcuate ligament of the abdominal
diaphragm:

a) The aorta

b) The oesophagus

c) The psoas major

d) The quadratus lumborum

e) The inferior vena cava

104. A posterior ulcer of the first part of the duodenum may penetrate the wall
and erode an artery running behind it, causing severe internal haemorrhage.
This artery is likely to be:

a) Hepatic

b) Right gastric

c) Gastroduodenal

d) Left gastric

e) Splenic

701 ‫صفحة‬ Dr. MOHAMED YAHIA 0900987639


QUESTION BANK OF Dr. MOHAMED YAHIA

105. A tumor originating from the sigmoid colon may reach the liver via the
blood stream. Which of the following veins would be first involved in the
pathway?

a) Inferior vena cava

b) External iliac vein

c) Splenic vein

d) Ascending lumbar veins

e) Left renal vein

106. Bleeding from the cystic artery during cholecystectomy can be controlled by
digital pressure on:

a) The fundus of the gall bladder

b) The region of Hartmann's pouch

c) The anterior wall of the foramen of Winslow

d) The lesser curvature of the stomach

e) The celiac artery

107. Which of the following structures forms the floor (inferior border) of the
inguinal canal:

a) Conjoint tendon

b) Inguinal ligament

c) Transversalis fascia

d) Peritoneum

702 ‫صفحة‬ Dr. MOHAMED YAHIA 0900987639


QUESTION BANK OF Dr. MOHAMED YAHIA

e) Aponeurosis of internal abdominal oblique

108. Which of the following groups of structures receives its arterial blood
supply from the superior mesenteric artery?

a) Ascending colon, transverse colon, descending colon, sigmoid colon

b) Left colic flexure, descending colon, ascending colon, rectum

c) Left colic flexure, descending colon, spleen, sigmoid colon

d) Jejunum, ileum, cecum, ascending colon

e) Descending colon, left colic flexure, rectum, sigmoid colon

109. Which of the following statements regarding circulation to abdominal


organs is false?

a) The common hepatic artery gives rise to the gastroduodenal artery and

propehepatic artery

b) Blood from the kidneys returns to the heart via the hepatic portal system

c) The superior mesenteric vein drains the ileum and jejunum

d) The superior portion of the rectum receives its blood from the inferior

mesenteric artery

e) The duodenum receives its arterial blood via the celiac trunk and the

superior mesenteric artery

110. Which of the following statements regarding the abdomen is false?

a) The falciform ligament is a dorsal mesentery

b) The bile duct enters the duodenum through the major duodenal papilla

c) The gastric arteries run parallel to the lesser curvature of the stomach

d) The epiploic foramen (of Winslow) is the opening into the lesser sac

e) The hepatic veins drain into the inferior vena cava

703 ‫صفحة‬ Dr. MOHAMED YAHIA 0900987639


QUESTION BANK OF Dr. MOHAMED YAHIA

111. Which of the following combinations is not a postal-systemic (portocaval)


anastomosis?

a) Left gastric veins and esophageal veins

b) Superior, middle and inferior rectal veins

c) Superior and inferior epigastric veins

d) Para-umbilical veins and epigastric veins

e) Colic veins and retroperitoneal veins

112. Which of the following statements regarding the posterior abdominal wall is
false?

a) The iliopsoas muscle flexes the hip

b) The quadratus lumborum muscle receives somatic motor innervation via the

lateral femoral cutaneous nerve

c) The medial and lateral arcuate ligaments serves as attachment sites to the

thoracic diaphragm

d) Lumbar arteries arising from the abdominal aorta, supply blood to the

posterior abdominal wall

e) The cisterna chyli is an enlargement at the inferior end of the thoracic duct

113. Which of the following statements regarding the kidney is false?

a) The superior (proximal), expanded end of the ureter is called the renal

pelvis

b) Renal arteries arise directly from the abdominal aorta

c) The right gonadal vein drains into the right renal vein

d) The suprarenal glands receive their blood supply from the inferior, middle,

and superior suprarenal arteries

e) The renal pyramids collect urine and empty it into the minor calyces via the

704 ‫صفحة‬ Dr. MOHAMED YAHIA 0900987639


QUESTION BANK OF Dr. MOHAMED YAHIA

renal papillae

114. Inferior to the arcuate line, which of the following structures lies posterior
to the rectus muscle?

a) Aponeurosis of external oblique

b) Deep inguinal ring

c) Aponeurosis of internal oblique

d) Internal oblique fascia

e) Transversalis fascia

115. Which of the following statements regarding the autonomic innervation of


abdominal organs is false?

a) Autonomic axons carried in the vagus nerve synapse in ganglia in the celiac

plexus

b) The superior hypogastric plexus is composed of autonomic nerves and

ganglia

c) Preganglionic cell bodies for axons comprising the least splanchnic nerves

lie in the spinal cord segment T12

d) The lumbar splanchnic nerves carry preganglionic sympathetic axons

e) The pelvic splanchnic nerves provide preganglionic parasympathetic axons

to the digestive tract beyond (distal to) the left colic flexure

116. Caput medusae signify:

a) Cirrhosis

b) Portal hypertension

c) Essential hypertension

d) Umbilical hernia

e) Epigastric hernia

705 ‫صفحة‬ Dr. MOHAMED YAHIA 0900987639


QUESTION BANK OF Dr. MOHAMED YAHIA

117. The ureter crosses all except:

a) Vas deferens

b) Uterine artery

c) Inferior mesenteric artery

d) Pelvic brim

e) Sacro-iliac joint

118. Which is true regarding the kidneys:

a) The anterior surface of the right kidney includes a colic area

b) The anterior surface of the left kidney does not include a colic area

c) The suprarenal, pancreatic and colic areas of the left kidney are covered

with peritoneum

d) The perirenal fascia is a synonym for renal fascia

e) The inferior poles may extend to the level of the first sacral vertebra

119. Which is false regarding the appendix:

a) In the majority of cases hangs down below the caecum

b) Receives arterial blood from the anterior caecal artery

c) May be over 20 cm long

d) Has a wider lumen in infants

e) Has a base which is said to lie at the point where the three taenia coli meet

120. There are a number of features which allow distinction between the large
and small intestine, including:

706 ‫صفحة‬ Dr. MOHAMED YAHIA 0900987639


QUESTION BANK OF Dr. MOHAMED YAHIA

a) The taenia coli of the large intestine, which are aggregates of circular

smooth muscle

b) The appendices epiploicae of the large intestine, which have no blood

supply

c) The haustrations of the large intestine, which are caused by contractions of

both layers of intramural smooth muscle

d) Peyer's patches will occur in both small & large intestines

e) Brunner's glands which are present in the duodenum

121. Which is incorrect regarding the pancreas:

a) Its venous drainage is via the splenic vein

b) The congenital malformation known as annular pancreas arises because of

malrotation of the ventral bud of the pancrease

c) It is supplied by the arterial blood via the celiac, superior mesenteric and

splenic arteries

d) The tip of the tail lies in the gastrosplenic ligament

e) The bile duct is sometimes embedded within its substance

122. Which is false about the superior mesenteric artery:

a) It arises about the level of L1

b) It has no branches which supply the large intestine

c) It does not supply the jejunum by its first branch

d) It emerges from the substance of the pancreas

e) It supplies the transverse colon by its last branch

123. The duodenum:

a) Has the common bile duct passing posterior to its first part

707 ‫صفحة‬ Dr. MOHAMED YAHIA 0900987639


QUESTION BANK OF Dr. MOHAMED YAHIA

b) Has posterior relationships in its third part with the inferior vena cava,

abdominal aorta and the common bile duct

c) Receives artrial blood from the superior pancreaticoduodenal, inferior

pancreaticoduodenal and inferior mesenteric artery

d) Has an interior that is thrown into folds throughout the length of the

duodenum

e) Is entirely retroperitoneal

124. A femoral hernia descends through the femoral canal, and the neck of the
hernial sac lies:

a) At the saphenous opening

b) Above and medial to the pubic tubercle

c) Below and lateral to the pubic tubercle

d) In the obturator canal

e) Lateral to the iliacus muscle

LOWER LIMB

1. Which of the following nerves innervates at least one muscle that acts on both
the hip and knee joints?

a) Ilioinguinal nerve

b) Femoral nerve

c) Saphenous nerve

d) Common peroneal nerve

e) Superficial peroneal nerve

708 ‫صفحة‬ Dr. MOHAMED YAHIA 0900987639


QUESTION BANK OF Dr. MOHAMED YAHIA

2. In walking, the hip bone of the suspended leg is raised by which of the
following muscles acting on the supported side of the body?

a) Gluteus maximus

b) Obturator internus

c) Gluteus medius

d) Obturator externus

e) Quadratus femoris

3. Which of the following muscles is a flexor of the thigh?

a) Superior gamellus

b) Adductor longus

c) Gracilis

d) Psoas

e) Obturator internus

4. Which of the following muscles dorsiflexes the foot at the ankle joint?

a) Peroneous longus

b) Extensor digitorum brevis

c) Tibialis posterior

d) Extensor hallucis brevis

e) Tibialis anterior

5. The following muscles invert the foot except the:

a) Tibialis anterior

b) Extensor hallucis longus

c) Extensor digitorum longus

709 ‫صفحة‬ Dr. MOHAMED YAHIA 0900987639


QUESTION BANK OF Dr. MOHAMED YAHIA

d) Peroneus tertius

e) Tibialis posterior

6. The following facts concerning the dorsalis pedis artery are correct except:

a) It is a continuation of the anterior tibial artery

b) It enters the sole of the foot by passing between the two heads of the first

dorsal interosseous muscle

c) It can be palpated on the dorsum of the foot between the tendons of tibialis

anterior and extensor hallucis longus muscle

d) It joins the lateral planter artery

e) On its lateral side lies the terminal part of the deep peroneal nerve

7. The following structures contribute to the boundaries of the popliteal fossa


except the:

a) Semimembranosus muscle

b) Plantaris

c) Biceps femoris

d) Medial head of the gastrocnemius muscle

e) Soleus

8. The following structures pass through the greater sciatic foramen except the:

a) Superior gluteal artery

b) Sciatic nerve

c) Obturator internus tendon

710 ‫صفحة‬ Dr. MOHAMED YAHIA 0900987639


QUESTION BANK OF Dr. MOHAMED YAHIA

d) Pudendal nerve

e) Inferior gluteal vein

9. The femoral ring is bounded by the following structures except the:

a) Femoral vein

b) Lacunar ligament

c) Superior ramus of the pubis

d) Femoral artery

e) Inguinal ligament

10. A femoral hernia descends through the femoral canal, and the NECK of the
hernial sac lies:

a) At the saphenous opening

b) Above and medial to the pubic tubercle

c) Below and lateral to the pubic tubercle

d) In the obturator canal

e) Lateral to the iliacus muscle

11. The following structures pass through the subsartorial canal except the:

a) Posterior division of the obturator nerve

b) Saphenous nerve

c) Femoral artery

d) Nerve to the vastus intermedius

e) Femoral vein

12. The floor of the femoral triangle is formed by the following muscles except
the:

a) Pectineus

711 ‫صفحة‬ Dr. MOHAMED YAHIA 0900987639


QUESTION BANK OF Dr. MOHAMED YAHIA

b) Adductor longus

c) Iliacus

d) Psoas

e) Adductor brevis

13. The peroneal artery is a branch of which artery?

a) Anterior tibial artery

b) Popliteal artery

c) Posterior tibial artery

d) Arcuate artery

e) Lateral planter artery

14. Which statement is NOT true of the ankle joint?

a) It is strengthened by the deltoid (medial collateral ligament)

b) It is a hinge joint

c) It is formed by the articulation of the talus and the distal ends of the tibia

and fibula

d) It is most stable in the fully plantar-flexed position

e) It is a synovial joint

15. "Unlocking" of the knee joint to permit flexion is caused by the action of
which muscle?

a) Vastus medialis

b) Articularis genu

712 ‫صفحة‬ Dr. MOHAMED YAHIA 0900987639


QUESTION BANK OF Dr. MOHAMED YAHIA

c) Gastrocnemius

d) Biceps femoris

e) Popliteus

16. In the adult, the chief arterial supply to the head of the femur is from the:

a) Superficial circumflex iliac artery

b) Obturator artery

c) Branches from the medial and lateral circumflex femoral arteries

d) Deep external pudendal artery

e) Inferior gluteal artery

17. The lymph drainage of the skin covering the ball of the big toe is into the:

a) Vertical group of superficial inguinal nodes

b) Popliteal nodes

c) Horizontal group superficial inguinal nodes

d) Axillary nodes

e) Internal iliac nodes

18. The lymph drainage of the skin covering the medial side of the knee joint is
into the:

a) Popliteal nodes

b) Internal iliac nodes

c) Vertical group of superficial inguinal nodes

d) Horizontal group of superficial inguinal nodes

e) Obturator nodes

713 ‫صفحة‬ Dr. MOHAMED YAHIA 0900987639


QUESTION BANK OF Dr. MOHAMED YAHIA

19. The lymph drainage of the skin of the buttock is into the:

a) Axillary nodes

b) Superior gluteal nodes

c) Vertical group of superficial inguinal nodes

d) Horizontal group of superficial inguinal nodes

e) Internal iliac nodes

20. The lymph drainage of the skin of the calf is into:

a) Vertical group of superficial inguinal nodes

b) Internal iliac nodes

c) Horizontal group of superficial inguina nodes

d) Popliteal nodes

e) Obturator nodes

21. Hyperextension of the hip joint is prevented by:

a) Obturator internus tendon

b) Ischiofemoral ligament

c) Tensor fascia lata muscle

d) Iliotibial tract

e) Ligamentum teres

22. If the dorsalis pedis artery is severed just proximal to its medial and lateral
tarsal branches, blood can still reach the dorsum of the foot through which of the
following vessels?

714 ‫صفحة‬ Dr. MOHAMED YAHIA 0900987639


QUESTION BANK OF Dr. MOHAMED YAHIA

a) The peroneal artery

b) The posterior tibial artery

c) The medial planter artery

d) The lateral planter artery

e) All of the above

23. In children, the chief femoral arterial supply to the head of the femur is
derived from:

a) The obturator artery

b) The internal pudendal artery

c) Branches from the medial and lateral circumflex humeral arteries

d) The deep circumflex iliac artery

e) The superficial circumflex iliac artery

24. The femoral ring is:

a) The opening in the deep fascia of the thigh for the great saphenous vein

b) The opening in the adductor magnus muscle for the femoral artery

c) The proximal opening in the femoral canal

d) The compartment in the femoral sheath for the femoral artery

e) The compartment in the femoral sheath for the femoral nerve

25. After a football injury, an orthopedic surgeon noted that the right tibia could
be moved anteriorly with excessive freedom when the knee was flexed. In this
patient, which ligament is most likely to be torn?

715 ‫صفحة‬ Dr. MOHAMED YAHIA 0900987639


QUESTION BANK OF Dr. MOHAMED YAHIA

a) The lateral collateral ligament

b) The posterior cruciate ligament

c) The anterior cruciate ligament

d) The medial collateral ligament

e) The patellar ligament

26. A femoral hernia has the following characteristics except:

a) It is more common in women than in men

b) The swelling occurs below and lateral to the pubic tubercle

c) It descends through the femoral canal

d) The neck is related immediately laterally to the femoral artery

e) The neck is related medially to the sharp edge of the lacunar ligament

27. The gastrocnemias and the soleus muscles have all the following features in
common except:

a) They are supplied by the tibial nerve

b) They are found in the posterior compartment of the leg

c) They arise from the femoral condyles and flex the knee joint

d) They insert via the tendo calaneus

e) They planter flex the ankle joint

28. All the following statements about the sartorius are correct except:

a) It flexes the leg at the knee joint

b) It flexes the thigh at the hip joint

c) It laterally rotates the thigh at the hip joint

d) It adducts the thigh at the hip joint

e) It attaches to the anterior superior iliac spine

716 ‫صفحة‬ Dr. MOHAMED YAHIA 0900987639


QUESTION BANK OF Dr. MOHAMED YAHIA

29. To lift the left foot off the ground while walking, which of the following
muscles play an important role?

a) The left gluteus medius muscle

b) The left gluteus maximus muscle

c) The right adductor longus muscle

d) The right gluteus medius muscle

e) None of the above

30. Rupture of the tendo calcaneus results in the inability to:

a) Dorsiflex the foot

b) Evert the foot

c) Invert the foot

d) Planterflex the foot

e) None of the above

31. A sprained ankle resulting from excessive eversion most likely demonstrates
a torn:

a) Talofibular ligament

b) Tendo calcaneus

c) Deltoid ligament

d) Interosseous ligament

e) Peroneal retinaculum

717 ‫صفحة‬ Dr. MOHAMED YAHIA 0900987639


QUESTION BANK OF Dr. MOHAMED YAHIA

32. If the foot is permanently dorsiflexed and everted, which nerve might be
injured?

a) The deep peroneal nerve

b) The superficial peroneal nerve

c) The common peroneal nerve

d) The tibial nerve

e) The obturator nerve

33. The femoral nerve arises from the following segments of the spinal cord:

a) L2-3

b) L4, 5; S1, 2 and 3

c) L2, 3 and 4

d) L1 and 2

e) L5; S1, 2 and 3

34. The dermantome present over the lateral side of the foot is:

a) S5

b) L3

c) S1

d) L4

e) L5

35. The following structures are transmitted through the lesser sciatic foramen
except:

a) The tendon of the obtarator internus muscle

b) The internal pudendal vessels

c) The nerve to the obtarator internus muscle

d) The pudendal nerve

718 ‫صفحة‬ Dr. MOHAMED YAHIA 0900987639


QUESTION BANK OF Dr. MOHAMED YAHIA

e) The inferior gluteal nerve

36. The femoral sheath is formed by which of the following layer(s) of fascia?

a) The pectineus fascia

b) The fascia iliaca and the fascia transversalis

c) The fascia lata and the membranous layer of the superficial fascia

d) The psoas fascia and the fatty layer of superficial fascia

e) The processus vaginalis

37. The following facts regarding the great saphenous vein are true except:

a) It arises on the dorsum of the foot

b) It enters the leg by passing anterior to the medial malleolus

c) It drains into the femoral vein approximately 1.5 inch (3.8 cm) below and

lateral to the pubic tubercle

d) It is accompanied by the saphenous nerve

e) It has no communication with the deep veins of the leg

38. Which of the following muscles everts the foot?

a) The tibialis posterior muscle

b) The flexor hallucis longus muscle

c) The peroneus longus muscle

d) The tibialis anterior muscle

719 ‫صفحة‬ Dr. MOHAMED YAHIA 0900987639


QUESTION BANK OF Dr. MOHAMED YAHIA

e) The flexor digitorum longus muscle

39. The lateral meniscus of the knee joint:

a) Has a thick inner border

b) Is strongly attached around its circumference to the tibia

c) Is more frequently torn than the medial meniscus

d) Is strongly attached to the lateral collateral ligament

e) Is attached by its anterior horn to the tibia in front of the intercondylar

eminence

40. A patient in the supine position with the hip and knee joints extended is
asked to abduct the lower limb against resistance provided by the physician. This
exercise tests which of the following muscles?

a) The semitendinosus muschle

b) The gluteus medius muscle

c) The pectineus muscle

d) The gracilis muscle

e) The semimembranosus muscle

41. The following facts regarding the obturator nerve are true except:

a) It originates from the lumbar plexus

b) It enters the thigh immediately beneath the inguinal ligament

c) It innervates the adductor muscles of the thigh

d) It divides into an anterior and posterior division

e) It supplies the skin on the medial side of the thigh

42. Injury to the common peroneal nerve results in an:

a) Inability to invert the foot

720 ‫صفحة‬ Dr. MOHAMED YAHIA 0900987639


QUESTION BANK OF Dr. MOHAMED YAHIA

b) Inability to planterflex the ankle

c) Inability to evert the foot

d) Inability to planterflex the big toe

e) Inability to feel skin sensation on the medial side of the leg

43. After a lesion of the tibial part of the sciatic nerve, some active flexion may
still be possible at the knee joint. The muscles responsible for this remaining
flexion include:

a) The short head of the biceps femoris muscle (because supplied by


common peroeal nerve)

b) The gastrocnemius muscle

c) The plantaris muscle ( wrong answer)

d) The popliteus muscle

e) The long head of the biceps femoris muscle

44. Which of the following muscles functions as an internal rotator of the hip:

a) Iliopsoas

b) Sartorius

c) Gluteus medius

d) Obturator externus

e) Gluteus maximus

45. The first metatarsal bone forms:

a) Medial longitudinal arch only

b) Medial and lateral longitudinal arches only

721 ‫صفحة‬ Dr. MOHAMED YAHIA 0900987639


QUESTION BANK OF Dr. MOHAMED YAHIA

c) Transverse arch only

d) Medial longitudinal and transverse arches

e) Lateral longitudinal and transverse arches

46. Calcaneus forms:

a) Medial longitudinal arch only

b) Medial and lateral longitudinal arches only

c) Transverse arch only

d) Medial longitudinal and transverse arches

e) Lateral longitudinal and transverse arches

47. Which of the following does not insert into the planter aspect of the foot?

a) Flexor digitorum longus

b) Peroneus tertias

c) Peroneus longus

d) Tibialis posterior

e) Flexor hallucis longus

48. Which of the following muscles does not arise from the calcaneus?

a) Flexor digitorum brevis

b) Extensor digitorum brevis

c) Flexor accessorius

d) Flexor hallucis brevis

e) Abductor hallucis

722 ‫صفحة‬ Dr. MOHAMED YAHIA 0900987639


QUESTION BANK OF Dr. MOHAMED YAHIA

49. The movement of eversion of the foot takes place at the following joints
except:

a) Subtalar joints

b) Ankle joints

c) Talocalcaneonavicular joint (false)

d) Calcaneocuboid joint

e) Transverse tarsal joints

50. The Great Saphenous Vein:

a) Lies anterior to the medial malleolus

b) Lies posterior to the central axis of the knee joint

c) Has communication with the short saphenous vein

d) Pierces the femoral sheath

e) All of the above

51. Which of the following nerve/vessel pairs are INCORRECT:

a) Sural nerve/ short saphenous vein

b) Superficial peroneal nerve/ peroneal artery

c) Tibial nerve/ posterior tibial artery

d) Deep peroneal nerve/ anterior tibial artery

e) Tibial nerve/ popliteal artery

52. Muscles attached to the ischial tuberosity include all except:

a) Inferior gamellus

723 ‫صفحة‬ Dr. MOHAMED YAHIA 0900987639


QUESTION BANK OF Dr. MOHAMED YAHIA

b) Adductor magnus

c) Long head of biceps femoris muscle

d) Gluteus medius

e) Semitendinousus

53. Select the incorrect statement about the gluteus maximus:

a) Gets its entire motor innervation from the inferior gluteal nerve

b) Is an important hip stabilizer in walking

c) Produces hip extension

d) Inserts primarily into the iliotibial tract

e) Takes a portion of its origin from the sacrotuberous ligament

54. Flexion of the knee can be produced by all of the following except:

a) Adductor magnus

b) Short head of biceps femoris

c) Gastrocnemius

d) Sartorius

e) Gracilis

55. The cruciate anastomosis of the hip includes all the following arteries except:

a) Medial femoral circumflex

b) Lateral femoral circumflex

c) Inferior gluteal

d) First perforating artery

e) Obturator artery

56. The foot receives cutaneous innervation from:

724 ‫صفحة‬ Dr. MOHAMED YAHIA 0900987639


QUESTION BANK OF Dr. MOHAMED YAHIA

a) Sural nerve

b) Medial planter nerve

c) Saphenous nerve

d) Deep peroneal nerve

e) All of the above

57. The fascia lata is an insertion for the following muscles:

a) Gluteus minimus

b) Tensor fascia lata

c) Both A and B

d) Neither A nor B

58. Muscles of the deep posterior compartment of the leg produce which of the
following?

a) Planter flexion

b) Inversion

c) Both A and B

d) Neither A nor B

59. Short head of biceps femoris is innervated by which of the following nerves:

a) Superficial peroneal nerve

b) Deep peroneal nerve

c) Both A and B

d) Neither A nor B

60. Tibialis posterior:

a) Takes its origin from the tibia

725 ‫صفحة‬ Dr. MOHAMED YAHIA 0900987639


QUESTION BANK OF Dr. MOHAMED YAHIA

b) Takes its origin from the fibula

c) Both A and B

d) Neither A nor B

61. The interossei of the foot

a) Are all involved in abduction of toes

b) Originate from adjacent side of proximal phalanges 2-4

c) Both A and B

d) Neither A nor B

62. Adductor magnus is innervated by which of the following?

a) Femoral nerve

b) Obturator nerve

c) Deep peroneal nerve

d) Lateral plantar nerve

e) None of the above

63. Small saphenous vein travels with which of the following nerves?

a) Sural nerve

b) Deep peroneal nerve

c) Saphenous nerve

d) Tibial nerve

726 ‫صفحة‬ Dr. MOHAMED YAHIA 0900987639


QUESTION BANK OF Dr. MOHAMED YAHIA

64. Posterior tibial artery travels with which of the following nerves?

a) Sural nerve

b) Deep peroneal nerve

c) Saphenous nerve

d) Tibial nerve

e) Superior gluteal nerve

65. Which of the following is involved in arterial anastomosis around the hip
joint?

a) Internal pudendal artery

b) Medial femoral circumflex artery

c) Both A and B

d) Neither A nor B

66. Which of the following muscles produce knee flexion?

a) Gracilis

b) Soleus

c) Both A and B

d) Neither A nor B

67. "Foot drop" can be caused by trauma to:

a) Common peroneal nerves

b) Femoral nerve

c) Both A and B

d) Neither A nor B

727 ‫صفحة‬ Dr. MOHAMED YAHIA 0900987639


QUESTION BANK OF Dr. MOHAMED YAHIA

68. The femoral sheath:

a) Contains 3 compartments: each for the femoral artery, vein and nerve

b) Saphenous opening on anterior surface for the great saphenous vein

c) Both A and B

d) Neither A nor B

69. Which is/are not innervated by lateral plantar nerves?

a) Adductor hallucis

b) Abductor digiti minimi

c) Flexor digitorum brevis

d) Fourth lumbrical

e) All of the above

70. Which of the following is a branch of the posterior tibial artery?

a) Dorsalis pedis artery

b) Peroneal artery

c) Both A and B

d) Neither A nor B

71. Structures traversing the lesser sciatic foramen include all of the following
except:

a) Pudendal nerve

b) Inferior gluteal nerve

c) Tendon of obturator internus

d) Internal pudendal artery

e) Internal pudendal vein

728 ‫صفحة‬ Dr. MOHAMED YAHIA 0900987639


QUESTION BANK OF Dr. MOHAMED YAHIA

72. Structures located in the subsartorial (adductor) canal include:

a) Saphenous nerve

b) Popliteal vein

c) Great saphenous vein

d) Profunda femoris artery

e) All of the above

73. The great sciatic foramen transmits all of the following except:

a) Superior gluteal artery

b) Inferior gluteal artery

c) Internal pudendal artery

d) External pudendal artery

74. The femoral triangle:

a) Has a portion of vastus lateralis in its lateral floor

b) Contains lymph nodes which drain skin of 7th thoracic dermatome

c) Has adductor longus muscle as its base

d) Contains femoral nerve

e) All of the above

75. Which muscle(s) is capable of producing flexion at one joint and extension at
a second joint?

a) Semimembranosus

b) Rectus femoris

c) Lumbricals of foot

d) Long head of biceps femoris

729 ‫صفحة‬ Dr. MOHAMED YAHIA 0900987639


QUESTION BANK OF Dr. MOHAMED YAHIA

e) All of the above

76. Sacrospinous ligament traverses the:

a) Greater sciatic foramen

b) Lesser sciatic foramen

c) Both A and B

d) Neither A nor B

77. Which can be found in the lateral compartment of the leg?

a) Peroneus brevis

b) Peroneus tertias

c) Both A and B

d) Neither A nor B

e) None of the above

78. Which of the following is not normally a branch of the profunda femoris
artery?

a) Obturator

b) Medial femoral circumflex

c) Lateral femoral circumflex

d) Its perforating artery

e) 4th perforating artery

79. The cruciate anastomosis:

a) Receives one of its major contributions from the obturator artery

b) Receives blood from the 1st perforating artery

c) Both A and B

d) Neither A nor B

730 ‫صفحة‬ Dr. MOHAMED YAHIA 0900987639


QUESTION BANK OF Dr. MOHAMED YAHIA

80. Position of the popliteal artery relative to the popliteal veins on the popliteal
surface of the femur is:

a) Medial

b) Lateral

c) Posterior

d) Anterior

e) Supeior

81. Position of the common peroneal nerve relative to the tendon of the
semitendinosus in the popliteal fossa:

a) Medial

b) Lateral

c) Posterior

d) Anterior

e) Superior

82. Obturator externus is innervated by which of the following nerves?

a) Ilioinguinal nerve

b) Femoral nerve

c) Obturator nerve

d) Lateral femoral nerve

e) Genitofemoral nerve

83. Soleus is:

a) Attached to the calcaneus

b) Attached to the proximal phalanx of medial toe

c) Attached to 5th metatarsal

731 ‫صفحة‬ Dr. MOHAMED YAHIA 0900987639


QUESTION BANK OF Dr. MOHAMED YAHIA

d) Both A and B

e) None of the above

84. The fascia lata is an insertion for the following muscle(s)?

a) Gluteus maximus

b) Gluteus medius

c) Both A and B

d) Neither A nor B

85. Adductor magnus muscle:

a) Derives blood supply from branches of profunda femoral artery

b) Gets nerve supply from femoral and obturator nerves

c) Inserts on the medial condyle of the tibia

d) Has partial origin from the ischial spine

e) All of the above

86. The femoral artery:

a) Has a common sheath with the femoral nerve

b) Begins midway between anterior superior iliac spine & pubic symphisis

c) Lies medial to the femoral vein

d) Has no superficial branches

e) Its profunda branch supplies the calf muscles

87. Select the incorrect statement about the femoral artery:

a) Lies on psoas major

b) Is continuation of the external iliac artery

c) Passes beneath the inguinal ligament

732 ‫صفحة‬ Dr. MOHAMED YAHIA 0900987639


QUESTION BANK OF Dr. MOHAMED YAHIA

d) Is used in cannulation

e) Lies medial to the femoral canal

88. Which of the following statements is incorrect regarding the adductor


magnus muscle?

a) Supplied by tibial part of sciatic nerve

b) Inserted into adductor tubercle

c) Supplied by anterior division of obturator nerve

d) Femoral nerves pass through a gap in the muscle

89. Select the correct statement about the adductor magnus:

a) Arises from the posterior inferior iliac spine

b) Inserts entirely into the liner aspera

c) Flexor of the knee joint

d) Has a double nerve supply

90. Which of the following muscles is attached to both tibia and fibula:

a) Tibialis posterior

b) Popliteus

c) Flexor pollicis longus

d) Both B and C

e) None of the above

91. Select the false statement regarding deep muscles of the calf:

a) Supplied by deep peroneal nerve

b) They flex the toes

733 ‫صفحة‬ Dr. MOHAMED YAHIA 0900987639


QUESTION BANK OF Dr. MOHAMED YAHIA

c) Are the main planterflexors of the ankle joint

d) Maintain the medial longitudinal arch of the foot

92. The short muscles of the first layer of the sole are:

a) Flexor digitorum brevis

b) Abductor hallucis

c) Abductor digit minimi

d) All of the above

e) None of the above

93. The muscles of the first layer of the sole are innervated by:

a) Medial planter nerve

b) Lateral planter nerve

c) Both A and B

d) Neither A nor B

94. Which of the following muscles is innervated by the tibial nerve?

a) Popliteus

b) Semimembranosus

c) Both A and B

d) Neither A nor B

95. The medial longitudinal arch of the foot involves the:

a) Spring ligament

b) Navicular bone

734 ‫صفحة‬ Dr. MOHAMED YAHIA 0900987639


QUESTION BANK OF Dr. MOHAMED YAHIA

c) Cuboid bone

d) Both A and B

e) All of the above

96. Which of the following muscles inserts on the fibula?

a) Flexor hallucis longus

b) Biceps femoris

c) Both A and B

d) Neither A nor B

97. All of the following lie within the femoral triangle except:

a) Femoral nerve

b) Profunda femoris artery

c) Obturator nerve

d) Long saphenous vein

e) Pectineus muscle

98. The small saphenous vein drains predominantly to:

a) The femoral vein

b) The tibial vein

c) The popliteal vein

d) The greater saphenous vein

e) The dorsal venous arch of the foot

735 ‫صفحة‬ Dr. MOHAMED YAHIA 0900987639


QUESTION BANK OF Dr. MOHAMED YAHIA

99. The only artery to enter the lesser sciatic foramen is the:

a) Sciatic artery

b) Inferior gluteal artery

c) Obturator artery

d) Internal pudendal artery

e) Deep femoral artery

100. Which of the following muscles does NOT receive its blood supply from
branches of the internal iliac artery:

a) Gluteus maximus

b) Superior gamellus

c) Vastus lateralis

d) Iliacus

e) Piriformis

101. The principal blood supply to the posterior compartment of the thigh comes
directly from:

a) The femoral artery

b) Perforating branches from the profunda artery

c) The obturator artery

d) The popliteal artery

e) The tibial artery

102. The sural nerve is a branch of the:

a) Femoral nerve

b) Tibial nerve

c) Common peroneal nerve

736 ‫صفحة‬ Dr. MOHAMED YAHIA 0900987639


QUESTION BANK OF Dr. MOHAMED YAHIA

d) Deep peroneal nerve

e) Obturator nerve

103. The femoral sheath is formed by which of the following layer(s) of fascia?

a) The pectineus fascia

b) The fascia lata and fascia transversalis

c) The fascia lata and the membranous layer of the superficial fascia

d) The psoas fascia and the fascia transversalis

e) The process vaginalis

104. Which of the following statements about the femoral nerve is false?

a) It is formed by dorsal division fibers of the lumbar plexus

b) It innervates the knee extensors

c) It lies medial to the femoral artery in the femoral triangle

d) It supplies sensory innervation to the medial side of the leg and foot via the
saphenous nerve

e) It innervates the sartorius muscle

105. The femoral artery passes from the anterior side of the thigh to the back of
the knee where it becomes the popliteal artery. It passes through an opening:

a) Between the long and short head of biceps femoris

b) Between abductor longus and adductor brevis

c) Between two parts of adductor magnus

d) Between semitendinosus and semimembranosus

737 ‫صفحة‬ Dr. MOHAMED YAHIA 0900987639


QUESTION BANK OF Dr. MOHAMED YAHIA

e) Between vastus medialis and lateralis

106. Blood supply to gluteal muscles comes primarily from:

a) The femoral artery

b) The obturator artery

c) The profundal femoris (deep femoral) artery

d) The popliteal artery

e) Branches of the internal iliac artery

107. Which of the following muscles does not cross the knee joint?

a) Semimembranosus

b) Semitendinosus

c) Gracilis

d) Sartorius

e) Adductor magnus

108. Which of the following is a symptom of a torn medial collateral ligament of


the knee?

a) The leg can be abducted

b) The leg can be adducted

c) Anterior drawer

d) Posterior drawer

e) Inability to flex the knee joint

109. The saphenous nerve is a branch of the:

a) Femoral nerve

b) Tibial nerve

738 ‫صفحة‬ Dr. MOHAMED YAHIA 0900987639


QUESTION BANK OF Dr. MOHAMED YAHIA

c) Common peroneal nerve (surl nerve)

d) Deep peroneal nerve

e) Obturator nerve

110. Which muscle(s) is/are capable of producing flexion at one joint and
extension at a second joint?

a) Semimembranosus

b) Rectus femoris

c) Vastus medialis

d) Both A and B

e) Both B and C

111. Nail bed of the middle toe is innervated by:

a) Branches of femoral nerve

b) Branches of tibial nerve

c) Branches of deep peroneal nerve

d) Branches of superficial peroneal nerve

112. Posterior femoral cutaneous nerve tranverses the:

a) Greater sciatic foramen

b) Lesser sciatic foramen

c) Both A and B

d) Neither A nor B

113. Sartorius produces which of the following actions:

a) Medial rotation

b) Adduction

739 ‫صفحة‬ Dr. MOHAMED YAHIA 0900987639


QUESTION BANK OF Dr. MOHAMED YAHIA

c) Both A and B

d) Neither A nor B

114. Which of the following would not be a good example for demonstration of
the ligamentous action (passive insufficiency) of muscles?

a) Semimembranosus

b) Semitendinosus

c) Soleus

d) Rectus femoris

e) Long head of biceps femoris

115. Anterior tibial artery:

a) Becomes dorsalis pedis artery in the foot

b) Lies between tibialis anterior and extensor hallucis longus tendons at the

ankle joint

c) Both A and B

d) Neither A nor B

116. Hip extension could be produced by:

a) Semimembranosus

b) Gluteus maximus

c) Both A and B

d) Neither A nor B

117. Structures which cross the ankle joint posterior to the medial malleolus
include all of the following except:

a) Tendon of flexor hallucis longus

b) Tendon of flexor digitorum longus

740 ‫صفحة‬ Dr. MOHAMED YAHIA 0900987639


QUESTION BANK OF Dr. MOHAMED YAHIA

c) Tendon of flexor digitorum brevis

d) Tibial nerve

e) Posterior tibial artery

118. Obturator externus is paralyzed by injury of which of the following nerves:

a) Nerve to obturator

b) Femoral nerve

c) Obturator nerve

d) Lateral femoral cutaneous nerve

e) Genitofemoral nerve

119. The close association of the insertion of sartorius, gracilis, and


semitendinousus muscles resembles the pes anserina (goose's foot). These
features would be found on the:

a) Lateral tibial condyle

b) Medial proximal shaft of the tibia

c) Lateral femoral condyle

d) Medial femoral condyle

e) Tibial tuberosity

120. A 25 year old sports man presented with (shin splints) and pain over the
medial calf. What sensory nerve innervates this dermatome?

a) The tibial nerve

b) The sural nerve

c) The deep peroneal nerve

d) The saphenous nerve

e) The superficial peroneal nerve

741 ‫صفحة‬ Dr. MOHAMED YAHIA 0900987639


QUESTION BANK OF Dr. MOHAMED YAHIA

121. A 55 year old lady had a fall at home. She presented to the casualty with a
shortened, externally rotated leg. She was told after taking x-rays that she had
subtrochanteric fracture of the proximal femur. Which of the following is correct
regarding the position of the proximal fragment:

a) It will be abducted and extended

b) It will be abducted and flexed

c) It will be adducted and extended

d) It will be adducted and internally rotated

122. Superior gluteal nerve:

a) Innervates gluteus medius

b) Lies in the interval between gluteus minimus and gluteus medius

c) Leaves the pelvis via the greater sciatic foramen

d) Lacks a cutaneous component

e) All of the above

123. The anterior compartment of the leg:

a) Contains muscles capable to produce inversion and eversio(p.tertiues)

b) Contains muscles all of which insert in the foot

c) Receives its blood as a branch of the popliteal artery

d) Is tightly compressed between the tibia and fibula by deep fascia

e) All of the above

124. All of the following nerves innervate the flexors acting on the knee joint
except :

a) Deep peroneal nerve

b) Femoral nerve

c) Obturator nerve

742 ‫صفحة‬ Dr. MOHAMED YAHIA 0900987639


QUESTION BANK OF Dr. MOHAMED YAHIA

d) Tibial nerve

e) Common peroneal nerve

125. All of the following are innervated by branches of the femoral nerve except:

a) Sartorius

b) Gracilis

c) Rectus femoris

d) Vastus medialis

e) Pectineus

126. Structures within the adductor (subartorial ; Hunter's) canal include all of
the following except:

a) Femoral artery

b) Great saphenous vein

c) Saphenous nerve

d) Nerve to vastus medialis

127. Regarding the lateral planter nerve:

a) Proximal part runs between first and second muscular layers of the foot

b) Distal part runs between second and third muscular layers of the foot

c) Supplies lateral 31/2 toes on the planter aspect

d) Is corresponding to the median nerve in the hand

e) Supplies first and second lumbricals

743 ‫صفحة‬ Dr. MOHAMED YAHIA 0900987639


QUESTION BANK OF Dr. MOHAMED YAHIA

128. In the popliteal fossa:

a) Popliteal artery is the deepest of its contents

b) Upper medial boundary is formed by sartorius and semitendinosus

c) Small saphenous vein pierces its roof to enter the popliteal vein

d) Upper lateral boundary is formed by lateral head of gastrocnemias

e) Plantaris lies in its floor

129. Midinguinal point:

a) Midway between anterior superior iliac spine and pubic tubercle

b) Midway between anterior superior iliac spine and pubic symphysis

c) Midway between anterior superior iliac spine and pubic crest

d) Femoral artery lies two centimeters medial to this point

e) Deep inguinal ring is five centimeters above this point

130. Which of the following ligaments attaches to the navicular and calcaneus?

a) Anterior tibiofibular ligament

b) Short plantar ligament

c) Anterior talofibular ligament

d) Long plantar ligament

e) Spring ligament

131. A blow to the head of the fibula is likely to result in damage to which nerve?

a) Tibial

b) Femoral

c) Saphenous

d) Common peroneal

744 ‫صفحة‬ Dr. MOHAMED YAHIA 0900987639


QUESTION BANK OF Dr. MOHAMED YAHIA

e) Sural

132. Which of the following is not a major weight bearing bone in the lower
limb?

a) The femur

b) The talus

c) The calcaneus

d) The tibia

e) The fibula

133. The femoral artery passes from the anterior compartment of the thigh to the
posterior aspect of the knee through which of the following gaps?

a) Between the long and short head of biceps femoris

b) Between adductor longus and adductor brevis

c) Between two parts of adductor magnus

d) Between semitendinosus and semimembranosus

e) Between vastus medialis and vastus lateralis

134. Which of the following vessel/nerve pairings (course together) is


INCORRECT:

a) Great saphenous vein/ Saphenous nerve

b) Small saphenous vein/ Sural nerve

c) Anterior tibial artery/ Deep peroneal nerve

d) Profunda femoris artery/ Femoral nerve

e) Posterior tibial artery/ Tibial nerve

135. Contents of the femoral triangle include all of the following except:

a) Profunda femoris artery

b) Great saphenous vein

745 ‫صفحة‬ Dr. MOHAMED YAHIA 0900987639


QUESTION BANK OF Dr. MOHAMED YAHIA

c) Femoral canal

d) Femoral sheath

e) Obturator nerve

136. Muscle(s) which could produce inversion include:

a) Tibialis anterior

b) Tibialis posterior

c) Extensor hallucis longus

d) Flexor hallucis longus

e) All of the above

137. Which of the following is/are not attached to the ischial tuberosity:

a) Short head of biceps femoris

b) Adductor magnus

c) Semitendinosus

d) Inferior gemellus

e) Long head of biceps femoris

138. The inguinal lymph nodes do not receive lymph from:

a) The big toe

b) The buttock

c) The anterior abdominal wall

746 ‫صفحة‬ Dr. MOHAMED YAHIA 0900987639


QUESTION BANK OF Dr. MOHAMED YAHIA

d) The scrotum

e) The testes

139. In the sole of the foot:

a) The lumbricals, which arise from the tendons of flexor digitorum brevis, lie

in the second layer of muscles

b) Both flexor digiti minimi brevis and longus are innervated by the lateral

plantar nerve

c) Adductor hallucis has a transverse and oblique head

d) The deepest layer contains only the interossei

e) The tendon of tibialis posterior is inserted into the base of the talus

140. Which is false concerning the extensor compartment of the leg:

a) Include tibialis anterior which arises from the lateral aspect of the
upper

fibula

b) Includes extensor hallucis longus, which is innervated by the deep peroneal

nerve

c) Includes deep peroneal nerve

d) Includes tibialis anterior whose tendon is a prominent landmark on the

medial aspect of the dorsum of the foot

e) Include peroneus tertius which is absent in a number of normal individuals

141. Which is false concerning the hip joint:

a) Receives a large portion of its blood supply from the artery in the ligament

of the head of the femur

b) Is innervated only by the sciatic nerve

747 ‫صفحة‬ Dr. MOHAMED YAHIA 0900987639


QUESTION BANK OF Dr. MOHAMED YAHIA

c) Receives a branch of the femoral nerve via the nerve to the quadratus

femoris

d) Receives blood via the trochanteric anastomosis which is derived from both
gluteal and both circumflex femoral arteries

e) Is a site of direct contact between the femoral vessels and the capsule of the

joint anteriorly

142. A ligament that is important in maintaining the medial portion of the


longitudinal arch is the:

a) Deltoid ligament

b) Calcaneofibula ligament

c) Long plantar ligament

d) Short plantar ligament

e) Plantar calcaneonavicula ligament (spring ligment)

143. Tibial nerve:

a) Lies in the deep posterior compartment of the leg

b) Contains the motor fibers for all the intrinsic plantar muscles of the foot

c) Both A and B

d) Neither A nor B

144. The cruciate anastomosis of the hip joint receives a primary contribution
from all of the following arteries except:

a) Medial femoral circumflex

b) Lateral femoral circumflex

c) Inferior epigastric

d) Inferior gluteal

e) First perforating artery

748 ‫صفحة‬ Dr. MOHAMED YAHIA 0900987639


QUESTION BANK OF Dr. MOHAMED YAHIA

145. Adductor hallucis is innervated by which of the following nerves?

a) Femoral nerve

b) Obturator nerve

c) Deep peroneal nerve

d) Lateral planter nerve

e) None of the above

146. Peroneus brevis is:

a) Attached to the calcaneus

b) Attached to the proximal phalanx of the medial toe

c) Attached to the fifth metatarsal

d) Both A and B

e) None of the above

147. Buttock lymph drainage to:

a) Vertical group of superficial inguinal nodes

b) Popliteal nodes

c) Horizontal group of superficial inguinal nodes

d) Axillary nodes

e) Node of the above

148. Foot drop is a condition in which the patient is unable to dorsiflex the foot at
the ankle; the plantar flexor muscles are unopposed. This condition could be
caused by trauma to:

a) Superficial peroneal nerve

b) Deep peroneal nerve

c) Sural nerve

749 ‫صفحة‬ Dr. MOHAMED YAHIA 0900987639


QUESTION BANK OF Dr. MOHAMED YAHIA

d) Tibial nerve

e) Saphenous nerve

EMBRYOLOGY

1. Which of the following synthesize progesterone?

a) Cytotrophoblast

b) Syncytiotrophoblast

c) Both A and B

d) Neither A or B

2. All occur during the morula stage except:

a) Attachment to the uterine epithelium takes place

b) The location of the cells dictates their fate

c) The outer cells pump fluid creating a fluid-filled blastocele

d) Tight junctions appear between cells at the periphery, isolating the inner

cells from the uterine fluid

3. During the first week of human development:

a) The zona pellicoda is formed

b) The cells of the embryo increase in number but become smaller

c) The cells of the embryo increase in size

d) Implantation is completed

e) None of the above

4. During the second week of human development:

a) The trophoblast differentiates into cytotrophoblast & syncytiotrophoblast

750 ‫صفحة‬ Dr. MOHAMED YAHIA 0900987639


QUESTION BANK OF Dr. MOHAMED YAHIA

b) The utero-placental circulation begins

c) The inner cell mass differentiation into hypoblast and epiblast

d) Primary stem villi appear

e) All of the above

5. As a result of transverse and longitudinal folding of the embryo:

a) The prochordal membrane is moved ventrally and caudally

b) Position of yolk sac becomes incorporated into embryonic body

c) The primitive groove is moved cranially & dorsally

d) Both A and B

e) All of the above

6. Which of the following is/are normally always interposed between fetal and
maternal blood:

a) Extraembronic mesoderm

b) Maternal endothelial cells

c) Syncytiotrophoblast

d) Both A and B

e) All of the above

7. Maternal blood normally comes into contact with:

a) Syncytiotrophoblast

b) Cytotrophoblast

751 ‫صفحة‬ Dr. MOHAMED YAHIA 0900987639


QUESTION BANK OF Dr. MOHAMED YAHIA

c) Fetal blood cells

d) Both A and B

e) All of the above

8. Which of the following is not a function of the yolk sac in humans?

a) Site of origin of primodial germ cells

b) Site of origin of fetal blood cells

c) Provides nourishment/metabolic reserves for the embryo

d) Site of origin of epithelium of the primitive gut

e) Facilitates transfer of nutrients into the early embryo

9. At the time of ovulation the endometrium is in the:

a) Menstral phase

b) Early proliferative phase

c) Late proliferative phase

d) Early secretory phase

e) Late secretory phase

10. Lines in the chorionic after the third developmental week is:

a) Cytotrophoblast

b) Syncytiotrophopblast

c) Both A and B

d) Neither A nor B

11. Always normally situated between maternal and fetal blood:

a) Cytotrophoblast

b) Syncytiotrophoblast

752 ‫صفحة‬ Dr. MOHAMED YAHIA 0900987639


QUESTION BANK OF Dr. MOHAMED YAHIA

c) Both A and B

d) Neither A nor B

12. Bleeding may occur near the 13th day of development:

a) May be confused with normal menstral bleeding

b) Is caused by increased blood flow into lacunar spaces

c) Both A and B

d) Neither A nor B

13. The extraembryonic membrane which contributes the most tissue to the
formation of the placenta is the:

a) Amnion

b) Chorion

c) Yolk sac

d) Connecting (body) stalk

e) Decidua

14. The extraembryonic epiblast gives rise to or contributes cells to which of the
following germ layers?

a) Ectoderm

b) Mesoderm

c) Endoderm

d) Both A and B

e) All of the above

15. The primitive streak:

753 ‫صفحة‬ Dr. MOHAMED YAHIA 0900987639


QUESTION BANK OF Dr. MOHAMED YAHIA

a) Is derived from the outer cells of the morula

b) Is formed during the second week in development

c) Persists as the cloacal membrane

d) Is the site of involution of epiblast cells to form mesoderm

16. The bilaminar germ disc:

a) Consists of hypoblast and epiblast

b) Is derived from trophoblast cells

c) Both A and B

d) Neither A nor B

17. Meiosis is completed within the gonads during gametogenesis in which sex?

a) Gametogenesis in human female

b) Gametogenesis in human male

c) Both A and B

d) Neither A nor B

18. Female pronucleus are fully formed:

a) Before fertilization

b) After fertilization

c) Both A and B

d) Neither A nor B

19. The conceptus is the:

a) Embryo/fetus

b) Amnion and Chorion

c) Deciduas

754 ‫صفحة‬ Dr. MOHAMED YAHIA 0900987639


QUESTION BANK OF Dr. MOHAMED YAHIA

d) Both A and B

e) All of the above

20. Functions of the placenta include all the following except:

a) Exchange of maternal and fetal blood

b) Exchange of nutrients and electrolytes

c) Hormone production

d) Gas exchange

e) Transmission of maternal antibodies

21. The secondary oocyte undergoes completion of the second maturation


division:

a) Before ovulation

b) At fertilization

c) After ovulation

d) Prenatally

e) At puberty

22. The ovarian follicular cells release:

a) Follicle stimulating hormone

b) Chorionic Gonadotrophin

c) Estrogen

d) Both A and B

755 ‫صفحة‬ Dr. MOHAMED YAHIA 0900987639


QUESTION BANK OF Dr. MOHAMED YAHIA

e) All of the above

23. The most common site for implantation in ectopic pregnancy is the::

a) Internal os of the uterus

b) Mesentary

c) Ovary

d) Oviduct

e) None of the above

24. At the time of initiation of implantation:

a) The endometrium is in the proliferative phase

b) The predominant ovarian hormone is progesterone

c) The conceptus is approximately one day "old" (postfertilization)

d) Both A and B

e) All of the above

25. All is true about primary oocytes except:

a) Are arrested in prophase of the first meiotic division

b) Continue to form after birth

c) Together, with surrounding epithelial cells, for a primordial follicle

d) Suffer from atretion until only approximately 40, 000 are left at puberty

26. Trophoblast and inner cell mass portions are evident in the:

a) Conceptus before implantation has started

b) Conceptus after implantation has started

c) Both A and B

d) Neither A nor B

756 ‫صفحة‬ Dr. MOHAMED YAHIA 0900987639


QUESTION BANK OF Dr. MOHAMED YAHIA

27. At fertilization:

a) The ovum completes the 2nd meiotic division

b) Cleavage is initiated

c) The sex of the individual is determined

d) The diploid number of chromosomes is restored

e) All of the above

28. All the following are derivatives of mesoderm except:

a) Dermatomes

b) Myotomes

c) Sclerotomes

d) Endothelium

e) Yolk sac

29. The prechordal plate:

a) Is a patent inducer region for future head structures

b) Becomes a portion of the oropharyngeal membrane

c) Is formed within the embryonic epiblast

d) Both A and B

e) All of the above

30. At the time of fertilization, the endometrium is in the:

a) Proliferative phase

b) Secretory phase

757 ‫صفحة‬ Dr. MOHAMED YAHIA 0900987639


QUESTION BANK OF Dr. MOHAMED YAHIA

c) Menstral phase

d) Both B and C

31. Which of the following contains abundant nutrient reserves?

a) Female gametes

b) Male gametes

c) Both A and B

d) Neither A nor B

32. Which of the following determines the genetic sex of the zygote?

a) Female gametes

b) Male gametes

c) Both A and B

d) Neither A nor B

33. Which of the following is found in the early placenta?

a) Cytotrophoblast

b) Syncytiotrophoblast

c) Both A and B

d) Neither A nor B

34. The space between the somatopleuric and splanchnopleuric extraembryonic


mesoderm is known as:

a) The primitive yolk sac

b) The secondary yolk sac

c) The blastocele

d) The amniotic cavity

758 ‫صفحة‬ Dr. MOHAMED YAHIA 0900987639


QUESTION BANK OF Dr. MOHAMED YAHIA

e) The extraembryonic coelom

35. As a result of fertilization:

a) Sex of the offspring is determined

b) The diploic number of chromosomes is restored

c) Cleavage divisions are initiated

d) Genetic heterogeneity is maintained

e) All of the above

36. At the time of the first missing menstral period, the conceptus is:

a) 3 days old

b) 5 days old

c) 3 weeks old

d) 5 weeks old

37. Primary oocytes have developed by the time of birt. From puberty to
menopause, these germ cells remain suspended in the meiotic prophase. The
oocyte of a mature follicle is induced to undergo the first meiotic division just
prior to ovulation as a result of which of the following hormonal stimuli:

a) The cessation of progesterone secretion

b) The gradual elevation of FSH titers

c) The low estrogen titers associated with the maturing follicle

d) The slow elevation of progesterone produced by luteal cells

e) The surge of luteinizing hormone initiated by high estrogen titers

759 ‫صفحة‬ Dr. MOHAMED YAHIA 0900987639


QUESTION BANK OF Dr. MOHAMED YAHIA

38. By the 4th day of development, fluid accumulation by the morula results is
separation of the cells into trophoblasts and embryoblasts, or inner cell mass.
From the 8th-12th day of development, the role of trophoblasts includes all of the
following except:

a) Enclosure of inner cell mass and blastocyst cavity

b) Formation of embryo proper

c) Invasion of endometrial epithelium

d) Production of hormones

e) Production of 2 distinct cell populations by differentiation

39. A 26 year old man had viral influenza with fever 39.5o for 3 days. Since
spermatogenesis cannot occur above a scrotal temperature of 35.5 o, he was left
with no viable sperm on his recovery. The time required for spermatogenesis,
spermiogenesis, passage of a viable sperm to the epididymis is approximately:

a) 3 days

b) 1 week

c) 5 weeks

d) 2 months

e) 4 months

40. A 25 year old female states that she has a very regular menstrual cycle that is
24 days in length. The projected time of ovulation for this woman would be
about the:

760 ‫صفحة‬ Dr. MOHAMED YAHIA 0900987639


QUESTION BANK OF Dr. MOHAMED YAHIA

a) 14th day following start of her last menstrual period

b) 12th day following end of her last menstrual period

c) 12th day following end of her last menstrual period

d) 10th day following end of her last menstrual period

e) 10th day following beginning of her last menstrual period

41. The chorionic villi of the placenta are most numerous in the region of the:

a) Decidua capsularis

b) Chorionic plate

c) Basal plate

d) Maternal septae

e) Decidua basalis

42. The following list contains components of the "placental barrier" during the
first trimester of pregnancy. Which one component is disappearing during late
pregnancy?

a) The endothelial lining of fetal capillaries

b) The cytotrophoblast

c) The syncytiotrophoblast

d) The basement membrane of fetal capillaries

e) None of the above

43. All of the following placental transport mechanism require the expenditure of
cellular energy except:

a) Active transport

b) Pinocytosis

c) Simple diffusion

d) Facilitated diffusion

761 ‫صفحة‬ Dr. MOHAMED YAHIA 0900987639


QUESTION BANK OF Dr. MOHAMED YAHIA

44. A full-term male infant has vomited 1 hour after suckling. There has been
failure to gait during the first 2 weeks postnatal. The vomitus is not bile-stained
and no respiratory dilation is evident. Wxamination reveals an abdomen neither
tense nor bloated. The most probable explaination is:

a) Congenital hypertrophic pyloric stenosis

b) Duodenal atresia

c) Patient ileal diverticulum

d) Imperforate anus

e) Tracheo-esophageal fistula

45. A fistula is found that runs between the palatine tonsil fossa and the skin of
the neck immediately anterior to the sternocleidomastoid muscle. Developmetal
failure occured:

a) With a rupture of the tissue between the third branchial cleft and third

pharyngeal pouch

b) With a failure to close the first branchial cleft

c) With a failure to obliterate the cervical sinus

d) Both A and B

e) All of the above

46. Which statement about fetal blood vessels is false:

762 ‫صفحة‬ Dr. MOHAMED YAHIA 0900987639


QUESTION BANK OF Dr. MOHAMED YAHIA

a) The ductus arteriosus carries blood from the aorta to the pulmonary trunk

b) The foramen ovale carries blood from the right atrium to the left atrium

c) The umbilical vein carries blood from the umbilical cord via the ductus

venosus to the inferior vena cave

d) Implantation is completed

e) None of the above

47. Regarding the blastocyst all the following are true except:

a) Has more cells than a morula

b) Has inner cell mass

c) Has syncytotrophoblast

d) Has trophoblast cells

e) May have zona pellucida

48. Derivatives of the mesodermal cell layer of the embryo include all the
following except:

a) Adrenal cortex

b) Gonads

c) Peritoneal serosa

d) Spleen

e) Liver parenchyma

49. In the developing embryo, the first bone to begin ossification is the:

a) First rib

b) Humerus

c) Tibia

d) Scapula

763 ‫صفحة‬ Dr. MOHAMED YAHIA 0900987639


QUESTION BANK OF Dr. MOHAMED YAHIA

e) Clavicle

50. Fallot's tetralogy includes all the following except:

a) Pulmonary stenosis

b) Aortic overrinding

c) Ventricular septal defect

d) Patency of ductus arteriosus

e) Hypertrophy of the right ventricle

51. The trachea develops from the:

a) Septum transversum

b) Fifth branchial arch

c) Thyroglossal canal

d) Pulmonary mesenchyme

e) Foregut

52. In the developing pancreas, the dorsal bud contributes all the following
except:

a) Body of the pancreas

b) Distal portion of the main pancreatic duct

c) Secondary pancreatic duct

d) Uncinate process

e) Upper half of the head of the pancreas

764 ‫صفحة‬ Dr. MOHAMED YAHIA 0900987639


QUESTION BANK OF Dr. MOHAMED YAHIA

53. All of the following structures contribute in development of diaphragm


except:

a) Body wall

b) Primitive mesoesophagus

c) Septum transversum

d) Pleuro-pericardial membrane

e) Pleuro-peritoneal membrane

54. Spina bifida is a failure of fusion between the:

a) Left and right halves of the vertebral column

b) Pedicle and centrum of the vertebra

c) Left and right laminae of the vertebra

d) Both A and B

e) All of the above

55. A urachal fistula represents a remnant of the:

a) Cloaca

b) Allantois

c) Urogenital sinus

d) Hindgut

e) Mesonephric duct

56. What is the origin of the mucosa of the middle ear:

a) Neural ectoderm

b) Surface ectoderm

c) Mesoderm

d) Endoderm

765 ‫صفحة‬ Dr. MOHAMED YAHIA 0900987639


QUESTION BANK OF Dr. MOHAMED YAHIA

e) None of the above

57. The left sixth aortic arch gives rise to the:

a) Hemiazygos vein

b) Coronary sinus

c) Ligamentum arteriosum

d) Ligamentum venosum

e) No remnants are found in this structure in the adult

58. The posterior pituitary is of which origin:

a) Endoderm

b) Mesoderm

c) Ectoderm

d) Endoderm and mesoderm

e) Endoderm and ectoderm

59. The parathyroid glands:

a) They are 3 in number

b) They secrete Calcitonia

c) They develop from the 2nd pharyngeal arch

d) They normally lie anterior to the thyroid gland

e) They develop from the 3rd and 4th pharyngeal arches (pouches)

60. Unilateral upper lip cleft is due to failure of fusion of:

a) Medial nasal processes

b) Maxillary and lateral nasal processes

c) Maxillary and medial nasal processes on one side

766 ‫صفحة‬ Dr. MOHAMED YAHIA 0900987639


QUESTION BANK OF Dr. MOHAMED YAHIA

d) Maxillary and mandibular processes

e) Medial and lateral nasal processes unilateralla

61. In which of the following is precursor cell mitosis evident after puberty:

a) Female gametes

b) Male gametes

c) Both A and B

d) Neither A nor B

62. What is the origin of the mucosa of the oral pharynx:

a) Branchial arch I

b) Branchial arch II

c) Branchial arch III

d) Branchial arch IV

e) Not of branchial arch origin

63. Which of the following is formed from portions of one sclerotome:

a) Vertebral centrum

b) Intervertebral disc (annulus fibrosus)

c) Both A and B

d) Neither A nor B

64. The clavicle is:

a) The first bone to begin ossification

b) The first bone to complete ossification

c) Intramembranous in origin

d) Both A and C

767 ‫صفحة‬ Dr. MOHAMED YAHIA 0900987639


QUESTION BANK OF Dr. MOHAMED YAHIA

e) All of the above

65. Limb musculature:

a) Relies on spinal nerve to aid the differentiation process

b) Begins as mesenchymal condensation

c) Is derived from somatic mesoderm

d) Splits into flexor and extensor compartments

e) All of the above

66. Cervical sinus develop from:

a) Pharyngeal pouch 1

b) Pharyngeal pouch 2

c) Pharyngeal pouch 3

d) Pharyngeal pouch 4

e) This structure is not derived from pharyngeal pouch

67. Parafollicular cells of the thyroid gland develop from:

a) Pharyngeal pouch 1

b) Pharyngeal pouch 2

c) Pharyngeal pouch 3

d) Pharyngeal pouch 4

e) This structure is not derived from pharyngeal pouch

68. During the third to sixth months of pregnancy, the structure primarily
responsible for erythropoiesis is the:

768 ‫صفحة‬ Dr. MOHAMED YAHIA 0900987639


QUESTION BANK OF Dr. MOHAMED YAHIA

a) Bone marrow

b) Liver

c) Spleen

d) Thymus

e) Yolk sac

69. Structures derived from the hind gut in the male include all the following
except the:

a) Allantois

b) Descending colon

c) Inferior portion of the anal canal

d) Prostatic urethtra

e) Urinary bladder

70. With regard to the formation of the placenta all of the following are false
except:

a) Tertiary chorionic villi differ from primary chorionic villi by the presence

of blood vessels

b) Primary chorionic villi contain only syncytiotrophoblast

c) The intervillous space contains fetal blood

d) The deciduas basalis and the chorion leave constitute the placenta

e) The umbilical cord contains a loose connective tissue called Wharton's jelly

769 ‫صفحة‬ Dr. MOHAMED YAHIA 0900987639


QUESTION BANK OF Dr. MOHAMED YAHIA

71. The endocardial cushions are involved in the formation of all of the following
except:

a) Right and left atrioventricular canals

b) Pectinate muscles

c) Atrioventricular valves

d) Membranous part of the interventricular septum

72. Which of the following is the most common type of congenital heart
malformation?

a) Dextrocardia

b) Atrial septal defect

c) Ectopic cordis

d) Ventricular septal defect

e) Transposition of the great vessels

73. The adult left atrium is derived embryologically primarily from the:

a) Right horn of the sinus venosus

b) Primitive pulmonary vein

c) Sinus venarum

d) Bulbus cordis

e) Ledt horn of the sinus venosus

74. The primitive aortic arch pattern is transformed into an adult arterial
arrangement. Which pair of arteries (embryonic/adult) is incorrect:

a) 6th arch/ pulmonary artery

b) Right 4th arch/ right subclavian

c) Left 4th arch/ left subclavian

d) 2nd arch/ stapedial artery

770 ‫صفحة‬ Dr. MOHAMED YAHIA 0900987639


QUESTION BANK OF Dr. MOHAMED YAHIA

e) 3rd arch/ common and internal carotid

75. Azygos vein is derived from the:

a) Right vitelline vein

b) Right subcardinal vein

c) Right anterior cardinal

d) Right supracardinal vein

e) All of the above

76. All of the following are true concerning development of the bronchi except:

a) Lung buds first form at the caudal end of the laryngotracheal tube

b) During the 5th week of development, the lung bud grows laterally into the

primitive pleural cavities (pericardioperitoneal canals)

c) Third generation bronchi supply bronchopulmonary segments

d) The pulmonary cartilage, smooth muscle, and connective tissue are derived
from the lung bud endoderm

e) By 24 weeks of development, respiratory bronchioles are present

77. Tracheoesophageal fistula with blind ending esophagus results in:

a) Polyhydraminos (more than usual amount of amniontic fluid)

b) Oligohydraminos (less than usual amount of amniotic fluid)

c) Both A and B

d) Neither A nor B

771 ‫صفحة‬ Dr. MOHAMED YAHIA 0900987639


QUESTION BANK OF Dr. MOHAMED YAHIA

78. The definitive diaphragm is formed by fusions of all except one of the
following:

a) Mesoesophagus

b) Septum transversum

c) Pleuroperitoneal membranes

d) Mesentery of duct of Cavier

e) Mesoderm of the body wall

79. Paraxial (somite) mesoderm gives rise to:

a) Muscles of mastication

b) Muscles in the stomach

c) Both A and B

d) Neither A nor B

80. The embryonic epiblast gives rise to or contributes cells to which of the
following germ layers?

a) Ectoderm

b) Mesoderm

c) Endoderm

d) Both A and B

e) All of the above

81. Gonia cells produced during gametogenesis are usually present in 65-year old
individuals of which sex:

a) Gametogenesis in human female

b) Gametogenesis in human male

c) Both A and B

772 ‫صفحة‬ Dr. MOHAMED YAHIA 0900987639


QUESTION BANK OF Dr. MOHAMED YAHIA

d) Neither A nor B

82. All of the following are true of amniotic fluid except:

a) It is derived from maternal blood

b) It is swallowed by the fetus from the beginning of the 5th month

c) It restricts fetal movement within the amniotic cavity

d) It absorbs jolts to the fetus

83. Epithelium of the tongue posterior to the sulcus terminals is derived from
the:

a) First branchial arch

b) Second branchial arch

c) Third branchial arch

d) Fourth branchial arch

e) Not of branchial arch origin

84. What is the origin of the superior parathyroid glands?

a) Pharyngeal pouch I

b) Pharyngeal pouch II

c) Pharyngeal pouch III

d) Pharyngeal pouch IV

e) Not of pharyngeal pouch origin

85. What is the origin of the mandible?

a) Branchial arch I

b) Branchial arch II

773 ‫صفحة‬ Dr. MOHAMED YAHIA 0900987639


QUESTION BANK OF Dr. MOHAMED YAHIA

c) Branchial arch III

d) Branchial arch IV

e) Not of the branchial arch origin

86. The thyroid gland:

a) Originates from an endodermal outgrowth at the base of the primordial

tongue

b) Is usually detached from the Thyroglossal duct in the adult

c) Both A and B

d) Neither A nor B

774 ‫صفحة‬ Dr. MOHAMED YAHIA 0900987639


‫‪QUESTION BANK OF Dr. MOHAMED YAHIA‬‬

‫أسأل هللا للجميع التوفيق و‬


‫النجاح‬
‫أوصيكم بالقراءة ثم حل‬
‫األسئلة ‪ -‬الهدف من االسئلة‬
‫فقط المراجعة‬
‫مع تحياتي‪:‬‬
‫د‪ -‬محمد يحيى ابوالقاسم ختم‬
‫‪REGISTRAR OF OBS & GYNAE .. R6‬‬

‫صفحة ‪775‬‬ ‫‪Dr. MOHAMED YAHIA 0900987639‬‬

You might also like